АдрСс: 105678, Π³. Москва, ШоссС Энтузиастов, Π΄. 55 (ΠšΠ°Ρ€Ρ‚Π° ΠΏΡ€ΠΎΠ΅Π·Π΄Π°)
ВрСмя Ρ€Π°Π±ΠΎΡ‚Ρ‹: ПН-ПВ: с 9.00 Π΄ΠΎ 18.00, Π‘Π‘: с 9.00 Π΄ΠΎ 14.00

Как ΡƒΠ²Π΅Π»ΠΈΡ‡ΠΈΡ‚ΡŒ Ρ‚Π΅ΠΏΠ»ΠΎΠΎΡ‚Π΄Π°Ρ‡Ρƒ ΠΊΠΎΠ½Π²Π΅ΠΊΡ‚ΠΎΡ€Π°: 3 простых Ρ€Π΅ΡˆΠ΅Π½ΠΈΡ ΠΊΠ°ΠΊ ΡƒΠ²Π΅Π»ΠΈΡ‡ΠΈΡ‚ΡŒ Ρ‚Π΅ΠΏΠ»ΠΎΠΎΡ‚Π΄Π°Ρ‡Ρƒ Π±Π°Ρ‚Π°Ρ€Π΅ΠΈ. ΠŸΠΎΠ²Ρ‹ΡˆΠ°Π΅ΠΌ Ρ‚Π΅ΠΏΠ»ΠΎΠ²ΡƒΡŽ ΠΎΡ‚Π΄Π°Ρ‡Ρƒ Π² ΠΎΡ‚ΠΎΠΏΠΈΡ‚Π΅Π»ΡŒΠ½Ρ‹ΠΉ ΠΏΠ΅Ρ€ΠΈΠΎΠ΄

Π‘ΠΎΠ΄Π΅Ρ€ΠΆΠ°Π½ΠΈΠ΅

Π’Π΅ΠΏΠ»ΠΎΠΎΡ‚Π΄Π°Ρ‡Π° Π²Π½ΡƒΡ‚Ρ€ΠΈΠΏΠΎΠ»ΡŒΠ½Ρ‹Ρ… ΠΊΠΎΠ½Π²Π΅ΠΊΡ‚ΠΎΡ€ΠΎΠ²

ΠŸΡ€Π°ΠΊΡ‚ΠΈΡ‡Π΅ΡΠΊΠΈ ΠΊΠ°ΠΆΠ΄Ρ‹ΠΉ Π³ΠΎΠ΄ Π½Π° отСчСствСнном ΡΡ‚Ρ€ΠΎΠΈΡ‚Π΅Π»ΡŒΠ½ΠΎΠΌ Ρ€Ρ‹Π½ΠΊΠ΅ ΠΏΠΎΡΠ²Π»ΡΡŽΡ‚ΡΡ Π½ΠΎΠ²Ρ‹Π΅ Ρ‚Π΅Ρ…Π½ΠΎΠ»ΠΎΠ³ΠΈΠΈ ΠΈ ΠΌΠ°Ρ‚Π΅Ρ€ΠΈΠ°Π»Ρ‹. Они ΠΊΠ°ΡΠ°ΡŽΡ‚ΡΡ самых Ρ€Π°Π·Π»ΠΈΡ‡Π½Ρ‹Ρ… сторон возвСдСния ΠΈ эксплуатации Π·Π΄Π°Π½ΠΈΠΉ ΠΈ сооруТСний. Π’ числС ΠΎΡ‚Π½ΠΎΡΠΈΡ‚Π΅Π»ΡŒΠ½ΠΎ Π½Π΅Π΄Π°Π²Π½ΠΎ ΠΏΠΎΡΠ²ΠΈΠ²ΡˆΠΈΡ…ΡΡ ΠΈ сразу ΠΏΠΎΠ»ΡƒΡ‡ΠΈΠ²ΡˆΠΈΡ… ΡΠ΅Ρ€ΡŒΠ΅Π·Π½ΠΎΠ΅ распространСниС Ρ‚Π΅Ρ…Π½ΠΎΠ»ΠΎΠ³ΠΈΠΉ слСдуСт ΠΎΡ‚ΠΌΠ΅Ρ‚ΠΈΡ‚ΡŒ способ отоплСния ΠΏΠΎΠΌΠ΅Ρ‰Π΅Π½ΠΈΠΉ ΠΏΡ€ΠΈ ΠΏΠΎΠΌΠΎΡ‰ΠΈ Π²Π½ΡƒΡ‚Ρ€ΠΈΠΏΠΎΠ»ΡŒΠ½Ρ‹Ρ… ΠΊΠΎΠ½Π²Π΅ΠΊΡ‚ΠΎΡ€ΠΎΠ².

Π’Π°ΠΆΠ½ΠΎ ΠΎΡ‚ΠΌΠ΅Ρ‚ΠΈΡ‚ΡŒ, Ρ‡Ρ‚ΠΎ ΠΏΡ€ΠΈΠΌΠ΅Π½Π΅Π½ΠΈΠ΅ этого Π²Π°Ρ€ΠΈΠ°Π½Ρ‚Π° ΠΎΠ±ΠΎΠ³Ρ€Π΅Π²Π° прСдоставляСт Π²Π»Π°Π΄Π΅Π»ΡŒΡ†Ρƒ Π΄ΠΎΠΌΠ°, офиса ΠΈΠ»ΠΈ Ρ‚ΠΎΡ€Π³ΠΎΠ²ΠΎΠ³ΠΎ Ρ†Π΅Π½Ρ‚Ρ€Π° большиС возмоТности Π² части создания ΡΡ‚ΠΈΠ»ΡŒΠ½Ρ‹Ρ… ΠΈ ΠΎΡ€ΠΈΠ³ΠΈΠ½Π°Π»ΡŒΠ½Ρ‹Ρ… ΠΈΠ½Ρ‚Π΅Ρ€ΡŒΠ΅Ρ€ΠΎΠ². Однако, Π·Π½Π°Ρ‡ΠΈΡ‚Π΅Π»ΡŒΠ½ΡƒΡŽ Ρ‡Π°ΡΡ‚ΡŒ ΠΏΠΎΡ‚Π΅Π½Ρ†ΠΈΠ°Π»ΡŒΠ½Ρ‹Ρ… ΠΏΠΎΡ‚Ρ€Π΅Π±ΠΈΡ‚Π΅Π»Π΅ΠΉ интСрСсуСт вопрос, насколько эффСктивным являСтся ΠΎΡ‚ΠΎΠΏΠ»Π΅Π½ΠΈΠ΅ с ΠΏΡ€ΠΈΠΌΠ΅Π½Π΅Π½ΠΈΠ΅ΠΌ Π²Π½ΡƒΡ‚Ρ€ΠΈΠΏΠΎΠ»ΡŒΠ½Ρ‹Ρ… ΠΊΠΎΠ½Π²Π΅ΠΊΡ‚ΠΎΡ€ΠΎΠ². Для ΠΎΡ‚Π²Π΅Ρ‚Π° Π½Π° Π½Π΅Π³ΠΎ Π½Π΅ΠΎΠ±Ρ…ΠΎΠ΄ΠΈΠΌΠΎ Ρ€Π°Π·ΠΎΠ±Ρ€Π°Ρ‚ΡŒΡΡ с понятиСм Ρ‚Π΅ΠΏΠ»ΠΎΠΎΡ‚Π΄Π°Ρ‡ΠΈ ΠΊΠΎΠ½Π²Π΅ΠΊΡ‚ΠΎΡ€ΠΎΠ² ΠΈ основными Ρ„Π°ΠΊΡ‚ΠΎΡ€Π°ΠΌΠΈ, ΠΎΡ‚ ΠΊΠΎΡ‚ΠΎΡ€Ρ‹Ρ… зависит этот ваТнСйший для систСмы отоплСния ΠΏΠΎΠΊΠ°Π·Π°Ρ‚Π΅Π»ΡŒ.

Β 

Π§Ρ‚ΠΎ Ρ‚Π°ΠΊΠΎΠ΅ Ρ‚Π΅ΠΏΠ»ΠΎΠΎΡ‚Π΄Π°Ρ‡Π° ΠΈ Ρ„Π°ΠΊΡ‚ΠΎΡ€Ρ‹, Π²Π»ΠΈΡΡŽΡ‰ΠΈΠ΅ Π½Π° этот ΠΏΠ°Ρ€Π°ΠΌΠ΅Ρ‚Ρ€

Π’Π΅ΠΏΠ»ΠΎΠΎΡ‚Π΄Π°Ρ‡Π° ΠΎΠ±ΠΎΠ³Ρ€Π΅Π²Π°Ρ‚Π΅Π»ΡŒΠ½ΠΎΠ³ΠΎ ΠΏΡ€ΠΈΠ±ΠΎΡ€Π° ΠΏΠΎΠΊΠ°Π·Ρ‹Π²Π°Π΅Ρ‚ Π΅Π³ΠΎ ΠΌΠΎΡ‰Π½ΠΎΡΡ‚ΡŒ.

Под этим ΠΏΠ°Ρ€Π°ΠΌΠ΅Ρ‚Ρ€ΠΎΠΌ понимаСтся количСство Ρ‚Π΅ΠΏΠ»Π°, ΠΊΠΎΡ‚ΠΎΡ€ΠΎΠ΅ выдСляСтся ΠΊΠΎΠ½Π²Π΅ΠΊΡ‚ΠΎΡ€ΠΎΠΌ ΠΈΠ»ΠΈ Π±Π°Ρ‚Π°Ρ€Π΅Π΅ΠΉ Π·Π° Π΅Π΄ΠΈΠ½ΠΈΡ†Ρƒ Π²Ρ€Π΅ΠΌΠ΅Π½ΠΈ. Π§Π΅ΠΌ Π²Ρ‹ΡˆΠ΅ Π·Π½Π°Ρ‡Π΅Π½ΠΈΠ΅ показатСля, Ρ‚Π΅ΠΌ Π±ΠΎΠ»Π΅Π΅ эффСктивным являСтся элСмСнт ΠΎΡ‚ΠΎΠΏΠΈΡ‚Π΅Π»ΡŒΠ½ΠΎΠΉ систСмы. Учитывая сказанноС, становится понятным, ΠΏΠΎΡ‡Π΅ΠΌΡƒ ΠΈΠΌΠ΅Π½Π½ΠΎ Ρ‚Π΅ΠΏΠ»ΠΎΠΎΡ‚Π΄Π°Ρ‡Π° выступаСт ΠΊΠ»ΡŽΡ‡Π΅Π²Ρ‹ΠΌ ΠΏΠΎΠΊΠ°Π·Π°Ρ‚Π΅Π»Π΅ΠΌ ΠΏΡ€ΠΈ Π²Ρ‹Π±ΠΎΡ€Π΅ подходящСго Π²Π°Ρ€ΠΈΠ°Π½Ρ‚Π° ΠΎΠ±ΠΎΠ³Ρ€Π΅Π²Π° здания ΠΈ ΠΊΠΎΠΌΠΏΠ»Π΅ΠΊΡ‚ΠΎΠ²Π°Π½ΠΈΠΈ систСмы отоплСния.

Для Ρ‚ΠΎΠ³ΠΎ, Ρ‡Ρ‚ΠΎΠ±Ρ‹ ΠΎΡ†Π΅Π½ΠΈΡ‚ΡŒ ΡƒΡ€ΠΎΠ²Π΅Π½ΡŒ Ρ‚Π΅ΠΏΠ»ΠΎΠΎΡ‚Π΄Π°Ρ‡ΠΈ Π²Π½ΡƒΡ‚Ρ€ΠΈΠΏΠΎΠ»ΡŒΠ½Ρ‹Ρ… ΠΊΠΎΠ½Π²Π΅ΠΊΡ‚ΠΎΡ€ΠΎΠ², Π½Π΅ΠΎΠ±Ρ…ΠΎΠ΄ΠΈΠΌΠΎ ΠΏΠΎΠ½ΠΈΠΌΠ°Ρ‚ΡŒ основныС Ρ„Π°ΠΊΡ‚ΠΎΡ€Ρ‹, ΠΊΠΎΡ‚ΠΎΡ€Ρ‹Π΅ Π²Π»ΠΈΡΡŽΡ‚ Π·Π° Π·Π½Π°Ρ‡Π΅Π½ΠΈΠ΅ этого Π²Π°ΠΆΠ½ΠΎΠ³ΠΎ ΠΏΠ°Ρ€Π°ΠΌΠ΅Ρ‚Ρ€Π°.

Π‘Ρ€Π΅Π΄ΠΈ Π½ΠΈΡ… Π½Π΅ΠΎΠ±Ρ…ΠΎΠ΄ΠΈΠΌΠΎ Π²Ρ‹Π΄Π΅Π»ΠΈΡ‚ΡŒ Ρ‚Ρ€ΠΈ Π½Π°ΠΈΠ±ΠΎΠ»Π΅Π΅ Π·Π½Π°Ρ‡ΠΈΠΌΡ‹Ρ…, Π° ΠΈΠΌΠ΅Π½Π½ΠΎ:

  1. ΠœΠ°Ρ‚Π΅Ρ€ΠΈΠ°Π» изготовлСния. НаибольшиС коэффициСнтом Ρ‚Π΅ΠΏΠ»ΠΎΠΎΡ‚Π΄Π°Ρ‡ΠΈ срСди ΠΌΠ΅Ρ‚Π°Π»Π»ΠΎΠ², ΠΈΡΠΏΠΎΠ»ΡŒΠ·ΡƒΠ΅ΠΌΡ‹Ρ… для изготовлСния ΠΎΡ‚ΠΎΠΏΠΈΡ‚Π΅Π»ΡŒΠ½Ρ‹Ρ… ΠΏΡ€ΠΈΠ±ΠΎΡ€ΠΎΠ², ΠΎΠ±Π»Π°Π΄Π°Π΅Ρ‚ мСдь. Π”Π°Π»Π΅Π΅ с Π·Π°ΠΌΠ΅Ρ‚Π½Ρ‹ΠΌ ΠΎΡ‚Ρ€Ρ‹Π²ΠΎΠΌ ΠΈΠ΄ΡƒΡ‚ алюминий, ΡΡ‚Π°Π»ΡŒ, Π° Π½Π° послСднСм мСстС — Ρ‡ΡƒΠ³ΡƒΠ½. Π“Π»Π°Π²Π½Ρ‹ΠΌ нСдостатком ΠΌΠ΅Π΄ΠΈ выступаСт Π½ΠΈΠ·ΠΊΠΈΠΉ ΡƒΡ€ΠΎΠ²Π΅Π½ΡŒ прочностных ΠΏΠ°Ρ€Π°ΠΌΠ΅Ρ‚Ρ€ΠΎΠ². Однако, конструктивныС особСнности встраиваСмых Π² ΠΏΠΎΠ» ΠΊΠΎΠ½Π²Π΅ΠΊΡ‚ΠΎΡ€ΠΎΠ² ΠΏΠΎΠ·Π²ΠΎΠ»ΡΡŽΡ‚ Π±Π΅Π· ΠΏΡ€ΠΎΠ±Π»Π΅ΠΌ ΠΈΡΠΏΠΎΠ»ΡŒΠ·ΠΎΠ²Π°Ρ‚ΡŒ этот ΠΌΠ΅Ρ‚Π°Π»Π», ΠΊΠΎΡ‚ΠΎΡ€Ρ‹ΠΉ ΠΎΠ±Ρ‹Ρ‡Π½ΠΎ дополняСтся алюминиСм.
    Π’ Ρ€Π΅Π·ΡƒΠ»ΡŒΡ‚Π°Ρ‚Π΅, Π²Π½ΡƒΡ‚Ρ€ΠΈΠΏΠΎΠ»ΡŒΠ½Ρ‹Π΅ Ρ€Π°Π΄ΠΈΠ°Ρ‚ΠΎΡ€Ρ‹ заслуТСнно ΡΡ‡ΠΈΡ‚Π°ΡŽΡ‚ΡΡ ΠΎΠ΄Π½ΠΈΠΌ ΠΈΠ· Π»ΡƒΡ‡ΡˆΠΈΡ… Π²Π°Ρ€ΠΈΠ°Π½Ρ‚ΠΎΠ² для ΠΎΠ±ΠΎΠ³Ρ€Π΅Π²Π° Π² ΠΏΠ»Π°Π½Π΅ уровня Ρ‚Π΅ΠΏΠ»ΠΎΠΎΡ‚Π΄Π°Ρ‡ΠΈ.
  2. Π Π°Π·ΠΌΠ΅Ρ‰Π΅Π½ΠΈΠ΅ ΠΈ Ρ‚ΠΈΠΏ ΠΏΠΎΠ΄ΠΊΠ»ΡŽΡ‡Π΅Π½ΠΈΡ ΠΊΠΎΠ½Π²Π΅ΠΊΡ‚ΠΎΡ€ΠΎΠ² ΠΈΠ»ΠΈ Π±Π°Ρ‚Π°Ρ€Π΅ΠΉ. Π“Π»Π°Π²Π½ΠΎΠ΅ ΠΏΡ€Π°Π²ΠΈΠ»ΠΎ, ΠΊΠΎΡ‚ΠΎΡ€ΠΎΠΌΡƒ ΡΠ»Π΅Π΄ΡƒΡŽΡ‚ ΠΏΡ€ΠΈ устройствС ΠΎΡ‚ΠΎΠΏΠΈΡ‚Π΅Π»ΡŒΠ½ΠΎΠΉ систСмы – располоТСниС Ρ€Π°Π΄ΠΈΠ°Ρ‚ΠΎΡ€ΠΎΠ² ΠΏΠΎ Π½Π°Ρ€ΡƒΠΆΠ½ΠΎΠΌΡƒ ΠΏΠ΅Ρ€ΠΈΠΌΠ΅Ρ‚Ρ€Ρƒ здания с ΠΎΠ±ΡΠ·Π°Ρ‚Π΅Π»ΡŒΠ½Ρ‹ΠΌ присутствиСм ΠΊΠΎΠ½Π²Π΅ΠΊΡ‚ΠΎΡ€Π° ΠΏΠΎΠ΄ ΠΊΠ°ΠΆΠ΄Ρ‹ΠΌ ΠΎΠΊΠ½ΠΎΠΌ. Π­Ρ‚ΠΎ ΡƒΡ‚Π²Π΅Ρ€ΠΆΠ΄Π΅Π½ΠΈΠ΅ являСтся справСдливым ΠΊΠ°ΠΊ для ΠΎΠ±Ρ‹Ρ‡Π½Ρ‹Ρ… Π±Π°Ρ‚Π°Ρ€Π΅ΠΉ, Ρ‚Π°ΠΊ ΠΈ для Π²Π½ΡƒΡ‚Ρ€ΠΈΠΏΠΎΠ»ΡŒΠ½Ρ‹Ρ… ΠΊΠΎΠ½Π²Π΅ΠΊΡ‚ΠΎΡ€ΠΎΠ². ΠŸΠΎΡΡ‚ΠΎΠΌΡƒ Π΄Π°Π½Π½Ρ‹ΠΉ Ρ„Π°ΠΊΡ‚ΠΎΡ€ Π½Π΅ влияСт Π½Π° ΡΠΎΠΎΡ‚Π½ΠΎΡˆΠ΅Π½ΠΈΠ΅ показатСля Ρ‚Π΅ΠΏΠ»ΠΎΠΎΡ‚Π΄Π°Ρ‡ΠΈ ΠΏΡ€ΠΈ сравнСнии Ρ€Π°Π·Π»ΠΈΡ‡Π½Ρ‹Ρ… Π²ΠΈΠ΄ΠΎΠ² ΠΏΡ€ΠΈΠ±ΠΎΡ€ΠΎΠ² отоплСния.
  3. Β 

  4. Π‘ΠΊΠΎΡ€ΠΎΡΡ‚ΡŒ ΠΈ Ρ‚Π΅ΠΌΠΏΠ΅Ρ€Π°Ρ‚ΡƒΡ€Π° тСплоноситСля. Π—Π°Π²ΠΈΡΠΈΠΌΠΎΡΡ‚ΡŒ Ρ‚Π΅ΠΏΠ»ΠΎΠΎΡ‚Π΄Π°Ρ‡ΠΈ ΠΎΡ‚ стСпСни Π½Π°Π³Ρ€Π΅Π²Π° тСплоноситСля ΠΈ скорости Π΅Π³ΠΎ пСрСмСщСния ΠΏΠΎ ΠΎΡ‚ΠΎΠΏΠΈΡ‚Π΅Π»ΡŒΠ½ΠΎΠΉ систСмС ΠΎΡ‡Π΅Π²ΠΈΠ΄Π½Π°. Но ΠΏΡ€ΠΈ рассмотрСнии этого Ρ„Π°ΠΊΡ‚ΠΎΡ€Π° ΠΊΡ€Π°ΠΉΠ½Π΅ Π²Π°ΠΆΠ½ΠΎ ΠΏΠΎΠ½ΠΈΠΌΠ°Ρ‚ΡŒ, Ρ‡Ρ‚ΠΎ Π±ΠΎΠ»Π΅Π΅ высокая Ρ‚Π΅ΠΌΠΏΠ΅Ρ€Π°Ρ‚ΡƒΡ€Π° сопровоТдаСтся ΠΊΠ°ΠΊ ростом эксплуатационных расходов Π½Π° ΠΎΠ±ΠΎΠ³Ρ€Π΅Π² здания, Ρ‚Π°ΠΊ ΠΈ ΡƒΠ²Π΅Π»ΠΈΡ‡Π΅Π½ΠΈΠ΅ΠΌ уровня ΠΏΠΎΡ‚Π΅Π½Ρ†ΠΈΠ°Π»ΡŒΠ½ΠΎΠΉ опасности ΠΏΠΎΠΆΠ°Ρ€Π°.

Π’Π°ΠΆΠ½ΠΎ ΠΎΡ‚ΠΌΠ΅Ρ‚ΠΈΡ‚ΡŒ, Ρ‡Ρ‚ΠΎ использованиС ΠΏΡ€ΠΈ ΠΈΠ·Π³ΠΎΡ‚ΠΎΠ²Π»Π΅Π½ΠΈΠΈ Π²Π½ΡƒΡ‚Ρ€ΠΈΠΏΠΎΠ»ΡŒΠ½Ρ‹Ρ… ΠΊΠΎΠ½Π²Π΅ΠΊΡ‚ΠΎΡ€ΠΎΠ² алюминия ΠΈ, Ρ‡Ρ‚ΠΎ особСнно Π²Π°ΠΆΠ½ΠΎ, ΠΌΠ΅Π΄ΠΈ, ΠΎΠ±Π»Π°Π΄Π°ΡŽΡ‰ΠΈΡ… высокой Ρ‚Π΅ΠΏΠ»ΠΎΠΎΡ‚Π΄Π°Ρ‡Π΅ΠΉ, позволяСт ΡƒΠΌΠ΅Π½ΡŒΡˆΠΈΡ‚ΡŒ Ρ‚Π΅ΠΌΠΏΠ΅Ρ€Π°Ρ‚ΡƒΡ€Ρƒ тСплоноситСля Π² систСмС, добившись ΠΏΡ€ΠΈ этом высокой эффСктивности ΠΎΠ±ΠΎΠ³Ρ€Π΅Π²Π°. Как слСдствиС, Π³Ρ€Π°ΠΌΠΎΡ‚Π½ΠΎΠ΅ ΠΏΡ€ΠΈΠΌΠ΅Π½Π΅Π½ΠΈΠ΅ΠΌ встраиваСмых Π² ΠΏΠΎΠ» Ρ€Π°Π΄ΠΈΠ°Ρ‚ΠΎΡ€ΠΎΠ² позволяСт ΡΠ½ΠΈΠ·ΠΈΡ‚ΡŒ расходы Π½Π° ΠΎΡ‚ΠΎΠΏΠ»Π΅Π½ΠΈΠ΅ ΠΈ ΡƒΠ²Π΅Π»ΠΈΡ‡ΠΈΡ‚ΡŒ ΡΡ‚Π΅ΠΏΠ΅Π½ΡŒ бСзопасности эксплуатации ΠΎΡ‚ΠΎΠΏΠΈΡ‚Π΅Π»ΡŒΠ½ΠΎΠΉ систСмы.

Π”Π°ΠΆΠ΅ повСрхностный Π°Π½Π°Π»ΠΈΠ· Ρ„Π°ΠΊΡ‚ΠΎΡ€ΠΎΠ², Π²Π»ΠΈΡΡŽΡ‰ΠΈΡ… Π½Π° ΡƒΡ€ΠΎΠ²Π΅Π½ΡŒ Ρ‚Π΅ΠΏΠ»ΠΎΠΎΡ‚Π΄Π°Ρ‡ΠΈ, наглядно ΠΏΠΎΠΊΠ°Π·Ρ‹Π²Π°Π΅Ρ‚, Ρ‡Ρ‚ΠΎ Π²Π½ΡƒΡ‚Ρ€ΠΈΠΏΠΎΠ»ΡŒΠ½Ρ‹Π΅ ΠΊΠΎΠ½Π²Π΅ΠΊΡ‚ΠΎΡ€Ρ‹ ΠΏΠΎ ΠΌΠ½ΠΎΠ³ΠΈΠΌ ΠΏΠ°Ρ€Π°ΠΌΠ΅Ρ‚Ρ€Π°ΠΌ Π½Π΅ Ρ‚ΠΎΠ»ΡŒΠΊΠΎ Π½Π΅ ΡƒΡΡ‚ΡƒΠΏΠ°ΡŽΡ‚ стандартным батарСям, Π½ΠΎ ΠΈ Π·Π°ΠΌΠ΅Ρ‚Π½ΠΎ прСвосходят ΠΈΡ…. Для Ρ‚ΠΎΠ³ΠΎ, Ρ‡Ρ‚ΠΎΠ±Ρ‹ ΡΠ΄Π΅Π»Π°Ρ‚ΡŒ Π±ΠΎΠ»Π΅Π΅ Π²Π·Π²Π΅ΡˆΠ΅Π½Π½Ρ‹Π΅ Π²Ρ‹Π²ΠΎΠ΄Ρ‹, трСбуСтся Π΄Π΅Ρ‚Π°Π»ΡŒΠ½ΠΎ ΠΈΠ·ΡƒΡ‡ΠΈΡ‚ΡŒ конструктивныС особСнности встраиваСмых Π² ΠΏΠΎΠ» Ρ€Π°Π΄ΠΈΠ°Ρ‚ΠΎΡ€ΠΎΠ² ΠΈ ΠΈΡ… влияниС Π½Π° ΡƒΡ€ΠΎΠ²Π΅Π½ΡŒ рассматриваСмого ΠΏΠ°Ρ€Π°ΠΌΠ΅Ρ‚Ρ€Π°.

ΠžΡΠΎΠ±Π΅Π½Π½ΠΎΡΡ‚ΠΈ Π²Π½ΡƒΡ‚Ρ€ΠΈΠΏΠΎΠ»ΡŒΠ½Ρ‹Ρ… ΠΊΠΎΠ½Π²Π΅ΠΊΡ‚ΠΎΡ€ΠΎΠ² ΠΏΡ€ΠΈΠΌΠ΅Π½ΠΈΡ‚Π΅Π»ΡŒΠ½ΠΎ ΠΊ ΠΏΠΎΠΊΠ°Π·Π°Ρ‚Π΅Π»ΡŽ Ρ‚Π΅ΠΏΠ»ΠΎΠΎΡ‚Π΄Π°Ρ‡ΠΈ

ΠšΠΎΠ½ΡΡ‚Ρ€ΡƒΠΊΡ†ΠΈΡ Π²Π½ΡƒΡ‚Ρ€ΠΈΠΏΠΎΠ»ΡŒΠ½ΠΎΠ³ΠΎ ΠΊΠΎΠ½Π²Π΅ΠΊΡ‚ΠΎΡ€Π° состоит ΠΈΠ· Ρ‚Ρ€Π΅Ρ… ΠΎΠ±ΡΠ·Π°Ρ‚Π΅Π»ΡŒΠ½Ρ‹Ρ… частСй, каТдая ΠΈΠ· ΠΊΠΎΡ‚ΠΎΡ€Ρ‹Ρ… ΠΈΠΌΠ΅Π΅Ρ‚ собствСнноС Ρ‡Π΅Ρ‚ΠΊΠΎ ΠΎΠΏΡ€Π΅Π΄Π΅Π»Π΅Π½Π½ΠΎΠ΅ Ρ„ΡƒΠ½ΠΊΡ†ΠΈΠΎΠ½Π°Π»ΡŒΠ½ΠΎΠ΅ Π½Π°Π·Π½Π°Ρ‡Π΅Π½ΠΈΠ΅, ΠΈΠΌΠ΅ΡŽΡ‰Π΅Π΅ ΠΊΡ€Π°ΠΉΠ½Π΅ Π²Π°ΠΆΠ½ΡƒΡŽ Ρ€ΠΎΠ»ΡŒ Π² ΠΎΠ±Ρ‰Π΅ΠΉ эффСктивности систСмы.

ΠŸΠΎΡΡ‚ΠΎΠΌΡƒ цСлСсообразно ΠΏΠΎΠ΄Ρ€ΠΎΠ±Π½Π΅Π΅ остановится Π½Π° всСх Ρ‚Ρ€Π΅Ρ…, ΡƒΠΊΠ°Π·Π°Π² влияниС Π½Π° ΡƒΡ€ΠΎΠ²Π΅Π½ΡŒ ΠΈΡ‚ΠΎΠ³ΠΎΠ²ΠΎΠΉ Ρ‚Π΅ΠΏΠ»ΠΎΠΎΡ‚Π΄Π°Ρ‡ΠΈ:

  • Π›ΠΎΡ‚ΠΎΠΊ. РазмСщаСтся Π² стяТкС ΠΏΠΎΠ»Π°. ОсновноС Ρ‚Ρ€Π΅Π±ΠΎΠ²Π°Π½ΠΈΠ΅ – ΠΏΡ€ΠΎΡ‡Π½ΠΎΡΡ‚ΡŒ, нСобходимая для Π½Π°Π΄Π΅ΠΆΠ½ΠΎΠ³ΠΎ располоТСния ΠΎΠ±ΠΎΠ³Ρ€Π΅Π²Π°Ρ‚Π΅Π»ΡŒΠ½ΠΎΠ³ΠΎ элСмСнта, ΠΈ ΡƒΡΡ‚ΠΎΠΉΡ‡ΠΈΠ²ΠΎΡΡ‚ΡŒ ΠΊ ΠΊΠΎΡ€Ρ€ΠΎΠ·ΠΈΠΈ, которая трСбуСтся для Π΄ΠΎΠ»Π³ΠΎΠ²Π΅Ρ‡Π½ΠΎΠΉ эксплуатации. ΠŸΠΎΡΡ‚ΠΎΠΌΡƒ Π»ΠΎΡ‚ΠΎΠΊ изготавливаСтся ΠΈΠ· ΠΎΡ†ΠΈΠ½ΠΊΠΎΠ²Π°Π½Π½ΠΎΠ³ΠΎ ΠΈΠ»ΠΈ Π½Π΅Ρ€ΠΆΠ°Π²Π΅ΡŽΡ‰Π΅Π³ΠΎ ΠΌΠ΅Ρ‚Π°Π»Π»Π°. Π­Ρ‚ΠΎ выступаСт Π΄ΠΎΠΏΠΎΠ»Π½ΠΈΡ‚Π΅Π»ΡŒΠ½Ρ‹ΠΌ плюсом с Ρ‚ΠΎΡ‡ΠΊΠΈ зрСния Ρ‚Π΅ΠΏΠ»ΠΎΠΎΡ‚Π΄Π°Ρ‡ΠΈ, Ρ‚Π°ΠΊ ΠΊΠ°ΠΊ Ρ‚Π΅ΠΏΠ»ΠΎ ΠΎΡ‚ ΠΊΠΎΠ½Π²Π΅ΠΊΡ‚ΠΎΡ€Π° отраТаСтся ΠΎΡ‚ повСрхности Π»ΠΎΡ‚ΠΊΠ° Π² сторону ΠΎΡ‚Π°ΠΏΠ»ΠΈΠ²Π°Π΅ΠΌΠΎΠ³ΠΎ помСщСния.
  • Β 

  • ΠžΠ±ΠΎΠ³Ρ€Π΅Π²Π°Ρ‚Π΅Π»ΡŒΠ½Ρ‹ΠΉ элСмСнт. ΠšΠ»ΡŽΡ‡Π΅Π²Π°Ρ Π΄Π΅Ρ‚Π°Π»ΡŒ систСмы отоплСния. РасполагаСтся Π²Π½ΡƒΡ‚Ρ€ΠΈ Π»ΠΎΡ‚ΠΊΠ°, изготавливаСтся ΠΈΠ· алюминия ΠΈ ΠΌΠ΅Π΄ΠΈ. ИмСнно поэтому ΠΎΠ±Π»Π°Π΄Π°Π΅Ρ‚ ΠΊΡ€Π°ΠΉΠ½Π΅ высоким ΡƒΡ€ΠΎΠ²Π½Π΅ΠΌ Ρ‚Π΅ΠΏΠ»ΠΎΠΎΡ‚Π΄Π°Ρ‡ΠΈ, Π·Π°ΠΌΠ΅Ρ‚Π½ΠΎ прСвосходящим Π±ΠΎΠ»ΡŒΡˆΡƒΡŽ Ρ‡Π°ΡΡ‚ΡŒ Π°Π»ΡŒΡ‚Π΅Ρ€Π½Π°Ρ‚ΠΈΠ²Π½Ρ‹Ρ… Π²Π°Ρ€ΠΈΠ°Π½Ρ‚ΠΎΠ² ΠΎΠ±ΠΎΠ³Ρ€Π΅Π²Π°.
  • Π”Π΅ΠΊΠΎΡ€Π°Ρ‚ΠΈΠ²Π½ΠΎ-защитная Ρ€Π΅ΡˆΠ΅Ρ‚ΠΊΠ°. Π€ΡƒΠ½ΠΊΡ†ΠΈΠΎΠ½Π°Π»ΡŒΠ½ΠΎΠ΅ Π½Π°Π·Π½Π°Ρ‡Π΅Π½ΠΈΠ΅ этого элСмСнта ΠΎΠ±ΠΎΠ³Ρ€Π΅Π²Π°Ρ‚Π΅Π»ΡŒΠ½ΠΎΠΉ систСмы – Π·Π°Ρ‰ΠΈΡ‚Π° ΠΊΠΎΠ½Π²Π΅ΠΊΡ‚ΠΎΡ€Π° ΠΎΡ‚ Π²Π½Π΅ΡˆΠ½ΠΈΡ… воздСйствий ΠΏΡ€ΠΈ ΠΎΠ΄Π½ΠΎΠ²Ρ€Π΅ΠΌΠ΅Π½Π½ΠΎΠΌ обСспСчСнии свободного Π²ΠΎΠ·Π΄ΡƒΡ…ΠΎΠΎΠ±ΠΌΠ΅Π½Π° ΠΌΠ΅ΠΆΠ΄Ρƒ пространством Π»ΠΎΡ‚ΠΊΠ° ΠΈ ΠΎΡ‚Π°ΠΏΠ»ΠΈΠ²Π°Π΅ΠΌΠΎΠ³ΠΎ помСщСния. Π Π΅ΡˆΠ΅Ρ‚ΠΊΠ° изготавливаСтся ΠΈΠ· Ρ€Π°Π·Π»ΠΈΡ‡Π½Ρ‹Ρ… ΠΌΠ°Ρ‚Π΅Ρ€ΠΈΠ°Π»ΠΎΠ², ΠΊΠ°ΠΊ ΠΎΠ±Π»Π°Π΄Π°ΡŽΡ‰ΠΈΡ… высокой Ρ‚Π΅ΠΏΠ»ΠΎΠΏΡ€ΠΎΠ²ΠΎΠ΄Π½ΠΎΡΡ‚ΡŒΡŽ, Π½Π°ΠΏΡ€ΠΈΠΌΠ΅Ρ€, Ρ€Π°Π·Π½Ρ‹Ρ… ΠΌΠ΅Ρ‚Π°Π»Π»ΠΎΠ², Ρ‚Π°ΠΊ ΠΈ ΠΈΠΌΠ΅ΡŽΡ‰ΠΈΡ… Π½ΠΈΠ·ΠΊΡƒΡŽ Ρ‚Π΅ΠΏΠ»ΠΎΠΎΡ‚Π΄Π°Ρ‡Ρƒ – пластика ΠΈΠ»ΠΈ дрСвСсины. НСсмотря Π½Π° это, Π½Π° ΠΎΠ±Ρ‰ΠΈΠΉ ΡƒΡ€ΠΎΠ²Π΅Π½ΡŒ Π΄Π°Π½Π½ΠΎΠ³ΠΎ ΠΏΠ°Ρ€Π°ΠΌΠ΅Ρ‚Ρ€Π° конструкции это практичСски Π½Π΅ влияСт, благодаря Ρ€Π΅ΡˆΠ΅Ρ‚Ρ‡Π°Ρ‚ΠΎΠΉ структурС элСмСнта.

Π’ΠΏΠ΅Ρ‡Π°Ρ‚Π»ΡΡŽΡ‰ΠΈΠ΅ ΠΏΠΎΠΊΠ°Π·Π°Ρ‚Π΅Π»ΠΈ Ρ‚Π΅ΠΏΠ»ΠΎΠΎΡ‚Π΄Π°Ρ‡ΠΈ систСмы ΠΎΠ±ΠΎΠ³Ρ€Π΅Π²Π° с Π²Π½ΡƒΡ‚Ρ€ΠΈΠΏΠΎΠ»ΡŒΠ½Ρ‹ΠΌΠΈ Ρ€Π°Π΄ΠΈΠ°Ρ‚ΠΎΡ€Π°ΠΌΠΈ ΠΎΠ±ΡŠΡΡΠ½ΡΡŽΡ‚ΡΡ ΠΏΡ€ΠΎΠ΄ΡƒΠΌΠ°Π½Π½ΠΎΠΉ конструкциСй ΠΊΠΎΠ½Π²Π΅ΠΊΡ‚ΠΎΡ€Π°, Π² ΠΊΠΎΡ‚ΠΎΡ€ΠΎΠΉ всС Ρ‚Ρ€ΠΈ элСмСнта ΠΊΡ€Π°ΠΉΠ½Π΅ ΡƒΠ΄Π°Ρ‡Π½ΠΎ Π΄ΠΎΠΏΠΎΠ»Π½ΡΡŽΡ‚ Π΄Ρ€ΡƒΠ³ Π΄Ρ€ΡƒΠ³Π°. Π’ Ρ€Π΅Π·ΡƒΠ»ΡŒΡ‚Π°Ρ‚Π΅, достигаСтся высокая ΡΡ„Ρ„Π΅ΠΊΡ‚ΠΈΠ²Π½ΠΎΡΡ‚ΡŒ отоплСния, Π² Π±ΠΎΠ»ΡŒΡˆΠΈΠ½ΡΡ‚Π²Π΅ случаСв прСвосходящая Π°Π»ΡŒΡ‚Π΅Ρ€Π½Π°Ρ‚ΠΈΠ²Π½Ρ‹Π΅ Π²Π°Ρ€ΠΈΠ°Π½Ρ‚Ρ‹.

Π Π΅ΠΊΠΎΠΌΠ΅Π½Π΄Π°Ρ†ΠΈΠΈ ΠΏΠΎ ΡƒΠ²Π΅Π»ΠΈΡ‡Π΅Π½ΠΈΡŽ Ρ‚Π΅ΠΏΠ»ΠΎΠΎΡ‚Π΄Π°Ρ‡ΠΈ встраиваСмых Π² ΠΏΠΎΠ» ΠΊΠΎΠ½Π²Π΅ΠΊΡ‚ΠΎΡ€ΠΎΠ²

ΠŸΡ€ΠΈΠ²Π΅Π΄Π΅Π½Π½Ρ‹ΠΉ Π²Ρ‹ΡˆΠ΅ Π°Π½Π°Π»ΠΈΠ· продСмонстрировал, Ρ‡Ρ‚ΠΎ систСма ΠΎΠ±ΠΎΠ³Ρ€Π΅Π²Π° с встраиваСмыми Π² ΠΏΠΎΠ» Ρ€Π°Π΄ΠΈΠ°Ρ‚ΠΎΡ€Π°ΠΌΠΈ Π²ΠΏΠΎΠ»Π½Π΅ заслуТСнно считаСтся ΠΎΠ΄Π½ΠΎΠΉ ΠΈΠ· самых эффСктивных. Однако, достиТСниС высокого уровня Ρ‚Π΅ΠΏΠ»ΠΎΠΎΡ‚Π΄Π°Ρ‡ΠΈ становится Π²ΠΎΠ·ΠΌΠΎΠΆΠ½Ρ‹ΠΌ Ρ‚ΠΎΠ»ΡŒΠΊΠΎ ΠΏΡ€ΠΈ Π³Ρ€Π°ΠΌΠΎΡ‚Π½ΠΎΠΌ Π²Ρ‹Π±ΠΎΡ€Π΅ ΠΈ ΠΏΠΎΡΠ»Π΅Π΄ΡƒΡŽΡ‰Π΅ΠΌ использовании Π²Π½ΡƒΡ‚Ρ€ΠΈΠΏΠΎΠ»ΡŒΠ½Ρ‹Ρ… ΠΊΠΎΠ½Π²Π΅ΠΊΡ‚ΠΎΡ€ΠΎΠ².

Для этого Π½Π΅ΠΎΠ±Ρ…ΠΎΠ΄ΠΈΠΌΠΎ ΡΠ»Π΅Π΄ΠΎΠ²Π°Ρ‚ΡŒ нСскольким достаточно простым рСкомСндациям. Π’ΠΎ-ΠΏΠ΅Ρ€Π²Ρ‹Ρ…, ΠΏΡ€ΠΈ ΠΏΠΎΠ΄Π±ΠΎΡ€Π΅ подходящСго Π²Π½ΡƒΡ‚Ρ€ΠΈΠΏΠΎΠ»ΡŒΠ½ΠΎΠ³ΠΎ ΠΊΠΎΠ½Π²Π΅ΠΊΡ‚ΠΎΡ€Π° цСлСсообразно Ρ€Π°ΡΡΠΌΠ°Ρ‚Ρ€ΠΈΠ²Π°Ρ‚ΡŒ Ρ‚ΠΎΠ»ΡŒΠΊΠΎ ΠΏΡ€ΠΎΠ΄ΡƒΠΊΡ†ΠΈΡŽ ΠΏΡ€ΠΎΠ²Π΅Ρ€Π΅Π½Π½Ρ‹Ρ…, Π΄Π°Π²Π½ΠΎ ΠΈ ΡƒΡΠΏΠ΅ΡˆΠ½ΠΎ Ρ€Π°Π±ΠΎΡ‚Π°ΡŽΡ‰ΠΈΡ… Π½Π° Ρ€Ρ‹Π½ΠΊΠ΅ ΠΊΠΎΠΌΠΏΠ°Π½ΠΈΠΉ-ΠΏΡ€ΠΎΠΈΠ·Π²ΠΎΠ΄ΠΈΡ‚Π΅Π»Π΅ΠΉ. Π­Ρ‚ΠΎ ΠΏΠΎΠ·Π²ΠΎΠ»ΠΈΡ‚ ΠΏΠΎΠ»ΡƒΡ‡ΠΈΡ‚ΡŒ качСствСнноС ΠΈΠ·Π΄Π΅Π»ΠΈΠ΅, ΠΏΠΎΠ»Π½ΠΎΡΡ‚ΡŒΡŽ ΠΎΡ‚Π²Π΅Ρ‡Π°ΡŽΡ‰Π΅Π΅ заявлСнным ΠΈΠ·Π³ΠΎΡ‚ΠΎΠ²ΠΈΡ‚Π΅Π»Π΅ΠΌ ΠΏΠ°Ρ€Π°ΠΌΠ΅Ρ‚Ρ€Π°ΠΌ.

Π’ΠΎ-Π²Ρ‚ΠΎΡ€Ρ‹Ρ…, ΡƒΠ²Π΅Π»ΠΈΡ‡ΠΈΡ‚ΡŒ ΠΏΠΎΠΊΠ°Π·Π°Ρ‚Π΅Π»ΡŒ Ρ‚Π΅ΠΏΠ»ΠΎΠΎΡ‚Π΄Π°Ρ‡ΠΈ ΠΌΠΎΠΆΠ΅Ρ‚ ΠΏΡ€ΠΈΠΎΠ±Ρ€Π΅Ρ‚Π΅Π½ΠΈΠ΅ ΠΊΠΎΠ½Π²Π΅ΠΊΡ‚ΠΎΡ€Π° с встроСнным вСнтилятором. ЕстСствСнно, ΠΏΠΎΠ΄ΠΎΠ±Π½ΠΎΠ΅ ΠΈΠ·Π΄Π΅Π»ΠΈΠ΅ обойдСтся Π΄ΠΎΡ€ΠΎΠΆΠ΅, Π½ΠΎ Π² процСссС эксплуатации ΡƒΠ²Π΅Π»ΠΈΡ‡Π΅Π½ΠΈΠ΅ стоимости компСнсируСтся ростом эффСктивности ΠΎΠ±ΠΎΠ³Ρ€Π΅Π²Π°.

Β 

Π’-Ρ‚Ρ€Π΅Ρ‚ΡŒΠΈΡ…, Π΅Ρ‰Π΅ ΠΎΠ΄Π½ΠΈΠΌ ΠΎΠ±ΡΠ·Π°Ρ‚Π΅Π»ΡŒΠ½Ρ‹ΠΌ условиСм достиТСния высокого уровня Ρ‚Π΅ΠΏΠ»ΠΎΠΎΡ‚Π΄Π°Ρ‡ΠΈ ΠΎΡ‚ΠΎΠΏΠΈΡ‚Π΅Π»ΡŒΠ½ΠΎΠΉ систСмы с Π²Π½ΡƒΡ‚Ρ€ΠΈΠΏΠΎΠ»ΡŒΠ½Ρ‹ΠΌΠΈ ΠΊΠΎΠ½Π²Π΅ΠΊΡ‚ΠΎΡ€Π°ΠΌΠΈ выступаСт ΠΏΡ€Π°Π²ΠΈΠ»ΡŒΠ½Ρ‹ΠΉ ΠΌΠΎΠ½Ρ‚Π°ΠΆ ΠΎΡ‚ΠΎΠΏΠΈΡ‚Π΅Π»ΡŒΠ½Ρ‹Ρ… ΠΏΡ€ΠΈΠ±ΠΎΡ€ΠΎΠ². Π’ Ρ…ΠΎΠ΄Π΅ Ρ€Π°Π±ΠΎΡ‚ Π΄ΠΎΠ»ΠΆΠ½Ρ‹ Π²Ρ‹ΠΏΠΎΠ»Π½ΡΡ‚ΡŒΡΡ всС тСхнологичСскиС трСбования, ΡΡ‚Ρ€ΠΎΠΈΡ‚Π΅Π»ΡŒΠ½Ρ‹Π΅ стандарты ΠΈ ΠΏΡ€Π°Π²ΠΈΠ»Π°. ΠŸΠΎΡΡ‚ΠΎΠΌΡƒ цСлСсообразно ΠΎΠ±Ρ€Π°Ρ‰Π°Ρ‚ΡŒΡΡ Π·Π° ΠΏΠΎΠΌΠΎΡ‰ΡŒΡŽ ΠΊ ΠΏΡ€ΠΎΡ„Π΅ΡΡΠΈΠΎΠ½Π°Π»ΡŒΠ½Ρ‹ΠΌ ΠΌΠΎΠ½Ρ‚Π°ΠΆΠ½ΠΈΠΊΠ°ΠΌ, Π»ΠΈΠ±ΠΎ Π²Ρ‹ΠΏΠΎΠ»Π½ΡΡ‚ΡŒ Ρ€Π°Π±ΠΎΡ‚Ρ‹ ΡΠ°ΠΌΠΎΡΡ‚ΠΎΡΡ‚Π΅Π»ΡŒΠ½ΠΎ, Π½ΠΎ ΠΏΡ€Π΅Π΄Π΅Π»ΡŒΠ½ΠΎ Π°ΠΊΠΊΡƒΡ€Π°Ρ‚Π½ΠΎ ΠΈ Ρ‚Ρ‰Π°Ρ‚Π΅Π»ΡŒΠ½ΠΎ соблюдая Ρ‚Π΅Ρ…Π½ΠΎΠ»ΠΎΠ³ΠΈΡŽ.

Π‘Π»Π΅Π΄ΠΎΠ²Π°Π½ΠΈΠ΅ пСрСчислСнным Π²Ρ‹ΡˆΠ΅ Π½Π΅Ρ…ΠΈΡ‚Ρ€Ρ‹ΠΌ рСкомСндациям ΠΏΠΎΠ·Π²ΠΎΠ»ΠΈΡ‚ ΠΏΠΎΠ»ΡƒΡ‡ΠΈΡ‚ΡŒ Π½Π° Π²Ρ‹Ρ…ΠΎΠ΄Ρ‹ ΡΡ„Ρ„Π΅ΠΊΡ‚ΠΈΠ²Π½ΡƒΡŽ систСму отоплСния с Π²Π½ΡƒΡ‚Ρ€ΠΈΠΏΠΎΠ»ΡŒΠ½Ρ‹ΠΌΠΈ ΠΊΠΎΠ½Π²Π΅ΠΊΡ‚ΠΎΡ€Π°ΠΌΠΈ. ΠŸΡ€ΠΈ этом Π²Π»Π°Π΄Π΅Π»ΡŒΡ†Ρƒ здания гарантируСтся высокий ΡƒΡ€ΠΎΠ²Π΅Π½ΡŒ Ρ‚Π΅ΠΏΠ»ΠΎΠΎΡ‚Π΄Π°Ρ‡ΠΈ, ΠΊΠΎΡ‚ΠΎΡ€Ρ‹ΠΉ сочСтаСтся с Ρ€Π°Π·ΡƒΠΌΠ½Ρ‹ΠΌΠΈ расходами Π½Π° ΠΎΠ±ΠΎΠ³Ρ€Π΅Π².

5 способов ΠΊΠ°ΠΊ ΠΏΠΎΠ²Ρ‹ΡΠΈΡ‚ΡŒ ΡΡ„Ρ„Π΅ΠΊΡ‚ΠΈΠ²Π½ΠΎΡΡ‚ΡŒ отоплСния

На Ρ‡Ρ‚Π΅Π½ΠΈΠ΅ 3 ΠΌΠΈΠ½ ΠŸΡ€ΠΎΡΠΌΠΎΡ‚Ρ€ΠΎΠ² 358 ΠžΠΏΡƒΠ±Π»ΠΈΠΊΠΎΠ²Π°Π½ΠΎ ОбновлСно

Π”ΠΎΠΌΠ° ΠΏΡ€ΠΎΡ…Π»Π°Π΄Π½ΠΎ, хотя ΠΎΡ‚ΠΎΠΏΠ»Π΅Π½ΠΈΠ΅ Π²ΠΊΠ»ΡŽΡ‡Π΅Π½ΠΎ? Π’ΠΎΠ·ΠΌΠΎΠΆΠ½ΠΎ, ΠΏΡ€ΠΎΠ±Π»Π΅ΠΌΠ° Π² Π½Π΅ΠΏΠΎΠ»Π½ΠΎΡ†Π΅Π½Π½ΠΎΠΉ Ρ‚Π΅ΠΏΠ»ΠΎΠΎΡ‚Π΄Π°Ρ‡Π΅ Ρ€Π°Π΄ΠΈΠ°Ρ‚ΠΎΡ€Π°. Π•Π΅ ΠΌΠΎΠΆΠ½ΠΎ ΡƒΠ²Π΅Π»ΠΈΡ‡ΠΈΡ‚ΡŒ ΠΏΠΎΡ€Π°Π±ΠΎΡ‚Π°Π² Π½Π°Π΄ ΠΎΡ‚ΠΎΠΏΠΈΡ‚Π΅Π»ΡŒΠ½ΠΎΠΉ систСмой ΠΈΠ»ΠΈ мСстом Π΅Π΅ располоТСния. Рассмотрим 5 способов, ΠΊΠ°ΠΊ ΠΏΠΎΠ²Ρ‹ΡΠΈΡ‚ΡŒ ΡΡ„Ρ„Π΅ΠΊΡ‚ΠΈΠ²Π½ΠΎΡΡ‚ΡŒ Π±Π°Ρ‚Π°Ρ€Π΅ΠΉ.

Π‘ΠΏΡƒΡΡ‚ΠΈΡ‚ΡŒ Π²ΠΎΠ·Π΄ΡƒΡ… ΠΈΠ»ΠΈ ΠΏΡ€ΠΎΡ‡ΠΈΡΡ‚ΠΈΡ‚ΡŒ сСкции

Π­Ρ‚ΠΎ Π½ΡƒΠΆΠ½ΠΎ, ΠΊΠΎΠ³Π΄Π°, Π² систСмС ΠΎΠ±Ρ€Π°Π·ΠΎΠ²Π°Π»Π°ΡΡŒ Π²ΠΎΠ·Π΄ΡƒΡˆΠ½Π°Ρ ΠΏΡ€ΠΎΠ±ΠΊΠ°. Она заняла мСсто, Π² ΠΊΠΎΡ‚ΠΎΡ€ΠΎΠ΅ Π΄ΠΎΠ»ΠΆΠ½Π° ΠΏΠΎΡΡ‚ΡƒΠΏΠ°Ρ‚ΡŒ горячая Π²ΠΎΠ΄Π°. НуТно Π΅Π΅ Π²Ρ‹ΠΏΡƒΡΡ‚ΠΈΡ‚ΡŒ, ΠΈ батарСя прогрССтся ΠΏΠΎΠ»Π½ΠΎΡΡ‚ΡŒΡŽ. На Ρ€Π°Π΄ΠΈΠ°Ρ‚ΠΎΡ€Π°Ρ… Π΅ΡΡ‚ΡŒ ΡΠΏΠ΅Ρ†ΠΈΠ°Π»ΡŒΠ½Ρ‹Π΅ отвСрстия для вывСдСния Π²ΠΎΠ·Π΄ΡƒΡ…Π°, Ρ‚Π°ΠΊΠΆΠ΅ примСняСтся ΠΊΡ€Π°Π½ МаСвского, ΠΊΠΎΡ‚ΠΎΡ€Ρ‹ΠΉ монтируСтся сбоку Π±Π°Ρ‚Π°Ρ€Π΅ΠΈ. Для Π°ΠΊΡ‚ΠΈΠ²Π°Ρ†ΠΈΠΈ спуска ΠΏΡ€ΠΎΠ±ΠΊΠΈ Π²Ρ€Π°Π΄ΠΈΠ°Ρ‚ΠΎΡ€Π΅ Π½ΡƒΠΆΠ½ΠΎ ΠΎΡ‚ΠΊΡ€ΡƒΡ‚ΠΈΡ‚ΡŒ ΠΊΡ€Π°Π½ ΠΎΡ‚Π²Π΅Ρ€Ρ‚ΠΊΠΎΠΉ ΠΈ ΠΏΠΎΠ΄ΡΡ‚Π°Π²ΠΈΡ‚ΡŒ Π΅ΠΌΠΊΠΎΡΡ‚ΡŒ, Π² ΠΊΠΎΡ‚ΠΎΡ€ΡƒΡŽ Π±ΡƒΠ΄Π΅Ρ‚ ΡΡ‚Π΅ΠΊΠ°Ρ‚ΡŒ Π²ΠΎΠ΄Π° послС Π²Ρ‹Ρ…ΠΎΠ΄Π° Π·Π°ΠΊΡƒΠΏΠΎΡ€ΠΊΠΈ. Π’Π°ΠΊΠΆΠ΅ ΡƒΡΡ‚Π°Π½Π°Π²Π»ΠΈΠ²Π°ΡŽΡ‚ автоматичСский Π²ΠΎΠ·Π΄ΡƒΡ…ΠΎΠΎΡ‚Π²ΠΎΠ΄Ρ‡ΠΈΠΊ, Ρƒ ΠΊΠΎΡ‚ΠΎΡ€ΠΎΠ³ΠΎ Π΅ΡΡ‚ΡŒ ΠΏΠΎΠΏΠ»Π°Π²ΠΎΠΊ, Π·Π°ΠΊΡ€Ρ‹Π²Π°ΡŽΡ‰ΠΈΠΉ отвСрстиС для Π²Ρ‹Ρ…ΠΎΠ΄Π° Π²ΠΎΠ·Π΄ΡƒΡ…Π°, ΠΊΠΎΡ‚ΠΎΡ€Ρ‹ΠΉ опускаСтся ΠΏΡ€ΠΈ Π΅Π³ΠΎ скапливании Π² систСмС ΠΈ Π΄Π°Π΅Ρ‚ Π΅ΠΌΡƒ Π²Ρ‹Ρ…ΠΎΠ΄. ВмСстС с этим устройством Π½ΡƒΠΆΠ½ΠΎ ΠΏΡ€ΠΈΠΌΠ΅Π½ΡΡ‚ΡŒ Ρ„ΠΈΠ»ΡŒΡ‚Ρ€Ρ‹ для Π²ΠΎΠ΄Ρ‹, ΠΏΠΎΠ΄Π°Π²Π°Π΅ΠΌΠΎΠΉ Π² Π±Π°Ρ‚Π°Ρ€Π΅ΡŽ.

Π’Π°ΠΊΠΆΠ΅ Ρ‚Π΅ΠΏΠ»ΠΎΠΎΡ‚Π΄Π°Ρ‡Π° ΠΌΠΎΠΆΠ΅Ρ‚ ΡΠ½ΠΈΠΆΠ°Ρ‚ΡŒΡΡ ΠΈΠ·-Π·Π° образования Ρ€ΠΆΠ°Π²Ρ‡ΠΈΠ½Ρ‹ ΠΈΠ»ΠΈ извСсткового Π½Π°Π»Π΅Ρ‚Π° Π² сСкциях. Π’Π°ΠΊΠΈΠ΅ засорСния ΡΠΎΠ·Π΄Π°ΡŽΡ‚ слой, ΠΏΠΎΠ³Π»ΠΎΡ‰Π°ΡŽΡ‰ΠΈΠΉ Ρ‚Π΅ΠΏΠ»ΠΎ, ΠΈ ΡΠ½ΠΈΠΆΠ°ΡŽΡ‚ объСм горячСй Π²ΠΎΠ΄Ρ‹, проходящСй ΠΏΠΎ ΠΎΡ‚ΠΎΠΏΠΈΡ‚Π΅Π»ΡŒΠ½ΠΎΠΉ систСмС. ΠŸΡ€ΠΎΡ‡ΠΈΡΡ‚ΠΊΠ° Π±Ρ‹Π²Π°Π΅Ρ‚ мСханичСской, химичСской, с ΠΏΡ€ΠΈΠΌΠ΅Π½Π΅Π½ΠΈΠ΅ΠΌ Π½Π°ΠΏΠΎΡ€Π°, ΡΠ»Π΅ΠΊΡ‚Ρ€ΠΎΠ³ΠΈΠ΄Ρ€ΠΎΠΈΠΌΠΏΡƒΠ»ΡŒΡΠ° ΠΈ Π΄Ρ€. ΠΌΠ΅Ρ‚ΠΎΠ΄ΠΎΠ². Π§Ρ‚ΠΎ Π²Ρ‹Π±Ρ€Π°Ρ‚ΡŒ, зависит ΠΎΡ‚ конструкции, ΠΌΠ°Ρ‚Π΅Ρ€ΠΈΠ°Π»Π° сСкций ΠΈ Π²ΠΈΠ΄Π° загрязнСний.

Π£ΡΠΊΠΎΡ€ΠΈΡ‚ΡŒ ΠΊΠΎΠ½Π²Π΅ΠΊΡ†ΠΈΡŽ

Π­Ρ‚ΠΎ ΠΌΠΎΠΆΠ΅Ρ‚ ΡΡ€Π°Π±ΠΎΡ‚Π°Ρ‚ΡŒ, ΠΊΠΎΠ³Π΄Π° Π±Π°Ρ‚Π°Ρ€Π΅ΠΈ горячиС, Π½ΠΎ ΠΏΠΎΠΌΠ΅Ρ‰Π΅Π½ΠΈΠ΅ Π½Π΅ прогрСваСтся. НуТно Π½Π°ΠΏΡ€Π°Π²ΠΈΡ‚ΡŒ Π½Π° Π½ΠΈΡ… вСнтилятор, ΠΈ ΠΎΠ½ ускорит ΠΊΠΎΠ½Π²Π΅ΠΊΡ†ΠΈΡŽ. Для этого Ρ‚Π°ΠΊΠΆΠ΅ ΠΈΡΠΏΠΎΠ»ΡŒΠ·ΡƒΡŽΡ‚ ΠΊΡƒΠ»Π΅Ρ€ систСмного Π±Π»ΠΎΠΊΠ° ΠΊΠΎΠΌΠΏΡŒΡŽΡ‚Π΅Ρ€Π°, ΠΊΠΎΡ‚ΠΎΡ€Ρ‹ΠΉ ΠΌΠΎΠΆΠ½ΠΎ ΡƒΡΡ‚Π°Π½ΠΎΠ²ΠΈΡ‚ΡŒ Π½Π΅ΠΏΠΎΠ΄Π°Π»Π΅ΠΊΡƒ ΠΎΡ‚ Ρ€Π°Π΄ΠΈΠ°Ρ‚ΠΎΡ€Π° ΠΈ Π½Π°ΠΏΡ€Π°Π²ΠΈΡ‚ΡŒ Π·Π°Π΄Π½Π΅ΠΉ Ρ‡Π°ΡΡ‚ΡŒΡŽ Π² Π΅Π³ΠΎ сторону. УскорСнный ΠΏΠΎΡ‚ΠΎΠΊ Π²ΠΎΠ·Π΄ΡƒΡ…Π° Π±ΡƒΠ΄Π΅Ρ‚ Π½Π°Π³Ρ€Π΅Π²Π°Ρ‚ΡŒΡΡ, ΠΈ Π΅Π³ΠΎ Ρ‚Π΅ΠΏΠ»Ρ‹Π΅ массы быстрСС заполнят ΠΏΠΎΠΌΠ΅Ρ‰Π΅Π½ΠΈΠ΅.

Π£ΡΡ‚Ρ€Π°Π½ΠΈΡ‚ΡŒ прСпятствия

Для красоты Π½Π΅ΠΊΠΎΡ‚ΠΎΡ€Ρ‹Π΅ Π·Π°ΠΊΡ€Ρ‹Π²Π°ΡŽΡ‚ Ρ€Π°Π΄ΠΈΠ°Ρ‚ΠΎΡ€Ρ‹ Π΄Π΅ΠΊΠΎΡ€Π°Ρ‚ΠΈΠ²Π½Ρ‹ΠΌΠΈ Ρ‰ΠΈΡ‚Π°ΠΌΠΈ. Они ΠΏΡ€Π΅ΠΏΡΡ‚ΡΡ‚Π²ΡƒΡŽΡ‚ ΠΊΠΎΠ½Π²Π΅ΠΊΡ†ΠΈΠΈ Π²ΠΎΠ·Π΄ΡƒΡ…Π° ΠΈ ΠΏΠΎΠ³Π»ΠΎΡ‰Π°ΡŽΡ‚ Ρ‚Π΅ΠΏΠ»ΠΎ. Если ΠΎΠ½ΠΈ Π½Π΅ΠΎΠ±Ρ…ΠΎΠ΄ΠΈΠΌΡ‹, Π½ΡƒΠΆΠ½ΠΎ Π·Π°ΠΌΠ΅Π½ΠΈΡ‚ΡŒ ΡΠΏΠ»ΠΎΡˆΠ½Ρ‹Π΅ конструкции Π½Π° Ρ€Π΅ΡˆΠ΅Ρ‚Ρ‡Π°Ρ‚Ρ‹Π΅ ΠΈΠ»ΠΈ Π΄Ρ‹Ρ€Ρ‡Π°Ρ‚Ρ‹Π΅. Π’Π°ΠΆΠ½ΠΎ ΡƒΡΡ‚Ρ€Π°Π½ΠΈΡ‚ΡŒ прСпятствия двиТСнию Π²ΠΎΠ·Π΄ΡƒΡ…Π° Π² Π²ΠΈΠ΄Π΅ Π²Ρ‹ΡΡ‚ΡƒΠΏΠ°ΡŽΡ‰ΠΈΡ… ΠΏΠΎΠ΄ΠΎΠΊΠΎΠ½Π½ΠΈΠΊΠΎΠ² ΠΈ ΠΏΠ»ΠΎΡ‚Π½Ρ‹Ρ… ΡˆΡ‚ΠΎΡ€, ΡΠ²ΠΈΡΠ°ΡŽΡ‰ΠΈΡ… Π½Π° Π±Π°Ρ‚Π°Ρ€Π΅ΡŽ.Π’ΠΎΠ³Π΄Π° процСсс Π΅Π³ΠΎ циркуляции Π±ΡƒΠ΄Π΅Ρ‚ ΠΌΠ°ΠΊΡΠΈΠΌΠ°Π»ΡŒΠ½Ρ‹ΠΌ, ΠΈ ΠΏΠΎΠΌΠ΅Ρ‰Π΅Π½ΠΈΠ΅ Π±ΡƒΠ΄Π΅Ρ‚ ΠΏΡ€ΠΎΠ³Ρ€Π΅Π²Π°Ρ‚ΡŒΡΡ Π»ΡƒΡ‡ΡˆΠ΅.

ΠΠ°ΠΏΡ€Π°Π²ΠΈΡ‚ΡŒ распространСниС Ρ‚Π΅ΠΏΠ»Π°

Π§Π°ΡΡ‚ΡŒ Ρ‚Π΅ΠΏΠ»Π° ΡƒΡ…ΠΎΠ΄ΠΈΡ‚ Π² стСну, Ρ‡Ρ‚ΠΎ Π·Π° Ρ€Π°Π΄ΠΈΠ°Ρ‚ΠΎΡ€ΠΎΠΌ, поэтому для Ρ‚ΠΎΠ³ΠΎ, Ρ‡Ρ‚ΠΎΠ±Ρ‹ ΠΎΠ½ΠΎ шло Ρ‚ΠΎΠ»ΡŒΠΊΠΎ Π² Π½Π°ΠΏΡ€Π°Π²Π»Π΅Π½ΠΈΠΈ помСщСния, Π΅Π΅ ΠΈΠ·ΠΎΠ»ΠΈΡ€ΡƒΡŽΡ‚. Для этого Π½Π° стСну Π½Π°ΠΊΠ»Π΅ΠΈΠ²Π°ΡŽΡ‚ вспСнСнный полиэтилСн со слоСм Ρ„ΠΎΠ»ΡŒΠ³ΠΈ с ΠΎΠ΄Π½ΠΎΠΉ стороны, ΠΎΡ‚Ρ€Π°ΠΆΠ°ΡŽΡ‰ΠΈΠΌ слоСм ΠΊ ΠΏΠΎΠΌΠ΅Ρ‰Π΅Π½ΠΈΡŽ, ΡƒΡ‚Π΅ΠΏΠ»ΠΈΡ‚Π΅Π»Π΅ΠΌ ΠΊ повСрхности. Он Π½Π΅ Π΄Π°Π΅Ρ‚ Ρ…ΠΎΠ»ΠΎΠ΄Ρƒ ΠΎΡ‚ стСны ΠΏΠΎΡΡ‚ΡƒΠΏΠ°Ρ‚ΡŒ Π² ΠΊΠΎΠΌΠ½Π°Ρ‚Ρƒ, Π° Ρ„ΠΎΠ»ΡŒΠ³Π° ΠΎΡ‚Ρ‚Π°Π»ΠΊΠΈΠ²Π°Π΅Ρ‚ Ρ‚Π΅ΠΏΠ»ΠΎ, исходящСС ΠΎΡ‚ Ρ€Π°Π΄ΠΈΠ°Ρ‚ΠΎΡ€Π°. Π­Ρ‚ΠΎΡ‚ ΠΌΠΎΠΌΠ΅Π½Ρ‚ ΠΌΠΎΠΆΠ½ΠΎ ΡƒΡ‡Π΅ΡΡ‚ΡŒ Π΅Ρ‰Π΅ ΠΏΡ€ΠΈ ΠΌΠΎΠ½Ρ‚Π°ΠΆΠ΅ ΠΎΡ‚ΠΎΠΏΠΈΡ‚Π΅Π»ΡŒΠ½ΠΎΠΉ систСмы, размСстив Π·Π° Π½Π΅ΠΉ рСбристый Ρ‰ΠΈΡ‚ ΠΈΠ· стали.

Π”ΠΎΠΏΠΎΠ»Π½ΠΈΡ‚Π΅Π»ΡŒΠ½ΠΎΠ΅ оснащСниС Ρ€Π°Π΄ΠΈΠ°Ρ‚ΠΎΡ€ΠΎΠ²

МоТно ΡƒΠ²Π΅Π»ΠΈΡ‡ΠΈΡ‚ΡŒ Ρ‚Π΅ΠΏΠ»ΠΎΠΎΡ‚Π΄Π°Ρ‡Ρƒ Π±Π°Ρ‚Π°Ρ€Π΅ΠΈ, Π½Π°Π΄Π΅Π² Π½Π° Π½Π΅Π΅ Π°Π»ΡŽΠΌΠΈΠ½ΠΈΠ΅Π²Ρ‹ΠΉ ΠΊΠΎΠΆΡƒΡ…. Π­Ρ‚ΠΎΡ‚ ΠΌΠ΅Ρ‚Π°Π»Π» быстро ΠΏΠΎΠ³Π»ΠΎΡ‰Π°Π΅Ρ‚ ΠΈ ΠΎΡ‚Π΄Π°Π΅Ρ‚ Ρ‚Π΅ΠΏΠ»ΠΎ. Если Π½Π΅ установлСн счСтчик Π½Π° ΠΎΡ‚ΠΎΠΏΠ»Π΅Π½ΠΈΠ΅, Π΄ΠΎΠ±Π°Π²Π»Π΅Π½ΠΈΠ΅ сСкций ΠΊ Ρ€Π°Π΄ΠΈΠ°Ρ‚ΠΎΡ€Ρƒ сдСлаСт Π½Π°Π³Ρ€Π΅Π²Π°Π΅ΠΌΡƒΡŽ ΠΏΠ»ΠΎΡ‰Π°Π΄ΡŒ большС, ΠΈ Π² ΠΊΠΎΠΌΠ½Π°Ρ‚Ρƒ станСт ΠΏΠΎΡΡ‚ΡƒΠΏΠ°Ρ‚ΡŒ большС Ρ‚Π΅ΠΏΠ»Π°.

Π’Π°ΠΊ ΠΆΠ΅ для Ρ‚Π΅ΠΏΠ»ΠΎΠΎΡ‚Π΄Π°Ρ‡ΠΈ ΠΈΠΌΠ΅Π΅Ρ‚ Π·Π½Π°Ρ‡Π΅Π½ΠΈΠ΅ Ρ†Π²Π΅Ρ‚ Ρ€Π°Π΄ΠΈΠ°Ρ‚ΠΎΡ€Π°: ΠΊΠΎΡ€ΠΈΡ‡Π½Π΅Π²Ρ‹ΠΉ ΠΈΠ»ΠΈ Π±Ρ€ΠΎΠ½Π·ΠΎΠ²Ρ‹ΠΉ Ρ†Π²Π΅Ρ‚ ΡƒΠ²Π΅Π»ΠΈΡ‡ΠΈΠ²Π°Π΅Ρ‚ Π΅Π΅ Π½Π° 20 – 25 % ΠΏΠΎ ΠΎΡ‚Π½ΠΎΡˆΠ΅Π½ΠΈΡŽ ΠΊ Π±Π΅Π»ΠΎΠΌΡƒ. ΠŸΠ΅Ρ€Π΅Π΄ покраской слСдуСт ΡƒΠ΄Π°Π»ΠΈΡ‚ΡŒ ΠΏΡ€Π΅Π΄Ρ‹Π΄ΡƒΡ‰ΠΈΠ΅ слои покрытия, ΠΊΠΎΡ‚ΠΎΡ€Ρ‹Π΅ часто Π΅ΡΡ‚ΡŒ Π½Π° старых Ρ‡ΡƒΠ³ΡƒΠ½Π½Ρ‹Ρ… батарСях, ΠΏΠΎΡ‚ΠΎΠΌΡƒ Ρ‡Ρ‚ΠΎ ΠΎΠ½ΠΈ ΠΏΠΎΠ³Π»ΠΎΡ‰Π°ΡŽΡ‚ Ρ‚Π΅ΠΏΠ»ΠΎ.

Если ΠΈΠ·Π±Π°Π²ΠΈΡ‚ΡŒ ΠΎΡ‚ΠΎΠΏΠΈΡ‚Π΅Π»ΡŒΠ½ΡƒΡŽ систСму Π΅Π΅ ΠΎΡ‚ Π²ΠΎΠ·Π΄ΡƒΡˆΠ½Ρ‹Ρ… ΠΏΡ€ΠΎΠ±ΠΎΠΊ, загрязнСний, ΡƒΡΡ‚Ρ€Π°Π½ΠΈΡ‚ΡŒ прСпятствия для ΠΊΠΎΠ½Π²Π΅ΠΊΡ†ΠΈΠΈ ΠΈ Π½Π°ΠΏΡ€Π°Π²ΠΈΡ‚ΡŒ распространСниС Ρ‚Π΅ΠΏΠ»Π°, Ρ‚Π΅ΠΌΠΏΠ΅Ρ€Π°Ρ‚ΡƒΡ€Π° Π²ΠΎΠ·Π΄ΡƒΡ…Π° Π² ΠΊΠΎΠΌΠ½Π°Ρ‚Π΅ повысится.

Π’Π΅ΠΏΠ»ΠΎΠΎΡ‚Π΄Π°Ρ‡Π° ΠΎΡ‚ΠΎΠΏΠΈΡ‚Π΅Π»ΡŒΠ½Ρ‹Ρ… ΠΏΡ€ΠΈΠ±ΠΎΡ€ΠΎΠ² — lovial.

narod.ru (ref 21.04.2015)
ΠšΠ²Π°Π»ΠΈΡ„ΠΈΡ†ΠΈΡ€ΠΎΠ²Π°Π½Π½Ρ‹ΠΉ мастСр ΠΎΠΊΠ°ΠΆΠ΅Ρ‚ услуги ΠΏΠΎ установкС, ΠΏΠΎΠ΄ΠΊΠ»ΡŽΡ‡Π΅Π½ΠΈΡŽ ΠΈ Ρ€Π΅ΠΌΠΎΠ½Ρ‚Ρƒ Π±Ρ‹Ρ‚ΠΎΠ²ΠΎΠΉ Ρ‚Π΅Ρ…Π½ΠΈΠΊΠΈ: ΡΡ‚ΠΈΡ€Π°Π»ΡŒΠ½Ρ‹Ρ…, посудомоСчных ΠΈ ΡΡƒΡˆΠΈΠ»ΡŒΠ½Ρ‹Ρ… машин, Π³Π°Π·ΠΎΠ²Ρ‹Ρ… ΠΈ элСктричСских ΠΏΠ»ΠΈΡ‚, ΠΏΡ€ΠΎΡ‚ΠΎΡ‡Π½Ρ‹Ρ… ΠΈ Смкостных Π²ΠΎΠ΄ΠΎΠ½Π°Π³Ρ€Π΅Π²Π°Ρ‚Π΅Π»Π΅ΠΉ, элСктричСских ΠΈ Π³Π°Π·ΠΎΠ²Ρ‹Ρ… Π΄ΡƒΡ…ΠΎΠ²Ρ‹Ρ… ΡˆΠΊΠ°Ρ„ΠΎΠ² ΠΈ ΠΌΠ½ΠΎΠ³ΠΎΠ³ΠΎ Π΄Ρ€ΡƒΠ³ΠΎΠ³ΠΎ Π² Π³ΠΎΡ€ΠΎΠ΄Π΅ КиСв ΠΈ Π±Π»ΠΈΠ·Π»Π΅ΠΆΠ°Ρ‰ΠΈΡ… Ρ€Π°ΠΉΠΎΠ½Π°Ρ…. Π’Π΅Π»Π΅Ρ„ΠΎΠ½ (099) 406-27-87 АлСксСй Β 

         Данная ΡΡ‚Π°Ρ‚ΡŒΡ Π±Ρ‹Π»Π° написана ΠΌΠ½ΠΎΠ³ΠΎ Π»Π΅Ρ‚ Π½Π°Π·Π°Π΄. Π’ настоящСС врСмя ΠΌΠ½ΠΎΠ³ΠΈΠ΅ ссылки Π½Π΅ Ρ€Π°Π±ΠΎΡ‚Π°ΡŽΡ‚. Π‘ΠΎ Π²Ρ€Π΅ΠΌΠ΅Π½Π΅ΠΌ ΡΡ‚Π°Ρ‚ΡŒΡ Π±ΡƒΠ΄Π΅Ρ‚ ΠΏΠ΅Ρ€Π΅Ρ€Π°Π±ΠΎΡ‚Π°Π½Π° ΠΈ Π΄ΠΎΠΏΠΎΠ»Π½Π΅Π½Π°. Β Β Β Β Β Β Β Π’ Π΄Π°Π½Π½ΠΎΠΉ ΡΡ‚Π°Ρ‚ΡŒΠ΅ я ΠΏΠΎΡΡ‚Π°Ρ€Π°ΡŽΡΡŒ ΠΎΠ±ΡŠΡΡΠ½ΠΈΡ‚ΡŒ, ΠΊΠ°ΠΊΠΈΠ΅ Π±Ρ‹Π²Π°ΡŽΡ‚ основныС разновидности ΠΎΡ‚ΠΎΠΏΠΈΡ‚Π΅Π»ΡŒΠ½Ρ‹Ρ… ΠΏΡ€ΠΈΠ±ΠΎΡ€ΠΎΠ² ΠΈ ΠΊΠ°ΠΊ ΠΎΠΏΡ€Π΅Π΄Π΅Π»ΡΡ‚ΡŒ ΠΈΡ… Ρ‚Π΅ΠΏΠ»ΠΎΠΎΡ‚Π΄Π°Ρ‡Ρƒ. ΠŸΡ€Π°ΠΊΡ‚ΠΈΡ‡Π΅ΡΠΊΠΈ ΠΌΠΎΠΆΠ΅Ρ‚Π΅ ΠΏΠΎΡΠΌΠΎΡ‚Ρ€Π΅Ρ‚ΡŒ здСсь.
Β Β Β Β Β Β Β ΠŸΠ΅Ρ€Π΅Π΄Π°Ρ‡Π° Ρ‚Π΅ΠΏΠ»Π° осущСствляСтся трСмя способами — Ρ‚Π΅ΠΏΠ»ΠΎΠΏΡ€ΠΎΠ²ΠΎΠ΄Π½ΠΎΡΡ‚ΡŒΡŽ (ΠΊΠΎΠ½Π΄ΡƒΠΊΡ‚ΠΈΠ²Π½ΠΎΡΡ‚ΡŒΡŽ), ΠΊΠΎΠ½Π²Π΅ΠΊΡ†ΠΈΠ΅ΠΉ (ΠΏΠ΅Ρ€Π΅ΠΌΠ΅ΡˆΠΈΠ²Π°Π½ΠΈΠ΅ΠΌ) ΠΈ ΠΈΠ·Π»ΡƒΡ‡Π΅Π½ΠΈΠ΅ΠΌ (Ρ€Π°Π΄ΠΈΠ°Ρ†ΠΈΠ΅ΠΉ).Π’ ΠΎΡ‚ΠΎΠΏΠΈΡ‚Π΅Π»ΡŒΠ½Ρ‹Ρ… ΠΏΡ€ΠΈΠ±ΠΎΡ€Π°Ρ… Ρ€Π°Π±ΠΎΡ‚Π°ΡŽΡ‚ всС Ρ‚Ρ€ΠΈ Π²ΠΈΠ΄Π° пСрСноса Ρ‚Π΅ΠΏΠ»Π°, Π½ΠΎ Π² зависимости ΠΎΡ‚ конструктивного исполнСния доля (Π²ΠΊΠ»Π°Π΄) Ρ‚ΠΎΠ³ΠΎ ΠΈΠ»ΠΈ ΠΈΠ½ΠΎΠ³ΠΎ способа Π² ΠΎΠ±Ρ‰ΡƒΡŽ Ρ‚Π΅ΠΏΠ»ΠΎΠΎΡ‚Π΄Π°Ρ‡Ρƒ ΠΎΡ‡Π΅Π½ΡŒ отличаСтся.
Β Β Β Β Β Β Β Π’ частности, ΠΈΠ½Ρ‚Π΅Π½ΡΠΈΠ²Π½ΠΎΡΡ‚ΡŒ лучистого (Ρ€Π°Π΄ΠΈΠ°Ρ†ΠΈΠΎΠ½Π½ΠΎΠ³ΠΎ) Ρ‚Π΅ΠΏΠ»ΠΎΠΎΠ±ΠΌΠ΅Π½Π° зависит ΠΎΡ‚ Ρ‡Π΅Ρ‚Π²Π΅Ρ€Ρ‚ΠΎΠΉ стСпСни Π°Π±ΡΠΎΠ»ΡŽΡ‚Π½ΠΎΠΉ (Π² ΠšΠ΅Π»ΡŒΠ²ΠΈΠ½Π°Ρ…) Ρ‚Π΅ΠΌΠΏΠ΅Ρ€Π°Ρ‚ΡƒΡ€Ρ‹ повСрхности, поэтому Ρƒ Ρ€Π°Π΄ΠΈΠ°Ρ‚ΠΎΡ€ΠΎΠ² ΠΎΡ‡Π΅Π½ΡŒ большая доля повСрхности ΠΊΠΎΠ½Ρ‚Π°ΠΊΡ‚ΠΈΡ€ΡƒΠ΅Ρ‚ нСпосрСдствСнно с тСплоноситСлСм. Для ΠΊΠΎΠ½Π²Π΅ΠΊΡ†ΠΈΠΈ Π·Π°Π²ΠΈΡΠΈΠΌΠΎΡΡ‚ΡŒ ΠΎΡ‚ Ρ‚Π΅ΠΌΠΏΠ΅Ρ€Π°Ρ‚ΡƒΡ€Ρ‹ линСйная, поэтому Ρ€Π°Ρ†ΠΈΠΎΠ½Π°Π»ΡŒΠ½Π΅Π΅ ΡƒΠ²Π΅Π»ΠΈΡ‡ΠΈΡ‚ΡŒ ΠΏΠ»ΠΎΡ‰Π°Π΄ΡŒ Π² нСсколько Ρ€Π°Π· ΠΎΡ€Π΅Π±Ρ€Π΅Π½ΠΈΠ΅ΠΌ — Ρ‚Π΅ΠΌΠΏΠ΅Ρ€Π°Ρ‚ΡƒΡ€Π° Ρ€Π΅Π±Π΅Ρ€ Π±ΡƒΠ΄Π΅Ρ‚ сущСствСнно Π½ΠΈΠΆΠ΅ Ρ‚Π΅ΠΌΠΏΠ΅Ρ€Π°Ρ‚ΡƒΡ€Ρ‹ тСплоноситСля, Π½ΠΎ Π·Π° счСт ΠΌΠ½ΠΎΠ³ΠΎΠΊΡ€Π°Ρ‚Π½ΠΎΠ³ΠΎ увСличСния ΠΏΠ»ΠΎΡ‰Π°Π΄ΠΈ ΡΡ„Ρ„Π΅ΠΊΡ‚ΠΈΠ²Π½ΠΎΡΡ‚ΡŒ возрастСт. ΠšΡ€ΠΎΠΌΠ΅ Ρ‚ΠΎΠ³ΠΎ, ΡƒΡΠΈΠ»Π΅Π½ΠΈΡŽ ΠΊΠΎΠ½Π²Π΅ΠΊΡ†ΠΈ ΠΎΡ‡Π΅Π½ΡŒ способствуСт ΡƒΠ²Π΅Π»ΠΈΡ‡Π΅Π½ΠΈΠ΅ скорости ΠΏΠΎΡ‚ΠΎΠΊΠ° Π²ΠΎΠ·Π΄ΡƒΡ…Π°, ΠΎΠ±Π΄ΡƒΠ²Π°ΡŽΡ‰Π΅Π³ΠΎ Ρ€Π΅Π±Ρ€Π°, Ρ‡Π΅Π³ΠΎ ΠΌΠΎΠΆΠ½ΠΎ Π΄ΠΎΠ±ΠΈΡ‚ΡŒΡΡ, ΡƒΠ²Π΅Π»ΠΈΡ‡ΠΈΠ² высоту ΠΊΠΎΠ½Π²Π΅ΠΊΡ‚ΠΈΠ²Π½Ρ‹Ρ… Ρ€Π΅Π±Π΅Ρ€.
Β Β Β Β Β Β Β Π’Π΅ΠΏΠ΅Ρ€ΡŒ Π΄Π°Π²Π°ΠΉΡ‚Π΅ Π½Π΅ΠΌΠ½ΠΎΠ³ΠΎ разбСрСмся с Ρ‚Π΅Ρ€ΠΌΠΈΠ½ΠΎΠ»ΠΎΠ³ΠΈΠ΅ΠΉ. Π Π°Π΄ΠΈΠ°Ρ‚ΠΎΡ€ΠΎΠΌ называСтся Ρ‚Π°ΠΊΠΎΠΉ ΠΎΡ‚ΠΎΠΏΠΈΡ‚Π΅Π»ΡŒΠ½Ρ‹ΠΉ ΠΏΡ€ΠΈΠ±ΠΎΡ€, Ρƒ ΠΊΠΎΡ‚ΠΎΡ€ΠΎΠ³ΠΎ доля Ρ‚Π΅ΠΏΠ»ΠΎΠΎΡ‚Π΄Π°Ρ‡ΠΈ ΠΈΠ·Π»ΡƒΡ‡Π΅Π½ΠΈΠ΅ΠΌ составляСт Π±ΠΎΠ»Π΅Π΅ 25%. Однако ΠΎΡ‡Π΅Π½ΡŒ часто (ΠΈ Π² Ρ‚ΠΎΡ€Π³ΡƒΡŽΡ‰ΠΈΡ… Ρ„ΠΈΡ€ΠΌΠ°Ρ…, ΠΈ Π² ΠΈΠ½Ρ‚Π΅Ρ€Π½Π΅Ρ‚Π΅) Ρ€Π°Π΄ΠΈΠ°Ρ‚ΠΎΡ€ΠΎΠΌ Π½Π°Π·Ρ‹Π²Π°ΡŽΡ‚ любой ΠΎΡ‚ΠΎΠΏΠΈΡ‚Π΅Π»ΡŒΠ½Ρ‹ΠΉ ΠΏΡ€ΠΈΠ±ΠΎΡ€, Ρ‡Ρ‚ΠΎ Π½Π΅ совсСм Π²Π΅Ρ€Π½ΠΎ. ΠšΡ€ΠΎΠΌΠ΅ Ρ‚ΠΎΠ³ΠΎ, ΠΏΡ€ΠΈ сильном ΠΈΠ·ΠΌΠ΅Π½Π΅Π½ΠΈΠΈ Ρ‚Π΅ΠΌΠΏΠ΅Ρ€Π°Ρ‚ΡƒΡ€Ρ‹ тСплоноситСля Π²ΠΎΠ·ΠΌΠΎΠΆΠ΅Π½ «ΠΏΠ΅Ρ€Π΅Ρ…ΠΎΠ΄» ΠΏΡ€ΠΈΠ±ΠΎΡ€Π° ΠΈΠ· ΠΊΠ°Ρ‚Π΅Π³ΠΎΡ€ΠΈΠΈ «Ρ€Π°Π΄ΠΈΠ°Ρ‚ΠΎΡ€» Π² ΠΊΠ°Ρ‚Π΅Π³ΠΎΡ€ΠΈΡŽ «Π½Π΅ Ρ€Π°Π΄ΠΈΠ°Ρ‚ΠΎΡ€» ΠΈ ΠΎΠ±Ρ€Π°Ρ‚Π½ΠΎ. Π’Π°ΠΊ Ρ‡Ρ‚ΠΎ Π½Π΅ Π±ΡƒΠ΄Π΅ΠΌ Π½Π° этом Π·Π°Ρ†ΠΈΠΊΠ»ΠΈΠ²Π°Ρ‚ΡŒΡΡ, ΠΌΠ΅Ρ‚ΠΎΠ΄ΠΈΠΊΠ° расчСта Ρ‚Π΅ΠΏΠ»ΠΎΠΎΡ‚Π΄Π°Ρ‡ΠΈ ΠΎΡ‚ названия, слава Π±ΠΎΠ³Ρƒ, Π½Π΅ зависит.

Π‘Ρ‚Π°Π»ΡŒΠ½Ρ‹Π΅ Ρ€Π°Π΄ΠΈΠ°Ρ‚ΠΎΡ€Ρ‹

       НаиболСС распространСнными ΠΈ ΠΎΡ‚Π½ΠΎΡΠΈΡ‚Π΅Π»ΡŒΠ½ΠΎ Π½Π΅Π΄ΠΎΡ€ΠΎΠ³ΠΈΠΌΠΈ Π² настоящСС врСмя ΡΠ²Π»ΡΡŽΡ‚ΡΡ ΡΡ‚Π°Π»ΡŒΠ½Ρ‹Π΅ ΠΏΠ°Π½Π΅Π»ΡŒΠ½Ρ‹Π΅ Ρ€Π°Π΄ΠΈΠ°Ρ‚ΠΎΡ€Ρ‹.
       БущСствуСт Π²Π΅Π»ΠΈΠΊΠΎΠ΅ мноТСство Ρ‚ΠΎΡ€Π³ΠΎΠ²Ρ‹Ρ… ΠΌΠ°Ρ€ΠΎΠΊ ΠΈ Π½Π°Π·Π²Π°Π½ΠΈΠΉ, Π½ΠΎ Π² Ρ€Π΅Π°Π»ΡŒΠ½ΠΎΡΡ‚ΠΈ ΠΏΠΎΡ‡Ρ‚ΠΈ всС ΠΎΠ½ΠΈ ΠΎΡ‡Π΅Π½ΡŒ ΠΏΠΎΡ…ΠΎΠΆΠΈ ΠΏΠΎ Π²Π½Π΅ΡˆΠ½Π΅ΠΌΡƒ Π²ΠΈΠ΄Ρƒ ΠΈ ΠΏΡ€ΠΎΡ‡ΠΈΠΌ характСристикам. ΠŸΡ€ΠΈΡ‡ΠΈΠ½Π° этого явлСния состоит Π² ΡΠ»Π΅Π΄ΡƒΡŽΡ‰Π΅ΠΌ: сущСствуСт финский ΠΊΠΎΠ½Ρ†Π΅Ρ€Π½ Rettig (ΠΏΠΎΠ΄Ρ€Π°Π·Π΄Π΅Π»Π΅Π½ΠΈΠ΅ Rettig Heating Group занимаСтся Ρ€Π°Π΄ΠΈΠ°Ρ‚ΠΎΡ€Π°ΠΌΠΈ), ΠΊΠΎΡ‚ΠΎΡ€Ρ‹ΠΉ ΠΏΡ€ΠΎΠΈΠ·Π²ΠΎΠ΄ΠΈΡ‚ Ρ€Π°Π΄ΠΈΠ°Ρ‚ΠΎΡ€Ρ‹ ΠΈ ΠΎΠ±ΠΎΡ€ΡƒΠ΄ΠΎΠ²Π°Π½ΠΈΠ΅ для ΠΈΡ… изготовлСния. ΠšΠΎΠ½Ρ†Π΅Ρ€Π½ строит (ΠΏΠΎΠΊΡƒΠΏΠ°Π΅Ρ‚) свои Π·Π°Π²ΠΎΠ΄Ρ‹ (Ρ‚ΠΎΡ€Π³ΠΎΠ²Ρ‹Π΅ ΠΌΠ°Ρ€ΠΊΠΈ Delta, Dianorm, Finimetal, LVI, Myson, Purmo, Radson, Thermopanel, Vogel & Noot, MMA & Myson Heating Controls, UWE, Jukova ΠΈ Π΄Ρ€., Π·Π°Π²ΠΎΠ΄Ρ‹ Π² Π¨Π²Π΅Ρ†ΠΈΠΈ, Π“Π΅Ρ€ΠΌΠ°Π½ΠΈΠΈ, Π‘Π΅Π»ΡŒΠ³ΠΈΠΈ, Π’Π΅Π»ΠΈΠΊΠΎΠ±Ρ€ΠΈΡ‚Π°Π½ΠΈΠΈ, Π€Ρ€Π°Π½Ρ†ΠΈΠΈ ΠΈ ПольшС), Π° Ρ‚Π°ΠΊΠΆΠ΅ поставляСт ΠΎΠ±ΠΎΡ€ΡƒΠ΄ΠΎΠ²Π°Π½ΠΈΠ΅ Π΄Ρ€ΡƒΠ³ΠΈΠΌ компаниям (Korado Π² Π§Π΅Ρ…ΠΈΠΈ, Korad Π² Π‘Π»ΠΎΠ²Π°ΠΊΠΈΠΈ ΠΈ Ρ‚.Π΄.). ΠŸΠΎΡΡ‚ΠΎΠΌΡƒ ΠΈΡ… характСристики довольно Π±Π»ΠΈΠ·ΠΊΠΈ. Π’ΠΏΠ»ΠΎΡ‚ΡŒ Π΄ΠΎ Ρ‚Π°Π±Π»ΠΈΡ† Ρ‚Π΅ΠΏΠ»ΠΎΠΎΡ‚Π΄Π°Ρ‡ΠΈ для Π½Π΅ΠΊΠΎΠ΅Π³ΠΎ усрСднСнного ΡΡ‚Π°Π»ΡŒΠ½ΠΎΠ³ΠΎ Ρ€Π°Π΄ΠΈΠ°Ρ‚ΠΎΡ€Π°. Однако Π½ΡŒΡŽΠ°Π½ΡΡ‹ Π΅ΡΡ‚ΡŒ ΠΈ, Π²ΠΎΠ·ΠΌΠΎΠΆΠ½ΠΎ, ΠΊΠΎΠ³Π΄Π°-Π½ΠΈΠ±ΡƒΠ΄ΡŒ я ΠΎ Π½ΠΈΡ… расскаТу.
Β Β Β Β Β Β Β ΠšΠΎΠ½ΡΡ‚Ρ€ΡƒΠΊΡ‚ΠΈΠ²Π½ΠΎ ΠΎΠ½ΠΈ ΠΏΡ€Π΅Π΄ΡΡ‚Π°Π²Π»ΡΡŽΡ‚ собой ΡˆΡ‚Π°ΠΌΠΏΠΎΠ²Π°Π½Π½Ρ‹Π΅ ΡΡ‚Π°Π»ΡŒΠ½Ρ‹Π΅ ΠΏΠ°Π½Π΅Π»ΠΈ, соСдинСнныС Π΄Ρ€ΡƒΠ³ с Π΄Ρ€ΡƒΠ³ΠΎΠΌ ΠΏΡ€ΠΈ ΠΏΠΎΠΌΠΎΡ‰ΠΈ сварки. Π’ΠΎΠ»Ρ‰ΠΈΠ½Π° листов ΠΏΠΎ СвропСйским Π½ΠΎΡ€ΠΌΠ°ΠΌ Π΄ΠΎΠ»ΠΆΠ½Π° Π±Ρ‹Ρ‚ΡŒ Π½Π΅ ΠΌΠ΅Π½Π΅Π΅ 1.11 ΠΌΠΌ, Π½Π° ΠΏΡ€Π°ΠΊΡ‚ΠΈΠΊΠ΅ ΠΎΠ½Π° колСблСтся ΠΎΡ‚ 1.12 Π΄ΠΎ 1.25 ΠΌΠΌ (СдинствСнноС извСстноС ΠΌΠ½Π΅ ΠΈΡΠΊΠ»ΡŽΡ‡Π΅Π½ΠΈΠ΅ — Kermi, ΠΎΠ½ΠΈ говорят ΠΎ Ρ‚ΠΎΠ»Ρ‰ΠΈΠ½Π΅ 2.5 ΠΌΠΌ), ΠΏΡ€ΠΈΡ‡Π΅ΠΌ Ρ‚ΠΎΠ»Ρ‰ΠΈΠ½Π° ΠΌΠ΅Ρ‚Π°Π»Π»Π° ΠΊΠΎΠ½ΠΊΡ€Π΅Ρ‚Π½ΠΎΠ³ΠΎ Ρ€Π°Π΄ΠΈΠ°Ρ‚ΠΎΡ€Π° зависит ΠΎΡ‚ ΠΊΠΎΠ½ΠΊΡ€Π΅Ρ‚Π½ΠΎΠ³ΠΎ ΠΌΠΎΡ‚ΠΊΠ° ΡΡ‚Π°Π»ΡŒΠ½ΠΎΠ³ΠΎ листа — ΠΏΡ€ΠΎΠΈΠ·Π²ΠΎΠ΄ΠΈΡ‚ΡŒ ΡΡ‚Π°Π»ΡŒΠ½Ρ‹Π΅ листы с Π³Π°Ρ€Π°Π½Ρ‚ΠΈΡ€ΠΎΠ²Π°Π½Π½ΠΎΠΉ Ρ‚ΠΎΡ‡Π½ΠΎΡΡ‚ΡŒΡŽ Π² сотыС Π΄ΠΎΠ»ΠΈ ΠΌΠΈΠ»Π»ΠΈΠΌΠ΅Ρ‚Ρ€Π° ΠΏΠΎΠΊΠ° Π½Π΅ Π½Π°ΡƒΡ‡ΠΈΠ»ΠΈΡΡŒ. А ΠΏΠΎΡΠΊΠΎΠ»ΡŒΠΊΡƒ ΠΌΠ΅Ρ‚Π°Π»Π» Ρ‚Ρ€Π°Π΄ΠΈΡ†ΠΈΠΎΠ½Π½ΠΎ покупаСтся Π½Π° вСс, Ρ‚ΠΎ Π±ΠΎΠ»ΡŒΡˆΠΈΠ½ΡΡ‚Π²ΠΎ Ρ€Π°Π΄ΠΈΠ°Ρ‚ΠΎΡ€ΠΎΠ² ΠΈΠΌΠ΅Π΅Ρ‚ Ρ‚ΠΎΠ»Ρ‰ΠΈΠ½Ρƒ стСнки большС 1.11, Π½ΠΎ Π΄ΠΎ 1.25 «Π½Π΅ дотягиваСт», Ρ‡Ρ‚ΠΎ Π±Ρ‹ Ρ‚Π°ΠΌ Π½ΠΈ Π³ΠΎΠ²ΠΎΡ€ΠΈΠ»ΠΈ ΠΌΠ΅Π½Π΅Π΄ΠΆΠ΅Ρ€Ρ‹…
       К панСлям ΠΌΠΎΠΆΠ΅Ρ‚ ΠΊΡ€Π΅ΠΏΠΈΡ‚ΡŒΡΡ конвСкторная Ρ‡Π°ΡΡ‚ΡŒ (Β«Π³Π°Ρ€ΠΌΠΎΡˆΠΊΠ°Β» ΠΈΠ· ΡΡ‚Π°Π»ΡŒΠ½ΠΎΠ³ΠΎ листа Ρ‚ΠΎΠ»Ρ‰ΠΈΠ½ΠΎΠΉ 0,5 ΠΌΠΌ). Π’ зависимости ΠΎΡ‚ ΠΊΠΎΠΌΠ±ΠΈΠ½Π°Ρ†ΠΈΠΉ Ρ€Π°Π΄ΠΈΠ°Ρ‚ΠΎΡ€Π½Ρ‹Ρ… ΠΏΠ°Π½Π΅Π»Π΅ΠΉ ΠΈ ΠΊΠΎΠ½Π²Π΅ΠΊΡ‚ΠΎΡ€ΠΎΠ² Ρ€Π°Π΄ΠΈΠ°Ρ‚ΠΎΡ€Ρ‹ дСлятся Π½Π° Ρ‚ΠΈΠΏΡ‹, прСдставлСнныС Π² Ρ‚Π°Π±Π»ΠΈΡ†Π΅. Π—Π°ΠΌΠ΅Ρ‡Ρƒ, Ρ‡Ρ‚ΠΎ Π±ΡƒΠΊΠ²Π΅Π½Π½Ρ‹Π΅ обозначСния Ρ‚ΠΈΠΏΠΎΠ² Ρƒ Ρ€Π°Π·Π½Ρ‹Ρ… ΠΈΠ·Π³ΠΎΡ‚ΠΎΠ²ΠΈΡ‚Π΅Π»Π΅ΠΉ ΠΌΠΎΠ³ΡƒΡ‚ ΠΎΡ‚Π»ΠΈΡ‡Π°Ρ‚ΡŒΡΡ, поэтому самым Π½Π°Π΄Π΅ΠΆΠ½Ρ‹ΠΌ Π±ΡƒΠ΄Π΅Ρ‚ Ρ†ΠΈΡ„Ρ€ΠΎΠ²ΠΎΠ΅ ΠΎΠ±ΠΎΠ·Π½Π°Ρ‡Π΅Π½ΠΈΠ΅. Π’ΠΈΠΏΡ‹ ΡΡ‚Π°Π»ΡŒΠ½Ρ‹Ρ… ΠΏΠ°Π½Π΅Π»ΡŒΠ½Ρ‹Ρ… Ρ€Π°Π΄ΠΈΠ°Ρ‚ΠΎΡ€ΠΎΠ²
       По Π³Π°Π±Π°Ρ€ΠΈΡ‚Π½Ρ‹ΠΌ Ρ€Π°Π·ΠΌΠ΅Ρ€Π°ΠΌ ΠΊΠ°Ρ€Ρ‚ΠΈΠ½Π° ΠΏΡ€ΠΈΠΌΠ΅Ρ€Π½ΠΎ ΡΠ»Π΅Π΄ΡƒΡŽΡ‰Π°Ρ:
Β Β Β Β Β Β Β Π’ΠΎΠ»Ρ‰ΠΈΠ½Π° (Π³Π»ΡƒΠ±ΠΈΠ½Π°) Ρ€Π°Π΄ΠΈΠ°Ρ‚ΠΎΡ€Π° зависит ΠΎΡ‚ Π΅Π³ΠΎ Ρ‚ΠΈΠΏΠ° (см. Ρ‚Π°Π±Π»ΠΈΡ‡ΠΊΡƒ Π²Ρ‹ΡˆΠ΅) ΠΈ составляСт ΠΏΡ€ΠΈΠΌΠ΅Ρ€Π½ΠΎ 50-60 ΠΌΠΌ для 10, 11 Ρ‚ΠΈΠΏΠ°, 100 ΠΌΠΌ для 20, 21, 22 Ρ‚ΠΈΠΏΠ° ΠΈ 150 ΠΌΠΌ для 33 Ρ‚ΠΈΠΏΠ°. Высота Ρ€Π°Π΄ΠΈΠ°Ρ‚ΠΎΡ€Π° ΠΎΠ±Ρ‹Ρ‡Π½ΠΎ соотвСтствуСт ряду Π² 300, 400, 500, 600 ΠΈ 900 ΠΌΠΌ, ΡˆΠΈΡ€ΠΈΠ½Π° Ρ€Π°Π΄ΠΈΠ°Ρ‚ΠΎΡ€Π° (Ρ‡Π°Ρ‰Π΅ всСго, Π½ΠΎ Π΅ΡΡ‚ΡŒ отклонСния Ρƒ Π½Π΅ΠΊΠΎΡ‚ΠΎΡ€Ρ‹Ρ… ΠΏΡ€ΠΎΠΈΠ·Π²ΠΎΠ΄ΠΈΡ‚Π΅Π»Π΅ΠΉ) ΠΈΠ΄Π΅Ρ‚ ΠΎΡ‚ 400 Π΄ΠΎ 1200 ΠΌΠΌ с шагом 100 ΠΌΠΌ, ΠΎΡ‚ 1300 Π΄ΠΎ 2000 ΠΌΠΌ — с шагом 200 ΠΌΠΌ, Π΄Π°Π»Π΅Π΅ 2300 ΠΌΠΌ, 2600 ΠΌΠΌ ΠΈ 3000 ΠΌΠΌ.
Β Β Β Β Β Β Β ΠŸΠΎΠ΄ΠΊΠ»ΡŽΡ‡Π΅Π½ΠΈΠ΅ Ρ‚Ρ€ΡƒΠ±ΠΎΠΏΡ€ΠΎΠ²ΠΎΠ΄ΠΎΠ² ΠΊ Ρ€Π°Π΄ΠΈΠ°Ρ‚ΠΎΡ€Ρƒ ΠΈΠΌΠ΅Π΅Ρ‚ нСсколько Π²Π°Ρ€ΠΈΠ°Π½Ρ‚ΠΎΠ² (ΠΊ соТалСнию, Ρƒ Ρ€Π°Π·Π½Ρ‹Ρ… Ρ„ΠΈΡ€ΠΌ обозначаСтся ΡΠΎΠ²Π΅Ρ€ΡˆΠ΅Π½Π½ΠΎ ΠΏΠΎ Ρ€Π°Π·Π½ΠΎΠΌΡƒ). ΠŸΡ€ΠΈ «Π±ΠΎΠΊΠΎΠ²ΠΎΠΌ» ΠΏΠΎΠ΄ΠΊΠ»ΡŽΡ‡Π΅Π½ΠΈΠΈ Ρ€Π°Π΄ΠΈΠ°Ρ‚ΠΎΡ€ ΠΈΠΌΠ΅Π΅Ρ‚ слСва ΠΈ справа ΠΏΠΎ Π΄Π²Π° Ρ€Π΅Π·ΡŒΠ±ΠΎΠ²Ρ‹Ρ… Π²Ρ‹Ρ…ΠΎΠ΄Π°. «ΠΠΈΠΆΠ½Π΅Π΅» ΠΏΠΎΠ΄ΠΊΠ»ΡŽΡ‡Π΅Π½ΠΈΠ΅ ΠΈΠΌΠ΅Π΅Ρ‚ Π΄Π²Π° Ρ€Π΅Π·ΡŒΠ±ΠΎΠ²Ρ‹Ρ… Π²Ρ‹Ρ…ΠΎΠ΄Π° снизу*ΠŸΡ€ΠΈ Π½Π°Π»ΠΈΡ‡ΠΈΠΈ Π½ΠΈΠΆΠ½Π΅Π³ΠΎ ΠΏΠΎΠ΄ΠΊΠ»ΡŽΡ‡Π΅Π½ΠΈΡ Π±ΠΎΠΊΠΎΠ²Ρ‹Π΅ ΠΏΠΎΠ΄ΠΊΠ»ΡŽΡ‡Π΅Π½ΠΈΡ, собствСнно говоря, Π½ΠΈΠΊΡƒΠ΄Π° Π½Π΅ Π΄Π΅Π²Π°ΡŽΡ‚ΡΡ — ΠΎΠ½ΠΈ Π·Π°Π³Π»ΡƒΡˆΠ΅Π½Ρ‹ ΠΏΡ€ΠΎΠ±ΠΊΠ°ΠΌΠΈ, Π½ΠΎ ΠΏΡ€ΠΈ нСобходимости ΠΈΠΌΠΈ ΠΌΠΎΠΆΠ½ΠΎ ΠΈ Π½ΡƒΠΆΠ½ΠΎ Π²ΠΎΡΠΏΠΎΠ»ΡŒΠ·ΠΎΠ²Π°Ρ‚ΡŒΡΡ. (ΠΎΠ±Ρ‹Ρ‡Π½ΠΎ справа, Π΅ΡΡ‚ΡŒ ΠΈ слСва, Π΅ΡΡ‚ΡŒ ΠΈ ΠΏΠΎ Ρ†Π΅Π½Ρ‚Ρ€Ρƒ, Π½ΠΎ эти Π±ΠΎΠ»Π΅Π΅ Ρ€Π΅Π΄ΠΊΠΈ). Один ΠΈΠ· Π½ΠΈΠΆΠ½ΠΈΡ… Π²Ρ‹Ρ…ΠΎΠ΄ΠΎΠ² (Π²Ρ…ΠΎΠ΄ Π² Ρ€Π°Π΄ΠΈΠ°Ρ‚ΠΎΡ€) поднимаСтся Ρ‚Ρ€ΡƒΠ±ΠΊΠΎΠΉ Π²Π²Π΅Ρ€Ρ… ΠΈ ΠΏΡ€ΠΎΡ…ΠΎΠ΄ΠΈΡ‚ Ρ‡Π΅Ρ€Π΅Π· встроСнный тСрмостатичСский Π²Π΅Π½Ρ‚ΠΈΠ»ΡŒ (достаточно ΡƒΡΡ‚Π°Π½ΠΎΠ²ΠΈΡ‚ΡŒ Π½Π° Π½Π΅Π³ΠΎ Ρ‚Π΅Ρ€ΠΌΠΎΠ³ΠΎΠ»ΠΎΠ²ΠΊΡƒ, ΠΈ Π²ΠΎΠ·ΠΌΠΎΠΆΠ½ΠΎΡΡ‚ΡŒ Ρ€Π΅Π³ΡƒΠ»ΠΈΡ€ΠΎΠ²ΠΊΠΈ Ρ‚Π΅ΠΌΠΏΠ΅Ρ€Π°Ρ‚ΡƒΡ€Ρ‹ Π² ΠΏΠΎΠΌΠ΅Ρ‰Π΅Π½ΠΈΠΈ ΡƒΠΆΠ΅ Π΅ΡΡ‚ΡŒ). Как ΠΏΡ€Π°Π²ΠΈΠ»ΠΎ, ΠΏΡ€ΠΈ Π½ΠΈΠΆΠ½Π΅ΠΌ ΠΏΠΎΠ΄ΠΊΠ»ΡŽΡ‡Π΅Π½ΠΈΠΈ Π² ΠΎΠ±ΠΎΠ·Π½Π°Ρ‡Π΅Π½ΠΈΠΈ ΠΏΠΎΠ΄ΠΊΠ»ΡŽΡ‡Π΅Π½ΠΈΡ Π² Ρ‚ΠΎΠΌ ΠΈΠ»ΠΈ ΠΈΠ½ΠΎΠΌ Π²ΠΈΠ΄Π΅ Ρ„ΠΈΠ³ΡƒΡ€ΠΈΡ€ΡƒΠ΅Ρ‚ Π±ΡƒΠΊΠ²Π° V (Ventil).
Β Β Β Β Β Β Β Π¦Π²Π΅Ρ‚ Ρ€Π°Π΄ΠΈΠ°Ρ‚ΠΎΡ€Π°, ΠΊΠ°ΠΊ ΠΏΡ€Π°Π²ΠΈΠ»ΠΎ, Π±Π΅Π»Ρ‹ΠΉ, ΠΎΠ΄Π½Π°ΠΊΠΎ Π΅ΡΡ‚ΡŒ Π²ΠΎΠ·ΠΌΠΎΠΆΠ½ΠΎΡΡ‚ΡŒ Π·Π°ΠΊΠ°Π·Π°Ρ‚ΡŒ Ρƒ производитСля Π΄Ρ€ΡƒΠ³ΠΈΠ΅ Ρ†Π²Π΅Ρ‚Π° согласно ΠΊΠ°Ρ‚Π°Π»ΠΎΠ³Π° Ρ†Π²Π΅Ρ‚ΠΎΠ² RAL (СстСствСнно, Π±ΡƒΠ΄ΡƒΡ‚ ΠΏΡ€ΠΎΡ†Π΅Π½Ρ‚ΠΎΠ² Π½Π° 20 Π΄ΠΎΡ€ΠΎΠΆΠ΅).
       Помимо ΡƒΠΊΠ°Π·Π°Π½Π½Ρ‹Ρ… Π²Ρ‹ΡˆΠ΅ Ρ‚ΠΈΠΏΠΎΠ², ΠΈΠ· стали производятся ΠΈ Ρ‚Π°ΠΊ Π½Π°Π·Ρ‹Π²Π°Π΅ΠΌΡ‹Π΅ «Π΄ΠΈΠ·Π°ΠΉΠ½-Ρ€Π°Π΄ΠΈΠ°Ρ‚ΠΎΡ€Ρ‹». Они ΠΌΠΎΠ³ΡƒΡ‚ ΠΈΠΌΠ΅Ρ‚ΡŒ ΡΠ°ΠΌΡƒΡŽ ΡΠΊΡΡ‚Ρ€Π°Π²Π°Π³Π°Π½Ρ‚Π½ΡƒΡŽ Ρ„ΠΎΡ€ΠΌΡƒ, Ρ†Π²Π΅Ρ‚ (Π²ΠΏΠ»ΠΎΡ‚ΡŒ Π΄ΠΎ ΡƒΠ·ΠΎΡ€ΠΎΠ²). Π•ΡΡ‚ΡŒ Π·Π°ΠΌΠ΅Ρ‡Π°Ρ‚Π΅Π»ΡŒΠ½Π°Ρ ΠΏΡ€ΠΎΠ³Ρ€Π°ΠΌΠΊΠ° ΠΎΡ‚ «Vogel & Noot», Π² ΠΊΠΎΡ‚ΠΎΡ€ΠΎΠΉ ΠΌΠΎΠΆΠ½ΠΎ Π²Ρ‹Π±Ρ€Π°Ρ‚ΡŒ ΠΈΠ½Ρ‚Π΅Ρ€ΡŒΠ΅Ρ€, Ρ†Π²Π΅Ρ‚ стСн, ΠΏΠΎΠ»Π° ΠΈ Π²ΡΠ»Π°ΡΡ‚ΡŒ ΠΏΠΎΠΈΠ³Ρ€Π°Ρ‚ΡŒΡΡ с ΠΏΠΎΠ΄Π±ΠΎΡ€ΠΎΠΌ Ρ‚ΠΈΠΏΠΎΠ² ΠΏΡ€ΠΈΠ±ΠΎΡ€ΠΎΠ², ΠΈΡ… Ρ†Π²Π΅Ρ‚ΠΎΠ² ΠΈ ΡƒΠ·ΠΎΡ€ΠΎΠ². На ΠΏΡ€ΠΈΠ²Π΅Π΄Π΅Π½Π½Ρ‹Ρ… Π½ΠΈΠΆΠ΅ ΠΊΠ»ΠΈΠΊΠ°Π±Π΅Π»ΡŒΠ½Ρ‹Ρ… слайдах я попытался ΠΏΠΎΠΊΠ°Π·Π°Ρ‚ΡŒ, ΠΊΠ°ΠΊ это ΠΏΡ€ΠΈΠΌΠ΅Ρ€Π½ΠΎ выглядит.
       К соТалСнию, Π΄Π°ΠΆΠ΅ послС «ΠΎΠ±Ρ€Π΅Π·Π°Π½ΠΈΡ» всСго лишнСго сиС «ΡΡ‡Π°ΡΡ‚ΠΈΠ΅» вСсит Π±Π΅Π· ΠΌΠ°Π»ΠΎΠ³ΠΎ 18 ΠΌΠ΅Π³Π°Π±Π°ΠΉΡ‚. Π‘ΠΊΠ°Ρ‡Π°Ρ‚ΡŒ ΠΏΡ€ΠΎΠ³Ρ€Π°ΠΌΠΊΡƒ ΠΌΠΎΠΆΠ½ΠΎ здСсь. НСкоторыС Π±Ρ€Π°ΡƒΠ·Π΅Ρ€Ρ‹ сразу ΠΏΡ‹Ρ‚Π°ΡŽΡ‚ΡΡ ΠΎΡ‚ΠΊΡ€Ρ‹Ρ‚ΡŒ Ρ„Π°ΠΉΠ», Ρ‚Π°ΠΊ Ρ‡Ρ‚ΠΎ Π»ΡƒΡ‡ΡˆΠ΅ ΠΊΠ»ΠΈΠΊΠ½ΡƒΡ‚ΡŒ Π½Π° ссылкС ΠΏΡ€Π°Π²ΠΎΠΉ ΠΊΠ½ΠΎΠΏΠΊΠΎΠΉ ΠΌΡ‹ΡˆΠΊΠΈ ΠΈ Π²Ρ‹Π±Ρ€Π°Ρ‚ΡŒ «Π‘ΠΎΡ…Ρ€Π°Π½ΠΈΡ‚ΡŒ Ρ„Π°ΠΉΠ» ΠΊΠ°ΠΊ…».
Β Β Β Β Β Β Β ΠžΡ‚ΠΊΡ€Ρ‹Π²Π°Π΅ΠΌ Ρ„Π°ΠΉΠ» Ρ„Π»Π΅Ρˆ-ΠΏΠ»Π΅Π΅Ρ€ΠΎΠΌ Π»ΠΈΠ±ΠΎ Π±Ρ€Π°ΡƒΠ·Π΅Ρ€ΠΎΠΌ. На Π²ΠΊΠ»Π°Π΄ΠΊΠ΅ «ΠŸΠΎΠΌΠ΅Ρ‰Π΅Π½ΠΈΠ΅» Π²Ρ‹Π±ΠΈΡ€Π°Π΅ΠΌ ΠΈΠ½Ρ‚Π΅Ρ€ΡŒΠ΅Ρ€, ΠΌΡ‹ΡˆΠΊΠΎΠΉ «Π²Ρ‹Π»Π°Π²Π»ΠΈΠ²Π°Π΅ΠΌ» Ρ†Π²Π΅Ρ‚ стСн ΠΈ Ρ†Π²Π΅Ρ‚ ΠΏΠΎΠ»Π°, На Π²ΠΊΠ»Π°Π΄ΠΊΠ΅ «Π Π°Π΄ΠΈΠ°Ρ‚ΠΎΡ€Ρ‹» Π²Ρ‹Π±ΠΈΡ€Π°Π΅ΠΌ ΠΏΡ€ΠΈΠ±ΠΎΡ€, Π½Π° Π²ΠΊΠ»Π°Π΄ΠΊΠ΅ «Ρ†Π²Π΅Ρ‚ Ρ€Π°Π΄ΠΈΠ°Ρ‚ΠΎΡ€Π°» Π²Ρ‹Π±ΠΈΡ€Π°Π΅ΠΌ Ρ†Π²Π΅Ρ‚. Π’ зависимости ΠΎΡ‚ Ρ‚ΠΈΠΏΠ° ΠΏΡ€ΠΈΠ±ΠΎΡ€Π° ΠΏΠΎΡΠ²Π»ΡΡŽΡ‚ΡΡ Π²ΠΊΠ»Π°Π΄ΠΊΠΈ «Π ΠΈΡΡƒΠ½ΠΎΠΊ» ΠΈ Ρ‚.Π΄. Π’ ΠΎΠ±Ρ‰Π΅ΠΌ, Π·Π°Π½ΡΡ‚ΡŒΡΡ Π±ΡƒΠ΄Π΅Ρ‚ Ρ‡Π΅ΠΌ!

       НС всС Π·Π°Π²ΠΎΠ΄Ρ‹ Π²Ρ‹ΠΏΡƒΡΠΊΠ°ΡŽΡ‚ всС Π²Ρ‹ΡˆΠ΅ΡƒΠΊΠ°Π·Π°Π½Π½Ρ‹Π΅ Ρ‚ΠΈΠΏΡ‹ ΠΈ Ρ€Π°Π·ΠΌΠ΅Ρ€Ρ‹, ΠΈ Ρ‚Π΅ΠΌ Π±ΠΎΠ»Π΅Π΅ Π½Π΅ всС Ρ‚ΠΎΡ€Π³ΡƒΡŽΡ‰ΠΈΠ΅ Ρ„ΠΈΡ€ΠΌΡ‹ «возят» вСсь Π½ΠΎΠΌΠ΅Π½ΠΊΠ»Π°Ρ‚ΡƒΡ€Π½Ρ‹ΠΉ ряд. НаиболСС Β«Ρ…ΠΎΠ΄ΠΎΠ²Ρ‹ΠΌΠΈΒ» ΡΠ²Π»ΡΡŽΡ‚ΡΡ 11, 22 ΠΈ 33 Ρ‚ΠΈΠΏΡ‹. ΠŸΠΎΡΡ‚ΠΎΠΌΡƒ Π²Ρ‹Π±ΠΎΡ€ Π»ΡŽΠ±Ρ‹Ρ… ΠΎΡ‚ΠΎΠΏΠΈΡ‚Π΅Π»ΡŒΠ½Ρ‹Ρ… ΠΏΡ€ΠΈΠ±ΠΎΡ€ΠΎΠ² Π»ΡƒΡ‡ΡˆΠ΅ всСго Π½Π°Ρ‡Π°Ρ‚ΡŒ с посСщСния 3 – 4 Ρ‚ΠΎΡ€Π³ΡƒΡŽΡ‰ΠΈΡ… ΠΎΡ‚ΠΎΠΏΠΈΡ‚Π΅Π»ΡŒΠ½Ρ‹ΠΌ ΠΎΠ±ΠΎΡ€ΡƒΠ΄ΠΎΠ²Π°Π½ΠΈΠ΅ΠΌ Ρ„ΠΈΡ€ΠΌ Π² окрСстностях ΠΈ изучСния ΠΈΡ… ассортимСнта.
Β Β Β Β Β Β Β ΠšΡ€Π΅ΠΏΠ»Π΅Π½ΠΈΠ΅ Ρ€Π°Π΄ΠΈΠ°Ρ‚ΠΎΡ€ΠΎΠ² ΠΊ стСнС ΠΈΠ»ΠΈ ΠΏΠΎΠ»Ρƒ осущСствляСтся, ΠΊΠ°ΠΊ ΠΏΡ€Π°Π²ΠΈΠ»ΠΎ, Ρ„ΠΈΡ€ΠΌΠ΅Π½Π½Ρ‹ΠΌΠΈ ΠΊΡ€Π΅ΠΏΠ΅ΠΆΠ½Ρ‹ΠΌΠΈ издСлиями. На Π·Π°Π΄Π½Π΅ΠΉ стСнкС Ρ€Π°Π΄ΠΈΠ°Ρ‚ΠΎΡ€Π° Π΅ΡΡ‚ΡŒ ΡΠΏΠ΅Ρ†ΠΈΠ°Π»ΡŒΠ½Ρ‹Π΅ пластины для ΠΊΡ€Π΅ΠΏΠ»Π΅Π½ΠΈΠΉ (4 ΡˆΡ‚ΡƒΠΊΠΈ ΠΏΡ€ΠΈ ΡˆΠΈΡ€ΠΈΠ½Π΅ Ρ€Π°Π΄ΠΈΠ°Ρ‚ΠΎΡ€Π° Π΄ΠΎ 1400 ΠΌΠΌ, 6 ΡˆΡ‚ΡƒΠΊ для Ρ€Π°Π΄ΠΈΠ°Ρ‚ΠΎΡ€ΠΎΠ² ΡˆΠΈΡ€ΠΈΠ½ΠΎΠΉ ΠΎΡ‚ 1600 ΠΌΠΌ.). ΠšΡ€Π΅ΠΏΠ»Π΅Π½ΠΈΠΉ ΠΌΠ½ΠΎΠ³ΠΎ самых Ρ€Π°Π·Π½ΠΎΠΎΠ±Ρ€Π°Π·Π½Ρ‹Ρ…, Π΅ΡΡ‚ΡŒ, Π½Π°ΠΏΡ€ΠΈΠΌΠ΅Ρ€, крСплСния, ΠΊΠΎΡ‚ΠΎΡ€Ρ‹Π΅ Ρ„ΠΈΠΊΡΠΈΡ€ΡƒΡŽΡ‚ΡΡ Π·Π° Π½ΠΈΠΆΠ½ΠΈΠΉ ΠΈ Π²Π΅Ρ€Ρ…Π½ΠΈΠΉ края Ρ€Π°Π΄ΠΈΠ°Ρ‚ΠΎΡ€Π°. Π’ связи с этим Π½Π΅ΠΊΠΎΡ‚ΠΎΡ€Ρ‹Π΅ Ρ€Π°Π΄ΠΈΠ°Ρ‚ΠΎΡ€Ρ‹ с Π½ΠΈΠΆΠ½ΠΈΠΌ ΠΏΠΎΠ΄ΠΊΠ»ΡŽΡ‡Π΅Π½ΠΈΠ΅ΠΌ Π²Ρ‹ΠΏΡƒΡΠΊΠ°ΡŽΡ‚ΡΡ Π±Π΅Π· ΠΊΡ€Π΅ΠΏΠ΅ΠΆΠ½Ρ‹Ρ… пластин (Ρ‚ΠΎ Π΅ΡΡ‚ΡŒ ΠΎΠ½ΠΈ ΠΌΠΎΠ³ΡƒΡ‚ Π±Ρ‹Ρ‚ΡŒ установлСны ΠΊΠ°ΠΊ ΠΏΠΎΠ΄ΠΊΠ»ΡŽΡ‡Π΅Π½ΠΈΠ΅ΠΌ справа, Ρ‚Π°ΠΊ ΠΈ ΠΏΠΎΠ΄ΠΊΠ»ΡŽΡ‡Π΅Π½ΠΈΠ΅ΠΌ слСва).

Π§ΡƒΠ³ΡƒΠ½Π½Ρ‹Π΅ Ρ€Π°Π΄ΠΈΠ°Ρ‚ΠΎΡ€Ρ‹

Β Β Β Β Β Β Β Π§ΡƒΠ³ΡƒΠ½ являСтся сплавом ΠΆΠ΅Π»Π΅Π·Π° с ΡƒΠ³Π»Π΅Ρ€ΠΎΠ΄ΠΎΠΌ. Он ΠΎΡ‡Π΅Π½ΡŒ быстро ΠΏΠ΅Ρ€Π΅Ρ…ΠΎΠ΄ΠΈΡ‚ ΠΈΠ· ΠΆΠΈΠ΄ΠΊΠΎΠ³ΠΎ состояния Π² Ρ‚Π²Π΅Ρ€Π΄ΠΎΠ΅, Π² Ρ€Π΅Π·ΡƒΠ»ΡŒΡ‚Π°Ρ‚Π΅ Π΅Π³ΠΎ практичСски Π½Π΅Π²ΠΎΠ·ΠΌΠΎΠΆΠ½ΠΎ ΠΎΠ±Ρ€Π°Π±Π°Ρ‚Ρ‹Π²Π°Ρ‚ΡŒ ΠΊΠΎΠ²ΠΊΠΎΠΉ, ΠΏΡ€ΠΎΠΊΠ°Ρ‚ΠΊΠΎΠΉ ΠΈ Ρ‚.Π΄., Π΄Π° ΠΈ сварка Π΅Π³ΠΎ Ρ‚ΠΎΠΆΠ΅ ΠΏΡ€ΠΎΠ±Π»Π΅ΠΌΠ°Ρ‚ΠΈΡ‡Π½Π°. ΠŸΠΎΡΡ‚ΠΎΠΌΡƒ ΠΎΡ‚ΠΎΠΏΠΈΡ‚Π΅Π»ΡŒΠ½Ρ‹Π΅ ΠΏΡ€ΠΈΠ±ΠΎΡ€Ρ‹ ΠΈΠ· Ρ‡ΡƒΠ³ΡƒΠ½Π°, ΠΊΠ°ΠΊ ΠΏΡ€Π°Π²ΠΈΠ»ΠΎ, ΠΎΡ‚Π»ΠΈΠ²Π°ΡŽΡ‚. А ΠΏΠΎΡΠΊΠΎΠ»ΡŒΠΊΡƒ ΠΏΡ€ΠΈ ΠΎΡ‚Π»ΠΈΠ²ΠΊΠ°Ρ… Π²ΠΎΠ·ΠΌΠΎΠΆΠ½Ρ‹ ΠΏΠΎΡ€ΠΎΠΊΠΈ (ΠΏΡƒΠ·Ρ‹Ρ€ΡŒΠΊΠΈ Π²ΠΎΠ·Π΄ΡƒΡ…Π°, ΠΊΠ°Π²Π΅Ρ€Π½Ρ‹, частицы шлака ΠΈ Ρ‚.ΠΏ.), Ρ‚ΠΎΠ»Ρ‰ΠΈΠ½Π° стСнок Ρ‡ΡƒΠ³ΡƒΠ½Π½Ρ‹Ρ… ΠΎΡ‚ΠΎΠΏΠΈΡ‚Π΅Π»ΡŒΠ½Ρ‹Ρ… ΠΏΡ€ΠΈΠ±ΠΎΡ€ΠΎΠ² составляСт 4 — 6 ΠΌΠΌ, Π² Ρ€Π΅Π·ΡƒΠ»ΡŒΡ‚Π°Ρ‚Π΅ Ρ‡Π΅Π³ΠΎ ΠΎΠ½ΠΈ ΠΈΠΌΠ΅ΡŽΡ‚ большой вСс, ΠΈ, ΠΊΠ°ΠΊ слСдствиС, Π²Ρ‹ΡΠΎΠΊΡƒΡŽ Ρ‚Π΅ΠΏΠ»ΠΎΠ΅ΠΌΠΊΠΎΡΡ‚ΡŒ самого Ρ€Π°Π΄ΠΈΠ°Ρ‚ΠΎΡ€Π°. Π‘ ΡƒΡ‡Π΅Ρ‚ΠΎΠΌ объСма Π²ΠΎΠ΄Ρ‹ Π² сСкции, Ρ‡ΡƒΠ³ΡƒΠ½Π½Ρ‹ΠΉ Ρ€Π°Π΄ΠΈΠ°Ρ‚ΠΎΡ€ являСтся довольно ΠΈΠ½Π΅Ρ€Ρ†ΠΈΠΎΠ½Π½Ρ‹ΠΌ ΠΏΡ€ΠΈΠ±ΠΎΡ€ΠΎΠΌ.
Β Β Β Β Β Β Β Π§ΡƒΠ³ΡƒΠ½ — достаточно Ρ…Ρ€ΡƒΠΏΠΊΠΈΠΉ ΠΌΠ°Ρ‚Π΅Ρ€ΠΈΠ°Π». Если Π²ΠΎΠ΄Π° Π·Π°ΠΌΠ΅Ρ€Π·Π½Π΅Ρ‚ (частично) Π² ΡΡ‚Π°Π»ΡŒΠ½ΠΎΠΌ Ρ€Π°Π΄ΠΈΠ°Ρ‚ΠΎΡ€Π΅, Ρ‚ΠΎ Π΅ΡΡ‚ΡŒ ΡˆΠ°Π½ΡΡ‹, Ρ‡Ρ‚ΠΎ Ρ€Π°Π΄ΠΈΠ°Ρ‚ΠΎΡ€ Π±ΡƒΠ΄Π΅Ρ‚ Π΄Π΅Ρ„ΠΎΡ€ΠΌΠΈΡ€ΠΎΠ²Π°Π½, Π½ΠΎ Π³Π΅Ρ€ΠΌΠ΅Ρ‚ΠΈΡ‡Π½ΠΎΡΡ‚ΡŒ сохранит. Π’ Ρ‡ΡƒΠ³ΡƒΠ½Π½ΠΎΠΌ — практичСски ΠΎΠ΄Π½ΠΎΠ·Π½Π°Ρ‡Π½ΠΎ Π»ΠΎΠΏΠ½Π΅Ρ‚… ΠšΡ€ΠΎΠΌΠ΅ Ρ‚ΠΎΠ³ΠΎ, Ρ‡ΡƒΠ³ΡƒΠ½ боится Π±ΠΎΠ»ΡŒΡˆΠΈΡ… ΠΏΠ΅Ρ€Π΅ΠΏΠ°Π΄ΠΎΠ² Ρ‚Π΅ΠΌΠΏΠ΅Ρ€Π°Ρ‚ΡƒΡ€ (Π²Ρ‹ΡˆΠ΅ 40 градусов). Π’ΠΎ Π΅ΡΡ‚ΡŒ Ссли Π² Ρ€Π°Π΄ΠΈΠ°Ρ‚ΠΎΡ€Π΅ Π²ΠΎΠ΄Π° 20 градусов, Π° Π² ΠΏΠΎΠ΄Π°Ρ‡Ρƒ Π²Π΄Ρ€ΡƒΠ³ зашло 80, Ρ€Π°Π΄ΠΈΠ°Ρ‚ΠΎΡ€ ΠΌΠΎΠΆΠ΅Ρ‚ этого Π½Π΅ пСрСнСсти…
Β Β Β Β Β Β Β Π§ΡƒΠ³ΡƒΠ½ Ρ€ΠΆΠ°Π²Π΅Π΅Ρ‚ Π½Π΅ΠΌΠ½ΠΎΠ³ΠΎ мСньшС, Ρ‡Π΅ΠΌ ΡΡ‚Π°Π»ΡŒ. Однако со Π²Ρ€Π΅ΠΌΠ΅Π½Π΅ΠΌ (особСнно ΠΏΠΎΠ΄ воздСйствиСм Π°Π½Ρ‚ΠΈΡ„Ρ€ΠΈΠ·ΠΎΠ²) ΠΈΠ· стСнок вымываСтся ΡƒΠ³Π»Π΅Ρ€ΠΎΠ΄ ΠΈ Π² Π²ΠΈΠ΄Π΅ Ρ‡Π΅Ρ€Π½ΠΎΠ³ΠΎ ΠΏΠΎΡ€ΠΎΡˆΠΊΠ° Π½Π°Ρ‡ΠΈΠ½Π°Π΅Ρ‚ ΠΎΡΠ΅Π΄Π°Ρ‚ΡŒ Π² систСмС отоплСния. ΠŸΠΎΡΡ‚ΠΎΠΌΡƒ Π² систСмах с Ρ‡ΡƒΠ³ΡƒΠ½Π½Ρ‹ΠΌΠΈ Ρ€Π°Π΄ΠΈΠ°Ρ‚ΠΎΡ€Π°ΠΌΠΈ, ИМΠ₯О, ΠΎΠΏΡ€Π°Π²Π΄Π°Π½ΠΎ ΠΏΡ€ΠΈΠΌΠ΅Π½Π΅Π½ΠΈΠ΅ Ρ„ΠΈΠ»ΡŒΡ‚Ρ€ΠΎΠ² с Ρ€Π°Π·ΠΌΠ΅Ρ€ΠΎΠΌ ячСйки 100 ΠΌΠΈΠΊΡ€ΠΎΠ½.
       Говоря ΠΎ Ρ€Π°Π·ΠΌΠ΅Ρ€Π°Ρ… Ρ‡ΡƒΠ³ΡƒΠ½Π½Ρ‹Ρ… Ρ€Π°Π΄ΠΈΠ°Ρ‚ΠΎΡ€ΠΎΠ², слСдуСт ΠΎΡ‚ΠΌΠ΅Ρ‚ΠΈΡ‚ΡŒ, Ρ‡Ρ‚ΠΎ Ρ‡Π°Ρ‰Π΅ всСго указываСтся «ΠΌΠ΅ΠΆΠΎΡΠ΅Π²ΠΎΠ΅ расстояниС» — расстояниС ΠΌΠ΅ΠΆΠ΄Ρƒ Ρ†Π΅Π½Ρ‚Ρ€Π°ΠΌΠΈ Π±ΠΎΠΊΠΎΠ²Ρ‹Ρ… 1″ Ρ€Π΅Π·ΡŒΠ±. Π‘Ρ‚Π°Π½Π΄Π°Ρ€Ρ‚Π½Ρ‹Ρ… мСТосСвых расстояний Π½Π΅ΠΌΠ½ΠΎΠ³ΠΎ: ИМΠ₯О, 350, 500 ΠΈ 1000 ΠΌΠΌ. Однако ΠΎΡ‚Π΄Π΅Π»ΡŒΠ½Ρ‹Π΅ ΠΏΡ€ΠΎΠΈΠ·Π²ΠΎΠ΄ΠΈΡ‚Π΅Π»ΠΈ ΠΌΠΎΠ³ΡƒΡ‚ ΠΏΡ€ΠΈΠ΄ΡƒΠΌΠ°Ρ‚ΡŒ свои…
Β Β Β Β Β Β Β Π§ΡƒΠ³ΡƒΠ½Π½Ρ‹Π΅ Ρ€Π°Π΄ΠΈΠ°Ρ‚ΠΎΡ€Ρ‹ относятся ΠΊ сСкционным ΠΏΡ€ΠΈΠ±ΠΎΡ€Π°ΠΌ, сСкции ΡΠΎΠ±ΠΈΡ€Π°ΡŽΡ‚ΡΡ Π΄Ρ€ΡƒΠ³ с Π΄Ρ€ΡƒΠ³ΠΎΠΌ ΠΏΡ€ΠΈ ΠΏΠΎΠΌΠΎΡ‰ΠΈ ΡΠΏΠ΅Ρ†ΠΈΠ°Π»ΡŒΠ½Ρ‹Ρ… Π½ΠΈΠΏΠΏΠ΅Π»Π΅ΠΉ с Π΄Π²ΠΎΠΉΠ½ΠΎΠΉ Ρ€Π΅Π·ΡŒΠ±ΠΎΠΉ. ΠŸΠΎΠ΄ΠΊΠ»ΡŽΡ‡Π΅Π½ΠΈΠ΅ Ρ‚Ρ€ΡƒΠ±ΠΎΠΏΡ€ΠΎΠ²ΠΎΠ΄ΠΎΠ² осущСствляСтся Ρ‡Π΅Ρ€Π΅Π· Ρ‚ΠΎΡ€Ρ†Π΅Π²Ρ‹Π΅ Π·Π°Π³Π»ΡƒΡˆΠΊΠΈ, ΠΊΠΎΡ‚ΠΎΡ€Ρ‹Π΅ ΠΌΠΎΠ³ΡƒΡ‚ ΠΈΠΌΠ΅Ρ‚ΡŒ Π²Π½ΡƒΡ‚Ρ€Π΅Π½Π½ΡŽΡŽ Ρ€Π΅Π·ΡŒΠ±Ρƒ 1/2″ ΠΈ 3/4″.
Β Β Β Β Β Β Β Π§ΡƒΠ³ΡƒΠ½Π½Ρ‹Π΅ Ρ€Π°Π΄ΠΈΠ°Ρ‚ΠΎΡ€Ρ‹, ΠΊΠ°ΠΊ ΠΏΡ€Π°Π²ΠΈΠ»ΠΎ, ΠΏΠΎΠ΄Π²Π΅ΡˆΠΈΠ²Π°ΡŽΡ‚ΡΡ Π½Π° ΡΠΏΠ΅Ρ†ΠΈΠ°Π»ΡŒΠ½Ρ‹Π΅ ΠΊΡ€ΡŽΠΊΠΈ, Π·Π°Π±ΠΈΠ²Π°Π΅ΠΌΡ‹Π΅ Π² стСну. Однако Π΅ΡΡ‚ΡŒ ΠΈ сСкции с ΠΎΡ‚Π»ΠΈΡ‚Ρ‹ΠΌΠΈ Π½ΠΎΠΆΠΊΠ°ΠΌΠΈ, ΠΈ Π΄Π°ΠΆΠ΅ ΠΎΡ‚Π΄Π΅Π»ΡŒΠ½ΠΎ подставки-Π½ΠΎΠΆΠΊΠΈ.
Β Β Β Β Β Β Β Π’ Π·Π°ΠΊΠ»ΡŽΡ‡Π΅Π½ΠΈΠ΅ ΠΎΡ‚ΠΌΠ΅Ρ‡Ρƒ, Ρ‡Ρ‚ΠΎ внСшний Π²ΠΈΠ΄ соврСмСнных Ρ‡ΡƒΠ³ΡƒΠ½Π½Ρ‹Ρ… Ρ€Π°Π΄ΠΈΠ°Ρ‚ΠΎΡ€ΠΎΠ² сильно измСнился. ЛицСвая ΠΏΠΎΠ²Π΅Ρ€Ρ…Π½ΠΎΡΡ‚ΡŒ Π½Π΅Ρ€Π΅Π΄ΠΊΠΎ дСлаСтся плоской, ΠΏΠ΅Ρ€Π΅Π΄ покраской ΡˆΠ»ΠΈΡ„ΡƒΠ΅Ρ‚ΡΡ. ВстрСчал (ΠΏΡ€Π°Π²Π΄Π°, Π½Π΅ Π² ΠΆΠΈΠ·Π½ΠΈ) сСкции с встроСнными тСрмостатичСскими ΠΊΠ»Π°ΠΏΠ°Π½Π°ΠΌΠΈ. Ну Π° ΡƒΠΆ Ρ€Π°Π·Π»ΠΈΡ‡Π½Ρ‹Ρ… Π΄ΠΈΠ·Π°ΠΉΠ½-Ρ€Π°Π΄ΠΈΠ°Ρ‚ΠΎΡ€ΠΎΠ² ΠΈΠ· Ρ‡ΡƒΠ³ΡƒΠ½Π°, с самыми Π·Π°ΠΌΡƒΠ΄Ρ€Π΅Π½Ρ‹ΠΌΠΈ Π·Π°Π²ΠΈΡ‚ΡƒΡˆΠΊΠ°ΠΌΠΈ Π½Π° повСрхности, Π²ΠΏΠ»ΠΎΡ‚ΡŒ Π΄ΠΎ ΠΏΠΎΠ·ΠΎΠ»ΠΎΡ‚Ρ‹, ΠΎΡ‡Π΅Π½ΡŒ ΠΌΠ½ΠΎΠ³ΠΎ…

ΠΠ»ΡŽΠΌΠΈΠ½ΠΈΠ΅Π²Ρ‹Π΅ ΠΈ бимСталличСскиС ΠΏΡ€ΠΈΠ±ΠΎΡ€Ρ‹

Β Β Β Β Β Β Β Π£ алюминия Π΅ΡΡ‚ΡŒ нСсколько ΠΎΡ‡Π΅Π½ΡŒ Π²Π°ΠΆΠ½Ρ‹Ρ… свойств: ΠΎΠ½ Π»Π΅Π³Ρ‡Π΅ стали (Ρ‡ΡƒΠ³ΡƒΠ½Π°) Π² Ρ€Π°Π·Ρ‹, Ρƒ Π½Π΅Π³ΠΎ ΠΎΡ‡Π΅Π½ΡŒ высокая Ρ‚Π΅ΠΏΠ»ΠΎΠΏΡ€ΠΎΠ²ΠΎΠ΄Π½ΠΎΡΡ‚ΡŒ. ΠŸΠΎΡΡ‚ΠΎΠΌΡƒ СстСствСнно, Ρ‡Ρ‚ΠΎ Π΅Π³ΠΎ ΠΏΡ€ΠΈΠΌΠ΅Π½ΡΡŽΡ‚ Π² ΠΎΡ‚ΠΎΠΏΠΈΡ‚Π΅Π»ΡŒΠ½Ρ‹Ρ… ΠΏΡ€ΠΈΠ±ΠΎΡ€Π°Ρ…. Но Π΅ΡΡ‚ΡŒ ΠΈ нСдостатки. Π’ΠΎ-ΠΏΠ΅Ρ€Π²Ρ‹Ρ…, алюминий — ΠΌΠ°Ρ‚Π΅Ρ€ΠΈΠ°Π» пластичный, ΠΈ ΠΏΡ€ΠΈ большом Π΄Π°Π²Π»Π΅Π½ΠΈΠΈ Π»Π΅Π³ΠΊΠΎ дСформируСтся. Π’ΠΎ Π²Ρ‚ΠΎΡ€Ρ‹Ρ…, алюминий — ΠΌΠ°Ρ‚Π΅Ρ€ΠΈΠ°Π» химичСски Π°ΠΊΡ‚ΠΈΠ²Π½Ρ‹ΠΉ. Богласно школьного курса Ρ…ΠΈΠΌΠΈΠΈ, алюминий Ρ€Π΅Π°Π³ΠΈΡ€ΡƒΠ΅Ρ‚ с Π²ΠΎΠ΄ΠΎΠΉ с Π²Ρ‹Π΄Π΅Π»Π΅Π½ΠΈΠ΅ΠΌ Π²ΠΎΠ΄ΠΎΡ€ΠΎΠ΄Π°, ΠΈ СдинствСнная ΠΏΡ€ΠΈΡ‡ΠΈΠ½Π°, ΠΏΠΎ ΠΊΠΎΡ‚ΠΎΡ€ΠΎΠΉ сущСствуСт алюминиСвая посуда ΠΈ Ρ‚.Π΄. — ΠΏΡ€ΠΈ ΠΊΠΎΠ½Ρ‚Π°ΠΊΡ‚Π΅ с кислородом алюминий Ρ‚ΡƒΡ‚ ΠΆΠ΅ покрываСтся окисной ΠΏΠ»Π΅Π½ΠΊΠΎΠΉ, которая ΠΈ Π½Π΅ пропускаСт Π²ΠΎΠ΄Ρƒ ΠΊ алюминию. Однако Π² тСплоноситСлС систСмы отоплСния, Π²ΠΎ-ΠΏΠ΅Ρ€Π²Ρ‹Ρ…, кислорода ΠΌΠ°Π»ΠΎ (с Π½ΠΈΠΌ ΡΠΏΠ΅Ρ†ΠΈΠ°Π»ΡŒΠ½ΠΎ Π±ΠΎΡ€ΡŽΡ‚ΡΡ, ΠΊΡ€ΠΎΠΌΠ΅ Ρ‚ΠΎΠ³ΠΎ, ΠΎΠ½ Π°ΠΊΡ‚ΠΈΠ²Π½ΠΎ выдСляСтся ΠΈΠ· Π²ΠΎΠ΄Ρ‹ ΠΏΡ€ΠΈ Π½Π°Π³Ρ€Π΅Π²Π΅ Π΄ΠΎ 80 — 85 градусов), Π²ΠΎ Π²Ρ‚ΠΎΡ€Ρ‹Ρ…, с Π²ΠΎΠ΄ΠΎΠΉ (Π° особСнно с Π°Π½Ρ‚ΠΈΡ„Ρ€ΠΈΠ·Π°ΠΌΠΈ) Π² систСму отоплСния ΠΏΠΎΠΏΠ°Π΄Π°Π΅Ρ‚ мноТСство Ρ€Π°Π·Π½Ρ‹Ρ… вСщСств. Для Ρ€Π°Π·Ρ€ΡƒΡˆΠ΅Π½ΠΈΡ систСмы ΠΈΡ…, СстСствСнно, Π½Π΅ Ρ…Π²Π°Ρ‚Π°Π΅Ρ‚, Π° Π²ΠΎΡ‚ для Ρ€Π°Π·Ρ€ΡƒΡˆΠ΅Π½ΠΈΡ Π½Π° нСбольшом участкС окисной ΠΏΠ»Π΅Π½ΠΊΠΈ ΠΌΠΎΠΆΠ΅Ρ‚ ΠΈ Ρ…Π²Π°Ρ‚ΠΈΡ‚ΡŒ… ΠšΡ€ΠΎΠΌΠ΅ Ρ‚ΠΎΠ³ΠΎ, ΠΏΡ€ΠΈ использовании Π² систСмС отоплСния ΠΌΠ°Ρ‚Π΅Ρ€ΠΈΠ°Π»ΠΎΠ² с Ρ€Π°Π·Π½Ρ‹ΠΌ элСктрохимичСским ΠΏΠΎΡ‚Π΅Π½Ρ†ΠΈΠ°Π»ΠΎΠΌ (алюминий, мСдь) Π½Π°Π±Π»ΡŽΠ΄Π°Π΅Ρ‚ΡΡ Ρ‚Π°ΠΊΠΆΠ΅ ΠΈ элСктрохимичСская коррозия. Π’ΠΎ всяком случаС, ΠΏΡ€ΠΎΠΈΠ·Π²ΠΎΠ΄ΠΈΡ‚Π΅Π»ΠΈ Π°Π»ΡŽΠΌΠΈΠ½ΠΈΠ΅Π²Ρ‹Ρ… Ρ€Π°Π΄ΠΈΠ°Ρ‚ΠΎΡ€ΠΎΠ² ΡƒΠΆΠ΅ ΠΏΡ€Π΅Π΄ΡƒΠΏΡ€Π΅ΠΆΠ΄Π°ΡŽΡ‚, Ρ‡Ρ‚ΠΎ ΠΊΡƒΡ€ΠΈΡ‚ΡŒ Π²ΠΎΠ·Π»Π΅ воздухосбросного вСнтиля нСльзя…
Β Β Β Β Β Β Β Π‘ нСдостатками, СстСствСнно, Π±ΠΎΡ€ΡŽΡ‚ΡΡ. Для компСнсации высокой пластичности увСличиваСтся Ρ‚ΠΎΠ»Ρ‰ΠΈΠ½Π° стСнок. ΠŸΡ€ΠΈΠΌΠ΅Π½ΡΡŽΡ‚ΡΡ сплавы алюминия с Π΄Ρ€ΡƒΠ³ΠΈΠΌΠΈ ΠΌΠ΅Ρ‚Π°Π»Π»Π°ΠΌΠΈ. Π•Ρ‰Π΅ ΠΎΠ΄ΠΈΠ½ ΠΈΠ· ΠΏΡƒΡ‚Π΅ΠΉ увСличСния ТСсткости — частичная ΠΈΠ»ΠΈ полная Π·Π°ΠΌΠ΅Π½Π° Π²Π½ΡƒΡ‚Ρ€Π΅Π½Π½Π΅ΠΉ повСрхности ΠΏΡ€ΠΈΠ±ΠΎΡ€Π° ΡΡ‚Π°Π»ΡŒΠ½Ρ‹ΠΌΠΈ вставками. ΠŸΡ€ΠΈ этом нСсколько сниТаСтся Ρ‚Π΅ΠΏΠ»ΠΎΠΎΡ‚Π΄Π°Ρ‡Π°, Π½ΠΎ Π²Ρ‹Π΄Π΅Ρ€ΠΆΠΈΠ²Π°Π΅ΠΌΠΎΠ΅ Π΄Π°Π²Π»Π΅Π½ΠΈΠ΅ растСт Π² Ρ€Π°Π·Ρ‹. Π­Ρ‚ΠΎ ΠΈ Π΅ΡΡ‚ΡŒ Ρ‚Π°ΠΊ Π½Π°Π·Ρ‹Π²Π°Π΅ΠΌΡ‹Π΅ бимСталличСскиС Ρ€Π°Π΄ΠΈΠ°Ρ‚ΠΎΡ€Ρ‹. Π’ случаС Π·Π°ΠΌΠ΅Π½Ρ‹ всСй Π²Π½ΡƒΡ‚Ρ€Π΅Π½Π½Π΅ΠΉ повСрхности Ρ‚Π΅ΠΏΠ»ΠΎΠ½ΠΎΡΠΈΡ‚Π΅Π»ΡŒ Π½Π΅ ΠΊΠΎΠ½Ρ‚Π°ΠΊΡ‚ΠΈΡ€ΡƒΠ΅Ρ‚ с алюминиСм, ΠΈ Π±ΠΎΠ»ΡŒΡˆΠΈΠ½ΡΡ‚Π²ΠΎ ΠΏΡ€ΠΎΠ±Π»Π΅ΠΌ алюминия Ρ€Π΅ΡˆΠ°Π΅Ρ‚ΡΡ. Если Π·Π°ΠΌΠ΅Π½Π° частичная (Π½Π°ΠΏΡ€ΠΈΠΌΠ΅Ρ€, ΡΡ‚Π°Π»ΡŒΠ½Ρ‹Π΅ Ρ‚Ρ€ΡƒΠ±ΠΊΠΈ Ρ‚ΠΎΠ»ΡŒΠΊΠΎ Π² Π²Π΅Ρ€Ρ‚ΠΈΠΊΠ°Π»ΡŒΠ½Ρ‹Ρ… частях ΠΏΡ€ΠΈΠ±ΠΎΡ€Π°) — ΠΏΡ€ΠΎΠ±Π»Π΅ΠΌΡ‹ Ρ€Π΅ΡˆΠ°ΡŽΡ‚ΡΡ частично.
Β Β Β Β Β Β Β ΠšΡ€Π΅ΠΏΠ»Π΅Π½ΠΈΠ΅ Ρ‚Π°ΠΊΠΈΡ… Ρ€Π°Π΄ΠΈΠ°Ρ‚ΠΎΡ€ΠΎΠ² ΠΎΠ±Ρ‹Ρ‡Π½ΠΎ Ρ‚ΠΎΠΆΠ΅ Π½Π° ΠΊΡ€ΡŽΠΊΠ°Ρ…, Π½ΠΎ Π²Π²ΠΈΠ΄Ρƒ Π·Π½Π°Ρ‡ΠΈΡ‚Π΅Π»ΡŒΠ½ΠΎ мСньшСго вСса (ΠΈ Ρ€Π°Π΄ΠΈΠ°Ρ‚ΠΎΡ€ΠΎΠ², ΠΈ Π²ΠΎΠ΄Ρ‹ Π² Π½ΠΈΡ…) ΠΊΡ€ΡŽΠΊΠΈ выглядят Π±ΠΎΠ»Π΅Π΅ Π°ΠΊΠΊΡƒΡ€Π°Ρ‚Π½ΠΎ ΠΈ практичСски Π½Π΅Π·Π°ΠΌΠ΅Ρ‚Π½Ρ‹.

ΠšΠΎΠ½Π²Π΅ΠΊΡ‚ΠΎΡ€Ρ‹

Β Β Β Β Β Β Β ΠŸΡ€Π°ΠΊΡ‚ΠΈΡ‡Π΅ΡΠΊΠΈ ΠΊΠΎΠ½Π²Π΅ΠΊΡ‚ΠΎΡ€ прСдставляСт ΠΈΠ· сСбя Ρ‚Ρ€ΡƒΠ±Ρƒ (нСсколько Ρ‚Ρ€ΡƒΠ±) с тСплоноситСлСм, Π½Π° ΠΊΠΎΡ‚ΠΎΡ€ΡƒΡŽ насаТСны ΠΌΠ½ΠΎΠ³ΠΎ-ΠΌΠ½ΠΎΠ³ΠΎ пластин, ΠΎΡ‚Π΄Π°ΡŽΡ‰ΠΈΡ… Ρ‚Π΅ΠΏΠ»ΠΎ Π²ΠΎΠ·Π΄ΡƒΡ…Ρƒ. ВсС это Π·Π°ΠΊΡ€Ρ‹Ρ‚ΠΎ ΠΊΠΎΠΆΡƒΡ…ΠΎΠΌ.
Β Β Β Β Β Β Β ΠšΠΎΠ½Π²Π΅ΠΊΡ‚ΠΎΡ€Ρ‹ совСтских Π²Ρ€Π΅ΠΌΠ΅Π½ ΠΈΠ·Π³ΠΎΡ‚ΠΎΠ²Π»ΡΠ»ΠΈΡΡŒ, ΠΊΠ°ΠΊ ΠΏΡ€Π°Π²ΠΈΠ»ΠΎ, ΠΈΠ· стали, соврСмСнныС ΠΈΠ·Π³ΠΎΡ‚Π°Π²Π»ΠΈΠ²Π°ΡŽΡ‚ΡΡ ΠΈΠ· стали ΠΈΠ»ΠΈ ΠΌΠ΅Π΄ΠΈ (Ρ‚Ρ€ΡƒΠ±Ρ‹), ΠΈ алюминия (пластины).
Β Β Β Β Β Β Β Π’Π΅ΠΏΠ»ΠΎΠΎΡ‚Π΄Π°Ρ‡Π° пластин ΠΊΠΎΠ½Π²Π΅ΠΊΡ‚ΠΎΡ€Π° Ρ‚Π΅ΠΌ Π²Ρ‹ΡˆΠ΅, Ρ‡Π΅ΠΌ большС Π²Π΅Ρ€Ρ‚ΠΈΠΊΠ°Π»ΡŒΠ½Ρ‹ΠΉ Ρ€Π°Π·ΠΌΠ΅Ρ€. Π’ связи с этим Ρ‚Π΅ΠΏΠ»ΠΎΠΎΡ‚Π΄Π°Ρ‡Π° встроСнного Π² ΠΏΠΎΠ» ΠΊΠΎΠ½Π²Π΅ΠΊΡ‚ΠΎΡ€Π°, ΠΊΠ°ΠΊ ΠΏΡ€Π°Π²ΠΈΠ»ΠΎ, Π½Π΅Π²Π΅Π»ΠΈΠΊΠ°, Π·Π° ΠΈΡΠΊΠ»ΡŽΡ‡Π΅Π½ΠΈΠ΅ΠΌ встроСнных Π² ΠΏΠΎΠ» ΠΊΠΎΠ½Π²Π΅ΠΊΡ‚ΠΎΡ€ΠΎΠ² с вСнтилятором Π²Π½ΡƒΡ‚Ρ€ΠΈ (Π»ΠΈΠ±ΠΎ подводящСй Π²ΠΎΠ·Π΄ΡƒΡ… Ρ‚Ρ€ΡƒΠ±ΠΎΠΉ ΠΎΡ‚ систСмы вСнтиляции). НаличиС вСнтилятора, Π² свою ΠΎΡ‡Π΅Ρ€Π΅Π΄ΡŒ, Ρ€Π΅Π·ΠΊΠΎ сниТаСт Π΄ΠΈΠ°ΠΏΠ°Π·ΠΎΠ½ примСнСния встроСнного ΠΊΠΎΠ½Π²Π΅ΠΊΡ‚ΠΎΡ€Π° (ΡˆΡƒΠΌ вСнтилятора Π² спальнС, элСктропроводка Π² Π²Π°Π½Π½ΠΎΠΉ ΠΊΠΎΠΌΠ½Π°Ρ‚Π΅ ΠΈ Ρ‚.Π΄.).
       Установка производится Π½Π° ΡΠΏΠ΅Ρ†ΠΈΠ°Π»ΡŒΠ½Ρ‹Ρ… Π½ΠΎΠΆΠΊΠ°Ρ…. МногиС встроСнныС ΠΌΠΎΠ΄Π΅Π»ΠΈ ΠΈΠΌΠ΅ΡŽΡ‚ ΡΠ²ΠΎΡ€Π°Ρ‡ΠΈΠ²Π°Π΅ΠΌΡƒΡŽ Π²Π΅Ρ€Ρ…Π½ΡŽΡŽ Ρ€Π΅ΡˆΠ΅Ρ‚ΠΊΡƒ, Ρƒ ΠΎΡ‚Π΄Π΅Π»ΡŒΠ½Ρ‹Ρ… ΠΏΡ€ΠΎΠΈΠ·Π²ΠΎΠ΄ΠΈΡ‚Π΅Π»Π΅ΠΉ эта Ρ€Π΅ΡˆΠ΅Ρ‚ΠΊΠ° ΠΌΠΎΠΆΠ΅Ρ‚ Π±Ρ‹Ρ‚ΡŒ Π²Ρ‹ΠΏΠΎΠ»Π½Π΅Π½Π° ΠΈΠ· Π΄Π΅Ρ€Π΅Π²Π° Ρ€Π°Π·Π½Ρ‹Ρ… ΠΏΠΎΡ€ΠΎΠ΄, ΠΌΠ΅Ρ‚Π°Π»Π»Π° (алюминий, ΡΡ‚Π°Π»ΡŒ) Ρ€Π°Π·Π»ΠΈΡ‡Π½Ρ‹Ρ… Ρ†Π²Π΅Ρ‚ΠΎΠ² — ΠΌΠΎΠΆΠ½ΠΎ ΠΏΠΎΠ΄ΠΎΠ±Ρ€Π°Ρ‚ΡŒ практичСски ΠΊ Π»ΡŽΠ±ΠΎΠΌΡƒ ΠΏΠΎΠ»Ρƒ (ΠΏΠ°Ρ€ΠΊΠ΅Ρ‚, ΠΌΡ€Π°ΠΌΠΎΡ€ ΠΈ Ρ‚.Π΄.).
Β Β Β Β Β Β Β Π—Π°ΠΏΠΎΡ€Π½ΠΎ-рСгулировочная Π°Ρ€ΠΌΠ°Ρ‚ΡƒΡ€Π° (тСрмостатичСскиС Π²Π΅Π½Ρ‚ΠΈΠ»ΠΈ, Ρ‚Π΅Ρ€ΠΌΠΎΠ³ΠΎΠ»ΠΎΠ²ΠΊΠΈ ΠΈ Ρ‚. Π΄.) Ρ‚Π°ΠΊΠΆΠ΅ Π΅ΡΡ‚ΡŒ.
Β Β Β Β Β Β Β Π“Π°Π±Π°Ρ€ΠΈΡ‚Ρ‹ Ρ‚Π°ΠΊΠΆΠ΅ самыС Ρ€Π°Π·Π»ΠΈΡ‡Π½Ρ‹Π΅. Π’ Π½Π΅ΠΊΠΎΡ‚ΠΎΡ€Ρ‹Ρ… Π²Π°Ρ€ΠΈΠ°Π½Ρ‚Π°Ρ… ΠΊΠΎΠ½Π²Π΅ΠΊΡ‚ΠΎΡ€Ρ‹ ΠΌΠΎΠΆΠ½ΠΎ ΡΡ‚Ρ‹ΠΊΠΎΠ²Π°Ρ‚ΡŒ Π΄Ρ€ΡƒΠ³ с Π΄Ρ€ΡƒΠ³ΠΎΠΌ, получая Π½Π΅ΠΏΡ€Π΅Ρ€Ρ‹Π²Π½Ρ‹ΠΉ ΠΏΡ€ΠΈΠ±ΠΎΡ€ практичСски любой Π΄Π»ΠΈΠ½Ρ‹. Π‘ΡƒΡ‰Π΅ΡΡ‚Π²ΡƒΡŽΡ‚ ΠΊΠΎΠ½Π²Π΅ΠΊΡ‚ΠΎΡ€Ρ‹, Π²Ρ‹ΠΏΠΎΠ»Π½Π΅Π½Π½Ρ‹Π΅ Π±ΡƒΠΊΠ²ΠΎΠΉ «Π“», Π° Ρ‚Π°ΠΊΠΆΠ΅ ΠΈΠ·ΠΎΠ³Π½ΡƒΡ‚Ρ‹Π΅.
       НаруТная ΠΎΡ‚Π΄Π΅Π»ΠΊΠ° настСнных ΠΊΠΎΠ½Π²Π΅ΠΊΡ‚ΠΎΡ€ΠΎΠ² самая разнообразная — ΠΎΡ‚ Π»Π°ΠΊΠΈΡ€ΠΎΠ²Π°Π½Π½ΠΎΠΉ дрСвСсины Π΄ΠΎ ΠΏΡ€ΠΈΡ€ΠΎΠ΄Π½ΠΎΠ³ΠΎ камня…

ΠžΠΏΡ€Π΅Π΄Π΅Π»Π΅Π½ΠΈΠ΅ Ρ‚Π΅ΠΏΠ»ΠΎΠΎΡ‚Π΄Π°Ρ‡ΠΈ ΠΎΡ‚ΠΎΠΏΠΈΡ‚Π΅Π»ΡŒΠ½ΠΎΠ³ΠΎ ΠΏΡ€ΠΈΠ±ΠΎΡ€Π°

Β Β Β Β Β Β Β ΠŸΠ΅Ρ€Π²Ρ‹ΠΉ вопрос, ΠΊΠΎΡ‚ΠΎΡ€Ρ‹ΠΉ Π·Π°Π΄Π°Π΅Ρ‚ Ρ‡Π΅Π»ΠΎΠ²Π΅ΠΊ, подбирая ΠΎΡ‚ΠΎΠΏΠΈΡ‚Π΅Π»ΡŒΠ½Ρ‹ΠΉ ΠΏΡ€ΠΈΠ±ΠΎΡ€ – Π° сколько ΠΎΠ½ Π΄Π°Π΅Ρ‚ Ρ‚Π΅ΠΏΠ»Π°? Π•ΡΡ‚ΡŒ масса Π²Π°Ρ€ΠΈΠ°Π½Ρ‚ΠΎΠ² опрСдСлСния, Π½Π΅ΠΊΠΎΡ‚ΠΎΡ€Ρ‹Π΅ ΠΈΠ· Π½ΠΈΡ… Π±ΡƒΠ΄ΡƒΡ‚ пСрСчислСны Π½ΠΈΠΆΠ΅ Π² порядкС возрастания слоТности.
       Как ΠΏΡ€Π°Π²ΠΈΠ»ΠΎ, Π² Ρ‚ΠΎΡ€Π³ΡƒΡŽΡ‰Π΅ΠΉ ΠΎΡ€Π³Π°Π½ΠΈΠ·Π°Ρ†ΠΈΠΈ для Π΄Π°Π½Π½ΠΎΠΉ ΠΌΠ°Ρ€ΠΊΠΈ ΠΏΡ€ΠΈΠ±ΠΎΡ€Π° имССтся Ρ‚Π°Π±Π»ΠΈΡ†Π° Ρ‚Π΅ΠΏΠ»ΠΎΠΎΡ‚Π΄Π°Ρ‡ΠΈ всСх Ρ‚ΠΈΠΏΠΎΡ€Π°Π·ΠΌΠ΅Ρ€ΠΎΠ² ΠΏΡ€ΠΈΠ±ΠΎΡ€Π° (Ρ‡Π°Ρ‰Π΅ всСго совмСщСнная с прайсом) — ΠΏΡ€ΠΈΠΌΠ΅Ρ€ ΠΏΡ€ΠΈΠ²Π΅Π΄Π΅Π½ Π½ΠΈΠΆΠ΅. Для сСкционных ΠΎΡ‚ΠΎΠΏΠΈΡ‚Π΅Π»ΡŒΠ½Ρ‹Ρ… ΠΏΡ€ΠΈΠ±ΠΎΡ€ΠΎΠ² — Ρ‚Π΅ΠΏΠ»ΠΎΠΎΡ‚Π΄Π°Ρ‡Π° ΠΎΠ΄Π½ΠΎΠΉ сСкции.
Π’ΠΈΠΏ101120212233
Высота300400500600900300400500600900500600900300400500600900300400500600900300400500600900
Π”Π»ΠΈΠ½Π°Β 
400Β Β 296355460Β Β 449533691482569739Β Β 571697937Β 6347648891229Β Β 109312751727
500226302377453589355462569675845610720935Β Β 716874117562679395611121538Β Β 136615812142
60027136245254370642655468281010147328641122Β Β 85910491409751952114713341846Β Β 163918972570
700Β Β 528634824497647796945118385410081309Β Β 1002122416448761110133815572153Β Β 191222132998
800Β Β 60372494256873991010801352976115214966829021146139818791001126915291779246114471825218625293426
900Β Β 679815105963983210231215152110981296168376910141289157321141126142717202002276816282053245928453856
1000Β Β 754905117771092411371350169012201440187085211271432174823491251158619112224307618092281273231614283
1100Β Β 8299961295781101612511485185913421584205793712401575192325841376174521022446338419772509300534774711
1200Β Β 905108614128521109136416202028146417282244102213521718209828191501190322932669369121712737327837935140
1400Β Β 1056126716489941294159218902366170820162618119315782005244732891751222026753114430625333193382544255996
1600Β Β 120614481883113614781819216027041952230429921363180322912797Β 2002253830583558492228943650437150586853
1800Β Β 13571629Β 1278166320472430Β 21962592Β 1534202925783146Β 2252285534404003Β 3256410649185690Β 
2000Β Β 15081810Β 1420184822742700Β 24402880Β 1704225428643496Β 2502317238224448Β 3618456254646322Β 
2300Β Β 17342082Β Β Β 26153105Β 28063312Β Β Β Β Β Β 2877364843955115Β 41615246Β Β Β 
2600Β Β 19602353Β Β Β 29563510Β 31723744Β Β Β Β Β Β 3253412449695782Β 47035931Β Β Β 
3000Β Β 22622715Β Β Β 34114050Β 36604320Β Β Β Β Β Β 3753475857336672Β 54276843Β Β Β 

Β Β Β Β Β Β Β Π“Π΄Π΅-Ρ‚ΠΎ Π½Π° этом ΠΆΠ΅ листикС Π΄ΠΎΠ»ΠΆΠ½Π° Π±Ρ‹Ρ‚ΡŒ Ρ„Ρ€Π°Π·Π° Ρ‚ΠΈΠΏΠ°:»Π’Π΅ΠΏΠ»ΠΎΠΎΡ‚Π΄Π°Ρ‡Π° ΡƒΠΊΠ°Π·Π°Π½Π° ΠΏΡ€ΠΈ Ρ‚Π΅ΠΌΠΏΠ΅Ρ€Π°Ρ‚ΡƒΡ€Π΅ (ΠΏΠΎΠ΄Π°Ρ‡Π°/ΠΎΠ±Ρ€Π°Ρ‚ΠΊΠ°/Π²ΠΎΠ·Π΄ΡƒΡ… помСщСния) 90/70/20Β°Π‘». Или просто написано «Π”Π»Ρ Ξ”Π’=60».
Β Β Β Β Β Β Β Π§Ρ‚ΠΎ Ρ‚Π°ΠΊΠΎΠ΅ Ξ”Π’ (Π”Π΅Π»ΡŒΡ‚Π° Вэ) ΠΈ Π·Π°Ρ‡Π΅ΠΌ ΠΎΠ½ΠΎ Π½Π°ΠΌ Π½ΡƒΠΆΠ½ΠΎ?
Β Β Β Β Β Β Β Ξ”Π’ опрСдСляСтся Ρ‡Π΅Ρ€Π΅Π· Ρ‚Π΅ΠΌΠΏΠ΅Ρ€Π°Ρ‚ΡƒΡ€Ρƒ ΠΏΠΎΠ΄Π°Ρ‡ΠΈ Π’ΠΏ (Π½Π° Π²Ρ…ΠΎΠ΄Π΅ Π² ΠΏΡ€ΠΈΠ±ΠΎΡ€), Ρ‚Π΅ΠΌΠΏΠ΅Ρ€Π°Ρ‚ΡƒΡ€Ρƒ ΠΎΠ±Ρ€Π°Ρ‚ΠΊΠΈ Π’ΠΎ (Π½Π° Π²Ρ‹Ρ…ΠΎΠ΄Π΅ ΠΈΠ· ΠΏΡ€ΠΈΠ±ΠΎΡ€Π°) ΠΈ Ρ‚Π΅ΠΌΠΏΠ΅Ρ€Π°Ρ‚ΡƒΡ€Ρƒ Π²ΠΎΠ·Π΄ΡƒΡ…Π° Π² ΠΏΠΎΠΌΠ΅Ρ‰Π΅Π½ΠΈΠΈ Π’Π². Π’ характСристиках ΠΎΡ‚ΠΎΠΏΠΈΡ‚Π΅Π»ΡŒΠ½Ρ‹Ρ… ΠΏΡ€ΠΈΠ±ΠΎΡ€ΠΎΠ² ΠΎΠ±Ρ‹Ρ‡Π½ΠΎ ΡƒΠΊΠ°Π·Ρ‹Π²Π°ΡŽΡ‚ всС Ρ‚Ρ€ΠΈ числа, Π½Π°ΠΏΡ€ΠΈΠΌΠ΅Ρ€: 90/70/20. Π’ΠΎ Π΅ΡΡ‚ΡŒ Π² ΠΏΡ€ΠΈΠ±ΠΎΡ€ Π·Π°Ρ…ΠΎΠ΄ΠΈΡ‚ Π²ΠΎΠ΄Π° ΠΏΡ€ΠΈ Ρ‚Π΅ΠΌΠΏΠ΅Ρ€Π°Ρ‚ΡƒΡ€Π΅ 90 градусов, ΡƒΡ…ΠΎΠ΄ΠΈΡ‚ ΠΏΡ€ΠΈ Ρ‚Π΅ΠΌΠΏΠ΅Ρ€Π°Ρ‚ΡƒΡ€Π΅ 70 градусов, Π° Ρ‚Π΅ΠΌΠΏΠ΅Ρ€Π°Ρ‚ΡƒΡ€Π° Π²ΠΎΠ·Π΄ΡƒΡ…Π° Π² ΠΏΠΎΠΌΠ΅Ρ‰Π΅Π½ΠΈΠΈ 20 градусов.
Β Β Β Β Β Β Β Π’Π°ΠΊΠΈΠΌ ΠΎΠ±Ρ€Π°Π·ΠΎΠΌ, для Π΄Π°Π½Π½ΠΎΠ³ΠΎ ΠΏΡ€ΠΈΠΌΠ΅Ρ€Π° Ξ”Π’ (Π”Π΅Π»ΡŒΡ‚Π° Вэ) = (90 + 70) / 2 – 20 = 60 градусов.*На самом Π΄Π΅Π»Π΅ Π±ΠΎΠ»Π΅Π΅ Ρ‚ΠΎΡ‡Π½ΠΎΠΉ являСтся Ξ”Π’ логарифмичСская, которая опрСдСляСтся ΠΏΠΎ Ρ„ΠΎΡ€ΠΌΡƒΠ»Π΅ Ξ”Π’ln = (TΠΏ – TΠΎ)/ln((TΠΏ – TΠ²)/(TΠΎ – TΠ²)), Π½ΠΎ расчСт ΠΏΡ€ΠΈ этом услоТняСтся, Π° Π²Ρ‹ΠΈΠ³Ρ€Ρ‹Ρˆ Π² точности Ρ‡Π°Ρ‰Π΅ всСго вСсьма Π½Π΅Π²Π΅Π»ΠΈΠΊ. Π’Π°ΠΊ, для ΠΏΠ°Ρ€Π°ΠΌΠ΅Ρ‚Ρ€ΠΎΠ² 90/70/20 Ξ”Π’ln = 59.44, Ρ‚.Π΅. Ρ€Π°Π·Π½ΠΈΡ†Π° всСго 0,56 градуса…
Β Β Β Β Β Β Β Π’Π΅ΠΏΠ΅Ρ€ΡŒ ΠΎΠΏΡ€Π΅Π΄Π΅Π»ΠΈΠΌ характСристики ΠΈΠΌΠ΅ΡŽΡ‰Π΅ΠΉΡΡ (ΠΈΠ»ΠΈ ΠΏΡ€ΠΎΠ΅ΠΊΡ‚ΠΈΡ€ΡƒΠ΅ΠΌΠΎΠΉ) систСмы.
Β Β Β Β Β Β Β Π’Π΅ΠΌΠΏΠ΅Ρ€Π°Ρ‚ΡƒΡ€Π° ΠΏΠΎΠ΄Π°Ρ‡ΠΈ Π² ΠΏΡ€ΠΈΠ±ΠΎΡ€ (для Π΄Π²ΡƒΡ…Ρ‚Ρ€ΡƒΠ±Π½ΠΎΠΉ схСмы отоплСния, ΠΊΠΎΠ³Π΄Π° ΠΊ ΠΊΠ°ΠΆΠ΄ΠΎΠΌΡƒ ΠΏΡ€ΠΈΠ±ΠΎΡ€Ρƒ подходят Π΄Π²Π΅ Ρ‚Ρ€ΡƒΠ±Ρ‹ – ΠΏΠΎΠ΄Π°Ρ‡Π° ΠΈ ΠΎΠ±Ρ€Π°Ρ‚ΠΊΠ°) опрСдСляСтся максимальной Ρ‚Π΅ΠΌΠΏΠ΅Ρ€Π°Ρ‚ΡƒΡ€ΠΎΠΉ тСплоноситСля Π½Π° Π²Ρ…ΠΎΠ΄Π΅ Π² Ρ‚Ρ€ΡƒΠ±ΠΎΠΏΡ€ΠΎΠ²ΠΎΠ΄Ρ‹ Π’Π°ΡˆΠ΅Π³ΠΎ Π΄ΠΎΠΌΠ°. Для многоэтаТСк, ΠΏΠΎΠ΄ΠΊΠ»ΡŽΡ‡Π΅Π½Π½Ρ‹Ρ… ΠΊ Ρ†Π΅Π½Ρ‚Ρ€Π°Π»ΠΈΠ·ΠΎΠ²Π°Π½Π½ΠΎΠΌΡƒ ΠΎΡ‚ΠΎΠΏΠ»Π΅Π½ΠΈΡŽ, максимальная Ρ‚Π΅ΠΌΠΏΠ΅Ρ€Π°Ρ‚ΡƒΡ€Π° Π½Π° Π²Ρ…ΠΎΠ΄Π΅ Π² систСму ΠΌΠΎΠΆΠ΅Ρ‚ Π΄ΠΎΡΡ‚ΠΈΠ³Π°Ρ‚ΡŒ 105 градусов ЦСльсия. Для ΠΈΠ½Π΄ΠΈΠ²ΠΈΠ΄ΡƒΠ°Π»ΡŒΠ½ΠΎΠ³ΠΎ отоплСния – согласно паспорта Π½Π° ΠΊΠΎΡ‚Π΅Π», для ΠΊΡ€Ρ‹ΡˆΠ½Ρ‹Ρ… ΠΈ ΠΏΡ€ΠΎΡ‡ΠΈΡ… ΠΊΠΎΡ‚Π΅Π»Π΅Π½ – Ρ‚Π°ΠΊΠΆΠ΅ ΠΏΠΎ Π½ΠΎΡ€ΠΌΠ°Ρ‚ΠΈΠ²Π½Ρ‹ΠΌ характСристикам. Π’ любом случаС Ρ‚Π΅ΠΌΠΏΠ΅Ρ€Π°Ρ‚ΡƒΡ€Ρƒ ΠΏΠΎΠ΄Π°Ρ‡ΠΈ слСдуСт ΠΏΡ€ΠΈΠ½ΠΈΠΌΠ°Ρ‚ΡŒ Π½Π° 5 градусов Π½ΠΈΠΆΠ΅ максимальной Ρ‚Π΅ΠΌΠΏΠ΅Ρ€Π°Ρ‚ΡƒΡ€Ρ‹, Π²Ρ‹Π΄Π°Π²Π°Π΅ΠΌΠΎΠΉ ΠΈΠ· систСмы Π² Π’Π°ΡˆΠΈ помСщСния – Π΄Π°Ρ‚Ρ‡ΠΈΠΊΠΈ Ρ‚Π΅ΠΌΠΏΠ΅Ρ€Π°Ρ‚ΡƒΡ€Ρ‹ ΠΈΠΌΠ΅ΡŽΡ‚ ΠΎΠΏΡ€Π΅Π΄Π΅Π»Π΅Π½Π½ΡƒΡŽ ΠΏΠΎΠ³Ρ€Π΅ΡˆΠ½ΠΎΡΡ‚ΡŒ, Ρ‡Π°ΡΡ‚ΡŒ Ρ‚Π΅ΠΏΠ»Π° ΡƒΠΉΠ΄Π΅Ρ‚ Ρ‡Π΅Ρ€Π΅Π· Ρ‚Ρ€ΡƒΠ±Ρ‹ Π΅Ρ‰Π΅ Π΄ΠΎ попадания Π² Ρ€Π°Π΄ΠΈΠ°Ρ‚ΠΎΡ€ ΠΈ Ρ‚.ΠΏ.
Β Β Β Β Β Β Β Π’Π΅ΠΌΠΏΠ΅Ρ€Π°Ρ‚ΡƒΡ€Π° ΠΎΠ±Ρ€Π°Ρ‚ΠΊΠΈ принимаСтся Π½ΠΈΠΆΠ΅ Ρ‚Π΅ΠΌΠΏΠ΅Ρ€Π°Ρ‚ΡƒΡ€Ρ‹ ΠΏΠΎΠ΄Π°Ρ‡ΠΈ Π½Π°: для частных Π΄ΠΎΠΌΠΎΠ² с большой Π΄Π»ΠΈΠ½ΠΎΠΉ Ρ‚Ρ€ΡƒΠ±, большим количСством ΠΏΡ€ΠΈΠ±ΠΎΡ€ΠΎΠ², Π²Π΅Π½Ρ‚ΠΈΠ»Π΅ΠΉ ΠΈ Ρ‚.ΠΏ. – 20 градусов; для частных Π΄ΠΎΠΌΠΎΠ² с нСбольшой Π΄Π»ΠΈΠ½ΠΎΠΉ Ρ‚Ρ€ΡƒΠ±, нСбольшим количСством ΠΏΡ€ΠΈΠ±ΠΎΡ€ΠΎΠ², Π²Π΅Π½Ρ‚ΠΈΠ»Π΅ΠΉ ΠΈ Ρ‚.ΠΏ. – 15 градусов; для ΠΊΠ²Π°Ρ€Ρ‚ΠΈΡ€ Π² многоэтаТках с большой Π΄Π»ΠΈΠ½ΠΎΠΉ Ρ‚Ρ€ΡƒΠ±, большим количСством ΠΏΡ€ΠΈΠ±ΠΎΡ€ΠΎΠ², Π²Π΅Π½Ρ‚ΠΈΠ»Π΅ΠΉ ΠΈ Ρ‚.ΠΏ. – 10 градусов; для ΠΊΠ²Π°Ρ€Ρ‚ΠΈΡ€ Π² многоэтаТках с нСбольшой Π΄Π»ΠΈΠ½ΠΎΠΉ Ρ‚Ρ€ΡƒΠ±, нСбольшим количСством ΠΏΡ€ΠΈΠ±ΠΎΡ€ΠΎΠ², Π²Π΅Π½Ρ‚ΠΈΠ»Π΅ΠΉ ΠΈ Ρ‚. ΠΏ. – 5 градусов.
Β Β Β Β Β Β Β Π’Π΅ΠΌΠΏΠ΅Ρ€Π°Ρ‚ΡƒΡ€Π° Π²ΠΎΠ·Π΄ΡƒΡ…Π° Π² ΠΏΠΎΠΌΠ΅Ρ‰Π΅Π½ΠΈΠΈ зависит ΠΎΡ‚ назначСния помСщСния. Для ΠΆΠΈΠ»Ρ‹Ρ… ΠΏΠΎΠΌΠ΅Ρ‰Π΅Π½ΠΈΠΉ — 20 — 24 градуса (большая Ρ†ΠΈΡ„Ρ€Π° — Π² ΡƒΠ³Π»ΠΎΠ²Ρ‹Ρ… помСщСниях), для Π²Π°Π½Π½Ρ‹Ρ… ΠΊΠΎΠΌΠ½Π°Ρ‚ — 25, для ΠΊΠΎΡ€ΠΈΠ΄ΠΎΡ€ΠΎΠ² ΠΈ Ρ‚.ΠΏ. 16 — 18 ΠΈ Ρ‚.Π΄.
Β Β Β Β Β Β Β Π’Π°ΠΊΠΈΠΌ ΠΎΠ±Ρ€Π°Π·ΠΎΠΌ, Π½Π°ΠΌ Π½Π΅ΠΎΠ±Ρ…ΠΎΠ΄ΠΈΠΌΠΎ ΠΎΠΏΡ€Π΅Π΄Π΅Π»ΠΈΡ‚ΡŒ Ρ‚Π΅ΠΏΠ»ΠΎΠΎΡ‚Π΄Π°Ρ‡Ρƒ Ρ€Π°Π΄ΠΈΠ°Ρ‚ΠΎΡ€Π° ΠΏΡ€ΠΈ Π½Π°ΡˆΠΈΡ… ΠΏΠ°Ρ€Π°ΠΌΠ΅Ρ‚Ρ€Π°Ρ… (Π½Π°ΠΏΡ€ΠΈΠΌΠ΅Ρ€, навСсной ΠΊΠΎΡ‚Π΅Π» с максимальной Ρ‚Π΅ΠΌΠΏΠ΅Ρ€Π°Ρ‚ΡƒΡ€ΠΎΠΉ ΠΏΠΎΠ΄Π°Ρ‡ΠΈ 80 градусов Π² ΡƒΠ³Π»ΠΎΠ²ΠΎΠΉ ΠΊΠΎΠΌΠ½Π°Ρ‚Π΅ Π΄Π²ΡƒΡ…ΠΊΠΎΠΌΠ½Π°Ρ‚Π½ΠΎΠΉ ΠΊΠ²Π°Ρ€Ρ‚ΠΈΡ€Ρ‹ многоэтаТного Π΄ΠΎΠΌΠ°): 75/65/24, Ξ”Π’ (Π”Π΅Π»ΡŒΡ‚Π° Вэ) = (75 + 65) / 2 – 24 = 46 градусов. Π’Π΅ΠΏΠ»ΠΎΠΏΠΎΡ‚Π΅Ρ€ΠΈ ΠΊΠΎΠΌΠ½Π°Ρ‚Ρ‹ ΡΠΎΡΡ‚Π°Π²Π»ΡΡŽΡ‚ 2000 Π’Ρ‚.
       А Ρ‚Π΅ΠΏΠ΅Ρ€ΡŒ считаСм Ρ€Π°Π·Π½Ρ‹ΠΌΠΈ способами (ΠΈΠ΄Π΅ΠΌ ΠΎΡ‚ простых ΠΊ слоТным). Бпособ β„– 1.
Β Β Β Β Β Β Β Π€ΠΈΡ€ΠΌΠ°-ΠΏΡ€ΠΎΠΈΠ·Π²ΠΎΠ΄ΠΈΡ‚Π΅Π»ΡŒ ΡƒΠΆΠ΅ всС посчитала. Π’ этом случаС имССтся Ρ‚Π°Π±Π»ΠΈΡ†Π° коэффициСнтов, аналогичная прСдставлСнной Π½ΠΈΠΆΠ΅:
Π’ΠΏΠ’ΠΎΠ’Π΅ΠΌΠΏΠ΅Ρ€Π°Ρ‚ΡƒΡ€Π° Π²ΠΎΠ·Π΄ΡƒΡ…Π° Π² ΠΏΠΎΠΌΠ΅Ρ‰Π΅Π½ΠΈΠΈ Π’Π²
10121518202224
95800,570,590,620,650,680,700,73
700,620,650,680,730,760,790,83
600,690,720,770,830,870,910,96
500,790,830,890,961,021,081,15
90800,590,610,640,680,710,740,77
750,620,640,680,720,750,780,82
700,650,670,720,760,800,830,87
650,680,710,760,810,850,890,93
600,720,760,810,870,910,961,01
550,770,810,870,930,981,041,10
500,830,870,931,011,071,141,21
85750,640,670,710,750,790,820,86
700,680,700,750,800,840,880,92
650,720,750,800,850,890,940,99
600,760,790,850,910,961,011,07
550,810,850,910,981,041,101,16
80700,710,740,790,840,880,930,97
650,750,780,840,900,940,991,05
600,800,830,890,961,011,071,13
550,850,890,961,041,101,161,24
500,910,961,041,131,201,281,37
75650,790,820,880,951,001,051,12
600,840,880,941,021,081,141,21
550,890,941,011,101,171,241,32
500,961,011,101,201,281,371,47

Β Β Β Β Β Β Β Π’ ΠΏΠ΅Ρ€Π²ΡƒΡŽ ΠΎΡ‡Π΅Ρ€Π΅Π΄ΡŒ опрСдСляСм, для ΠΊΠ°ΠΊΠΈΡ… ΠΏΠ°Ρ€Π°ΠΌΠ΅Ρ‚Ρ€ΠΎΠ² эта Ρ‚Π°Π±Π»ΠΈΡ†Π°. Для этого Π½Π°Ρ…ΠΎΠ΄ΠΈΠΌ Π² Π½Π΅ΠΉ коэффициСнт, Ρ€Π°Π²Π½Ρ‹ΠΉ 1. Π’Π°ΠΊΠΈΠΌ ΠΎΠ±Ρ€Π°Π·ΠΎΠΌ, данная Ρ‚Π°Π±Π»ΠΈΡ†Π° составлСна для исходных ΠΏΠ°Ρ€Π°ΠΌΠ΅Ρ‚Ρ€ΠΎΠ² 75/65/20. Если Π±Ρ‹ Ρ‚Π°Π±Π»ΠΈΡ†Π° с Ρ‚Π΅ΠΏΠ»ΠΎΠΎΡ‚Π΄Π°Ρ‡Π°ΠΌΠΈ Π±Ρ‹Π»Π° составлСна для этих ΠΏΠ°Ρ€Π°ΠΌΠ΅Ρ‚Ρ€ΠΎΠ², Ρ‚ΠΎ расчСт достаточно прост: ΠΌΡ‹ Π±Ρ‹ нашли коэффициСнт для Π½Π°ΡˆΠΈΡ… ΠΏΠ°Ρ€Π°ΠΌΠ΅Ρ‚Ρ€ΠΎΠ² (для ΠΏΡ€ΠΈΠ²Π΅Π΄Π΅Π½Π½ΠΎΠ³ΠΎ Π²Ρ‹ΡˆΠ΅ ΠΏΡ€ΠΈΠΌΠ΅Ρ€Π° ΠΎΠ½ Ρ€Π°Π²Π΅Π½ 1,12) ΠΈ искали Π±Ρ‹ Π² Ρ‚Π°Π±Π»ΠΈΡ†Π΅ Ρ‚Π΅ΠΏΠ»ΠΎΠΎΡ‚Π΄Π°Ρ‡ Ρ€Π°Π΄ΠΈΠ°Ρ‚ΠΎΡ€ с Ρ‚Π΅ΠΏΠ»ΠΎΠΎΡ‚Π΄Π°Ρ‡Π΅ΠΉ, Ρ€Π°Π²Π½ΠΎΠΉ ΠΈΠ»ΠΈ Π½Π΅ΠΌΠ½ΠΎΠ³ΠΎ большСй 2000 β€’ 1.12 = 2240 Π’Ρ‚ (Π½Π°ΠΏΡ€ΠΈΠΌΠ΅Ρ€, 10 Ρ‚ΠΈΠΏ высотой 500 ΠΈ Π΄Π»ΠΈΠ½ΠΎΠΉ 3000 ΠΌΠΌ с Ρ‚Π°Π±Π»ΠΈΡ‡Π½ΠΎΠΉ Ρ‚Π΅ΠΏΠ»ΠΎΠΎΡ‚Π΄Π°Ρ‡Π΅ΠΉ 2262 Π’Ρ‚). РСально ΠΆΠ΅ ΠΏΡ€ΠΈ Π·Π°Π΄Π°Π½Π½Ρ‹Ρ… условиях ΠΎΠ½ ΠΌΠΎΠΆΠ΅Ρ‚ ΠΎΡ‚Π΄Π°Ρ‚ΡŒ 2262 / 1,12 = 2019 Π’Ρ‚.
       Однако Π½Π°ΠΌ Π½Π΅ ΠΏΠΎΠ²Π΅Π·Π»ΠΎ. Π’Π°Π±Π»ΠΈΡ†Π° Ρ‚Π΅ΠΏΠ»ΠΎΠΎΡ‚Π΄Π°Ρ‡ составлСна для ΠΏΠ°Ρ€Π°ΠΌΠ΅Ρ‚Ρ€ΠΎΠ² 90/70/20. Π’ΠΎΠ³Π΄Π° Π½Π°ΠΌ придСтся Π½Π°ΠΉΡ‚ΠΈ Ρ‚Π°ΠΊΠΆΠ΅ ΠΈ коэффициСнт для 90/70/20, ΠΊΠΎΡ‚ΠΎΡ€Ρ‹ΠΉ составляСт 0,8. Π”Π΅Π»ΠΈΠΌ 1,12 Π½Π° число, ΠΎΠ±Ρ€Π°Ρ‚Π½ΠΎΠ΅ 0,8 (1,25) ΠΈ ΠΏΠΎΠ»ΡƒΡ‡Π°Π΅ΠΌ 1,4. А Ρ‚Π΅ΠΏΠ΅Ρ€ΡŒ ΠΈΡ‰Π΅ΠΌ Π² Ρ‚Π°Π±Π»ΠΈΡ†Π΅ Ρ‚Π΅ΠΏΠ»ΠΎΠΎΡ‚Π΄Π°Ρ‡ для ΠΏΠ°Ρ€Π°ΠΌΠ΅Ρ‚Ρ€ΠΎΠ² 90/70/20 Ρ€Π°Π΄ΠΈΠ°Ρ‚ΠΎΡ€ с ΠΌΠΎΡ‰Π½ΠΎΡΡ‚ΡŒΡŽ, Π² 1,4 Ρ€Π°Π·Π° большСй Ρ‚Π΅ΠΏΠ»ΠΎΠΏΠΎΡ‚Π΅Ρ€ΡŒ Π² ΠΏΠΎΠΌΠ΅Ρ‰Π΅Π½ΠΈΠΈ: 2000 β€’ 1,4 = 2800 Π’Ρ‚. ΠŸΠΎΠ΄Ρ…ΠΎΠ΄ΠΈΡ‚, Π½Π°ΠΏΡ€ΠΈΠΌΠ΅Ρ€, 22 Ρ‚ΠΈΠΏ 400 Π½Π° 1800 ΠΌΠΌ с Ρ‚Π΅ΠΏΠ»ΠΎΠΎΡ‚Π΄Π°Ρ‡Π΅ΠΉ 2855 Π’Ρ‚. РСальная Ρ‚Π΅ΠΏΠ»ΠΎΠΎΡ‚Π΄Π°Ρ‡Π° ΠΏΡ€ΠΈ Π½Π°ΡˆΠΈΡ… условиях составит 2855 / 1,4 = 2039 Π’Ρ‚.
Β Β Β Β Β Β Β ΠžΠ±Ρ‰Π΅Π΅ Π·Π°ΠΌΠ΅Ρ‡Π°Π½ΠΈΠ΅: ΠΏΡ€ΠΈ Π²Ρ‹Π±ΠΎΡ€Π΅ Ρ€Π°Π΄ΠΈΠ°Ρ‚ΠΎΡ€Π° допустимым являСтся Π½Π΅Ρ…Π²Π°Ρ‚ΠΊΠ° мощности Π΄ΠΎ 5 % (Π½ΠΎ Π½Π΅ Π±ΠΎΠ»Π΅Π΅ 60 Π’Ρ‚). ΠŸΡ€ΠΈ условии использования Ρ€Π΅Π³ΡƒΠ»ΠΈΡ€ΡƒΡŽΡ‰ΠΈΡ… устройств (ΠΊΠ»Π°ΠΏΠ°Π½Π° с Ρ‚Π΅Ρ€ΠΌΠΎΠ³ΠΎΠ»ΠΎΠ²ΠΊΠ°ΠΌΠΈ ΠΈ Ρ‚.Π΄.) Π²Π΅Π»ΠΈΡ‡ΠΈΠ½Π° Π²ΠΎΠ·ΠΌΠΎΠΆΠ½ΠΎΠ³ΠΎ ΠΏΡ€Π΅Π²Ρ‹ΡˆΠ΅Π½ΠΈΡ мощности Ρ‚Π΅ΠΏΠ»ΠΎΠΎΡ‚Π΄Π°Ρ‡ΠΈ зависит Ρ‚ΠΎΠ»ΡŒΠΊΠΎ ΠΎΡ‚ Вас — Π±ΠΎΠ»Π΅Π΅ ΠΌΠΎΡ‰Π½Ρ‹Π΅ Ρ€Π°Π΄ΠΈΠ°Ρ‚ΠΎΡ€Ρ‹ Π΄ΠΎΡ€ΠΎΠΆΠ΅. Бпособ β„– 2.
Β Β Β Β Β Β Β ΠŸΡ€ΠΎΠΈΠ·Π²ΠΎΠ΄ΠΈΡ‚Π΅Π»ΡŒ ΠΏΡ€ΠΈΠ²ΠΎΠ΄ΠΈΡ‚ Π² описании ΠΏΡ€ΠΈΠ±ΠΎΡ€Π° ΡƒΡ€Π°Π²Π½Π΅Π½ΠΈΠ΅ Ρ‚Π΅ΠΏΠ»ΠΎΠ²ΠΎΠΉ характСристики, ΠΊΠΎΡ‚ΠΎΡ€ΠΎΠ΅ выглядит, Π½Π°ΠΏΡ€ΠΈΠΌΠ΅Ρ€, Ρ‚Π°ΠΊ: Π€ = К β€’ Ξ”Π’n β€’ qb β€’ L β€’ Ξ΅,        К ΡƒΡ€Π°Π²Π½Π΅Π½ΠΈΡŽ ΠΏΡ€ΠΈΠ»Π°Π³Π°ΡŽΡ‚ΡΡ значСния коэффициСнтов ΠΏΡ€ΠΈ Ρ€Π°Π·Π½Ρ‹Ρ… Ρ‚ΠΈΠΏΠ°Ρ…, Π΄Π»ΠΈΠ½Π°Ρ… ΠΈ высотах ΠΏΡ€ΠΈΠ±ΠΎΡ€Π°, Π° Ρ‚Π°ΠΊΠΆΠ΅ Π΅Ρ‰Π΅ ΠΊΡƒΡ‡Π° Ρ„ΠΎΡ€ΠΌΡƒΠ» для опрСдСлСния коэффициСнтов. Π”Π΅Π»ΠΎ это Π΄ΠΎΠ»Π³ΠΎΠ΅ ΠΈ ΠΌΡƒΡ‚ΠΎΡ€Π½ΠΎΠ΅, поэтому ΠΏΠΎΠΉΠ΄Π΅ΠΌ ΠΏΠΎ сокращСнному Π²Π°Ρ€ΠΈΠ°Π½Ρ‚Ρƒ.
Β Β Β Β Β Β Β Π‘Ρ‡ΠΈΡ‚Π°Π΅ΠΌ, Ρ‡Ρ‚ΠΎ ΠΏΡ€ΠΎΡ‡ΠΈΠ΅ Ρ€Π°Π²Π½Ρ‹Π΅ условия ΠΈΠ΄Π΅Π½Ρ‚ΠΈΡ‡Π½Ρ‹, ΠΈ влияниС Π½Π° Ρ‚Π΅ΠΏΠ»ΠΎΠΎΡ‚Π΄Π°Ρ‡Ρƒ ΠΎΠΊΠ°Π·Ρ‹Π²Π°Π΅Ρ‚ Ρ‚ΠΎΠ»ΡŒΠΊΠΎ Ξ”Π’. Π’ этом случаС для ΠΏΠΎΠ΄Π±ΠΎΡ€Π° Ρ€Π°Π΄ΠΈΠ°Ρ‚ΠΎΡ€Π° ΠΌΠΎΠΆΠ½ΠΎ ΠΈΡΠΏΠΎΠ»ΡŒΠ·ΠΎΠ²Π°Ρ‚ΡŒ Ρ„ΠΎΡ€ΠΌΡƒΠ»Ρƒ WΡ‚ = WΡ„ β€’ (Ξ”Π’Ρ‚ / Ξ”Π’Ρ„)n, Π³Π΄Π΅ WΡ„ — фактичСская Ρ‚Π΅ΠΏΠ»ΠΎΠΎΡ‚Π΄Π°Ρ‡Π° Ρ€Π°Π΄ΠΈΠ°Ρ‚ΠΎΡ€Π°, WΡ‚ — табличная Ρ‚Π΅ΠΏΠ»ΠΎΠΎΡ‚Π΄Π°Ρ‡Π° Ρ€Π°Π΄ΠΈΠ°Ρ‚ΠΎΡ€Π°, Ξ”Π’Ρ„ — фактичСскоС Π·Π½Π°Ρ‡Π΅Π½ΠΈΠ΅, Ξ”Π’Ρ‚ — Ρ‚Π°Π±Π»ΠΈΡ‡Π½ΠΎΠ΅ Π·Π½Π°Ρ‡Π΅Π½ΠΈΠ΅, n — ΠΏΠΎΠΊΠ°Π·Π°Ρ‚Π΅Π»ΡŒ стСпСни. Π•Π³ΠΎ ΠΏΡ€ΠΎΠΈΠ·Π²ΠΎΠ΄ΠΈΡ‚Π΅Π»ΠΈ Ρ‚Π°ΠΊΠΆΠ΅ Π΄Π°ΡŽΡ‚ Π² характСристиках. Если Π΅Π³ΠΎ Π½Π΅Ρ‚ – ΠΏΡ€ΠΈΠΌΠ΅Ρ€Π½ΠΎ опрСдСляСм ΡΠ»Π΅Π΄ΡƒΡŽΡ‰ΠΈΠΌ ΠΎΠ±Ρ€Π°Π·ΠΎΠΌ: ΠΏΠ°Π½Π΅Π»ΡŒΠ½Ρ‹Π΅ Ρ€Π°Π΄ΠΈΠ°Ρ‚ΠΎΡ€Ρ‹ 1,2 — 1,3, Ρ‡ΡƒΠ³ΡƒΠ½Π½Ρ‹Π΅ Ρ€Π°Π΄ΠΈΠ°Ρ‚ΠΎΡ€Ρ‹ 1,3, ΠΊΠΎΠ½Π²Π΅ΠΊΡ‚ΠΎΡ€Ρ‹ 1,4, ΠΊΠΎΠ½Π²Π΅ΠΊΡ‚ΠΎΡ€Ρ‹ настСнныС с ΠΊΠΎΠΆΡƒΡ…ΠΎΠΌ 1,3, ΠΊΠΎΠ½Π²Π΅ΠΊΡ‚ΠΎΡ€Ρ‹ настСнныС Π±Π΅Π· ΠΊΠΎΠΆΡƒΡ…Π° 1,2, Ρ‚Ρ€ΡƒΠ±Ρ‹ ΠΎΡ‚ΠΎΠΏΠΈΡ‚Π΅Π»ΡŒΠ½Ρ‹Π΅ Ρ‡ΡƒΠ³ΡƒΠ½Π½Ρ‹Π΅ 1,25, Ρ€Π°Π΄ΠΈΠ°Ρ‚ΠΎΡ€Ρ‹ ΠΈΠ· Π³Π»Π°Π΄ΠΊΠΈΡ… Ρ‚Ρ€ΡƒΠ± 1,25, Π°Π»ΡŽΠΌΠΈΠ½ΠΈΠ΅Π²Ρ‹Π΅ Ρ€Π°Π΄ΠΈΠ°Ρ‚ΠΎΡ€Ρ‹ 1,25.
       Для нашСго ΠΏΡ€ΠΈΠΌΠ΅Ρ€Π° WΡ‚ = WΡ„ β€’ (60 / 46)1,3 = WΡ„ β€’ 1,4125 (Π²Ρ‹ΡˆΠ΅ Π±Ρ‹Π»ΠΎ Π±Π»ΠΈΠ·ΠΊΠΎΠ΅ число — 1,4). Π’.Π΅. Π² Ρ‚Π°Π±Π»ΠΈΡ†Π΅ Ρ‚ΠΏΠ΅Π»ΠΎΠΎΡ‚Π΄Π°Ρ‡ΠΈ с ΠΏΠ°Ρ€Π°ΠΌΠ΅Ρ‚Ρ€Π°ΠΌΠΈ 90/70/20 Π½ΡƒΠΆΠ½ΠΎ ΠΈΡΠΊΠ°Ρ‚ΡŒ Ρ€Π°Π΄ΠΈΠ°Ρ‚ΠΎΡ€ с ΠΌΠΎΡ‰Π½ΠΎΡΡ‚ΡŒΡŽ Π² 1,4125 Ρ€Π°Π· Π²Ρ‹ΡˆΠ΅ Ρ‚Ρ€Π΅Π±ΡƒΠ΅ΠΌΠΎΠΉ для ΠΎΠ±ΠΎΠ³Ρ€Π΅Π²Π° помСщСния. Для нашСго ΠΏΡ€ΠΈΠΌΠ΅Ρ€Π° WΡ‚ = 2000 β€’ 1,4125 = 2825 Π’Ρ‚.
       Для ΠΏΡ€ΠΎΠ²Π΅Ρ€ΠΊΠΈ ΠΎΠΏΡ€Π΅Π΄Π΅Π»ΠΈΠΌ коэффициСнт пСрСрасчСта для Ρ‚Π°Π±Π»ΠΈΡ†Ρ‹ с Ρ‚Π΅ΠΏΠ»ΠΎΠΎΡ‚Π΄Π°Ρ‡Π΅ΠΉ ΠΏΡ€ΠΈ ΠΏΠ°Ρ€Π°ΠΌΠ΅Ρ‚Ρ€Π°Ρ… 75/65/20. Ξ”Π’Ρ‚ ΠΏΡ€ΠΈ этом составит 50.
Β Β Β Β Β Β Β WΡ‚ = WΡ„ β€’ (50 / 46)1,3 = WΡ„ β€’ 1,1145, Ρ‡Ρ‚ΠΎ ΠΏΠΎΡ‡Ρ‚ΠΈ совпадаСт с Π½Π°ΠΉΠ΄Π΅Π½Π½Ρ‹ΠΌ Π²Ρ‹ΡˆΠ΅ Π² Ρ‚Π°Π±Π»ΠΈΡ†Π΅ коэффициСнтом 1,12. Бпособ β„– 3.
Β Β Β Β Β Β Β Π’Π°ΠΌ ΡƒΠ΄Π°Π»ΠΎΡΡŒ (ΠΈΠ»ΠΈ ΠΏΡ€ΠΎΠΈΠ·Π²ΠΎΠ΄ΠΈΡ‚Π΅Π»ΡŒ Π΄Π°Π») ΡƒΠ·Π½Π°Ρ‚ΡŒ Ρ€Π°ΡΡ‡Π΅Ρ‚Π½ΡƒΡŽ Ρ‚Π΅ΠΏΠ»ΠΎΠΎΡ‚Π΄Π°Ρ‡Ρƒ ΠΏΡ€ΠΈΠ±ΠΎΡ€Π° ΠΏΡ€ΠΈ Π΄Π²ΡƒΡ… Ρ€Π°Π·Π½Ρ‹Ρ… ΠΏΠ°Ρ€Π°ΠΌΠ΅Ρ‚Ρ€Π°Ρ… тСплоноситСля. НапримСр, Ρ‚Π΅ΠΏΠ»ΠΎΠΎΡ‚Π΄Π°Ρ‡Π° ΠΏΡ€ΠΈ Ξ”Π’1=50 составляСт Q1=2000 Π’Ρ‚, Π° ΠΏΡ€ΠΈ Ξ”Π’2=70 — Q2=2500 Π’Ρ‚. Π’ этом случаС ΠΌΠΎΠΆΠ½ΠΎ ΠΎΠΏΡ€Π΅Π΄Π΅Π»ΠΈΡ‚ΡŒ ΠΏΠΎΠΊΠ°Π·Π°Ρ‚Π΅Π»ΡŒ стСпСни ΠΏΠΎ ΡΠ»Π΅Π΄ΡƒΡŽΡ‰Π΅ΠΉ Ρ„ΠΎΡ€ΠΌΡƒΠ»Π΅: n=log(Ξ”Π’2/Ξ”Π’1)(Q2/Q1). Данная Ρ„ΠΎΡ€ΠΌΡƒΠ»Π° Π² ячСйкС Excel выглядит Ρ‚Π°ΠΊ: =LOG(Q2/Q1;Ξ”Π’2/Ξ”Π’1). РазумССтся, Π² Ρ„ΠΎΡ€ΠΌΡƒΠ»Π΅ Π΄ΠΎΠ»ΠΆΠ½Ρ‹ Π±Ρ‹Ρ‚ΡŒ Π²Π²Π΅Π΄Π΅Π½Ρ‹ Π»ΠΈΠ±ΠΎ числовыС значСния, Π»ΠΈΠ±ΠΎ ссылки Π½Π° ячСйки, Ρ‚Π°ΠΊΠΈΠ΅ значСния содСрТащиС.
Β Β Β Β Β Β Β Π’ΠΎΡ‚, собствСнно, ΠΏΠΎΠΊΠ° ΠΈ всС. Вопросы, замСчания ΠΈ прСдлоТСния ΠΌΠΎΠΆΠ½ΠΎ ΠΎΡΡ‚Π°Π²ΠΈΡ‚ΡŒ Π² Π²ΠΈΠ΄Π΅ коммСнтария.

ΠŸΠΎΡΡ‚ΠΎΡΠ½Π½Ρ‹ΠΉ адрСс страницы
http://lovial.narod.ru/statyi/teplo/radiator.html
Β 
Β 

Π Π°Π΄ΠΈΠ°Ρ‚ΠΎΡ€Ρ‹ отоплСния. β€” ПолСзная информация

Β 

Β Π Π°Π΄ΠΈΠ°Ρ‚ΠΎΡ€Ρ‹ отоплСния.

На российском Ρ€Ρ‹Π½ΠΊΠ΅ прСдставлСны Ρ€Π°Π·Π»ΠΈΡ‡Π½Ρ‹Π΅ Ρ€Π°Π΄ΠΈΠ°Ρ‚ΠΎΡ€Ρ‹ отоплСния, отСчСствСнных и иностранных ΠΏΡ€ΠΎΠΈΠ·Π²ΠΎΠ΄ΠΈΡ‚Π΅Π»Π΅ΠΉ. Π Π°Π΄ΠΈΠ°Ρ‚ΠΎΡ€Ρ‹ Π΄ΠΎΠ»ΠΆΠ½Ρ‹ ΡΠΎΠΎΡ‚Π²Π΅Ρ‚ΡΡ‚Π²ΠΎΠ²Π°Ρ‚ΡŒ Π“ΠžΠ‘Π’ 31311-2005. Π§Π°Ρ‰Π΅ всСго в систСмах отоплСния Π·Π΄Π°Π½ΠΈΠΉ и сооруТСний ΠΏΡ€ΠΈΠΌΠ΅Π½ΡΡŽΡ‚ΡΡ Π°Π»ΡŽΠΌΠΈΠ½ΠΈΠ΅Π²Ρ‹Π΅, бимСталличСскиС ΠΈΒ ΡΡ‚Π°Π»ΡŒΠ½Ρ‹Π΅ Ρ€Π°Π΄ΠΈΠ°Ρ‚ΠΎΡ€Ρ‹.

Β Π‘Ρ‚Π°Π»ΡŒΠ½Ρ‹Π΅ Ρ€Π°Π΄ΠΈΠ°Ρ‚ΠΎΡ€Ρ‹ отоплСния.

Β 

Π’Ρ‹ΠΏΡƒΡΠΊΠ°ΡŽΡ‚ΡΡ Ρ€Π°Π΄ΠΈΠ°Ρ‚ΠΎΡ€Ρ‹ Π΄Π²ΡƒΡ… Ρ‚ΠΈΠΏΠΎΠ²:

1)Β ΠΏΠ°Π½Π΅Π»ΡŒΠ½Ρ‹Π΅;

2)Β Ρ‚Ρ€ΡƒΠ±Ρ‡Π°Ρ‚Ρ‹Π΅.

Π’Ρ€ΡƒΠ±Ρ‡Π°Ρ‚Ρ‹Π΅ Ρ€Π°Π΄ΠΈΠ°Ρ‚ΠΎΡ€Ρ‹ достаточно Ρ‚Ρ€ΡƒΠ΄ΠΎΡ‘ΠΌΠΊΠΈ Π²Β ΠΈΠ·Π³ΠΎΡ‚ΠΎΠ²Π»Π΅Π½ΠΈΠΈ ΠΈΒ ΠΈΠΌΠ΅ΡŽΡ‚ Π±ΠΎΠ»Π΅Π΅ Π²Ρ‹ΡΠΎΠΊΡƒΡŽ Ρ†Π΅Π½Ρƒ, в пСрСсчётС Π½Π°Β ΠΌΠΎΡ‰Π½ΠΎΡΡ‚ΡŒ, Ρ‡Π΅ΠΌΒ ΡΡ‚Π°Π»ΡŒΠ½Ρ‹Π΅ ΠΏΠ°Π½Π΅Π»ΡŒΠ½Ρ‹Π΅ Ρ€Π°Π΄ΠΈΠ°Ρ‚ΠΎΡ€Ρ‹.

Π‘ΠΎΠ»ΡŒΡˆΠΈΠΌ плюсом ΡΡ‚Π°Π»ΡŒΠ½Ρ‹Ρ… ΠΏΠ°Π½Π΅Π»ΡŒΠ½Ρ‹Ρ… Ρ€Π°Π΄ΠΈΠ°Ρ‚ΠΎΡ€ΠΎΠ² являСтся их низкая ΡΡ‚ΠΎΠΈΠΌΠΎΡΡ‚ΡŒ. Π‘Ρ‚Π°Π»ΡŒΠ½ΠΎΠΉ ΠΏΠ°Π½Π΅Π»ΡŒΠ½Ρ‹ΠΉ Ρ€Π°Π΄ΠΈΠ°Ρ‚ΠΎΡ€ дСшСвлС бимСталличСского и алюминиСвого Π½Π°Β 30-80%, Π°Π½Π°Π»ΠΎΠ³ΠΈΡ‡Π½ΠΎΠ³ΠΎ ΠΏΠΎΒ Ρ‚Π΅ΠΏΠ»ΠΎΠ²ΠΎΠΉ мощности. Π”Π°Π½Π½Ρ‹ΠΉ Ρ€Π°Π΄ΠΈΠ°Ρ‚ΠΎΡ€ ΠΈΠ΄Π΅Π°Π»Π΅Π½ для примСнСния в зданиях ΡΒ ΠΈΠ½Π΄ΠΈΠ²ΠΈΠ΄ΡƒΠ°Π»ΡŒΠ½Ρ‹ΠΌ ΠΎΡ‚ΠΎΠΏΠ»Π΅Π½ΠΈΠ΅ΠΌ, ΡΒ ΠΊΡ€Ρ‹ΡˆΠ½Ρ‹ΠΌΠΈ ΠΊΠΎΡ‚Π΅Π»ΡŒΠ½Ρ‹ΠΌΠΈ, Π²Β ΠΎΡ‚ΠΎΠΏΠ»Π΅Π½ΠΈΠΈ Тилых или производствСнных комплСксов ΡΒ ΠΈΠ½Π΄ΠΈΠ²ΠΈΠ΄ΡƒΠ°Π»ΡŒΠ½ΠΎΠΉ ΠΊΠΎΡ‚Π΅Π»ΡŒΠ½ΠΎΠΉ. ΠŸΠΎΡΠΊΠΎΠ»ΡŒΠΊΡƒ ΡΡ‚Π°Π»ΡŒ ΠΌΠΎΠΆΠ΅Ρ‚ Π±Ρ‹Ρ‚ΡŒ ΠΏΠΎΠ΄Π²Π΅Ρ€ΠΆΠ΅Π½Π° ΠΊΠΎΡ€Ρ€ΠΎΠ·ΠΈΠΈ ΠΈΠ·-Π·Π° свободного кислорода, растворСнного Π²Β Π²ΠΎΠ΄Π΅, Π’Π°ΠΌΒ Π½Π΅ΠΎΠ±Ρ…ΠΎΠ΄ΠΈΠΌΠΎ ΠΎΠ±Π΅ΡΠΏΠ΅Ρ‡ΠΈΡ‚ΡŒ достаточный ΡƒΡ€ΠΎΠ²Π΅Π½ΡŒ Π²ΠΎΠ΄ΠΎΠΏΠΎΠ΄Π³ΠΎΡ‚ΠΎΠ²ΠΊΠΈ ΠΈΒ Π΄ΠΎΠ±ΠΈΡ‚ΡŒΡΡ отсутствия постоянной ΠΏΠΎΠ΄ΠΏΠΈΡ‚ΠΊΠΈ свСТСй Π²ΠΎΠ΄ΠΎΠΏΡ€ΠΎΠ²ΠΎΠ΄Π½ΠΎΠΉ Π²ΠΎΠ΄Ρ‹ Π²Β Π’Π°ΡˆΡƒ систСму отоплСния. Π”Π»ΡΒ ΡΡ‚Π°Π»ΡŒΠ½Ρ‹Ρ… ΠΏΠ°Π½Π΅Π»ΡŒΠ½Ρ‹Ρ… Ρ€Π°Π΄ΠΈΠ°Ρ‚ΠΎΡ€ΠΎΠ² pH тСплоноситСля Π΄ΠΎΠ»ΠΆΠ΅Π½ Π±Ρ‹Ρ‚ΡŒ Π½Π΅ΠΉΡ‚Ρ€Π°Π»ΡŒΠ½Ρ‹ΠΌ.

Высокая Ρ‚Π΅Ρ…Π½ΠΎΠ»ΠΎΠ³ΠΈΡ‡Π½ΠΎΡΡ‚ΡŒ стали Π½Π΅Β Ρ‚Ρ€Π΅Π±ΡƒΠ΅Ρ‚ Π΄ΠΎΠΊΠ°Π·Π°Ρ‚Π΅Π»ΡŒΡΡ‚Π². Π­Ρ‚ΠΎΡ‚ пластичный, ΠΏΡ€ΠΎΡ‡Π½Ρ‹ΠΉ, Π³ΠΈΠ±ΠΊΠΈΠΉ ΠΈΒ ΠΊΠΎΠ²ΠΊΠΈΠΉ ΠΌΠ°Ρ‚Π΅Ρ€ΠΈΠ°Π» Ρ…ΠΎΡ€ΠΎΡˆΠΎ поддаСтся сваркС, Π°Β Ρ‚Π°ΠΊΠΆΠ΅ Π·Π°ΠΌΠ΅Ρ‡Π°Ρ‚Π΅Π»ΡŒΠ½ΠΎ ΠΏΡ€ΠΎΠ²ΠΎΠ΄ΠΈΡ‚ Ρ‚Π΅ΠΏΠ»ΠΎ.

ΠŸΡ€ΠΈ ΠΈΠ·Π³ΠΎΡ‚ΠΎΠ²Π»Π΅Π½ΠΈΠΈ ΡΡ‚Π°Π»ΡŒΠ½Ρ‹Ρ… ΠΏΠ°Π½Π΅Π»ΡŒΠ½Ρ‹Ρ… Ρ€Π°Π΄ΠΈΠ°Ρ‚ΠΎΡ€ΠΎΠ² примСняСтся низкоуглСродистая ΡΡ‚Π°Π»ΡŒ.

ПослС изготовлСния Ρ€Π°Π΄ΠΈΠ°Ρ‚ΠΎΡ€Π°, Π΅Π³ΠΎΒ ΠΏΠΎΠ²Π΅Ρ€Ρ…Π½ΠΎΡΡ‚ΡŒ обрабатываСтся составами, Π·Π°Ρ‰ΠΈΡ‰Π°ΡŽΡ‰ΠΈΠΌΠΈ ΡΡ‚Π°Π»ΡŒ ΠΎΡ‚Β ΠΊΠΎΡ€Ρ€ΠΎΠ·ΠΈΠΈ, Ρ€Π°Π΄ΠΈΠ°Ρ‚ΠΎΡ€ грунтуСтся ΠΈΒ ΠΎΠΊΡ€Π°ΡˆΠΈΠ²Π°Π΅Ρ‚ΡΡ.

Π‘Ρ‚Π°Π»ΡŒΠ½ΠΎΠΉ ΠΏΠ°Π½Π΅Π»ΡŒΠ½Ρ‹ΠΉ Ρ€Π°Π΄ΠΈΠ°Ρ‚ΠΎΡ€ прСдставляСт собой ΠΏΡ€ΡΠΌΠΎΡƒΠ³ΠΎΠ»ΡŒΠ½ΡƒΡŽ панСль, ΡΠΎΡΡ‚ΠΎΡΡ‰ΡƒΡŽ ΠΈΠ·Β Π΄Π²ΡƒΡ… сварСнных вмСстС ΡΡ‚Π°Π»ΡŒΠ½Ρ‹Ρ… листов, ΡΒ Π²Ρ‹ΡˆΡ‚Π°ΠΌΠΏΠΎΠ²Π°Π½Π½Ρ‹ΠΌΠΈ углублСниями, при сваркС ΠΎΠ±Ρ€Π°Π·ΡƒΡŽΡ‰ΠΈΠΌΠΈ ΠΊΠ°Π½Π°Π»Ρ‹ для циркуляции тСплоноситСля. Для увСличСния Ρ‚Π΅ΠΏΠ»ΠΎΠΎΡ‚Π΄Π°Ρ‡ΠΈ, ΠΊΒ Ρ‚Ρ‹Π»ΡŒΠ½ΠΎΠΉ сторонС ΠΏΠ°Π½Π΅Π»ΠΈ привариваСтся ΠΊΠΎΠ½Π²Π΅ΠΊΡ‚ΠΎΡ€. ΠšΠΎΠ½Π²Π΅ΠΊΡ‚ΠΎΡ€ прСдставляСт из сСбя Π±ΠΎΠ»Π΅Π΅ Ρ‚ΠΎΠ½ΠΊΠΈΠΉ ΡˆΡ‚Π°ΠΌΠΏΠΎΠ²Π°Π½Π½Ρ‹ΠΉ ΠΏ-ΠΎΠ±Ρ€Π°Π·Π½Ρ‹ΠΉ лист.Β Π‘Ρ‚Π°Π»ΡŒΠ½ΠΎΠΉ ΠΏΠ°Π½Π΅Π»ΡŒΠ½Ρ‹ΠΉ Ρ€Π°Π΄ΠΈΠ°Ρ‚ΠΎΡ€ отоплСния ΠΌΠΎΠΆΠ΅Ρ‚ Π±Ρ‹Ρ‚ΡŒ сварСн ΠΈΠ·Β ΠΎΠ΄Π½ΠΎΠΉ, Π΄Π²ΡƒΡ…Β ΠΈΠ»ΠΈΒ Ρ‚Ρ€Ρ‘Ρ… ΠΏΠ°Π½Π΅Π»Π΅ΠΉ, соСдинСнных ΠΌΠ΅ΠΆΠ΄Ρƒ собой ΡΒ ΠΏΠΎΠΌΠΎΡ‰ΡŒΡŽ ΡΡ‚Π°Π»ΡŒΠ½Ρ‹Ρ… ΠΏΠ°Ρ‚Ρ€ΡƒΠ±ΠΊΠΎΠ². Если в ряд соСдинСны нСсколько ΠΏΠ°Π½Π΅Π»Π΅ΠΉ, то с обоих сторон и с вСрху ΠΈΡ…Β Π½Π°ΠΊΡ€Ρ‹Π²Π°ΡŽΡ‚ ΠΊΠΎΠΆΡƒΡ…Π°ΠΌΠΈ. Π’Π΅Ρ€Ρ…Π½ΠΈΠΉ ΠΊΠΎΠΆΡƒΡ… Π²Ρ‹ΠΏΠΎΠ»Π½Π΅Π½ Π²Β Π²ΠΈΠ΄Π΅ воздуховыпускной Ρ€Π΅ΡˆΠ΅Ρ‚ΠΊΠΈ, которая нС прСпятствуСт свободному двиТСнию Π²ΠΎΠ·Π΄ΡƒΡ…Π° (ΠΊΠΎΠ½Π²Π΅ΠΊΡ†ΠΈΠΈ).

Π’ зависимости от количСства Π½Π°Π³Ρ€Π΅Π²Π°Ρ‚Π΅Π»ΡŒΠ½Ρ‹Ρ… ΠΈΒ ΠΊΠΎΠ½Π²Π΅ΠΊΡ‚ΠΎΡ€Π½Ρ‹Ρ… ΠΏΠ°Π½Π΅Π»Π΅ΠΉ, находящихся Π²Π½ΡƒΡ‚Ρ€ΠΈ Ρ€Π°Π΄ΠΈΠ°Ρ‚ΠΎΡ€ΠΎΠ² сущСствуСт ΡΡƒΡ‰Π΅ΡΡ‚Π²ΡƒΡŽΡ‚ ΡΠ»Π΅Π΄ΡƒΡŽΡ‰ΠΈΠ΅ Ρ‚ΠΈΠΏΡ‹ ΡΡ‚Π°Π»ΡŒΠ½Ρ‹Ρ… ΠΏΠ°Π½Π΅Π»ΡŒΠ½Ρ‹Ρ… Ρ€Π°Π΄ΠΈΠ°Ρ‚ΠΎΡ€ΠΎΠ²:

1) Π’ΠΈΠΏΒ 10- это однорядный Ρ€Π°Π΄ΠΈΠ°Ρ‚ΠΎΡ€ Π±Π΅Π·Β ΠΊΠΎΠ½Π²Π΅ΠΊΡ‚ΠΎΡ€Π° ΠΈΒ Π±Π΅Π·Β ΠΎΠ±Π»ΠΈΡ†ΠΎΠ²ΠΊΠΈ,

2) Π’ΠΈΠΏΒ 11- однорядный Ρ€Π°Π΄ΠΈΠ°Ρ‚ΠΎΡ€ с одним ΠΊΠΎΠ½Π²Π΅ΠΊΡ‚ΠΎΡ€ΠΎΠΌ, с боковыми ΠΎΠ±Π»ΠΈΡ†ΠΎΠ²ΠΊΠ°ΠΌΠΈ ΠΈΒ Π²Π΅Ρ€Ρ…Π½Π΅ΠΉ Ρ€Π΅ΡˆΠ΅Ρ‚ΠΊΠΎΠΉ,

3) Π’ΠΈΠΏΒ 20- двухрядный Ρ€Π°Π΄ΠΈΠ°Ρ‚ΠΎΡ€ Π±Π΅Π·Β ΠΊΠΎΠ½Π²Π΅ΠΊΡ‚ΠΎΡ€Π°, с боковыми ΠΎΠ±Π»ΠΈΡ†ΠΎΠ²ΠΊΠ°ΠΌΠΈ ΠΈΒ Π²Π΅Ρ€Ρ…Π½Π΅ΠΉ Ρ€Π΅ΡˆΠ΅Ρ‚ΠΊΠΎΠΉ,

4) Π’ΠΈΠΏΒ 21Β β€” двухрядный Ρ€Π°Π΄ΠΈΠ°Ρ‚ΠΎΡ€ с одним ΠΊΠΎΠ½Π²Π΅ΠΊΡ‚ΠΎΡ€ΠΎΠΌ, с боковыми ΠΎΠ±Π»ΠΈΡ†ΠΎΠ²ΠΊΠ°ΠΌΠΈ ΠΈΒ Π²Π΅Ρ€Ρ…Π½Π΅ΠΉ Ρ€Π΅ΡˆΠ΅Ρ‚ΠΊΠΎΠΉ

5) Π’ΠΈΠΏΒ 22- двухрядный Ρ€Π°Π΄ΠΈΠ°Ρ‚ΠΎΡ€ с двумя ΠΊΠΎΠ½Π²Π΅ΠΊΡ‚ΠΎΡ€Π°ΠΌΠΈ, с боковыми ΠΎΠ±Π»ΠΈΡ†ΠΎΠ²ΠΊΠ°ΠΌΠΈ ΠΈΒ Π²Π΅Ρ€Ρ…Π½Π΅ΠΉ Ρ€Π΅ΡˆΠ΅Ρ‚ΠΊΠΎΠΉ

6) Π’ΠΈΠΏΒ 30- трСхрядный, Π±Π΅Π·Β ΠΊΠΎΠ½Π²Π΅ΠΊΡ‚ΠΎΡ€ΠΎΠ², с боковыми ΠΎΠ±Π»ΠΈΡ†ΠΎΠ²ΠΊΠ°ΠΌΠΈ ΠΈΒ Π²Π΅Ρ€Ρ…Π½Π΅ΠΉ Ρ€Π΅ΡˆΠ΅Ρ‚ΠΊΠΎΠΉ,

7) Π’ΠΈΠΏΒ 33- трСхрядный Ρ€Π°Π΄ΠΈΠ°Ρ‚ΠΎΡ€ с трСмя ΠΊΠΎΠ½Π²Π΅ΠΊΡ‚ΠΎΡ€Π°ΠΌΠΈ, с боковыми ΠΎΠ±Π»ΠΈΡ†ΠΎΠ²ΠΊΠ°ΠΌΠΈ ΠΈΒ Π²Π΅Ρ€Ρ…Π½Π΅ΠΉ Ρ€Π΅ΡˆΠ΅Ρ‚ΠΊΠΎΠΉ.

НаибольшСС распространСниС в России ΠΏΠΎΠ»ΡƒΡ‡ΠΈΠ»ΠΈ Ρ‚ΠΈΠΏΒ 22 (66% ΠΏΡ€ΠΎΠ΄Π°Π²Π°Π΅ΠΌΡ‹Ρ… Ρ€Π°Π΄ΠΈΠ°Ρ‚ΠΎΡ€ΠΎΠ²), Ρ‚ΠΈΠΏΒ 11 (19% ΠΏΡ€ΠΎΠ΄Π°Π²Π°Π΅ΠΌΡ‹Ρ… Ρ€Π°Π΄ΠΈΠ°Ρ‚ΠΎΡ€ΠΎΠ²), Ρ‚ΠΈΠΏΒ 21 (9-12%).Β ΠžΡ‚Β Ρ‚ΠΈΠΏΠ° Ρ€Π°Π΄ΠΈΠ°Ρ‚ΠΎΡ€Π° Π±ΡƒΠ΄Π΅Ρ‚ Π·Π°Π²ΠΈΡΠ΅Ρ‚ΡŒ Π΅Π³ΠΎΒ Ρ‚Π΅ΠΏΠ»ΠΎΠΎΡ‚Π΄Π°Ρ‡Π°.Β Π‘ ΠΎΡ‚Π΄Π°Ρ‡Π΅ΠΉ Ρ‚Π΅ΠΏΠ»Π° ΡƒΒ ΡΡ‚Π°Π»ΡŒΠ½Ρ‹Ρ… Ρ€Π°Π΄ΠΈΠ°Ρ‚ΠΎΡ€ΠΎΠ² Π΄Π΅Π»Π° обстоят совсСм Π½Π΅ΠΏΠ»ΠΎΡ…ΠΎ – ΠΏΠΎΠΊΠ°Π·Π°Ρ‚Π΅Π»ΡŒ Ρ‚Π΅ΠΏΠ»ΠΎΠΎΡ‚Π΄Π°Ρ‡ΠΈ Π²Π°Ρ€ΡŒΠΈΡ€ΡƒΠ΅Ρ‚ΡΡ ΠΎΡ‚Β 210 Π΄ΠΎΒ 10 000 Π’Π°Ρ‚Ρ‚, а доля Ρ‚Π΅ΠΏΠ»ΠΎΠ²ΠΎΠ³ΠΎ ΠΏΠΎΡ‚ΠΎΠΊΠ°, ΠΏΠ΅Ρ€Π΅Π΄Π°Π²Π°Π΅ΠΌΠΎΠ³ΠΎ ΠΊΠΎΠ½Π²Π΅ΠΊΡ†ΠΈΠ΅ΠΉ, ΠΌΠΎΠΆΠ΅Ρ‚ Π΄ΠΎΡ…ΠΎΠ΄ΠΈΡ‚ΡŒ Π΄ΠΎΒ 75%. Зависит этот ΠΏΠ°Ρ€Π°ΠΌΠ΅Ρ‚Ρ€ ΠΎΡ‚Β Π³Π°Π±Π°Ρ€ΠΈΡ‚ΠΎΠ² Ρ€Π°Π΄ΠΈΠ°Ρ‚ΠΎΡ€Π° (Ρ‚ΠΈΠΏΠ°, высоты, Π΄Π»ΠΈΠ½Ρ‹), ΠΎΡ‚Β ΠΏΡ€ΠΈΠΌΠ΅Π½Ρ‘Π½Π½Ρ‹Ρ… ΠΏΡ€ΠΎΠΈΠ·Π²ΠΎΠ΄ΠΈΡ‚Π΅Π»Π΅ΠΌ Π½ΠΎΡƒ-Ρ…Π°Ρƒ, ΠΏΠΎΠ²Ρ‹ΡˆΠ°ΡŽΡ‰ΠΈΡ… ΠΊΠΎΠ½Π²Π΅ΠΊΡ†ΠΈΡŽ Ρ‚Π΅ΠΏΠ»ΠΎΠ²ΠΎΠ³ΠΎ ΠΏΡ€ΠΈΠ±ΠΎΡ€Π°. Π—Π°ΠΌΠ΅Ρ‚ΠΈΠΌ, Ρ‡Ρ‚ΠΎΒ Π±ΠΎΠ»ΡŒΡˆΠΈΠΌ плюсом Π΄Π°Π½Π½Ρ‹Ρ… ΠΏΡ€ΠΈΠ±ΠΎΡ€ΠΎΠ² являСтся малая ΠΈΠ½Π΅Ρ€Ρ†ΠΈΠΎΠ½Π½ΠΎΡΡ‚ΡŒ. ΠžΠ½ΠΈΒ ΠΎΡ‡Π΅Π½ΡŒ быстро Π½Π°Π³Ρ€Π΅Π²Π°ΡŽΡ‚ΡΡ ΠΈΒ Π½Π°Ρ‡ΠΈΠ½Π°ΡŽΡ‚ ΠΎΡ‚Π΄Π°Π²Π°Ρ‚ΡŒ Ρ‚Π΅ΠΏΠ»ΠΎ ΠΏΠΎΠΌΠ΅Ρ‰Π΅Π½ΠΈΡŽ. МаксимальноС Ρ€Π°Π±ΠΎΡ‡Π΅Π΅ Π΄Π°Π²Π»Π΅Π½ΠΈΠ΅ ΡƒΒ Π΄Π°Π½Π½ΠΎΠ³ΠΎ Π²ΠΈΠ΄Π° Ρ€Π°Π΄ΠΈΠ°Ρ‚ΠΎΡ€ΠΎΠ² колСблСтся Π²Β ΠΏΡ€Π΅Π΄Π΅Π»Π°Ρ… ΠΎΡ‚Β 8 Π΄ΠΎΒ 10 атмосфСр. ΠžΠΏΡ€Π΅ΡΡΠΎΠ²ΠΎΡ‡Π½ΠΎΠ΅ Π΄Π°Π²Π»Π΅Π½ΠΈΠ΅ (максимальноС Ρ€Π°Π±ΠΎΡ‡Π΅Π΅) составляСт 12-15 атмосфСр. МаксимальноС Π΄Π°Π²Π»Π΅Π½ΠΈΠ΅ Ρ€Π°Π·Ρ€ΡƒΡˆΠ΅Π½ΠΈΡ Ρ‚Π°ΠΊΠΎΠ³ΠΎ Ρ€Π°Π΄ΠΈΠ°Ρ‚ΠΎΡ€Π° ΠΎΠ±Ρ‹Ρ‡Π½ΠΎ составляСт 20-25 атмосфСр. Максимальная Ρ‚Π΅ΠΌΠΏΠ΅Ρ€Π°Ρ‚ΡƒΡ€Π° горячСй Π²ΠΎΠ΄Ρ‹, ΠΊΠΎΡ‚ΠΎΡ€ΡƒΡŽ ΠΌΠΎΠ³ΡƒΡ‚ Π²Ρ‹Π΄Π΅Ρ€ΠΆΠ°Ρ‚ΡŒ ΡΡ‚Π°Π»ΡŒΠ½Ρ‹Π΅ Π±Π°Ρ‚Π°Ρ€Π΅ΠΈ – ΠΎΡ‚Β 110 Π΄ΠΎΒ 120 градусов. Π‘Ρ‚Π°Π»ΡŒΠ½Ρ‹Π΅ Ρ€Π°Π΄ΠΈΠ°Ρ‚ΠΎΡ€Ρ‹ ΠΌΠΎΠ³ΡƒΡ‚ ΠΈΠΌΠ΅Ρ‚ΡŒ ΠΊΠ°ΠΊΒ Π±ΠΎΠΊΠΎΠ²ΠΎΠ΅ Ρ‚Π°ΠΊΒ ΠΈΒ Π½ΠΈΠΆΠ½Π΅Π΅ ΠΏΠΎΠ΄ΠΊΠ»ΡŽΡ‡Π΅Π½ΠΈΠ΅. Π Π°Π΄ΠΈΠ°Ρ‚ΠΎΡ€ с ниТним ΠΏΠΎΠ΄ΠΊΠ»ΡŽΡ‡Π΅Π½ΠΈΠ΅ΠΌ ΠΈΠΌΠ΅Π΅Ρ‚ ΠΈΠ½ΡƒΡŽ ΠΊΠΎΠ½ΡΡ‚Ρ€ΡƒΠΊΡ†ΠΈΡŽ ΠΏΠΎΠ΄Π°Ρ‡ΠΈ ΠΈΒ ΠΎΡ‚Π²ΠΎΠ΄Π° тСплоноситСля ΠΎΡ‚Β ΠΏΡ€ΠΈΠ±ΠΎΡ€Π°, Ρ‡Π΅ΠΌΒ Ρ€Π°Π΄ΠΈΠ°Ρ‚ΠΎΡ€ с боковым ΠΏΠΎΠ΄ΠΊΠ»ΡŽΡ‡Π΅Π½ΠΈΠ΅ΠΌ. Π‘Ρ‚Π°Π»ΡŒΠ½ΠΎΠΉ ΠΏΠ°Π½Π΅Π»ΡŒΠ½Ρ‹ΠΉ Ρ€Π°Π΄ΠΈΠ°Ρ‚ΠΎΡ€ с ниТним ΠΏΠΎΠ΄ΠΊΠ»ΡŽΡ‡Π΅Π½ΠΈΠ΅ΠΌ примСняСтся Ρ‚ΠΎΠ»ΡŒΠΊΠΎ с автоматичСским Ρ‚Π΅Ρ€ΠΌΠΎΡ€Π΅Π³ΡƒΠ»ΠΈΡ€ΡƒΡŽΡ‰ΠΈΠΌ ΠΊΠ»Π°ΠΏΠ°Π½ΠΎΠΌ. Подвод ΠΈΒ ΠΎΡ‚Π±ΠΎΡ€ тСплоноситСля ΠΊΒ Ρ‚Π°ΠΊΠΈΠΌ Ρ‚Π΅ΠΏΠ»ΠΎΠ²Ρ‹ΠΌ ΠΏΡ€ΠΈΠ±ΠΎΡ€Π°ΠΌ производится ΡΒ ΠΏΠΎΠΌΠΎΡ‰ΡŒΡŽ ΡƒΠ·Π»Π° Π½ΠΈΠΆΠ½Π΅Π³ΠΎ ΠΏΠΎΠ΄ΠΊΠ»ΡŽΡ‡Π΅Π½ΠΈΡ с мСТосСвым расстояниСм 50 ΠΌΠΌ. Установка Ρ€Π°Π΄ΠΈΠ°Ρ‚ΠΎΡ€Π° с ниТним ΠΏΠΎΠ΄ΠΊΠ»ΡŽΡ‡Π΅Π½ΠΈΠ΅ΠΌ позволяСт ΠΏΡ€ΠΎΠΈΠ·Π²ΠΎΠ΄ΠΈΡ‚ΡŒ ΠΌΠΎΠ½Ρ‚Π°ΠΆ ΠΊΒ Π½Π΅ΠΌΡƒ скрытой Ρ‚Ρ€ΡƒΠ±Π½ΠΎΠΉ ΠΏΠΎΠ΄Π²ΠΎΠ΄ΠΊΠΈ, ΠΌΠΎΠ½Ρ‚ΠΈΡ€ΡƒΠ΅ΠΌΠΎΠΉ в стСны ΠΈ (ΠΈΠ»ΠΈ) пол здания, Ρ‡Ρ‚ΠΎΒ ΡƒΠ»ΡƒΡ‡ΡˆΠ°Π΅Ρ‚ эстСтичный внСшний Π²ΠΈΠ΄. ΠžΠ±Ρ‹Ρ‡Π½ΠΎ, любой Π΄ΠΈΠ·Π°ΠΉΠ½-ΠΏΡ€ΠΎΠ΅ΠΊΡ‚ помСщСния, ΠΏΡ€Π΅Π΄ΠΏΠΎΠ»Π°Π³Π°Π΅Ρ‚ установку Π΄Π°Π½Π½Ρ‹Ρ… ΠΏΡ€ΠΈΠ±ΠΎΡ€ΠΎΠ².

 МСТосСвоС расстояниС Π²Π°ΠΆΠ½ΠΎ для радиаторов ΠΈΠΌΠ΅Π½Π½ΠΎ с боковым ΠΏΠΎΠ΄ΠΊΠ»ΡŽΡ‡Π΅Π½ΠΈΠ΅ΠΌ. ΠžΠ½ΠΎΒ ΠΎΠΏΡ€Π΅Π΄Π΅Π»ΡΠ΅Ρ‚, Π½Π°Β ΠΊΠ°ΠΊΠΎΠΌ расстоянии Π²Π΅Ρ€Ρ…Π½ΠΈΠΉ ΠΏΠΎΠ΄Π°ΡŽΡ‰ΠΈΠΉ Ρ‚Ρ€ΡƒΠ±Π½Ρ‹ΠΉ ΠΊΠΎΠ»Π»Π΅ΠΊΡ‚ΠΎΡ€ находится ΠΎΡ‚Β Π½ΠΈΠΆΠ½Π΅Π³ΠΎ, отводящСго. Π­Ρ‚ΠΎΒ Π½Π΅ΠΎΠ±Ρ…ΠΎΠ΄ΠΈΠΌΠΎ ΡƒΡ‡ΠΈΡ‚Ρ‹Π²Π°Ρ‚ΡŒ ΠΏΡ€ΠΈΒ ΠΌΠΎΠ½Ρ‚Π°ΠΆΠ΅ Ρ€Π°Π΄ΠΈΠ°Ρ‚ΠΎΡ€Π°. Π‘Ρ‚Π°Π»ΡŒΠ½Ρ‹Π΅ ΠΏΠ°Π½Π΅Π»ΡŒΠ½Ρ‹Π΅ Ρ€Π°Π΄ΠΈΠ°Ρ‚ΠΎΡ€Ρ‹ с боковым ΠΏΠΎΠ΄ΠΊΠ»ΡŽΡ‡Π΅Π½ΠΈΠ΅ΠΌ, в зависимости ΠΎΡ‚Β ΠΌΠΎΠ΄Π΅Π»ΠΈ, Ρ‚ΠΈΠΏΠ° и производитСля, ΠΌΠΎΠ³ΡƒΡ‚ ΠΈΠΌΠ΅Ρ‚ΡŒ мСТосСвоС расстояниС Ρ€Π°Π²Π½ΠΎΠ΅ высотС Ρ€Π°Π΄ΠΈΠ°Ρ‚ΠΎΡ€Π°, минус 50Β β€” 70 ΠΌΠΌ. Β 

Β 

Β ΠΠ»ΡŽΠΌΠΈΠ½ΠΈΠ΅Π²Ρ‹Π΅ Ρ€Π°Π΄ΠΈΠ°Ρ‚ΠΎΡ€Ρ‹.

Β 

ΠΠ»ΡŽΠΌΠΈΠ½ΠΈΠ΅Π²Ρ‹Π΅ Ρ€Π°Π΄ΠΈΠ°Ρ‚ΠΎΡ€Ρ‹ отоплСния заняли ΠΏΡ€ΠΎΡ‡Π½ΠΎΠ΅ мСсто Π½Π°Β Ρ€Ρ‹Π½ΠΊΠ΅ ΠΎΡ‚ΠΎΠΏΠΈΡ‚Π΅Π»ΡŒΠ½Ρ‹Ρ… систСм. Π”Π°Π½Π½Ρ‹ΠΉ Ρ‚ΠΈΠΏΒ Ρ€Π°Π΄ΠΈΠ°Ρ‚ΠΎΡ€ΠΎΠ² ΠΈΠ·Π³ΠΎΡ‚ΠΎΠ²Π»Π΅Π½ из сплава алюминия с крСмниСвыми Π΄ΠΎΠ±Π°Π²ΠΊΠ°ΠΌΠΈ, Π²Ρ‹ΠΏΠΎΠ»Π½Π΅Π½Π½Ρ‹ΠΌΠΈ Π²Β Π²ΠΈΠ΄Π΅ ΠΎΡ‚Π΄Π΅Π»ΡŒΠ½Ρ‹Ρ… сСкций. ΠŸΡ€ΠΈΠ±Π»ΠΈΠ·ΠΈΡ‚Π΅Π»ΡŒΠ½ΠΎ ΠΏΠΎΠ»ΠΎΠ²ΠΈΠ½Ρƒ своСго Ρ‚Π΅ΠΏΠ»Π° Π°Π»ΡŽΠΌΠΈΠ½ΠΈΠ΅Π²Ρ‹Π΅ Ρ€Π°Π΄ΠΈΠ°Ρ‚ΠΎΡ€Ρ‹ ΠΎΡ‚Π΄Π°ΡŽΡ‚ ΡΒ ΠΏΠΎΠΌΠΎΡ‰ΡŒΡŽ излучСния, Π²Ρ‚ΠΎΡ€ΡƒΡŽ ΠΏΠΎΠ»ΠΎΠ²ΠΈΠ½Ρƒ ΡΒ ΠΏΠΎΠΌΠΎΡ‰ΡŒΡŽ ΠΊΠΎΠ½Π²Π΅ΠΊΡ†ΠΈΠΈ (Π²ΠΎΠ·Π΄ΡƒΡˆΠ½Π°Ρ циркуляция снизу — Π²Π²Π΅Ρ€Ρ…). ΠŸΡ€ΠΈΠΌΠ΅Π½Π΅Π½ΠΈΠ΅ Ρ…ΠΎΡ€ΠΎΡˆΠΎ Ρ€Π°Π·Π²ΠΈΡ‚Ρ‹Ρ… повСрхностСй Π²Β Π²ΠΈΠ΄Π΅ Ρ‚ΠΎΠ½ΠΊΠΈΡ… Ρ€Π΅Π±Π΅Ρ€, ΠΊΠΎΡ‚ΠΎΡ€Ρ‹Π΅ располоТСны Π²ΠΎΒ Π²Π½ΡƒΡ‚Ρ€Π΅Π½Π½Π΅ΠΉ части сСкций, ΡƒΠ²Π΅Π»ΠΈΡ‡ΠΈΠ²Π°ΡŽΡ‚, ΠΈΒ Π±Π΅Π·Β Ρ‚ΠΎΠ³ΠΎ довольно Π²Ρ‹ΡΠΎΠΊΡƒΡŽ, Ρ‚Π΅ΠΏΠ»ΠΎΠΎΡ‚Π΄Π°Ρ‡Ρƒ Π°Π»ΡŽΠΌΠΈΠ½ΠΈΠ΅Π²Ρ‹Ρ… Ρ€Π°Π΄ΠΈΠ°Ρ‚ΠΎΡ€ΠΎΠ². ΠΠ»ΡŽΠΌΠΈΠ½ΠΈΠ΅Π²Ρ‹Π΅ Ρ€Π°Π΄ΠΈΠ°Ρ‚ΠΎΡ€Ρ‹ отоплСния ΠΈΠΌΠ΅ΡŽΡ‚ ΡΠ»Π΅Π΄ΡƒΡŽΡ‰ΠΈΠ΅ тСхничСскиС характСристики: Ρ€Π°Π±ΠΎΡ‡Π΅Π΅ Π΄Π°Π²Π»Π΅Π½ΠΈΠ΅ Π±ΠΎΠ»Π΅Π΅ 12 атмосфСр, ΠΈΡΠΏΡ‹Ρ‚Π°Ρ‚Π΅Π»ΡŒΠ½ΠΎΠ΅ Π±ΠΎΠ»Π΅Π΅ 18 атмосфСр. Максимальной Ρ‚Π΅ΠΌΠΏΠ΅Ρ€Π°Ρ‚ΡƒΡ€ΠΎΠΉ тСплоноситСля ΡΠ²Π»ΡΡŽΡ‚ΡΡ Ρ‚Π΅ΠΌΠΏΠ΅Ρ€Π°Ρ‚ΡƒΡ€Ρ‹ порядка 110-120Β°Π‘.Β Π“Π»Π°Π²Π½Ρ‹ΠΌΠΈ ΠΏΠΎΠ»ΠΎΠΆΠΈΡ‚Π΅Π»ΡŒΠ½Ρ‹ΠΌΠΈ качСствами Π°Π»ΡŽΠΌΠΈΠ½ΠΈΠ΅Π²Ρ‹Ρ… Ρ€Π°Π΄ΠΈΠ°Ρ‚ΠΎΡ€ΠΎΠ² отоплСния ΡΠ²Π»ΡΡŽΡ‚ΡΡ ΠΈΡ…Β Π½Π΅Π±ΠΎΠ»ΡŒΡˆΠΎΠΉ вСс, простота в установкС, высокий ΡƒΡ€ΠΎΠ²Π΅Π½ΡŒ Ρ‚Π΅ΠΏΠ»ΠΎΠΎΡ‚Π΄Π°Ρ‡ΠΈ и соврСмСнный Π΄ΠΈΠ·Π°ΠΉΠ½, ΠΊΠΎΡ‚ΠΎΡ€Ρ‹ΠΉ Π΄Π°Π΅Ρ‚ Π²ΠΎΠ·ΠΌΠΎΠΆΠ½ΠΎΡΡ‚ΡŒ установки Ρ€Π°Π΄ΠΈΠ°Ρ‚ΠΎΡ€ΠΎΠ² в помСщСниях с высокими эстСтичСскими трСбованиями. Β ΠžΡΠ½ΠΎΠ²Π½Ρ‹Π΅ нСдостатки: коррозия алюминия Π²Β Π²ΠΎΠ΄Π½ΠΎΠΉ срСдС, ΠΊΠΎΠ½Ρ†Π΅Π½Ρ‚Ρ€ΠΈΡ€ΠΎΠ²Π°Π½ΠΈΠ΅ Ρ‚Π΅ΠΏΠ»Π° Π½Π°Β Ρ€Π΅Π±Ρ€Π°Ρ… Ρ€Π°Π΄ΠΈΠ°Ρ‚ΠΎΡ€ΠΎΠ², низкая конвСктивная ΡΠΏΠΎΡΠΎΠ±Π½ΠΎΡΡ‚ΡŒ и высокая Π²Π΅Ρ€ΠΎΡΡ‚Π½ΠΎΡΡ‚ΡŒ газообразования с выдСлСниСм Π²ΠΎΠ΄ΠΎΡ€ΠΎΠ΄Π°.

ΠΠ»ΡŽΠΌΠΈΠ½ΠΈΠ΅Π²Ρ‹Π΅ Ρ€Π°Π΄ΠΈΠ°Ρ‚ΠΎΡ€Ρ‹ Ρ‡Π°Ρ‰Π΅ всСго дСлят на три основныС Ρ‚ΠΈΠΏΠ°: Π»ΠΈΡ‚Ρ‹Π΅ ΡΒ Ρ†Π΅Π»ΡŒΠ½Ρ‹ΠΌΠΈ сСкциями, экструдированныС с мСханичСски соСдинСнным Π½Π°Π±ΠΎΡ€ΠΎΠΌ сСкций, ΠΊΠΎΠΌΠ±ΠΈΠ½ΠΈΡ€ΠΎΠ²Π°Π½Π½Ρ‹Π΅ – Π»ΠΈΡ‚Ρ‹Π΅ с мСханичСски соСдинСнным Π½Π°Π±ΠΎΡ€ΠΎΠΌ сСкций.

Β 

БимСталличСскиС Ρ€Π°Π΄ΠΈΠ°Ρ‚ΠΎΡ€Ρ‹.

Π‘Π΅ΠΊΡ†ΠΈΠΎΠ½Π½Ρ‹Π΅ Ρ€Π°Π΄ΠΈΠ°Ρ‚ΠΎΡ€Ρ‹ ΠΎΠ±Π»Π°Π΄Π°ΡŽΡ‚ высокой ΠΏΠΎΠΏΡƒΠ»ΡΡ€Π½ΠΎΡΡ‚ΡŒΡŽ срСди ΠΏΠΎΡ‚Ρ€Π΅Π±ΠΈΡ‚Π΅Π»Π΅ΠΉ. Π‘Ρ‚ΠΎΠΈΠΌΠΎΡΡ‚ΡŒ Ρ€Π°Π΄ΠΈΠ°Ρ‚ΠΎΡ€Π° зависит от вСса ΠΎΠ΄Π½ΠΎΠΉ сСкции, количСства сСкций, мощности ΠΎΠ΄Π½ΠΎΠΉ сСкции, страны производства Ρ€Π°Π΄ΠΈΠ°Ρ‚ΠΎΡ€Π° и от производитСля ΠΏΡ€ΠΎΠ΄ΡƒΠΊΡ†ΠΈΠΈ.Β  БимСталличСскиС Ρ€Π°Π΄ΠΈΠ°Ρ‚ΠΎΡ€Ρ‹ ΠΎΡ‚ΠΎΠΏΠ»Π΅Π½ΠΈΡΒ ΠΈΠ·Π³ΠΎΡ‚Π°Π²Π»ΠΈΠ²Π°ΡŽΡ‚ΡΡ ΠΈΠ·Β Π΄Π²ΡƒΡ… Ρ€Π°Π·Π»ΠΈΡ‡Π½Ρ‹Ρ… ΠΌΠ΅Ρ‚Π°Π»Π»ΠΎΠ²: сплава алюминия и стали. Благодаря Π½Π°Π»ΠΈΡ‡ΠΈΡŽ сплава алюминия, обСспСчиваСтся эффСктивный Ρ‚Π΅ΠΏΠ»ΠΎΠ²ΠΎΠΉ ΠΏΠΎΡ‚ΠΎΠΊ, что позволяСт Ρ€Π°Π²Π½ΠΎΠΌΠ΅Ρ€Π½ΠΎ ΠΏΡ€ΠΎΠ³Ρ€Π΅Ρ‚ΡŒ вСсь корпус Ρ€Π°Π΄ΠΈΠ°Ρ‚ΠΎΡ€Π°.Β Π‘Π΅Ρ€Π΄Ρ†Π΅Π²ΠΈΠ½Π°, изготовлСнная из стали, позволяСт Π·Π°Ρ‰ΠΈΡ‚ΠΈΡ‚ΡŒ Ρ€Π°Π΄ΠΈΠ°Ρ‚ΠΎΡ€ от воздСйствия Ρ‰Π΅Π»ΠΎΡ‡ΠΈ ΠΈΒ Π΄Ρ€ΡƒΠ³ΠΎΠ³ΠΎ агрСссивного окруТСния, ΠΏΡ€ΠΈΡΡƒΡ‚ΡΡ‚Π²ΡƒΡŽΡ‰Π΅Π³ΠΎ Π²Β ΠΎΡ‚ΠΎΠΏΠΈΡ‚Π΅Π»ΡŒΠ½Ρ‹Ρ… систСмах. Β Π‘Ρ‚Π°Π»ΡŒΠ½ΠΎΠ΅ сСрдСчник ΡƒΒ ΠΏΠΎΠ»Π½ΠΎ-бимСталличСских Ρ€Π°Π΄ΠΈΠ°Ρ‚ΠΎΡ€ΠΎΠ² Π΄Π°Π΅Ρ‚ Π²ΠΎΠ·ΠΌΠΎΠΆΠ½ΠΎΡΡ‚ΡŒ ΠΈΡΠΏΠΎΠ»ΡŒΠ·ΠΎΠ²Π°Ρ‚ΡŒ Ρ€Π°Π΄ΠΈΠ°Ρ‚ΠΎΡ€Ρ‹ при высоком Π΄Π°Π²Π»Π΅Π½ΠΈΠΈ, Π·Π½Π°Ρ‡Π΅Π½ΠΈΠ΅ ΠΊΠΎΡ‚ΠΎΡ€ΠΎΠ³ΠΎ ΠΌΠΎΠΆΠ΅Ρ‚ Π΄ΠΎΡΡ‚ΠΈΠ³Π°Ρ‚ΡŒ порядка 40 атмосфСр.Β Π£Π΄Π°Ρ‡Π½ΠΎΠ΅ сочСтаниС тСхничСских характСристик позволяСт ΠΈΡΠΏΠΎΠ»ΡŒΠ·ΠΎΠ²Π°Ρ‚ΡŒ их для отоплСния ΠΏΠΎΠΌΠ΅Ρ‰Π΅Π½ΠΈΠΉ в высотных зданиях, Π³Π΄Π΅Β ΠΏΠΎΠ΄Π°Ρ‡Π° Π²ΠΎΠ΄Ρ‹ осущСствляСтся под довольно высокими давлСниями. БпСциалисты ΡΡ‡ΠΈΡ‚Π°ΡŽΡ‚, Ρ‡Ρ‚ΠΎΒ ΠΈΠΌΠ΅Π½Π½ΠΎ бимСталличСскиС Ρ€Π°Π΄ΠΈΠ°Ρ‚ΠΎΡ€Ρ‹ отоплСния Π»ΡƒΡ‡ΡˆΠ΅ всСго приспособлСны Π΄Π»ΡΒ ΠΎΡ‚ΠΎΠΏΠΈΡ‚Π΅Π»ΡŒΠ½Ρ‹Ρ… систСм отСчСствСнных многоэтаТных Π΄ΠΎΠΌΠΎΠ² с подачСй тСплоноситСля ΠΎΡ‚Β Π’Π­Π¦ (Ρ†Π΅Π½Ρ‚Ρ€Π°Π»ΡŒΠ½ΠΎΠ΅ ΠΎΡ‚ΠΎΠΏΠ»Π΅Π½ΠΈΠ΅). Помимо явного тСхничСского прСимущСства бимСталличСских Ρ€Π°Π΄ΠΈΠ°Ρ‚ΠΎΡ€ΠΎΠ² и высокой устойчивости к процСссам ΠΊΠΎΡ€Ρ€ΠΎΠ·ΠΈΠΈ, ΠΎΡ‚ΠΎΠΏΠΈΡ‚Π΅Π»ΡŒΠ½Ρ‹Π΅ ΠΏΡ€ΠΈΠ±ΠΎΡ€Ρ‹ Π΄Π°Π½Π½ΠΎΠ³ΠΎ Ρ‚ΠΈΠΏΠ° ΠΈΠΌΠ΅ΡŽΡ‚ нСбольшиС ΠΊΠΎΠΌΠΏΠ°ΠΊΡ‚Π½Ρ‹Π΅ Ρ€Π°Π·ΠΌΠ΅Ρ€Ρ‹. Π’Π°ΠΊΠΈΠΌ ΠΎΠ±Ρ€Π°Π·ΠΎΠΌ, основными прСимущСствами бимСталличСских Ρ€Π°Π΄ΠΈΠ°Ρ‚ΠΎΡ€ΠΎΠ² отоплСния ΠΏΠ΅Ρ€Π΅Π΄ Π΄Ρ€ΡƒΠ³ΠΈΠΌΠΈ ΡΠ²Π»ΡΡŽΡ‚ΡΡ ΠΈΡ…:

1) высокий ΡƒΡ€ΠΎΠ²Π΅Π½ΡŒ тСплопроводности;

2) высокая Π½Π°Π΄Π΅ΠΆΠ½ΠΎΡΡ‚ΡŒ;

3) достаточная ΡƒΡΡ‚ΠΎΠΉΡ‡ΠΈΠ²ΠΎΡΡ‚ΡŒ ΠΊΒ ΠΊΠΎΡ€Ρ€ΠΎΠ·ΠΈΠΈ ΠΌΠ΅Ρ‚Π°Π»Π»Π°;

4) Π½Π΅ΠΏΡ€ΠΈΡ…ΠΎΡ‚Π»ΠΈΠ²ΠΎΡΡ‚ΡŒ в использовании;

5) отсутствиС слоТностСй в процСссС установки.

Благодаря использованию ΠΏΡ€ΠΈΒ ΠΈΠ·Π³ΠΎΡ‚ΠΎΠ²Π»Π΅Π½ΠΈΠΈ Ρ€Π°Π΄ΠΈΠ°Ρ‚ΠΎΡ€Π° сплавов алюминия, по своСй способности ΠΎΡ‚Π΄Π°Π²Π°Ρ‚ΡŒ Ρ‚Π΅ΠΏΠ»ΠΎ бимСталличСскиС Ρ€Π°Π΄ΠΈΠ°Ρ‚ΠΎΡ€Ρ‹ ΠΏΡ€ΠΈΠ±Π»ΠΈΠΆΠ°ΡŽΡ‚ΡΡ ΠΊΒ ΠΎΡ‚ΠΎΠΏΠΈΡ‚Π΅Π»ΡŒΠ½Ρ‹ΠΌ Ρ€Π°Π΄ΠΈΠ°Ρ‚ΠΎΡ€Π°ΠΌ ΠΈΠ·Β Π°Π»ΡŽΠΌΠΈΠ½ΠΈΠ΅Π²Ρ‹Ρ… сплавов.Β  Π”Π°Π²Π»Π΅Π½ΠΈΠ΅ Ρ€Π°Π·Ρ€Ρ‹Π²Π° Ρ‚Π°ΠΊΠΎΠ³ΠΎ Ρ€Π°Π΄ΠΈΠ°Ρ‚ΠΎΡ€Π° составляСт ΠΏΡ€ΠΈΠ±Π»ΠΈΠ·ΠΈΡ‚Π΅Π»ΡŒΠ½ΠΎ 90 атмосфСр.Β 

Β 

Как ΠΏΡ€Π°Π²ΠΈΠ»ΡŒΠ½ΠΎ Π²Ρ‹Π±Ρ€Π°Ρ‚ΡŒ Ρ€Π°Π΄ΠΈΠ°Ρ‚ΠΎΡ€.

Β 

Π§Ρ‚ΠΎΠ±Ρ‹ ΠΏΡ€Π°Π²ΠΈΠ»ΡŒΠ½ΠΎ Π²Ρ‹Π±Ρ€Π°Ρ‚ΡŒ Ρ€Π°Π΄ΠΈΠ°Ρ‚ΠΎΡ€, Π½ΡƒΠΆΠ½ΠΎ ΡƒΡ‡Π΅ΡΡ‚ΡŒ ΡΠ»Π΅Π΄ΡƒΡŽΡ‰ΠΈΠ΅ ΠΏΠ°Ρ€Π°ΠΌΠ΅Ρ‚Ρ€Ρ‹: Π²Π½ΡƒΡ‚Ρ€Π΅Π½Π½ΠΈΠΉ объСм помСщСния,Β ΠΌΠ°Ρ‚Π΅Ρ€ΠΈΠ°Π», ΠΈΠ·Β ΠΊΠΎΡ‚ΠΎΡ€ΠΎΠ³ΠΎ построСн Π΄ΠΎΠΌ (ΠΊΠΈΡ€ΠΏΠΈΡ‡, Π±Π΅Ρ‚ΠΎΠ½ ΠΈΒ Ρ‚.Π΄.), количСство ΠΎΠΊΠΎΠ½ Π²Β ΠΊΠΎΠΌΠ½Π°Ρ‚Π΅ ΠΈΒ ΠΈΡ…Β Ρ€Π°Π·ΠΌΠ΅Ρ€, качСство стСклопакСтов, количСство Π½Π°Ρ€ΡƒΠΆΠ½Ρ‹Ρ… стСн (угловая ΠΊΠ²Π°Ρ€Ρ‚ΠΈΡ€Π° нуТдаСтся в усилСнном ΠΎΠ±ΠΎΠ³Ρ€Π΅Π²Π΅), Π°Β Ρ‚Π°ΠΊΠΆΠ΅ ΠΈΡ…Β ΡƒΡ‚Π΅ΠΏΠ»Π΅Π½ΠΈΠ΅, сторона, Π½Π°Β ΠΊΠΎΡ‚ΠΎΡ€ΡƒΡŽ выходят ΠΎΠΊΠ½Π°.

Β 

ΠŸΠΎΠ΄ΠΊΠ»ΡŽΡ‡Π΅Π½ΠΈΠ΅ Ρ€Π°Π΄ΠΈΠ°Ρ‚ΠΎΡ€Π° отоплСния.

Β 

ΠžΡ‡Π΅Π½ΡŒ Π²Π°ΠΆΠ½Ρ‹ΠΌ ΠΌΠΎΠΌΠ΅Π½Ρ‚ΠΎΠΌ выполнСния ΠΌΠΎΠ½Ρ‚Π°ΠΆΠ° систСмы отоплСния Π±ΡƒΠ΄Π΅Ρ‚ ΠΏΡ€Π°Π²ΠΈΠ»ΡŒΠ½ΠΎΠ΅ ΠΏΠΎΠ΄ΠΊΠ»ΡŽΡ‡Π΅Π½ΠΈΠ΅ Ρ€Π°Π΄ΠΈΠ°Ρ‚ΠΎΡ€ΠΎΠ² отоплСния с учСтом их прСимущСств и нСдостатков. БистСмы отоплСния ΠΌΠΎΠ³ΡƒΡ‚ Π±Ρ‹Ρ‚ΡŒ Π΄Π²ΡƒΡ… Π²ΠΈΠ΄ΠΎΠ²: двухтрубная и однотрубная.Β ΠžΡΠ½ΠΎΠ²Π½Ρ‹ΠΌ ΠΎΡ‚Π»ΠΈΡ‡ΠΈΠ΅ΠΌ ΠΎΠ΄Π½ΠΎΡ‚Ρ€ΡƒΠ±Π½Ρ‹Ρ… систСм отоплСния являСтся Ρ‚ΠΎ, Ρ‡Ρ‚ΠΎΒ ΠΏΠΎΠ΄Π°Ρ‡Π° горячСй Π²ΠΎΠ΄Ρ‹ Π²Β Π·Π΄Π°Π½ΠΈΠ΅ ΠΎΠ±Ρ‹Ρ‡Π½ΠΎ осущСствляСтся свСрху, Π·Π°Ρ‚Π΅ΠΌ ΠΎΠ½Π°Β Ρ‚Π΅Ρ‡Π΅Ρ‚ Π²Π½ΠΈΠ·, отдавая Ρ‚Π΅ΠΏΠ»ΠΎ Ρ‡Π΅Ρ€Π΅Π· ΠΎΡ‚ΠΎΠΏΠΈΡ‚Π΅Π»ΡŒΠ½Ρ‹Π΅ ΠΏΡ€ΠΈΠ±ΠΎΡ€Ρ‹, ΠΊΠΎΡ‚ΠΎΡ€Ρ‹Π΅ установлСны Π²Β ΠΊΠ²Π°Ρ€Ρ‚ΠΈΡ€Π΅. Π’Π°ΠΊΠΎΠΉ способ доставки тСплоноситСля ΠΈΡΠΏΠΎΠ»ΡŒΠ·ΡƒΠ΅Ρ‚ΡΡ Π²Β Π±ΠΎΠ»ΡŒΡˆΠΈΠ½ΡΡ‚Π²Π΅ Ρ‚ΠΈΠΏΠΎΠ²Ρ‹Ρ… ΠΊΠ²Π°Ρ€Ρ‚ΠΈΡ€. Π Π°Π·Π½ΠΎΠ²ΠΈΠ΄Π½ΠΎΡΡ‚ΡŒΡŽ Ρ‚Π°ΠΊΠΎΠΉ систСмы являСтся однотрубная Π³ΠΎΡ€ΠΈΠ·ΠΎΠ½Ρ‚Π°Π»ΡŒΠ½Π°Ρ (Π² ΠΏΡ€Π΅Π΄Π΅Π»Π°Ρ… ΠΎΠ΄Π½ΠΎΠ³ΠΎ этаТа здания). ΠšΒ Π½Π΅Π΄ΠΎΡΡ‚Π°Ρ‚ΠΊΠ°ΠΌ Ρ‚Π°ΠΊΠΎΠΉ систСмы ΠΌΠΎΠΆΠ½ΠΎ отнСсти Π½Π΅Π²ΠΎΠ·ΠΌΠΎΠΆΠ½ΠΎΡΡ‚ΡŒ рСгулирования Ρ‚Π΅ΠΌΠΏΠ΅Ρ€Π°Ρ‚ΡƒΡ€Ρ‹ ΠΏΡ€ΠΈΠ±ΠΎΡ€ΠΎΠ² отоплСния. Для этого понадобится установка ΡΠΏΠ΅Ρ†ΠΈΠ°Π»ΡŒΠ½Ρ‹Ρ… конструктивных элСмСнтов. Π’Π°ΠΊΠΆΠ΅ Π½ΡƒΠΆΠ½ΠΎ ΡƒΡ‡ΠΈΡ‚Ρ‹Π²Π°Ρ‚ΡŒ, что в этом случаС Π²ΠΎΠ΄Π°, ΠΏΠΎΡΡ‚ΡƒΠΏΠ°ΡŽΡ‰Π°Ρ Π²Β ΠΎΡ‚ΠΎΠΏΠΈΡ‚Π΅Π»ΡŒΠ½Ρ‹Π΅ ΠΏΡ€ΠΈΠ±ΠΎΡ€Ρ‹ Π½Π°Β Π½ΠΈΠΆΠ½ΠΈΡ… этаТах Π±ΡƒΠ΄Π΅Ρ‚ Π·Π½Π°Ρ‡ΠΈΡ‚Π΅Π»ΡŒΠ½ΠΎ Ρ…ΠΎΠ»ΠΎΠ΄Π½Π΅Π΅, Ρ‡Π΅ΠΌΒ Π½Π°Β Π²Π΅Ρ€Ρ…Π½ΠΈΡ…. Во ТС самоС ΠΏΡ€ΠΎΠΈΠ·ΠΎΠΉΠ΄Π΅Ρ‚ Π²Β Π³ΠΎΡ€ΠΈΠ·ΠΎΠ½Ρ‚Π°Π»ΡŒΠ½ΠΎΠΉ ΠΎΠ΄Π½ΠΎΡ‚Ρ€ΡƒΠ±Π½ΠΎΠΉ систСмС отоплСния, с послСдними Ρ€Π°Π΄ΠΈΠ°Ρ‚ΠΎΡ€Π°ΠΌΠΈ. Π§Ρ‚ΠΎΠ±Ρ‹ ΠΈΠ·Π±Π°Π²ΠΈΡ‚ΡŒΡΡ от этого нСдостатка, Π²Β ΠΎΠ΄Π½ΠΎΡ‚Ρ€ΡƒΠ±Π½ΠΎΠΉ Π³ΠΎΡ€ΠΈΠ·ΠΎΠ½Ρ‚Π°Π»ΡŒΠ½ΠΎΠΉ систСмС примСняСтся ΠŸΠ΅Ρ‚Π»Ρ Π’ΠΈΡ…Π΅Π»ΡŒΠΌΠ°Π½Π°.

Β 

ΠŸΡ€ΠΈ Π΄Π²ΡƒΡ…Ρ‚Ρ€ΡƒΠ±Π½ΠΎΠΉ систСмС отоплСния ΠΏΠΎΠ΄Π°Ρ‡Π° горячСй Π²ΠΎΠ΄Ρ‹ осущСствляСтся ΠΏΠΎΒ ΠΎΠ΄Π½ΠΎΠΉ Ρ‚Ρ€ΡƒΠ±Π΅, Π°Β ΠΎΡ‚Π²ΠΎΠ΄ Π²ΠΎΠ΄Ρ‹ β€” ΠΏΠΎΒ Π΄Ρ€ΡƒΠ³ΠΎΠΉ (ΠΎΠ±Ρ€Π°Ρ‚Π½ΠΎΠΉ Ρ‚Ρ€ΡƒΠ±Π΅). ΠŸΠΎΠ΄ΠΊΠ»ΡŽΡ‡Π΅Π½ΠΈΠ΅ Ρ€Π°Π΄ΠΈΠ°Ρ‚ΠΎΡ€ΠΎΠ² Π²Β Ρ‚Π°ΠΊΠΎΠΌ случаС производится ΠΏΠ°Ρ€Π°Π»Π»Π΅Π»ΡŒΠ½ΠΎ. Эта систСма вСсьма ΡƒΡΠΏΠ΅ΡˆΠ½ΠΎ ΠΈΡΠΏΠΎΠ»ΡŒΠ·ΡƒΠ΅Ρ‚ΡΡ Π²Β ΠΊΠΎΡ‚Ρ‚Π΅Π΄ΠΆΠ°Ρ…, частных Π΄ΠΎΠΌΠ°Ρ…. ΠšΒ ΠΏΠΎΠ»ΠΎΠΆΠΈΡ‚Π΅Π»ΡŒΠ½Ρ‹ΠΌ ΠΌΠΎΠΌΠ΅Π½Ρ‚Π°ΠΌ слСдуСт отнСсти Ρ‚ΠΎΡ‚Β Ρ„Π°ΠΊΡ‚, что она обСспСчиваСт ΠΎΠ΄ΠΈΠ½Π°ΠΊΠΎΠ²ΡƒΡŽ Ρ‚Π΅ΠΌΠΏΠ΅Ρ€Π°Ρ‚ΡƒΡ€Ρƒ Π½Π°Β Π²Ρ…ΠΎΠ΄Π΅ всСх Π½Π°Π³Ρ€Π΅Π²Π°Ρ‚Π΅Π»ΡŒΠ½Ρ‹Ρ… ΠΏΡ€ΠΈΠ±ΠΎΡ€Π°Ρ… Π²Β Π΄ΠΎΠΌΠ΅, Ρ‡Ρ‚ΠΎΒ ΡƒΠΏΡ€ΠΎΡ‰Π°Π΅Ρ‚ Ρ‚Π΅ΠΏΠ»ΠΎΠ²ΠΎΠΉ расчёт ΠΈΒ ΠΌΠΎΠ½Ρ‚Π°ΠΆ систСмы отоплСния.

Β 

НСкоторыС схСмы ΠΏΠΎΠ΄ΠΊΠ»ΡŽΡ‡Π΅Π½ΠΈΡ Ρ€Π°Π΄ΠΈΠ°Ρ‚ΠΎΡ€ΠΎΠ² отоплСния.

Β 

ΠžΠ΄Π½ΠΎΡΡ‚ΠΎΡ€ΠΎΠ½Π½Π΅Π΅ Π±ΠΎΠΊΠΎΠ²ΠΎΠ΅ ΠΏΠΎΠ΄ΠΊΠ»ΡŽΡ‡Π΅Π½ΠΈΠ΅ Ρ€Π°Π΄ΠΈΠ°Ρ‚ΠΎΡ€Π°.Β Β 

НаиболСС распространСнным способом Ρ€Π°Π·Π²ΠΎΠ΄ΠΊΠΈ Π±Π°Ρ‚Π°Ρ€Π΅ΠΉ считаСтся одностороннСС Π±ΠΎΠΊΠΎΠ²ΠΎΠ΅ ΠΏΠΎΠ΄ΠΊΠ»ΡŽΡ‡Π΅Π½ΠΈΠ΅. ΠŸΠΎΠ΄ΠΊΠ»ΡŽΡ‡Π΅Π½ΠΈΠ΅ подводящСй Ρ‚Ρ€ΡƒΠ±Ρ‹ производится Π²Β Π²Π΅Ρ€Ρ…Π½ΠΈΠΉ ΠΏΠ°Ρ‚Ρ€ΡƒΠ±ΠΎΠΊ, а отводящСй β€” Π²Β Π½ΠΈΠΆΠ½ΠΈΠΉ ΠΏΠ°Ρ‚Ρ€ΡƒΠ±ΠΎΠΊ. Это способствуСт ΠΏΠΎΠ»ΡƒΡ‡Π΅Π½ΠΈΡŽ максимальной Ρ‚Π΅ΠΏΠ»ΠΎΠΎΡ‚Π΄Π°Ρ‡ΠΈ.

Π”ΠΈΠ°Π³ΠΎΠ½Π°Π»ΡŒΠ½ΠΎΠ΅ Π±ΠΎΠΊΠΎΠ²ΠΎΠ΅ ΠΏΠΎΠ΄ΠΊΠ»ΡŽΡ‡Π΅Π½ΠΈΠ΅ Ρ€Π°Π΄ΠΈΠ°Ρ‚ΠΎΡ€Π°.Β Β 

ΠœΠ°ΠΊΡΠΈΠΌΠ°Π»ΡŒΠ½Ρ‹ΠΉ эффСкт ΠΎΡ‚Β Π΄ΠΈΠ°Π³ΠΎΠ½Π°Π»ΡŒΠ½ΠΎΠ³ΠΎ ΠΏΠΎΠ΄ΠΊΠ»ΡŽΡ‡Π΅Π½ΠΈΡ Π±Π°Ρ‚Π°Ρ€Π΅ΠΉ ΠΏΠΎΠ»ΡƒΡ‡Π°ΡŽΡ‚ в случаС установки Ρ€Π°Π΄ΠΈΠ°Ρ‚ΠΎΡ€ΠΎΠ² большой Π΄Π»ΠΈΠ½Ρ‹. Π­Ρ‚ΠΎΡ‚ способ Π΄Π°Π΅Ρ‚ Π²ΠΎΠ·ΠΌΠΎΠΆΠ½ΠΎΡΡ‚ΡŒ Ρ€Π°Π²Π½ΠΎΠΌΠ΅Ρ€Π½ΠΎΠ³ΠΎ ΠΏΡ€ΠΎΠ³Ρ€Π΅Π²Π° всСго Ρ€Π°Π΄ΠΈΠ°Ρ‚ΠΎΡ€Π°. ΠŸΡ€ΠΈΒ Π΄ΠΈΠ°Π³ΠΎΠ½Π°Π»ΡŒΠ½ΠΎΠΌ ΠΏΠΎΠ΄ΠΊΠ»ΡŽΡ‡Π΅Π½ΠΈΠΈ производится ΠΏΠΎΠ΄ΠΊΠ»ΡŽΡ‡Π΅Π½ΠΈΠ΅ ΠΏΠΈΡ‚Π°ΡŽΡ‰Π΅ΠΉ Ρ‚Ρ€ΡƒΠ±Ρ‹ ΠΊΒ Π²Π΅Ρ€Ρ…Π½Π΅ΠΌΡƒ ΠΏΠ°Ρ‚Ρ€ΡƒΠ±ΠΊΡƒ с одной стороны, а отводящСй β€” ΠΊΒ Π½ΠΈΠΆΠ½Π΅ΠΌΡƒ, Π½ΠΎΒ Π½Π°Β Π΄Ρ€ΡƒΠ³ΠΎΠΉ сторонС Π±Π°Ρ‚Π°Ρ€Π΅ΠΈ.

Β 

НиТнСС ΠΏΠΎΠ΄ΠΊΠ»ΡŽΡ‡Π΅Π½ΠΈΠ΅ Ρ€Π°Π΄ΠΈΠ°Ρ‚ΠΎΡ€Π° с боковой ΠΏΠΎΠ΄Π²ΠΎΠ΄ΠΊΠΎΠΉ тСплоноситСля.

.

  НиТний способ ΠΏΠΎΠ΄ΠΊΠ»ΡŽΡ‡Π΅Π½ΠΈΡ ΠΏΡ€ΠΈΠ΅ΠΌΠ»Π΅ΠΌ Ρ‚ΠΎΠ»ΡŒΠΊΠΎ в случаС, Ссли систСма отоплСния спрятана в пол или стСны здания. Π’Π΅ΠΏΠ»ΠΎΠΎΡ‚Π΄Π°Ρ‡Π° в этом случаС Π±ΡƒΠ΄Π΅Ρ‚ мСньшС, Ρ‡Π΅ΠΌΒ ΠΏΡ€ΠΈΒ Π±ΠΎΠΊΠΎΠ²ΠΎΠΉ схСмС ΠΏΠΎΠ΄ΠΊΠ»ΡŽΡ‡Π΅Π½ΠΈΡ ΠΈΒ ΠΏΠΎΡ‚Ρ€Π΅Π±ΡƒΠ΅Ρ‚ Ρ‚ΠΎΡ‡Π½ΠΎΠ³ΠΎ расчёта ΠΏΡ€ΠΈΠ±ΠΎΡ€Π°, а в случаС установки Π°Π»ΡŽΠΌΠΈΠ½ΠΈΠ΅Π²ΠΎΠ³ΠΎΒ ΠΈΠ»ΠΈΒ Π±ΠΈΠΌΠ΅Ρ‚Π°Π»Π»ΠΈΡ‡Π΅ΡΠΊΠΎΠ³ΠΎ Ρ€Π°Π΄ΠΈΠ°Ρ‚ΠΎΡ€Π° – установки Π΄ΠΎΠΏΠΎΠ»Π½ΠΈΡ‚Π΅Π»ΡŒΠ½Ρ‹Ρ… устройств, ΠΏΠ΅Ρ€Π΅Ρ€Π°ΡΠΏΡ€Π΅Π΄Π΅Π»ΡΡŽΡ‰ΠΈΡ… ΠΏΠΎΡ‚ΠΎΠΊ тСплоноситСля Π²Π½ΡƒΡ‚Ρ€ΠΈ Ρ€Π°Π΄ΠΈΠ°Ρ‚ΠΎΡ€Π°.

Β 

Π’ Π·Π°ΠΊΠ»ΡŽΡ‡Π΅Π½ΠΈΠ΅ слСдуСт Π΅Ρ‰Π΅Β Ρ€Π°Π·Β Π²ΡΠΏΠΎΠΌΠ½ΠΈΡ‚ΡŒ самыС Π²Π°ΠΆΠ½Ρ‹Π΅ ΠΏΡ€Π°Π²ΠΈΠ»Π° ΠΌΠΎΠ½Ρ‚Π°ΠΆΠ° Ρ€Π°Π΄ΠΈΠ°Ρ‚ΠΎΡ€ΠΎΠ²: установка нСпосрСдствСнно ΠΏΠΎΠ΄Β ΠΎΠΊΠ½Π°ΠΌΠΈ Π±ΡƒΠ΄Π΅Ρ‚ ΠΏΡ€Π΅ΠΏΡΡ‚ΡΡ‚Π²ΠΎΠ²Π°Ρ‚ΡŒ ΠΏΡ€ΠΎΠ½ΠΈΠΊΠ½ΠΎΠ²Π΅Π½ΠΈΡŽ Π²Β ΠΊΠΎΠΌΠ½Π°Ρ‚Ρƒ Ρ…ΠΎΠ»ΠΎΠ΄Π½ΠΎΠ³ΠΎ Π²ΠΎΠ·Π΄ΡƒΡ…Π°. ΠžΠΏΡ‚ΠΈΠΌΠ°Π»ΡŒΠ½ΡƒΡŽ Ρ‚Π΅ΠΏΠ»ΠΎΠΎΡ‚Π΄Π°Ρ‡Ρƒ ΠΌΠΎΠΆΠ½ΠΎ ΠΏΠΎΠ»ΡƒΡ‡ΠΈΡ‚ΡŒ ΠΏΡ€ΠΈΒ Π²Ρ‹ΠΏΠΎΠ»Π½Π΅Π½ΠΈΠΈ ΡΠ»Π΅Π΄ΡƒΡŽΡ‰ΠΈΡ… Ρ‚Ρ€Π΅Π±ΠΎΠ²Π°Π½ΠΈΠΉ к установкС: расстояниС ΠΎΡ‚Β Π±Π°Ρ‚Π°Ρ€Π΅ΠΈ Π΄ΠΎΒ ΠΏΠΎΠ»Π° Π½Π΅Β Π΄ΠΎΠ»ΠΆΠ½ΠΎ Π±Ρ‹Ρ‚ΡŒ мСньшС 10-12 см,Β Ρ€Π°Π΄ΠΈΠ°Ρ‚ΠΎΡ€ Π΄ΠΎΠ»ΠΆΠ΅Π½ Π±Ρ‹Ρ‚ΡŒ располоТСн Π²Β 3-5 см от стСны, вСрхняя Ρ‡Π°ΡΡ‚ΡŒ ниши или подоконника Π΄ΠΎΠ»ΠΆΠ½Ρ‹ Π±Ρ‹Ρ‚ΡŒ на расстоянии ΠΏΡ€ΠΈΠΌΠ΅Ρ€Π½ΠΎ 10 см от радиатора.

Β 

Как Π»Π΅Π³ΠΊΠΎ ΠΈ быстро ΡƒΠ²Π΅Π»ΠΈΡ‡ΠΈΡ‚ΡŒ Ρ‚Π΅ΠΏΠ»ΠΎΠΎΡ‚Π΄Π°Ρ‡Ρƒ Π΄ΠΎΠΌΠ°ΡˆΠ½ΠΈΡ… Π±Π°Ρ‚Π°Ρ€Π΅ΠΉ. 4 эффСктивных способа, ΠΏΠΎΠ·Π²ΠΎΠ»ΡΡŽΡ‰ΠΈΡ… ΠΏΠΎΠ²Ρ‹ΡΠΈΡ‚ΡŒ Ρ‚Π΅ΠΌΠΏΠ΅Ρ€Π°Ρ‚ΡƒΡ€Ρƒ Π² Π΄ΠΎΠΌΠ΅ | Π”Π°Ρ‡Π½Ρ‹ΠΉ Π‘Ρ‚Ρ€ΠΎΠΉΠ Π΅ΠΌΠΎΠ½Ρ‚

ΠžΡ‚ Ρ‚Π΅ΠΏΠ»ΠΎΠΎΡ‚Π΄Π°Ρ‡ΠΈ Ρ€Π°Π΄ΠΈΠ°Ρ‚ΠΎΡ€ΠΎΠ² зависит общая ΡΡ„Ρ„Π΅ΠΊΡ‚ΠΈΠ²Π½ΠΎΡΡ‚ΡŒ ΠΎΡ‚ΠΎΠΏΠΈΡ‚Π΅Π»ΡŒΠ½ΠΎΠΉ систСмы. Π’.Π΅. ΠΏΡ€ΠΈ Π²Ρ‹Π±ΠΎΡ€Π΅ элСмСнтов отоплСния, Π½ΡƒΠΆΠ½ΠΎ ΠΎΠ±ΡΠ·Π°Ρ‚Π΅Π»ΡŒΠ½ΠΎ ΠΎΠ±Ρ€Π°Ρ‰Π°Ρ‚ΡŒ Π²Π½ΠΈΠΌΠ°Π½ΠΈΠ΅ Π½Π° этот ΠΏΠ°Ρ€Π°ΠΌΠ΅Ρ‚Ρ€.

БлучаСтся, Ρ‡Ρ‚ΠΎ Ρ€Π°Π΄ΠΈΠ°Ρ‚ΠΎΡ€Ρ‹ ΠΊΡƒΠΏΠΈΠ»ΠΈ, ΡƒΠΆΠ΅ Π΄Π°ΠΆΠ΅ успСли ΠΈΡ… ΡƒΡΡ‚Π°Π½ΠΎΠ²ΠΈΡ‚ΡŒ, ΠΈ Ρ‚ΡƒΡ‚ ΡƒΠ·Π½Π°Π»ΠΈ, Ρ‡Ρ‚ΠΎ ΠΏΠΎ паспорту Ρƒ Π½ΠΈΡ… Π½Π΅ слишком большая Ρ‚Π΅ΠΏΠ»ΠΎΠΎΡ‚Π΄Π°Ρ‡Π°. НСприятно, Π½ΠΎ Π½Π΅ ΡΠΌΠ΅Ρ€Ρ‚Π΅Π»ΡŒΠ½ΠΎ. ΠžΠΊΠ°Π·Ρ‹Π²Π°Π΅Ρ‚ΡΡ, с этой ΠΏΡ€ΠΎΠ±Π»Π΅ΠΌΠΎΠΉ ΠΌΠΎΠΆΠ½ΠΎ ΡΠΏΡ€Π°Π²ΠΈΡ‚ΡŒΡΡ.

Π•ΡΡ‚ΡŒ нСсколько элСмСнтарных ΠΈ эффСктивных способов, ΠΊΠΎΡ‚ΠΎΡ€Ρ‹Π΅ ΠΏΠΎΠΌΠΎΠ³ΡƒΡ‚ Π·Π½Π°Ρ‡ΠΈΡ‚Π΅Π»ΡŒΠ½ΠΎ ΡƒΠ²Π΅Π»ΠΈΡ‡ΠΈΡ‚ΡŒ Ρ‚Π΅ΠΏΠ»ΠΎΠΎΡ‚Π΄Π°Ρ‡Ρƒ Ρ€Π°Π΄ΠΈΠ°Ρ‚ΠΎΡ€ΠΎΠ². Плюс Π² Ρ‚ΠΎΠΌ, Ρ‡Ρ‚ΠΎ способы эти Π½Π΅ Π·Π°Ρ‚Ρ€Π°Ρ‚Π½Ρ‹Π΅.

ΠŸΡ€Π΅Π΄Π»Π°Π³Π°ΡŽ Ρ€Π°Π·Π»ΠΎΠΆΠΈΡ‚ΡŒ всё Β«ΠΏΠΎ ΠΏΠΎΠ»ΠΎΡ‡ΠΊΠ°ΠΌΒ»:

1. Π’ΠΎΠ·Π΄ΡƒΡˆΠ½Π°Ρ конвСкция, Ρ‚.Π΅. циркуляция

Π˜ΡΡ…ΠΎΠ΄Ρ ΠΈΠ· Π»ΠΈΡ‡Π½ΠΎΠ³ΠΎ ΠΎΠΏΡ‹Ρ‚Π°, ΠΌΠΎΠ³Ρƒ ΡΠΊΠ°Π·Π°Ρ‚ΡŒ, Ρ‡Ρ‚ΠΎ ΠΈΠ½ΠΎΠ³Π΄Π° Π½ΠΈΠ·ΠΊΠΈΠ΅ ΠΏΠΎΠΊΠ°Π·Π°Ρ‚Π΅Π»ΠΈ Ρ‚Π΅ΠΏΠ»ΠΎΠΎΡ‚Π΄Π°Ρ‡ΠΈ Π² ΠΊΠ²Π°Ρ€Ρ‚ΠΈΡ€Π΅ ΠΎΠ±ΡƒΡΠ»Π°Π²Π»ΠΈΠ²Π°ΡŽΡ‚ΡΡ чСрСсчур ΠΏΠ»ΠΎΡ‚Π½Ρ‹ΠΌΠΈ ΡˆΡ‚ΠΎΡ€Π°ΠΌΠΈ ΠΈ Π½Π°Π»ΠΈΡ‡ΠΈΠ΅ΠΌ Π΄Π΅ΠΊΠΎΡ€Π°Ρ‚ΠΈΠ²Π½Ρ‹Ρ… Ρ‰ΠΈΡ‚ΠΎΠ². НСкоторыС ΠΏΡ€Π΅Π΄ΠΏΠΎΡ‡ΠΈΡ‚Π°ΡŽΡ‚ Π½Π΅ ΠΏΠΎΡ€Ρ‚ΠΈΡ‚ΡŒ внСшний Π²ΠΈΠ΄ помСщСния батарСями ΠΈ прячут ΠΈΡ… Π² красивыС Β«ΠΊΠΎΡ€ΠΎΠ±ΠΎΡ‡ΠΊΠΈΒ». Π’ Ρ€Π΅Π·ΡƒΠ»ΡŒΡ‚Π°Ρ‚Π΅ конвСкция Π½Π°Ρ€ΡƒΡˆΠ°Π΅Ρ‚ΡΡ, ΠΏΠΎΡ‚ΠΎΠΊΠΈ Π²ΠΎΠ·Π΄ΡƒΡ…Π° проходят Ρ‡Π΅Ρ€Π΅Π· ΠΏΡ€ΠΎΠ»Ρ‘Ρ‚Ρ‹ Ρ€Π°Π΄ΠΈΠ°Ρ‚ΠΎΡ€Π°, Π½ΠΎ Π½Π΅ Π²ΠΎΠ·Π²Ρ€Π°Ρ‰Π°ΡŽΡ‚ΡΡ Π² ΠΊΠΎΠΌΠ½Π°Ρ‚Ρƒ Π² Π½Π΅ΠΎΠ±Ρ…ΠΎΠ΄ΠΈΠΌΠΎΠΌ количСствС.

К Π°Π½Π°Π»ΠΎΠ³ΠΈΡ‡Π½Ρ‹ΠΌ ΠΏΡ€ΠΎΠ±Π»Π΅ΠΌΠ°ΠΌ я отнСсу слишком ΡˆΠΈΡ€ΠΎΠΊΠΈΠ΅ ΠΏΠΎΠ΄ΠΎΠΊΠΎΠ½Π½ΠΈΠΊΠΈ ΠΈ ΡΠΏΠ΅Ρ†ΠΈΠ°Π»ΡŒΠ½Ρ‹Π΅ ΡƒΠ³Π»ΡƒΠ±Π»Ρ‘Π½Π½Ρ‹Π΅ ниши Π² стСнС, Π² ΠΊΠΎΡ‚ΠΎΡ€Ρ‹Π΅ Π°ΠΊΠΊΡƒΡ€Π°Ρ‚Π½ΠΎ ΡƒΠΊΠ»Π°Π΄Ρ‹Π²Π°ΡŽΡ‚ Π±Π°Ρ‚Π°Ρ€Π΅ΠΈ.

ΠŸΠΎΠ»ΡƒΡ‡Π°Π΅Ρ‚ΡΡ, Π½ΡƒΠΆΠ½ΠΎ Π²Ρ‹Π±Ρ€Π°Ρ‚ΡŒ, Ρ‡Ρ‚ΠΎ для вас Π²Π°ΠΆΠ½Π΅Π΅: красота ΠΈΠ»ΠΈ Ρ‚Π΅ΠΏΠ»ΠΎ. Если ΠΏΠ΅Ρ€Π²ΠΎΠ΅, Ρ‚ΠΎ смСло маскируйтС Ρ€Π°Π΄ΠΈΠ°Ρ‚ΠΎΡ€Ρ‹, Ρ‚ΠΎΠ»ΡŒΠΊΠΎ Π½Π΅ Π·Π°Π±Ρ‹Π²Π°ΠΉΡ‚Π΅ Ρ‚Π΅ΠΏΠ»Π΅Π΅ ΠΎΠ΄Π΅Π²Π°Ρ‚ΡŒΡΡ Π² ΠΊΠ²Π°Ρ€Ρ‚ΠΈΡ€Π΅. Если Π²Ρ‚ΠΎΡ€ΠΎΠ΅ – ΡΠ²Ρ‹ΠΊΠ°ΠΉΡ‚Π΅ΡΡŒ с ΠΌΡ‹ΡΠ»ΡŒΡŽ, Ρ‡Ρ‚ΠΎ Π±Π΅Π»Ρ‹Π΅ Π±Π°Ρ‚Π°Ρ€Π΅ΠΈ – это Π½Π΅ Ρ‚Π°ΠΊ ΡƒΠΆ старомодно ΠΈ нСкрасиво.

2. Π’Π΅Π»ΠΈΡ‡ΠΈΠ½Π° отступов Ρ€Π°Π΄ΠΈΠ°Ρ‚ΠΎΡ€Π°

ΠŸΠΎΠΊΡƒΠΏΠ°Ρ Ρ€Π°Π΄ΠΈΠ°Ρ‚ΠΎΡ€, Π½Π΅ Π½ΡƒΠΆΠ½ΠΎ сразу Π²Ρ‹ΠΊΠΈΠ΄Ρ‹Π²Π°Ρ‚ΡŒ ΠΈΠ½ΡΡ‚Ρ€ΡƒΠΊΡ†ΠΈΡŽ, ΠΊΠ°ΠΊ это Π΄Π΅Π»Π°ΡŽΡ‚ ΠΌΠ½ΠΎΠ³ΠΈΠ΅. Π’ Π½Π΅ΠΉ ΡƒΠΊΠ°Π·Π°Π½Ρ‹ тСхничСскиС трСбования ΠΏΠΎ ΠΌΠΎΠ½Ρ‚Π°ΠΆΡƒ элСмСнтов ΠΎΡ‚ΠΎΠΏΠΈΡ‚Π΅Π»ΡŒΠ½ΠΎΠΉ систСмы: отступы ΠΎΡ‚ ΠΏΠΎΠ»Π°, стСны ΠΈ ΠΏΠΎΠ΄ΠΎΠΊΠΎΠ½Π½ΠΈΠΊΠ°, ΠΊΠΎΡ‚ΠΎΡ€Ρ‹Π΅ Π½Π΅ΠΎΠ±Ρ…ΠΎΠ΄ΠΈΠΌΠΎ ΡΠΎΠ±Π»ΡŽΠ΄Π°Ρ‚ΡŒ.

Π˜Π½ΡΡ‚Ρ€ΡƒΠΊΡ†ΠΈΡ гласит, Ρ‡Ρ‚ΠΎ ΠΏΡ€ΠΈ установкС Ρ€Π°Π΄ΠΈΠ°Ρ‚ΠΎΡ€Π°, Π½ΡƒΠΆΠ½ΠΎ ΠΎΡ‚ΡΡ‚ΡƒΠΏΠ°Ρ‚ΡŒ ΠΎΡ‚ стСны ΠΌΠΈΠ½ΠΈΠΌΡƒΠΌ 50 ΠΌΠΈΠ»Π»ΠΈΠΌΠ΅Ρ‚Ρ€ΠΎΠ². ИмСнно Ρ‚Π°ΠΊΠΎΠ΅ расстояниС ΠΏΠΎΠΌΠΎΠ³Π°Π΅Ρ‚ Π²ΠΎΠ·Π΄ΡƒΡˆΠ½Ρ‹ΠΌ ΠΏΠΎΡ‚ΠΎΠΊΠ°ΠΌ свободно ΠΏΡ€ΠΎΠ½ΠΈΠΊΠ°Ρ‚ΡŒ Π² отсСки Π±Π°Ρ‚Π°Ρ€Π΅ΠΈ, Π° Π·Π°Ρ‚Π΅ΠΌ Π²ΠΎΠ·Π²Ρ€Π°Ρ‰Π°Ρ‚ΡŒΡΡ Π² ΠΏΠΎΠΌΠ΅Ρ‰Π΅Π½ΠΈΠ΅, прогрСвая Π΅Π³ΠΎ. ΠžΡ‚ΡΡ‚ΡƒΠΏ Ρ€Π°Π΄ΠΈΠ°Ρ‚ΠΎΡ€Π° ΠΎΡ‚ ΠΏΠΎΠ»Π° ΠΈ ΠΏΠΎΠ΄ΠΎΠΊΠΎΠ½Π½ΠΈΠΊΠ° Ρ‚Π°ΠΊΠΆΠ΅ Π½Π΅ Π΄ΠΎΠ»ΠΆΠ΅Π½ Π±Ρ‹Ρ‚ΡŒ мСньшС 50 ΠΌΠΈΠ»Π»ΠΈΠΌΠ΅Ρ‚Ρ€ΠΎΠ².

ДопустимыС отступы Ρ€Π°Π΄ΠΈΠ°Ρ‚ΠΎΡ€ΠΎΠ²

ДопустимыС отступы Ρ€Π°Π΄ΠΈΠ°Ρ‚ΠΎΡ€ΠΎΠ²

Когда ΠΏΠΎΠ΄ΠΎΠΊΠΎΠ½Π½ΠΈΠΊ ΡˆΠΈΡ€Π΅ Ρ€Π°Π΄ΠΈΠ°Ρ‚ΠΎΡ€Π°, Π½ΡƒΠΆΠ½ΠΎ ΠΎΠΏΡƒΡΡ‚ΠΈΡ‚ΡŒ Π±Π°Ρ‚Π°Ρ€Π΅ΡŽ Π½ΠΈΠΆΠ΅ ΠΈΠ· расчёта: Π½Π° 1 сантимСтр выступа ΠΏΠΎΠ΄ΠΎΠΊΠΎΠ½Π½ΠΈΠΊΠ° – 2,5 сантимСтра отступа. Π’ ΠΏΡ€ΠΎΡ‚ΠΈΠ²Π½ΠΎΠΌ случаС циркуляция Π±ΡƒΠ΄Π΅Ρ‚ Π½Π°Ρ€ΡƒΡˆΠ΅Π½Π°.

3. Π­ΠΊΡ€Π°Π½Ρ‹ ΠΈΠ· Ρ„ΠΎΠ»ΡŒΠ³ΠΈ, ΠΎΡ‚Ρ€Π°ΠΆΠ°ΡŽΡ‰ΠΈΠ΅ Ρ‚Π΅ΠΏΠ»ΠΎ

Π€ΠΎΠ»ΡŒΠ³ΠΈΡ€ΠΎΠ²Π°Π½Π½Ρ‹Π΅ экраны способны Π² нСсколько Ρ€Π°Π· ΡƒΠ²Π΅Π»ΠΈΡ‡ΠΈΡ‚ΡŒ ΠšΠŸΠ” всСй ΠΎΡ‚ΠΎΠΏΠΈΡ‚Π΅Π»ΡŒΠ½ΠΎΠΉ систСмы. ΠšΠΎΠ½ΡΡ‚Ρ€ΡƒΠΊΡ†ΠΈΡ ΠΈΠ³Ρ€Π°Π΅Ρ‚ Ρ€ΠΎΠ»ΡŒ своСобразного Π±Π°Ρ€ΡŒΠ΅Ρ€Π°, установлСнного ΠΌΠ΅ΠΆΠ΄Ρƒ Ρ€Π°Π΄ΠΈΠ°Ρ‚ΠΎΡ€ΠΎΠΌ ΠΈ Ρ…ΠΎΠ»ΠΎΠ΄Π½ΠΎΠΉ стСнкой. Благодаря установкС Ρ‚Π°ΠΊΠΈΡ… экранов, общая Ρ‚Π΅ΠΌΠΏΠ΅Ρ€Π°Ρ‚ΡƒΡ€Π° Π² ΠΊΠΎΠΌΠ½Π°Ρ‚Π΅ ΠΌΠΎΠΆΠ΅Ρ‚ ΠΏΠΎΠ²Ρ‹ΡΠΈΡ‚ΡŒΡΡ Π½Π° 2 – 3 градуса.

4. ИзмСнСниС способа ΠΏΠΎΠ΄ΠΊΠ»ΡŽΡ‡Π΅Π½ΠΈΡ Ρ€Π°Π΄ΠΈΠ°Ρ‚ΠΎΡ€ΠΎΠ²

Π‘ΡƒΡ‰Π΅ΡΡ‚Π²ΡƒΡŽΡ‚ Ρ€Π°Π·Π½ΠΎΠΎΠ±Ρ€Π°Π·Π½Ρ‹Π΅ Π²Π°Ρ€ΠΈΠ°Π½Ρ‚Ρ‹ Ρ€Π°Π΄ΠΈΠ°Ρ‚ΠΎΡ€ΠΎΠ². БСгодня ΠΌΠΎΠΆΠ½ΠΎ ΡƒΡΡ‚Π°Π½ΠΎΠ²ΠΈΡ‚ΡŒ Π±Π°Ρ‚Π°Ρ€Π΅ΠΈ Π±ΠΎΠΊΠΎΠ²Ρ‹ΠΌ, Π΄ΠΈΠ°Π³ΠΎΠ½Π°Π»ΡŒΠ½Ρ‹ΠΌ Π»ΠΈΠ±ΠΎ Π½ΠΈΠΆΠ½ΠΈΠΌ способом.

Π§Ρ‚ΠΎΠ±Ρ‹ Π΄ΠΎΠ±ΠΈΡ‚ΡŒΡΡ Π»ΡƒΡ‡ΡˆΠ΅ΠΉ Ρ‚Π΅ΠΏΠ»ΠΎΠΎΡ‚Π΄Π°Ρ‡ΠΈ, слСдуСт ΠΏΠΎΠ΄ΠΊΠ»ΡŽΡ‡Π°Ρ‚ΡŒ Ρ€Π°Π΄ΠΈΠ°Ρ‚ΠΎΡ€Ρ‹ Π΄ΠΈΠ°Π³ΠΎΠ½Π°Π»ΡŒΠ½Ρ‹ΠΌ способом. Π’ Ρ‚Π°ΠΊΠΎΠΌ случаС Ρ‚Π΅ΠΏΠ»ΠΎΠ½ΠΎΡΠΈΡ‚Π΅Π»ΡŒ Π±ΡƒΠ΄Π΅Ρ‚ Ρ€Π°ΡΡ‚Π΅ΠΊΠ°Ρ‚ΡŒΡΡ ΠΏΠΎ всСм сСкциям Ρ€Π°Π²Π½ΠΎΠΌΠ΅Ρ€Π½ΠΎ. ΠšΡ€ΠΎΠΌΠ΅ Ρ‚ΠΎΠ³ΠΎ, ΠΏΡ€ΠΈ диагональном ΠΏΠΎΠ΄ΠΊΠ»ΡŽΡ‡Π΅Π½ΠΈΠΈ, Π² Ρ€Π°Π΄ΠΈΠ°Ρ‚ΠΎΡ€Π°Ρ… практичСски ΠΎΡ‚ΡΡƒΡ‚ΡΡ‚Π²ΡƒΡŽΡ‚ Π·ΠΎΠ½Ρ‹, Π³Π΄Π΅ ΡΠΊΠ°ΠΏΠ»ΠΈΠ²Π°ΡŽΡ‚ΡΡ загрязнСния.

Из всСх способов ΠΏΠΎΠ΄ΠΊΠ»ΡŽΡ‡Π΅Π½ΠΈΡ ΠΏΡ€Π΅Π΄ΠΏΠΎΡ‡Ρ‚Π΅Π½ΠΈΠ΅ слСдуСт ΠΎΡ‚Π΄Π°Π²Π°Ρ‚ΡŒ Π΄ΠΈΠ°Π³ΠΎΠ½Π°Π»ΡŒΠ½ΠΎΠΌΡƒ.

Π’ΠΎΡΠΏΠΎΠ»ΡŒΠ·ΠΎΠ²Π°Π²ΡˆΠΈΡΡŒ этими совСтами, Π²Ρ‹ смоТСтС ΠΏΠΎΠ²Ρ‹ΡΠΈΡ‚ΡŒ Ρ‚Π΅ΠΏΠ»ΠΎΠΎΡ‚Π΄Π°Ρ‡Ρƒ своих Π±Π°Ρ‚Π°Ρ€Π΅ΠΉ ΠΈ ΡƒΠ»ΡƒΡ‡ΡˆΠΈΡ‚ΡŒ Ρ€Π°Π±ΠΎΡ‚Ρƒ ΠΎΡ‚ΠΎΠΏΠΈΡ‚Π΅Π»ΡŒΠ½ΠΎΠΉ систСмы.

Π£Π²Π°ΠΆΠ°Π΅ΠΌΡ‹ΠΉ Ρ‡ΠΈΡ‚Π°Ρ‚Π΅Π»ΡŒ! НадСюсь, Ρ‡Ρ‚ΠΎ данная ΡΡ‚Π°Ρ‚ΡŒΡ Π±Ρ‹Π»Π° Π²Π°ΠΌ интСрСсна ΠΈ ΠΏΠΎΠ»Π΅Π·Π½Π°.

Если Π½Π΅ слоТно, ΠΎΡ‚Π±Π»Π°Π³ΠΎΠ΄Π°Ρ€ΠΈΡ‚Π΅, поТалуйста Π»Π°ΠΉΠΊΠΎΠΌ πŸ‘ ΠΈ подпиской Π½Π° ΠΊΠ°Π½Π°Π».

О Ρ€Π°Π΄ΠΈΠ°Ρ‚ΠΎΡ€Π°Ρ…, Π±ΠΎΠΉΠ»Π΅Ρ€Π°Ρ… ΠΈ Ρ‚Π΅ΠΏΠ»Ρ‹Ρ… ΠΏΠΎΠ»Π°Ρ…

Π¨Π°Π³ Π²Ρ‚ΠΎΡ€ΠΎΠΉ: Π’Ρ‹Π±ΠΎΡ€ Ρ€Π°Π΄ΠΈΠ°Ρ‚ΠΎΡ€ΠΎΠ²

Бамая близкая ΠΊ Π½Π°ΠΌ Ρ‡Π°ΡΡ‚ΡŒ ΠΎΡ‚ΠΎΠΏΠΈΡ‚Π΅Π»ΡŒΠ½ΠΎΠΉ систСмы, ΠΊΠΎΡ‚ΠΎΡ€ΡƒΡŽ ΠΌΡ‹ Π²ΠΈΠ΄ΠΈΠΌ Π΅ΠΆΠ΅Π΄Π½Π΅Π²Π½ΠΎ – это ΠΎΡ‚ΠΎΠΏΠΈΡ‚Π΅Π»ΡŒΠ½Ρ‹Π΅ ΠΏΡ€ΠΈΠ±ΠΎΡ€Ρ‹ Ρ€Π°Π΄ΠΈΠ°Ρ‚ΠΎΡ€Ρ‹ (ΠΈΠ»ΠΈ попросту Π±Π°Ρ‚Π°Ρ€Π΅ΠΈ ). ИмСнно с ΠΈΡ… Π²Ρ‹Π±ΠΎΡ€Π° ΠΈ размСщСния начинаСтся созданиС ΠΏΡ€ΠΎΠ΅ΠΊΡ‚Π° ΠΈ ΠΌΠΎΠ½Ρ‚Π°ΠΆ ΠΎΡ‚ΠΎΠΏΠΈΡ‚Π΅Π»ΡŒΠ½ΠΎΠΉ систСмы. ВсС ΠΎΡ‚ΠΎΠΏΠΈΡ‚Π΅Π»ΡŒΠ½Ρ‹Π΅ ΠΏΡ€ΠΈΠ±ΠΎΡ€Ρ‹ ΠΌΠΎΠΆΠ½ΠΎ условно Ρ€Π°Π·Π΄Π΅Π»ΠΈΡ‚ΡŒ Π½Π° Ρ€Π°Π΄ΠΈΠ°Ρ‚ΠΎΡ€Ρ‹ ΠΈ ΠΊΠΎΠ½Π²Π΅ΠΊΡ‚ΠΎΡ€Ρ‹:

  • Π Π°Π΄ΠΈΠ°Ρ‚ΠΎΡ€Ρ‹ — ΠΏΠΎ своСй конструкции ΠΈΠΌΠ΅ΡŽΡ‚ ΠΎΡ‚Π½ΠΎΡΠΈΡ‚Π΅Π»ΡŒΠ½ΠΎ большой объСм ΠΈ постоянно содСрТат ΠΌΠ½ΠΎΠ³ΠΎ горячСго тСплоноситСля. Π—Π° счСт этого ΠΎΠ½ΠΈ ΠΎΡ‚Π΄Π°ΡŽΡ‚ Ρ‚Π΅ΠΏΠ»ΠΎ прСимущСствСнно Π² Π²ΠΈΠ΄Π΅ излучСния (ΠΊΠ°ΠΌΠΈΠ½Π½Ρ‹ΠΉ эффСкт). К Π½ΠΈΠΌ ΠΌΠΎΠΆΠ½ΠΎ отнСсти ΠΏΡ€ΠΈΠ²Ρ‹Ρ‡Π½Ρ‹Π΅ Π½Π°ΠΌ Ρ‡ΡƒΠ³ΡƒΠ½Π½Ρ‹Π΅, Π°Π»Π»ΡŽΠΌΠΈΠ½ΠΈΠ΅Π²Ρ‹Π΅ ΠΈ ΡΡ‚Π°Π»ΡŒΠ½Ρ‹Π΅ Ρ€Π°Π΄ΠΈΠ°Ρ‚ΠΎΡ€Ρ‹.
  • ΠšΠΎΠ½Π²Π΅ΠΊΡ‚ΠΎΡ€Ρ‹ — ΠΎΡ‚Π΄Π°ΡŽΡ‚ Ρ‚Π΅ΠΏΠ»ΠΎ Π² основном Π·Π° счСт циркуляции Π²ΠΎΠ·Π΄ΡƒΡ…Π° Ρ‡Π΅Ρ€Π΅Π· Π½ΠΈΡ…. По Ρ‚Ρ€ΡƒΠ±Π΅ ΠΊΠΎΠ½Π²Π΅ΠΊΡ‚ΠΎΡ€Π° двиТСтся Ρ‚Π΅ΠΏΠ»ΠΎΠ½ΠΎΡΠΈΡ‚Π΅Π»ΡŒ, нагрСвая повСрхности «Π½Π°Π΄Π΅Ρ‚ΠΎΠΉ» Π½Π° Π½Π΅Π³ΠΎ «Π³Π°Ρ€ΠΌΠΎΡˆΠΊΠΈ». Π’ΠΎΠ·Π΄ΡƒΡ… ΠΏΡ€ΠΎΡ…ΠΎΠ΄ΠΈΡ‚ сквозь ΠΊΠΎΠ½Π²Π΅ΠΊΡ‚ΠΎΡ€ снизу Π²Π²Π΅Ρ€Ρ…, Π½Π°Π³Ρ€Π΅Π²Π°ΡΡΡŒ ΠΎΡ‚ многочислСнных Ρ‚Π΅ΠΏΠ»Ρ‹Ρ… повСрхностСй. ΠšΠΎΠ½Π²Π΅ΠΊΡ‚ΠΎΡ€Π° ΠΈΠΌΠ΅ΡŽΡ‚ Ρ€Π°Π·Π½Ρ‹Π΅ Ρ„ΠΎΡ€ΠΌΡ‹ ΠΈ ΠΌΠΎΠ³ΡƒΡ‚ Ρ‚Π°ΠΊΠΆΠ΅ Ρ€Π°Π·ΠΌΠ΅Ρ‰Π°Ρ‚ΡŒΡΡ Π½Π° стСнС Π° Ρ‚Π°ΠΊΠΆΠ΅ Π±Ρ‹Ρ‚ΡŒ встраиваСмыС Π² ΠΏΠΎΠ», Ρ‡Ρ‚ΠΎ ΡƒΠ΄ΠΎΠ±Π½ΠΎ, Π½Π°ΠΏΡ€ΠΈΠΌΠ΅Ρ€ ΠΏΡ€ΠΈ Π±ΠΎΠ»ΡŒΡˆΠΈΡ… Π²ΠΈΡ‚Ρ€Π°ΠΆΠ½Ρ‹Ρ… ΠΎΠΊΠ½Π°Ρ….

А Ρ‚Π°ΠΊΠΆΠ΅ ΡΡƒΡ‰Π΅ΡΡ‚Π²ΡƒΡŽΡ‚ ΠΎΡ‚ΠΎΠΏΠΈΡ‚Π΅Π»ΡŒΠ½Ρ‹Π΅ ΠΏΡ€ΠΈΠ±ΠΎΡ€Ρ‹, ΡΠΎΠ΅Π΄ΠΈΠ½ΡΡŽΡ‰ΠΈΠ΅ Π² сСбС свойства Ρ€Π°Π΄ΠΈΠ°Ρ‚ΠΎΡ€ΠΎΠ² ΠΈ ΠΊΠΎΠ½Π²Π΅ΠΊΡ‚ΠΎΡ€ΠΎΠ² (это ΡΡ‚Π°Π»ΡŒΠ½Ρ‹Π΅ ΠΏΠ°Π½Π΅Π»ΡŒΠ½Ρ‹Π΅ Ρ€Π°Π΄ΠΈΠ°Ρ‚ΠΎΡ€Ρ‹ Ρ‚ΠΈΠΏΠ° Buderus, Kermi ΠΈ Ρ‚.Π΄.), Π² ΠΈΡ… плоскиС Π½Π°ΠΊΠΎΠΏΠΈΡ‚Π΅Π»ΡŒΠ½Ρ‹Π΅ ΠΏΠ°Π½Π΅Π»ΠΈ поступаСт большая масса Ρ‚Π΅ΠΏΠ»ΠΎΠΉ Π²ΠΎΠ΄Ρ‹ ΠΈ, Π² Ρ‚ΠΎ ΠΆΠ΅ врСмя, Π²Π½ΡƒΡ‚Ρ€ΠΈ Ρƒ Π½ΠΈΡ… Π΅ΡΡ‚ΡŒ рСбристыС ΠΊΠΎΠ½Π²Π΅ΠΊΡ†ΠΈΠΎΠ½Π½Ρ‹Π΅ повСрхности. Π’Π°ΠΊΠΈΠ΅ Ρ€Π°Π΄ΠΈΠ°Ρ‚ΠΎΡ€Ρ‹ ΠΎΠ±Π»Π°Π΄Π°ΡŽΡ‚ самыми Π»ΡƒΡ‡ΡˆΠΈΠΌΠΈ показатСлями ΠšΠŸΠ” , ΠΏΠΎΡ‚ΠΎΠΌΡƒ Ρ‡Ρ‚ΠΎ Π² Π½ΠΈΡ… ΡΠΎΡ‡Π΅Ρ‚Π°ΡŽΡ‚ΡΡ ΠΎΠ±Π° Π²Π°Ρ€ΠΈΠ°Π½Ρ‚Π° Ρ‚Π΅ΠΏΠ»ΠΎΠΎΡ‚Π΄Π°Ρ‡ΠΈ — ΠΈΠ·Π»ΡƒΡ‡Π΅Π½ΠΈΠ΅ ΠΈ конвСкция.

Β 

КакиС Π±Ρ‹Π²Π°ΡŽΡ‚ Ρ€Π°Π΄ΠΈΠ°Ρ‚ΠΎΡ€Ρ‹?

Π Π°Π΄ΠΈΠ°Ρ‚ΠΎΡ€Ρ‹ Π±Ρ‹Π²Π°ΡŽΡ‚ Ρ‡ΡƒΠ³ΡƒΠ½Π½Ρ‹Π΅, Π°Π»ΡŽΠΌΠΈΠ½ΠΈΠ΅Π²Ρ‹Π΅, ΡΡ‚Π°Π»ΡŒΠ½Ρ‹Π΅Β  ΠΏΠ°Π½Π΅Π»ΡŒΠ½Ρ‹Π΅ (ΡˆΡ‚Π°ΠΌΠΏΠΎΠ²Π°Π½Π½Ρ‹Π΅)ΠΈ, Ρ‚Π°ΠΊ Π½Π°Π·Ρ‹Π²Π°Π΅ΠΌΡ‹Π΅, бимСталличСскиС.

  • Π§ΡƒΠ³ΡƒΠ½Π½Ρ‹Π΅ — Ρ…ΠΎΡ€ΠΎΡˆΠΎ ΠΎΡ‚Π΄Π°ΡŽΡ‚ Ρ‚Π΅ΠΏΠ»ΠΎ ΠΈ ΡΠΎΠΏΡ€ΠΎΡ‚ΠΈΠ²Π»ΡΡŽΡ‚ΡΡ Ρ€ΠΆΠ°Π²Ρ‡ΠΈΠ½Π΅, ΠΌΠΎΠ³ΡƒΡ‚ Π²Ρ‹Π΄Π΅Ρ€ΠΆΠΈΠ²Π°Ρ‚ΡŒ довольно высокоС Π΄Π°Π²Π»Π΅Π½ΠΈΠ΅ Π² систСмС, Π½ΠΎ ΠΎΠ½ΠΈ тяТСлыС ΠΈ Π½Π΅ всСгда ΡΠΎΠΎΡ‚Π²Π΅Ρ‚ΡΡ‚Π²ΡƒΡŽΡ‚ соврСмСнным трСбованиям Π΄ΠΈΠ·Π°ΠΉΠ½Π°, хотя Π±Ρ‹Π²Π°ΡŽΡ‚ ΠΈ ΠΎΡ‡Π΅Π½ΡŒ красивыС, Ρ‚Π°ΠΊ Π½Π°Π·Ρ‹Π²Π°Π΅ΠΌΡ‹Π΅ Ρ€Π΅Ρ‚Ρ€ΠΎ-Ρ€Π°Π΄ΠΈΠ°Ρ‚ΠΎΡ€Ρ‹
  • ΠΠ»ΡŽΠΌΠΈΠ½ΠΈΠ΅Π²Ρ‹Π΅ , сСкционныС- Π»Π΅Π³ΠΊΠΈΠ΅, ΠΎΠ±Π»Π°Π΄Π°ΡŽΡ‚ высокой Ρ‚Π΅ΠΏΠ»ΠΎΠΎΡ‚Π΄Π°Ρ‡Π΅ΠΉ, красивы, Π½ΠΎ качСствСнныС довольно Π΄ΠΎΡ€ΠΎΠ³ΠΈ ΠΈ ΠΈΠ½ΠΎΠ³Π΄Π° Π½Π΅ Π²Ρ‹Π΄Π΅Ρ€ΠΆΠΈΠ²Π°ΡŽΡ‚ высокого давлСния Π² систСмС. Π’Π°ΠΊΠΆΠ΅ ΠΈΠ· минусов Ρ‚ΠΎ, Ρ‡Ρ‚ΠΎ сСкции Ρ‚Π°ΠΊΠΈΡ… Ρ€Π°Π΄ΠΈΠ°Ρ‚ΠΎΡ€ΠΎΠ² собраны Ρ‡Π΅Ρ€Π΅Π· ΠΏΡ€ΠΎΠΊΠ»Π°Π΄ΠΊΠΈ, ΠΊΠΎΡ‚ΠΎΡ€Ρ‹Π΅ ΠΈΠ½ΠΎΠ³Π΄Π° Π΄Π°ΡŽΡ‚ Ρ‚Π΅Ρ‡ΡŒ.
  • БимСталличСскиС — состоят ΠΈΠ· ΡΡ‚Π°Π»ΡŒΠ½ΠΎΠΉ Ρ‚Ρ€ΡƒΠ±Ρ‹, ΠΏΠΎ ΠΊΠΎΡ‚ΠΎΡ€ΠΎΠΉ Π΄ΠΎΠ»ΠΆΠ΅Π½ Π΄Π²ΠΈΠ³Π°Ρ‚ΡŒΡΡ Ρ‚Π΅ΠΏΠ»ΠΎΠ½ΠΎΡΠΈΡ‚Π΅Π»ΡŒ, ΠΈ алюминиСвого корпуса. Π‘Ρ‚Π°Π»ΡŒΠ½Π°Ρ Ρ‚Ρ€ΡƒΠ±Π° Π²Ρ‹Π΄Π΅Ρ€ΠΆΠΈΠ²Π°Π΅Ρ‚ высокоС Π΄Π°Π²Π»Π΅Π½ΠΈΠ΅, Π° Π°Π»ΡŽΠΌΠΈΠ½ΠΈΠ΅Π²Ρ‹Π΅ сСкции Π»Π΅Π³ΠΊΠΎ ΠΎΡ‚Π΄Π°ΡŽΡ‚ Ρ‚Π΅ΠΏΠ»ΠΎ. Π’Π°ΠΊΠΈΠ΅ Ρ€Π°Π΄ΠΈΠ°Ρ‚ΠΎΡ€Ρ‹ появились Π½Π΅Π΄Π°Π²Π½ΠΎ, внСшнС ΠΎΠ½ΠΈ сильно походят Π½Π° сСкционныС Π°Π»ΡŽΠΌΠΈΠ½ΠΈΠ΅Π²Ρ‹Π΅, Π½ΠΎ ΠΏΠΎ Ρ†Π΅Π½Π΅ Π΄ΠΎΡ€ΠΎΠΆΠ΅. Π₯ΠΎΡ€ΠΎΡˆΠΎ Π²Ρ‹Π΄Π΅Ρ€ΠΆΠΈΠ²Π°ΡŽΡ‚ высокоС Π΄Π°Π²Π»Π΅Π½ΠΈΠ΅.
  • Π‘Ρ‚Π°Π»ΡŒΠ½Ρ‹Π΅ ΠΏΠ°Π½Π΅Π»ΡŒΠ½Ρ‹Π΅ Β ΠΈΠ»ΠΈ ΡˆΡ‚Π°ΠΌΠΏΠΎΠ²Π°Π½Π½Ρ‹Π΅ — ΠΎΠΏΡ‚ΠΈΠΌΠ°Π»ΡŒΠ½Ρ‹ ΠΏΠΎ Ρ†Π΅Π½Π΅, ΠΎΠ±Π»Π°Π΄Π°ΡŽΡ‚ высокой Ρ‚Π΅ΠΏΠ»ΠΎΠΎΡ‚Π΄Π°Ρ‡Π΅ΠΉ. Π’ настоящСС врСмя ΠΎΠ½ΠΈ Π½Π°ΠΈΠ±ΠΎΠ»Π΅Π΅ популярны. Π Π°Π΄ΠΈΠ°Ρ‚ΠΎΡ€Ρ‹ этого Ρ‚ΠΈΠΏΠ°, выпускаСмыС Ρ€Π°Π·Π½Ρ‹ΠΌΠΈ Ρ„ΠΈΡ€ΠΌΠ°ΠΌΠΈ, ΠΈΠΌΠ΅ΡŽΡ‚ ΠΎΠ±Ρ‰ΠΈΠΉ стандарт ΠΈ ΠΏΠΎΡ…ΠΎΠΆΠΈ ΠΏΠΎ Π²Π½Π΅ΡˆΠ½Π΅ΠΌΡƒ Π²ΠΈΠ΄Ρƒ. Π’Π°ΠΊΠΈΠ΅ Ρ€Π°Π΄ΠΈΠ°Ρ‚ΠΎΡ€Ρ‹ производятся ΠΈΠ· высококачСствСнной Ρ…ΠΎΠ»ΠΎΠ΄Π½ΠΎΠΊΠ°Ρ‚Π°Π½ΠΎΠΉ стали. Они состоят ΠΈΠ· Π΄Π²ΡƒΡ… ΠΈΠ»ΠΈ Ρ‚Ρ€Π΅Ρ… плоских ΠΏΠ°Π½Π΅Π»Π΅ΠΉ, Π²Π½ΡƒΡ‚Ρ€ΡŒ ΠΊΠΎΡ‚ΠΎΡ€Ρ‹Ρ… поступаСт Ρ‚Π΅ΠΏΠ»ΠΎΠ½ΠΎΡΠΈΡ‚Π΅Π»ΡŒ, ΠΈ рСбристых повСрхностСй ΠΌΠ΅ΠΆΠ΄Ρƒ Π½ΠΈΠΌΠΈ, Π½Π°Π³Ρ€Π΅Π²Π°ΡŽΡ‰ΠΈΡ…ΡΡ ΠΎΡ‚ ΠΏΠ°Π½Π΅Π»Π΅ΠΉ. РСбристыС повСрхности располоТСны Ρ‚Π°ΠΊ, Ρ‡Ρ‚ΠΎΠ±Ρ‹ Π²Π΅Ρ€Ρ‚ΠΈΠΊΠ°Π»ΡŒΠ½Ρ‹ΠΉ ΠΏΠΎΡ‚ΠΎΠΊ Π²ΠΎΠ·Π΄ΡƒΡ…Π° свободно ΠΏΡ€ΠΎΡ…ΠΎΠ΄ΠΈΠ» ΠΌΠ΅ΠΆΠ΄Ρƒ Π½ΠΈΠΌΠΈ. Π‘ΠΎΠ»ΡŒΡˆΠΈΠ΅ Ρ‚Π΅ΠΏΠ»Ρ‹Π΅ ΠΏΠ°Π½Π΅Π»ΠΈ ΠΎΡ‚Π΄Π°ΡŽΡ‚ Ρ‚Π΅ΠΏΠ»ΠΎ прСимущСствСнно Π·Π° счСт излучСния, Π° рСбристыС повСрхности — Π·Π° счСт ΠΊΠΎΠ½Π²Π΅ΠΊΡ†ΠΈΠΈ. Π’Π°ΠΊΠΈΠ΅ Ρ€Π°Π΄ΠΈΠ°Ρ‚ΠΎΡ€Ρ‹ Π±Ρ‹Π²Π°ΡŽΡ‚ с Π½ΠΈΠΆΠ½ΠΈΠΌ ΠΏΠΎΠ΄ΠΊΠ»ΡŽΡ‡Π΅Π½ΠΈΠ΅ΠΌ ΠΈ с Π±ΠΎΠΊΠΎΠ²Ρ‹ΠΌ ΠΏΠΎΠ΄ΠΊΠ»ΡŽΡ‡Π΅Π½ΠΈΠ΅ΠΌ.Β 
    Π Π°Π΄ΠΈΠ°Ρ‚ΠΎΡ€Ρ‹ с Π½ΠΈΠΆΠ½ΠΈΠΌ ΠΏΠΎΠ΄ΠΊΠ»ΡŽΡ‡Π΅Π½ΠΈΠ΅ΠΌ Π±ΠΎΠ»Π΅Π΅ эстСтичны ΠΈ просты Π² ΠΌΠΎΠ½Ρ‚Π°ΠΆΠ΅. Π£ Ρ€Π°Π΄ΠΈΠ°Ρ‚ΠΎΡ€ΠΎΠ² этого Ρ‚ΠΈΠΏΠ° Π΅ΡΡ‚ΡŒ Ρ‚Π°ΠΊΠΆΠ΅ встроСнный тСрмостатичСский Π²Π΅Π½Ρ‚ΠΈΠ»ΡŒ, Π½Π° ΠΊΠΎΡ‚ΠΎΡ€Ρ‹ΠΉ ΠΌΠΎΠΆΠ½ΠΎ ΡƒΡΡ‚Π°Π½ΠΎΠ²ΠΈΡ‚ΡŒ тСрморСгулятор, автоматичСски ΠΏΠΎΠ΄Π΄Π΅Ρ€ΠΆΠΈΠ²Π°ΡŽΡ‰ΠΈΠΉ Π² ΠΏΠΎΠΌΠ΅Ρ‰Π΅Π½ΠΈΠΈ Π·Π°Π΄Π°Π½Π½ΡƒΡŽ Ρ‚Π΅ΠΌΠΏΠ΅Ρ€Π°Ρ‚ΡƒΡ€Ρƒ

 ВсС, Π±Π΅Π· ΠΈΡΠΊΠ»ΡŽΡ‡Π΅Π½ΠΈΡ Ρ€Π°Π΄ΠΈΠ°Ρ‚ΠΎΡ€Ρ‹ сильно ΡΡƒΡˆΠ°Ρ‚ Π²ΠΎΠ·Π΄ΡƒΡ… Π² ΠΏΠΎΠΌΠ΅Ρ‰Π΅Π½ΠΈΠΈ, мСняя ΠΌΠΈΠΊΡ€ΠΎ-ΠΊΠ»ΠΈΠΌΠ°Ρ‚, Ρ‡Ρ‚ΠΎ Π½Π΅ совсСм Ρ…ΠΎΡ€ΠΎΡˆΠΎ для Π·Π΄ΠΎΡ€ΠΎΠ²ΡŒΡ Ρ‡Π΅Π»ΠΎΠ²Π΅ΠΊΠ°. ΠŸΠΎΡΡ‚ΠΎΠΌΡƒ Π΅Ρ‰Π΅ хочСтся ΡΠΊΠ°Π·Π°Ρ‚ΡŒ:

Β 

О Ρ‚Π΅ΠΌΠΏΠ΅Ρ€Π°Ρ‚ΡƒΡ€Π΅ ΠΎΡ‚ΠΎΠΏΠΈΡ‚Π΅Π»ΡŒΠ½Ρ‹Ρ… ΠΏΡ€ΠΈΠ±ΠΎΡ€ΠΎΠ²

ΠŸΡ€ΠΈ ΠΎΠ±ΠΎΠ³Ρ€Π΅Π²Π΅ ΠΏΠΎΠΌΠ΅Ρ‰Π΅Π½ΠΈΠΉ с ΠΏΠΎΠΌΠΎΡ‰ΡŒΡŽ Ρ€Π°Π΄ΠΈΠ°Ρ‚ΠΎΡ€ΠΎΠ² всСгда Π΅ΡΡ‚ΡŒ Π²Ρ‹Π±ΠΎΡ€: Π»ΠΈΠ±ΠΎ ΡƒΡΡ‚Π°Π½ΠΎΠ²ΠΈΡ‚ΡŒ нСбольшиС Ρ€Π°Π΄ΠΈΠ°Ρ‚ΠΎΡ€Ρ‹(ΡΡΠΊΠΎΠ½ΠΎΠΌΠΈΡ‚ΡŒ Π½Π° Ρ†Π΅Π½Π΅) ΠΈ ΡƒΠ²Π΅Π»ΠΈΡ‡ΠΈΠ²Π°Ρ‚ΡŒ Ρ‚Π΅ΠΏΠ»ΠΎΠΎΡ‚Π΄Π°Ρ‡Ρƒ ΠΎΡ‚ Π½ΠΈΡ…, ΠΏΠΎΠ²Ρ‹ΡˆΠ°Ρ Ρ‚Π΅ΠΌΠΏΠ΅Ρ€Π°Ρ‚ΡƒΡ€Ρƒ тСплоноситСля (высокотСмпСратурноС ΠΎΡ‚ΠΎΠΏΠ»Π΅Π½ΠΈΠ΅), Π»ΠΈΠ±ΠΎ, Π½Π°ΠΎΠ±ΠΎΡ€ΠΎΡ‚, ΡΡ‚Π°Ρ€Π°Ρ‚ΡŒΡΡ ΠΏΡ€ΠΈ Ρ‚ΠΎΠΉ ΠΆΠ΅ Ρ‚Π΅ΠΏΠ»ΠΎΠΎΡ‚Π΄Π°Ρ‡Π΅ ΡƒΠ²Π΅Π»ΠΈΡ‡ΠΈΡ‚ΡŒ Ρ€Π°Π·ΠΌΠ΅Ρ€Ρ‹ Ρ€Π°Π΄ΠΈΠ°Ρ‚ΠΎΡ€Π°, Π½ΠΎ Π²Π·Π°ΠΌΠ΅Π½ ΠΏΠΎΠ»ΡƒΡ‡ΠΈΡ‚ΡŒ Π±ΠΎΠ»Π΅Π΅ Π½ΠΈΠ·ΠΊΡƒΡŽ Ρ‚Π΅ΠΌΠΏΠ΅Ρ€Π°Ρ‚ΡƒΡ€Ρƒ Π΅Π³ΠΎ повСрхности (Π½ΠΈΠ·ΠΊΠΎΡ‚Π΅ΠΌΠΏΠ΅Ρ€Π°Ρ‚ΡƒΡ€Π½ΠΎΠ΅ ΠΎΡ‚ΠΎΠΏΠ»Π΅Π½ΠΈΠ΅).

Если ΠΎΡ‚ΠΎΠΏΠ»Π΅Π½ΠΈΠ΅ высокотСмпСратурноС, Ρ€Π°Π΄ΠΈΠ°Ρ‚ΠΎΡ€Ρ‹ ΠΏΡ‹ΡˆΡƒΡ‚ ΠΆΠ°Ρ€ΠΎΠΌ ΠΈ ΠΊ Π½ΠΈΠΌ Π½Π΅Π²ΠΎΠ·ΠΌΠΎΠΆΠ½ΠΎ ΠΏΡ€ΠΈΠΊΠΎΡΠ½ΡƒΡ‚ΡŒΡΡ. Π­Ρ‚ΠΎ нСэкономично, с Ρ‚ΠΎΡ‡ΠΊΠΈ зрСния расхода Ρ‚ΠΎΠΏΠ»ΠΈΠ²Π° ΠΈ Ρƒ Ρ‚Π°ΠΊΠΎΠΉ систСмы Π½Π΅Ρ‚ запаса рСгулирования. К Ρ‚ΠΎΠΌΡƒ ΠΆΠ΅, Ссли Ρ‚Π΅ΠΌΠΏΠ΅Ρ€Π°Ρ‚ΡƒΡ€Π° Π½Π° Ρ€Π°Π΄ΠΈΠ°Ρ‚ΠΎΡ€Π΅ высокая, Ρ‚ΠΎ Π²Π»Π°ΠΆΠ½ΠΎΡΡ‚ΡŒ Π²ΠΎΠ·Π΄ΡƒΡ…Π° Π² Ρ‚Π°ΠΊΠΎΠΌ ΠΏΠΎΠΌΠ΅Ρ‰Π΅Π½ΠΈΠΈ ΠΎΡ‡Π΅Π½ΡŒ низкая, Ρ‡Ρ‚ΠΎ ΠΏΡ€ΠΈΠ²ΠΎΠ΄ΠΈΡ‚ ΠΊ ΠΏΠ΅Ρ€Π΅ΡΡƒΡˆΠΈΠ²Π°Π½ΠΈΡŽ слизистых ΠΎΠ±ΠΎΠ»ΠΎΡ‡Π΅ΠΊ носа. Π•Ρ‰Π΅ ΠΎΡ‚ высокой Ρ‚Π΅ΠΌΠΏΠ΅Ρ€Π°Ρ‚ΡƒΡ€Ρ‹ Ρ€Π°Π΄ΠΈΠ°Ρ‚ΠΎΡ€Π° начинаСтся Ρ€Π°Π·Π»ΠΎΠΆΠ΅Π½ΠΈΠ΅ органичСской ΠΏΡ‹Π»ΠΈ, которая, ΠΊΠ°ΠΊ ΠΏΡ€Π°Π²ΠΈΠ»ΠΎ, присутствуСт Π² любом ΠΏΠΎΠΌΠ΅Ρ‰Π΅Π½ΠΈΠΈ. ΠŸΡ€ΠΎΠ΄ΡƒΠΊΡ‚Ρ‹ этого разлоТСния Π²Ρ‹Π΄Π΅Π»ΡΡŽΡ‚ΡΡ Π² Π²ΠΎΠ·Π΄ΡƒΡ… ΠΈ Π²Π΄Ρ‹Ρ…Π°ΡŽΡ‚ΡΡ людьми, находящимися Π² ΠΏΠΎΠΌΠ΅Ρ‰Π΅Π½ΠΈΠΈ.

ΠŸΡ€ΠΈ Π½ΠΈΠ·ΠΊΠΎΡ‚Π΅ΠΌΠΏΠ΅Ρ€Π°Ρ‚ΡƒΡ€Π½ΠΎΠΌ ΠΎΡ‚ΠΎΠΏΠ»Π΅Π½ΠΈΠΈ Ρ€Π°Π΄ΠΈΠ°Ρ‚ΠΎΡ€Ρ‹ слСгка Ρ‚Π΅ΠΏΠ»Ρ‹Π΅, Π½ΠΎ ΠΈ Π² ΠΊΠΎΠΌΠ½Π°Ρ‚Π΅ Ρ‚Π΅ΠΏΠ»ΠΎ. Π­Ρ‚ΠΎ ΠΊΠΎΠΌΡ„ΠΎΡ€Ρ‚Π½ΠΎ, бСзопасно ΠΈ позволяСт ΡΡΠΊΠΎΠ½ΠΎΠΌΠΈΡ‚ΡŒ, Ρ‚Π°ΠΊ ΠΊΠ°ΠΊ ΠΊΠΎΡ‚Π΅Π» Ρ€Π°Π±ΠΎΡ‚Π°Π΅Ρ‚ Π½Π΅ Π½Π° ΠΏΠΎΠ»Π½ΡƒΡŽ ΠΌΠΎΡ‰Π½ΠΎΡΡ‚ΡŒ. ИсслСдования ΠΏΠΎΠΊΠ°Π·Π°Π»ΠΈ, Ρ‡Ρ‚ΠΎ Π½Π°ΠΈΠ±ΠΎΠ»Π΅Π΅ комфортная для Ρ‡Π΅Π»ΠΎΠ²Π΅ΠΊΠ° Ρ‚Π΅ΠΌΠΏΠ΅Ρ€Π°Ρ‚ΡƒΡ€Π° отоплСния — 37 градусов.

Π’Ρ‹Π²ΠΎΠ΄: Π‘ΠΎΠ»ΡŒΡˆΠΎΠΉ ΠΈ Ρ‚Π΅ΠΏΠ»Ρ‹ΠΉ Ρ€Π°Π΄ΠΈΠ°Ρ‚ΠΎΡ€ Π»ΡƒΡ‡ΡˆΠ΅ малСнького ΠΈ горячСго!

Β 

Π“Π΄Π΅ ΠΈ ΠΊΠ°ΠΊ Ρ€Π°Π·ΠΌΠ΅ΡΡ‚ΠΈΡ‚ΡŒ Ρ€Π°Π΄ΠΈΠ°Ρ‚ΠΎΡ€?

РазмСщаСтся Ρ€Π°Π΄ΠΈΠ°Ρ‚ΠΎΡ€, ΠΊΠ°ΠΊ ΠΏΡ€Π°Π²ΠΈΠ»ΠΎ, Π½Π° стСнС ΠΏΠΎΠ΄ ΠΎΠΊΠ½ΠΎΠΌ для создания Ρ‚Π°ΠΊ Π½Π°Π·Ρ‹Π²Π°Π΅ΠΌΠΎΠΉ «Ρ‚Π΅ΠΏΠ»ΠΎΠ²ΠΎΠΉ завСсы». Π’ΠΎΠ·Π΄ΡƒΡ… ΠΎΠΊΠΎΠ»ΠΎ Ρ€Π°Π΄ΠΈΠ°Ρ‚ΠΎΡ€Π° нагрСваСтся, становится Π»Π΅Π³Ρ‡Π΅ ΠΈ поднимаСтся Π²Π²Π΅Ρ€Ρ…. Восходящий ΠΏΠΎΡ‚ΠΎΠΊ Ρ‚Π΅ΠΏΠ»ΠΎΠ³ΠΎ Π²ΠΎΠ·Π΄ΡƒΡ…Π° ΠΎΡ‚ Ρ€Π°Π΄ΠΈΠ°Ρ‚ΠΎΡ€Π° Π±Π»ΠΎΠΊΠΈΡ€ΡƒΠ΅Ρ‚ Π΄Π²ΠΈΠΆΠ΅Π½ΠΈΠ΅ Ρ…ΠΎΠ»ΠΎΠ΄Π½ΠΎΠ³ΠΎ Π²ΠΎΠ·Π΄ΡƒΡ…Π° ΠΎΡ‚ ΠΎΠΊΠ½Π° Π² Π·Π°ΠΌΠΊΠ½ΡƒΡ‚ΠΎΠΌ пространствС ΠΏΠ΅Ρ€Π΅Π΄ ΠΎΠΊΠ½ΠΎΠΌ.

РасчСт мощности Ρ€Π°Π΄ΠΈΠ°Ρ‚ΠΎΡ€Π°

Для Π²Ρ‹Π±ΠΎΡ€Π° Ρ‚Π΅ΠΏΠ»ΠΎΠ²ΠΎΠΉ мощности Ρ€Π°Π΄ΠΈΠ°Ρ‚ΠΎΡ€Π°, достаточной для ΠΊΠ°ΠΆΠ΄ΠΎΠΉ ΠΊΠΎΠΌΠ½Π°Ρ‚Ρ‹, Π² климатичСском поясС НиТнСго Новгорода ΠΌΠΎΠΆΠ½ΠΎ ΡΠ»Π΅Π΄ΠΎΠ²Π°Ρ‚ΡŒ простому ΠΏΡ€Π°Π²ΠΈΠ»Ρƒ:

  • Π² ΠΊΠΎΠΌΠ½Π°Ρ‚Π΅ с ΠΎΠ΄Π½ΠΎΠΉ Π½Π°Ρ€ΡƒΠΆΠ½ΠΎΠΉ стСной ΠΈ ΠΎΠ΄Π½ΠΈΠΌ ΠΎΠΊΠ½ΠΎΠΌ ΠΎΠ΄Π½ΠΎΠ³ΠΎ ΠΊΠΈΠ»ΠΎΠ²Π°Ρ‚Ρ‚Π° (1 ΠΊΠ’Ρ‚) Ρ‚Π΅ΠΏΠ»ΠΎΠ²ΠΎΠΉ мощности Ρ€Π°Π΄ΠΈΠ°Ρ‚ΠΎΡ€Π°      достаточно для отоплСния 10 ΠΊΠ².ΠΌ. ΠΆΠΈΠ»ΠΎΠΉ ΠΏΠ»ΠΎΡ‰Π°Π΄ΠΈ;
  • Ссли Π² ΠΊΠΎΠΌΠ½Π°Ρ‚Π΅ Π΄Π²Π΅ Π½Π°Ρ€ΡƒΠΆΠ½Ρ‹Π΅ стСны ΠΈ ΠΎΠ΄Π½ΠΎ ΠΎΠΊΠ½ΠΎ, Ρ‚ΠΎ для отоплСния 10 ΠΊΠ².ΠΌ. трСбуСтся 1,2 ΠΊΠ’Ρ‚ Ρ‚Π΅ΠΏΠ»ΠΎΠ²ΠΎΠΉ мощности;
  • Ссли Π² ΠΊΠΎΠΌΠ½Π°Ρ‚Π΅ Π΄Π²Π΅ Π½Π°Ρ€ΡƒΠΆΠ½Ρ‹Ρ… стСны ΠΈ Π΄Π²Π° ΠΎΠΊΠ½Π°, для отоплСния 10 ΠΊΠ².ΠΌ. трСбуСтся 1,3 ΠΊΠ’Ρ‚ Ρ‚Π΅ΠΏΠ»ΠΎΠ²ΠΎΠΉ мощности.

Π‘ΡƒΡ‰Π΅ΡΡ‚Π²ΡƒΡŽΡ‚ Π±ΠΎΠ»Π΅Π΅ Ρ‚ΠΎΡ‡Π½Ρ‹Π΅ расчСты Π½Π΅ΠΎΠ±Ρ…ΠΎΠ΄ΠΈΠΌΠΎΠΉ мощности Ρ€Π°Π΄ΠΈΠ°Ρ‚ΠΎΡ€ΠΎΠ², ΠΊΠΎΡ‚ΠΎΡ€Ρ‹ΠΌΠΈ Ρ€ΡƒΠΊΠΎΠ²ΠΎΠ΄ΡΡ‚Π²ΡƒΡŽΡ‚ΡΡ спСциалисты-ΠΏΡ€ΠΎΠ΅ΠΊΡ‚ΠΈΡ€ΠΎΠ²Ρ‰ΠΈΠΊΠΈ, Π½ΠΎ для Π³Ρ€ΡƒΠ±ΠΎΠΉ ΠΎΡ†Π΅Π½ΠΊΠΈ достаточно ΠΈ ΠΏΡ€Π΅Π΄Π»ΠΎΠΆΠ΅Π½Π½ΠΎΠ³ΠΎ простого ΠΌΠ΅Ρ‚ΠΎΠ΄Π°.

БСйчас Π½Π° Ρ€Ρ‹Π½ΠΊΠ΅ ΠΎΡ‚ΠΎΠΏΠΈΡ‚Π΅Π»ΡŒΠ½ΠΎΠ³ΠΎ оборудования ΠΏΡ€Π΅Π΄Π»ΠΎΠΆΠ΅Π½ΠΎ ΠΎΡ‡Π΅Π½ΡŒ большоС количСство Ρ€Π°Π΄ΠΈΠ°Ρ‚ΠΎΡ€ΠΎΠ² Ρ€Π°Π·Π»ΠΈΡ‡Π½Ρ‹Ρ… ΠΏΡ€ΠΎΠΈΠ·Π²ΠΎΠ΄ΠΈΡ‚Π΅Π»Π΅ΠΉ, Ρ„ΠΎΡ€ΠΌ, ΠΎΡ‚Π»ΠΈΡ‡Π°ΡŽΡ‰ΠΈΡ…ΡΡ Π΄Ρ€ΡƒΠ³ ΠΎΡ‚ Π΄Ρ€ΡƒΠ³Π° Π² ΠΏΠ΅Ρ€Π²ΡƒΡŽ ΠΎΡ‡Π΅Ρ€Π΅Π΄ΡŒ внСшним Π²ΠΈΠ΄ΠΎΠΌ (Π΄ΠΈΠ·Π°ΠΉΠ½ΠΎΠΌ), Ρ‚Π°ΠΊΠΆΠ΅ ΠΌΠ°Ρ‚Π΅Ρ€ΠΈΠ°Π»ΠΎΠΌ, Ρ‚Π΅ΠΏΠ»ΠΎΠΎΡ‚Π΄Π°Ρ‡Π΅ΠΉ ΠΈ ΠΊΠΎΠ½Π΅Ρ‡Π½ΠΎ ΠΆΠ΅ Ρ†Π΅Π½ΠΎΠΉ. Π§Π΅ΠΌΡƒ Ρ‚ΡƒΡ‚ ΠΎΡ‚Π΄Π°Ρ‚ΡŒ ΠΏΡ€Π΅Π΄ΠΏΠΎΡ‡Ρ‚Π΅Π½ΠΈΠ΅? Волько Π½Π΅ Ρ†Π΅Π½Π΅!

НС Ρ€Π°Π· ΡƒΠΆΠ΅ ΠΏΡ€ΠΎΠ²Π΅Ρ€Π΅Π½ΠΎ Π½Π° Π³ΠΎΡ€ΡŒΠΊΠΎΠΌ ΠΎΠΏΡ‹Ρ‚Π΅ ΠΌΠ½ΠΎΠ³ΠΈΡ… людСй, Ρ‡Ρ‚ΠΎ Π΄Π΅ΡˆΠ΅Π²Ρ‹Π΅ Ρ€Π°Π΄ΠΈΠ°Ρ‚ΠΎΡ€Ρ‹ ΠΊΡ€Π°ΠΉΠ½Π΅ Π½Π΅ Π½Π°Π΄Π΅ΠΆΠ½Ρ‹! ΠŸΠΎΡΠ»Π΅Π΄ΡΡ‚Π²ΠΈΡ Ρ‚Π°ΠΊΠΈΡ… ΠΏΡ€ΠΎΠ²Π΅Ρ€ΠΎΠΊ ΠΌΠΎΠ³ΡƒΡ‚ ΡΡ‚Π°Ρ‚ΡŒ ΠΎΡ‡Π΅Π½ΡŒ ΠΏΠ»Π°Ρ‡Π΅Π²Π½Ρ‹ΠΌΠΈ ΠΊΠ°ΠΊ для Π²Π½ΡƒΡ‚Ρ€Π΅Π½Π½Π΅ΠΉ ΠΎΡ‚Π΄Π΅Π»ΠΊΠΈ Ρ‚Π°ΠΊ ΠΈ для самих ΠΎΠ±ΠΈΡ‚Π°Ρ‚Π΅Π»Π΅ΠΉ Π΄ΠΎΠΌΠ°, особСнно Π² Ρ…ΠΎΠ»ΠΎΠ΄Π½ΠΎΠ΅ Π·ΠΈΠΌΠ½Π΅Π΅ врСмя.

Π’Ρ‹Π±Ρ€Π°Ρ‚ΡŒ Ρ…ΠΎΡ€ΠΎΡˆΠΈΠ΅, Π΄Π΅ΠΉΡΡ‚Π²ΠΈΡ‚Π΅Π»ΡŒΠ½ΠΎ Π½Π°Π΄Π΅ΠΆΠ½Ρ‹Π΅ ΠΎΡ‚ΠΎΠΏΠΈΡ‚Π΅Π»ΡŒΠ½Ρ‹Π΅ ΠΏΡ€ΠΈΠ±ΠΎΡ€Ρ‹, Ρ‚ΠΎ Π΅ΡΡ‚ΡŒ Ρ€Π°Π΄ΠΈΠ°Ρ‚ΠΎΡ€Ρ‹, ΠΊΠΎΠ½Π²Π΅ΠΊΡ‚ΠΎΡ€Ρ‹ ΠΈΒ  ΠΏΠΎΠ»ΠΎΡ‚Π΅Π½Ρ†Π΅ΡΡƒΡˆΠΈΡ‚Π΅Π»ΠΈ, Π° Ρ‚Π°ΠΊΠΆΠ΅ ΠΏΡ€Π°Π²ΠΈΠ»ΡŒΠ½ΠΎ Ρ€Π°ΡΡΡ‡ΠΈΡ‚Π°Ρ‚ΡŒ ΠΈΡ… ΠΏΠΎ мощности Π’Π°ΠΌ всСгда ΠΏΠΎΠΌΠΎΠ³ΡƒΡ‚ спСциалисты нашСй ΠΊΠΎΠΌΠΏΠ°Π½ΠΈΠΈ.

Β 

Π¨Π°Π³ Ρ‚Ρ€Π΅Ρ‚ΠΈΠΉ: Горячая Π²ΠΎΠ΄Π°

Π£ ΠΊΠΎΠΌΠΏΠ°Π½ΠΈΠΈ Buderus (ГСрмания) ΠΎΡ‚Π½ΠΎΡΠΈΡ‚Π΅Π»ΡŒΠ½ΠΎ этого Π΅ΡΡ‚ΡŒ слоган: Β«Π•ΡΡ‚ΡŒ Π»ΠΈ Ρ‡Ρ‚ΠΎ-Ρ‚ΠΎ Π±ΠΎΠ»Π΅Π΅ Ρ†Π΅Π½Π½ΠΎΠ΅, Ρ‡Π΅ΠΌ Π²ΠΎΠ΄Π°? Волько горячая Π²ΠΎΠ΄Π°!Β»

Β 

Как ΠΏΠΎΠ»ΡƒΡ‡ΠΈΡ‚ΡŒ Π³ΠΎΡ€ΡΡ‡ΡƒΡŽ Π²ΠΎΠ΄Ρƒ для ΠΊΡƒΡ…Π½ΠΈ ΠΈ Π΄ΡƒΡˆΠ°?

Горячая Π²ΠΎΠ΄Π° для Π±Ρ‹Ρ‚ΠΎΠ²Ρ‹Ρ… Π½ΡƒΠΆΠ΄ ΠΈ горячая Π²ΠΎΠ΄Π° Π² систСмС отоплСния – Π½Π΅ ΠΎΠ΄Π½ΠΎ ΠΈ Ρ‚ΠΎΠΆΠ΅. БистСма отоплСния находится ΠΏΠΎ Π΄Π°Π²Π»Π΅Π½ΠΈΠ΅ΠΌ, ΠΎΠ½Π° Π·Π°ΠΌΠΊΠ½ΡƒΡ‚Π° ΠΈ ΠΈΠ· Π½Π΅Π΅ Π½Π΅ Π΄ΠΎΠ»ΠΆΠ½ΠΎ Π½ΠΈΡ‡Π΅Π³ΠΎ Π²Ρ‹Π»ΠΈΠ²Π°Ρ‚ΡŒΡΡ. Π’Π΅ΠΏΠ»ΠΎΠ½ΠΎΡΠΈΡ‚Π΅Π»ΡŒ систСмы отоплСния (Π² Ρ‚ΠΎΠΌ числС Π°Π½Ρ‚ΠΈΡ„Ρ€ΠΈΠ·Ρ‹) Π½ΠΈΠ³Π΄Π΅ Π½Π΅ ΠΈΠΌΠ΅Π΅Ρ‚ прямого ΠΊΠΎΠ½Ρ‚Π°ΠΊΡ‚Π° с Π±Ρ‹Ρ‚ΠΎΠ²ΠΎΠΉ (ΠΏΠΈΡ‚ΡŒΠ΅Π²ΠΎΠΉ) Π²ΠΎΠ΄ΠΎΠΉ. Горячая Π²ΠΎΠ΄Π° для Π±Ρ‹Ρ‚ΠΎΠ²Ρ‹Ρ… Π½ΡƒΠΆΠ΄ получаСтся ΠΏΡƒΡ‚Π΅ΠΌ Π½Π°Π³Ρ€Π΅Π²Π° Ρ…ΠΎΠ»ΠΎΠ΄Π½ΠΎΠΉ (ΠΏΠΈΡ‚ΡŒΠ΅Π²ΠΎΠΉ) Π²ΠΎΠ΄Ρ‹ ΠΈΠ· скваТины ΠΈΠ»ΠΈ Π²ΠΎΠ΄ΠΎΠΏΡ€ΠΎΠ²ΠΎΠ΄Π° ΠΈ послС использования ΠΏΠΎ Π½Π°Π·Π½Π°Ρ‡Π΅Π½ΠΈΡŽ Π±Π΅Π·Π²ΠΎΠ·Π²Ρ€Π°Ρ‚Π½ΠΎ сливаСтся Π² ΠΊΠ°Π½Π°Π»ΠΈΠ·Π°Ρ†ΠΈΡŽ.

Π•ΡΡ‚ΡŒ нСсколько Π²ΠΈΠ΄ΠΎΠ² устройств, ΠΏΠΎ ΠΏΡ€ΠΈΠ½Ρ†ΠΈΠΏΡƒ дСйствия — ΠΏΡ€ΠΎΡ‚ΠΎΡ‡Π½Ρ‹Ρ… ΠΈΠ»ΠΈ Π½Π°ΠΊΠΎΠΏΠΈΡ‚Π΅Π»ΡŒΠ½Ρ‹Ρ…, Π½Π°Π³Ρ€Π΅Π²Π°ΡŽΡ‰ΠΈΡ… Π²ΠΎΠ΄Ρƒ для Π±Ρ‹Ρ‚ΠΎΠ²Ρ‹Ρ… Π½ΡƒΠΆΠ΄:

  • элСктронагрСватСли;
  • Π³Π°Π·ΠΎΠ²Ρ‹Π΅ Π½Π°Π³Ρ€Π΅Π²Π°Ρ‚Π΅Π»ΠΈ;
  • Π°ΠΏΠΏΠ°Ρ€Π°Ρ‚Ρ‹ косвСнного Π½Π°Π³Ρ€Π΅Π²Π° Π±Ρ‹Ρ‚ΠΎΠ²ΠΎΠΉ горячСй Π²ΠΎΠ΄Ρ‹ ΠΎΡ‚ тСплоноситСля систСмы отоплСния.
  • Π’ ΠΏΡ€ΠΎΡ‚ΠΎΡ‡Π½Ρ‹Ρ… водонагрСватСлях Π²ΠΎΠ΄Π° нагрСваСтся ΠΏΠΎ ΠΌΠ΅Ρ€Π΅ продвиТСния ΠΌΠΈΠΌΠΎ Ρ‚Π΅ΠΏΠ»ΠΎΠΏΠ΅Ρ€Π΅Π΄Π°ΡŽΡ‰ΠΈΡ… элСмСнтов: это элСктричСскиС тэны, ΠΌΠ΅Π΄Π½Ρ‹Π΅ Ρ‚Ρ€ΡƒΠ±Ρ‹ Π³Π°Π·ΠΎΠ²Ρ‹Ρ… ΠΊΠΎΠ»ΠΎΠ½ΠΎΠΊ ΠΈΠ»ΠΈ ячСистыС Ρ‚Π΅ΠΏΠ»ΠΎΠΎΠ±ΠΌΠ΅Π½Π½ΠΈΠΊΠΈ косвСнного Π½Π°Π³Ρ€Π΅Π²Π°. ΠŸΡ€ΠΈ этом Ρ‡Ρ‚ΠΎΠ±Ρ‹ ΠΏΠΎΠ»ΡƒΡ‡ΠΈΡ‚ΡŒ Π΄Π΅ΠΉΡΡ‚Π²ΠΈΡ‚Π΅Π»ΡŒΠ½ΠΎ Π³ΠΎΡ€ΡΡ‡ΡƒΡŽ Π²ΠΎΠ΄Ρƒ, Π° Π½Π΅ Ρ‡ΡƒΡ‚ΡŒ Ρ‚Π΅ΠΏΠ»ΡƒΡŽ, Π½ΡƒΠΆΠ½Π° довольно большая ΠΌΠΎΡ‰Π½ΠΎΡΡ‚ΡŒ Ρ‚Π΅ΠΏΠ»ΠΎΠΏΠ΅Ρ€Π΅Π΄Π°Ρ‡ΠΈ, ΠΈΠ»ΠΈ Π²ΠΎΠ΄Π° Π΄ΠΎΠ»ΠΆΠ½Π° Ρ‚Π΅Ρ‡ΡŒ ΠΌΠ΅Π΄Π»Π΅Π½Π½ΠΎ. Π­Ρ‚ΠΎΡ‚ способ Π½Π΅ ΡƒΠ΄ΠΎΠ±Π΅Π½, ΠΊΠΎΠ³Π΄Π° имССтся довольно большая ΠΏΠΎΡ‚Ρ€Π΅Π±Π½ΠΎΡΡ‚ΡŒ Π² горячСй Π²ΠΎΠ΄Π΅, Ρ‚ΠΎ Π΅ΡΡ‚ΡŒ сразу нСсколько ΠΎΠ΄Π½ΠΎΠ²Ρ€Π΅ΠΌΠ΅Π½Π½ΠΎ Ρ€Π°Π±ΠΎΡ‚Π°ΡŽΡ‰ΠΈΡ… Ρ‚ΠΎΡ‡Π΅ΠΊ Ρ€Π°Π·Π±ΠΎΡ€Π° горячСй Π²ΠΎΠ΄Ρ‹ (Π½Π°ΠΏΡ€ΠΈΠΌΠ΅Ρ€ Π² Π΄ΠΎΠΌΠ΅ нСсколько сан. ΡƒΠ·Π»ΠΎΠ², кухня) Ρ‚Π°ΠΊ ΠΊΠ°ΠΊ ΠΏΡ€ΠΎΠΈΠ·Π²ΠΎΠ΄ΠΈΡ‚Π΅Π»ΡŒΠ½ΠΎΡΡ‚ΡŒ Ρƒ ΠΏΡ€ΠΎΡ‚ΠΎΡ‡Π½Ρ‹Ρ… Π²ΠΎΠ΄ΠΎΠ½Π°Π³Ρ€Π΅Π²Π°Ρ‚Π΅Π»Π΅ΠΉ достаточно малая. Но ΠΈ Ρ‚ΡƒΡ‚ Π΅ΡΡ‚ΡŒ ΠΈΡΠΊΠ»ΡŽΡ‡Π΅Π½ΠΈΡ
  • ΠΠ°ΠΊΠΎΠΏΠΈΡ‚Π΅Π»ΡŒΠ½Ρ‹ΠΉ Π²ΠΎΠ΄ΠΎΠ½Π°Π³Ρ€Π΅Π²Π°Ρ‚Π΅Π»ΡŒ (Π±ΠΎΠΉΠ»Π΅Ρ€) отличаСтся ΠΎΡ‚ ΠΏΡ€ΠΎΡ‚ΠΎΡ‡Π½ΠΎΠ³ΠΎ Π½Π°ΠΌΠ½ΠΎΠ³ΠΎ большим объСмом запасаСмой горячСй Π²ΠΎΠ΄Ρ‹, ΠΎΠ½ Π² Π±Ρ‹Ρ‚Ρƒ ΡƒΠ΄ΠΎΠ±Π½Π΅ΠΉ, Ρ‚Π°ΠΊ ΠΊΠ°ΠΊ Π½Π°Π³Ρ€Π΅Π² Π²ΠΎΠ΄Ρ‹ Π΄ΠΎ Π·Π°Π΄Π°Π½Π½ΠΎΠΉ Ρ‚Π΅ΠΌΠΏΠ΅Ρ€Π°Ρ‚ΡƒΡ€Ρ‹ происходит Π·Π°Ρ€Π°Π½Π΅Π΅. Π’ Π±ΠΎΠΉΠ»Π΅Ρ€Π΅ постоянно находится горячая Π²ΠΎΠ΄Π°, Π° ΠΏΠΎ ΠΌΠ΅Ρ€Π΅ расхода Π² Π½Π΅Π³ΠΎ поступаСт холодная ΠΈ подогрСваСтся Π΄ΠΎ Π½ΡƒΠΆΠ½ΠΎΠΉ Ρ‚Π΅ΠΌΠΏΠ΅Ρ€Π°Ρ‚ΡƒΡ€Ρ‹. НапримСр, Π±ΠΎΠΉΠ»Π΅Ρ€Π° Π΅ΠΌΠΊΠΎΡΡ‚ΡŒΡŽ 200 Π»ΠΈΡ‚Ρ€ΠΎΠ² достаточно для сСмьи ΠΈΠ· 4 Ρ‡Π΅Π»ΠΎΠ²Π΅ΠΊ.

ΠžΠ΄Π½ΠΎΠ·Π½Π°Ρ‡Π½ΠΎ ΡΠΊΠ°Π·Π°Ρ‚ΡŒ, Ρ‡Ρ‚ΠΎΒ  Π½Π°ΠΊΠΎΠΏΠΈΡ‚Π΅Π»ΡŒΠ½Ρ‹ΠΉ Π²ΠΎΠ΄ΠΎΠ½Π°Π³Ρ€Π΅Π²Π°Ρ‚Π΅Π»ΡŒ ΡƒΠ΄ΠΎΠ±Π½Π΅Π΅ Β ΠΏΡ€ΠΎΡ‚ΠΎΡ‡Π½ΠΎΠ³ΠΎ нСльзя ΠΏΠΎΡ‚ΠΎΠΌΡƒ Ρ‡Ρ‚ΠΎ, Π½Π°ΠΏΡ€ΠΈΠΌΠ΅Ρ€ Ρƒ ΠΊΠΎΠΌΠΏΠ°Π½ΠΈΠΈΒ  Π‘Π’Π‘ (ШвСция)Β  Π΅ΡΡ‚ΡŒ Ρ‚Π°ΠΊ Π½Π°Π·Ρ‹Π²Π°Π΅ΠΌΡ‹Π΅ высокоскоростныС Ρ‚Π΅ΠΏΠ»ΠΎΠΎΠ±ΠΌΠ΅Π½Π½Ρ‹Π΅ ΡƒΠ·Π»Ρ‹, ΠΊΠΎΡ‚ΠΎΡ€Ρ‹Π΅ встроСны ΠΈΠ»ΠΈ ΠΎΡ‚Π΄Π΅Π»ΡŒΠ½ΠΎ ΠΏΠΎΠ΄ΠΊΠ»ΡŽΡ‡Π°Π΅ΠΌΡ‹ ΠΊ ΠΊΠΎΡ‚Π»Ρƒ ΠΈ Π² ΠΏΡ€ΠΎΡ‚ΠΎΡ‡Π½ΠΎΠΌ Ρ€Π΅ΠΆΠΈΠΌΠ΅ ΠΌΠΎΠ³ΡƒΡ‚ ΠΏΡ€ΠΈΠ³ΠΎΡ‚ΠΎΠ²Π»ΡΡ‚ΡŒ ΠΎΡ‚ 500 Π΄ΠΎ 1500 Π»ΠΈΡ‚Ρ€ΠΎΠ² горячСй Π²ΠΎΠ΄Ρ‹ Π² час, замСняя Π±ΠΎΠΉΠ»Π΅Ρ€Ρ‹Β  Π΄ΠΎ 200 Π».

Но Ссли Ρƒ вас большой Π΄ΠΎΠΌ ΠΈ большая сСмья ΠΈ расход горячСй Π²ΠΎΠ΄Ρ‹ Π±ΡƒΠ΄Π΅Ρ‚ достаточно высоким, Ρ‚ΠΎ Ρ‚ΡƒΡ‚ ΡƒΠΆ Π’Π°Ρˆ Π²Ρ‹Π±ΠΎΡ€ Π΄ΠΎΠ»ΠΆΠ΅Π½ Π±Ρ‹Ρ‚ΡŒ ΠΎΠ΄Π½ΠΎΠ·Π½Π°Ρ‡Π½Ρ‹ΠΌ – Π±ΠΎΠΉΠ»Π΅Ρ€ косвСнного Π½Π°Π³Ρ€Π΅Π²Π°.Β  Π‘ΠΎΠΉΠ»Π΅Ρ€Ρ‹ Π±Ρ‹Π²Π°ΡŽΡ‚ Ρ€Π°Π·Π½ΠΎΠΉ Смкости( Π΄ΠΎ 2000 Π»ΠΈΡ‚Ρ€ΠΎΠ²), Π½ΠΎ ΠΏΡ€ΠΈΠ½Ρ†ΠΈΠΏ Π½Π°Π³Ρ€Π΅Π²Π° Π²ΠΎΠ΄Ρ‹ Ρƒ Π½ΠΈΡ… ΠΎΠ΄ΠΈΠ½Π°ΠΊΠΎΠ². Π‘ΠΎΠΉΠ»Π΅Ρ€ – это Ρ‚Π΅ΠΏΠ»ΠΎΠΈΠ·ΠΎΠ»ΠΈΡ€ΠΎΠ²Π°Π½Π½Ρ‹ΠΉ Π±Π°ΠΊ. Π’Π½ΡƒΡ‚Ρ€ΠΈ Ρ‡Π΅Ρ€Π΅Π· Π½Π΅Π³ΠΎ ΠΏΡ€ΠΎΡ…ΠΎΠ΄ΠΈΡ‚ ΡΠΏΠΈΡ€Π°Π»ΡŒΡŽ Ρ‚Ρ€ΡƒΠ±Π°, Π² ΠΊΠΎΡ‚ΠΎΡ€ΠΎΠΉ двигаСтся горячий Ρ‚Π΅ΠΏΠ»ΠΎΠ½ΠΎΡΠΈΡ‚Π΅Π»ΡŒ ΠΈΠ· систСмы отоплСния. Π‘Π½ΠΈΠ·Ρƒ ΠΏΠΎ Ρ‚Ρ€ΡƒΠ±Π΅ Π² Π±ΠΎΠΉΠ»Π΅Ρ€ поступаСт холодная Π²ΠΎΠ΄Π°, Ρ‚Π΅ΠΏΠ»ΠΎ ΠΎΡ‚ тСплоноситСля систСмы отоплСния пСрСдаСтся Ρ…ΠΎΠ»ΠΎΠ΄Π½ΠΎΠΉ Π±Ρ‹Ρ‚ΠΎΠ²ΠΎΠΉ Π²ΠΎΠ΄Π΅ Ρ‡Π΅Ρ€Π΅Π· стСнки ΡΠΏΠΈΡ€Π°Π»ΡŒΠ½ΠΎΠΉ Ρ‚Ρ€ΡƒΠ±Ρ‹ (Ρ‚Π΅ΠΏΠ»ΠΎΠΎΠ±ΠΌΠ΅Π½Π½ΠΈΠΊΠ° Π±ΠΎΠΉΠ»Π΅Ρ€Π°) .К Π²Π΅Ρ€Ρ…Π½Π΅ΠΉ части Π±ΠΎΠΉΠ»Π΅Ρ€Π° подсоСдиняСтся Π΅Ρ‰Π΅ ΠΎΠ΄Π½Π° Ρ‚Ρ€ΡƒΠ±Π°, для Π²Ρ‹Ρ…ΠΎΠ΄Π° горячСй Π²ΠΎΠ΄Ρ‹. К Π±ΠΎΠΉΠ»Π΅Ρ€Ρƒ ΠΏΠΎΠ΄ΠΊΠ»ΡŽΡ‡Π°ΡŽΡ‚ Ρ‚Π΅Ρ€ΠΌΠΎΠΌΠ΅Ρ‚Ρ€ для контроля Ρ‚Π΅ΠΌΠΏΠ΅Ρ€Π°Ρ‚ΡƒΡ€Ρ‹ Π²ΠΎΠ΄Ρ‹ Π² Π½Π΅ΠΌ.

Π’Π½ΡƒΡ‚Ρ€Π΅Π½Π½Π΅Π΅ ΠΏΠΎΠΊΡ€Ρ‹Ρ‚ΠΈΠ΅ Π±ΠΎΠΉΠ»Π΅Ρ€Π° ΠΌΠΎΠΆΠ΅Ρ‚ Π±Ρ‹Ρ‚ΡŒ Ρ€Π°Π·Π½Ρ‹ΠΌ, Π½ΠΎ ΠΎΠ±Ρ‹Ρ‡Π½ΠΎ это ΡΠΏΠ΅Ρ†ΠΈΠ°Π»ΡŒΠ½Π°Ρ эмаль ΠΈΠ»ΠΈ Π½Π΅Ρ€ΠΆΠ°Π²Π΅ΠΉΠΊΠ°. Π—Π΄Π΅ΡΡŒ Ρ‚ΠΎΠΆΠ΅ хочСтся ΡΠΊΠ°Π·Π°Ρ‚ΡŒ, Ρ‡Ρ‚ΠΎ ΠΎΡ‚ качСства Π²Π½ΡƒΡ‚Ρ€Π΅Π½Π½Π΅Π³ΠΎ покрытия Π±ΠΎΠΉΠ»Π΅Ρ€Π° сильно зависит качСство ΠΏΠΎΠ»ΡƒΡ‡Π°Π΅ΠΌΠΎΠΉ Π²ΠΎΠ΄Ρ‹ ΠΈ СстСствСнно Ρ†Π΅Π½Π°( Ρ‡Π΅ΠΌ ΠΏΠΎΠΊΡ€Ρ‹Ρ‚ΠΈΠ΅ Π»ΡƒΡ‡ΡˆΠ΅, Ρ‚Π΅ΠΌ Π±ΠΎΠΉΠ»Π΅Ρ€ Π΄ΠΎΡ€ΠΎΠΆΠ΅). Π£ Π½Π΅ΠΊΠΎΡ‚ΠΎΡ€Ρ‹Ρ… ΠΏΡ€ΠΎΠΈΠ·Π²ΠΎΠ΄ΠΈΡ‚Π΅Π»Π΅ΠΉ Π²Π½ΡƒΡ‚Ρ€Π΅Π½Π½Π΅Π΅ ΠΏΠΎΠΊΡ€Ρ‹Ρ‚ΠΈΠ΅ ΠΈΠΌΠ΅Π΅Ρ‚ Π΄Π°ΠΆΠ΅ Π°Π½Ρ‚ΠΈ-Π±Π°ΠΊΡ‚Π΅Ρ€ΠΈΠ°Π»ΡŒΠ½ΡƒΡŽ основу. ΠŸΡ€ΠΈ Π²Ρ‹Π±ΠΎΡ€Π΅ Π±ΠΎΠΉΠ»Π΅Ρ€Π° ΠΎΠ±ΡΠ·Π°Ρ‚Π΅Π»ΡŒΠ½ΠΎ ΠΏΠΎΠΈΠ½Ρ‚Π΅Ρ€Π΅ΡΡƒΠΉΡ‚Π΅ΡΡŒ ΠΈΠ· Ρ‡Π΅Π³ΠΎ ΠΎΠ½ Π²Π½ΡƒΡ‚Ρ€ΠΈ сдСлан, ΠΈ ΠΎΠ±ΡΠ·Π°Ρ‚Π΅Π»ΡŒΠ½ΠΎ ΠΎΡ‚Π΄Π°ΠΉΡ‚Π΅ ΠΏΡ€Π΅Π΄ΠΏΠΎΡ‡Ρ‚Π΅Π½ΠΈΠ΅ извСстным ΠΈ Π·Π°Ρ€Π΅ΠΊΠΎΠΌΠ΅Π½Π΄ΠΎΠ²Π°Π²ΡˆΠΈΠΌ сСбя производитСлям.

Наша компания ΠΌΠΎΠΆΠ΅Ρ‚ Ρ‚Π°ΠΊΠΆΠ΅ ΠΏΠΎΠΌΠΎΡ‡ΡŒ Π² ΠΏΠΎΠ΄Π±ΠΎΡ€Π΅ Π½ΡƒΠΆΠ½ΠΎΠ³ΠΎ Π’Π°ΠΌ Π±ΠΎΠΉΠ»Π΅Ρ€Π°, ΡƒΠ²Π΅Ρ€Π΅Π½Π½ΠΎ ΠΏΠΎΡ€Π΅ΠΊΠΎΠΌΠ΅Π½Π΄ΠΎΠ²Π°Π² самыС Π½Π°Π΄Π΅ΠΆΠ½Ρ‹Π΅ ΠΈ Π»ΡƒΡ‡ΡˆΠΈΠ΅ Π½Π° Ρ€Ρ‹Π½ΠΊΠ΅ ΠΎΡ‚ΠΎΠΏΠΈΡ‚Π΅Π»ΡŒΠ½ΠΎΠ³ΠΎ оборудования.

Β 

Π¨Π°Π³ Ρ‡Π΅Ρ‚Π²Π΅Ρ€Ρ‚Ρ‹ΠΉ: Π’Π΅ΠΏΠ»Ρ‹Π΅ ΠΏΠΎΠ»Ρ‹

Π’ΠΎΠ·ΠΌΠΎΠΆΠ½ΠΎ Π’Ρ‹ Π·Π°Ρ…ΠΎΡ‚ΠΈΡ‚Π΅ Π΅Ρ‰Π΅ большС ΡƒΠ²Π΅Π»ΠΈΡ‡ΠΈΡ‚ΡŒ ΠΊΠΎΠΌΡ„ΠΎΡ€Ρ‚ Π² вашСм Π΄ΠΎΠΌΠ΅ ΠΏΡƒΡ‚Π΅ΠΌ ΠΏΠΎΠ΄ΠΎΠ³Ρ€Π΅Π²Π° ΠΏΠΎΠ»Π°, Π½Π°ΠΏΡ€ΠΈΠΌΠ΅Ρ€ Π² Π²Π°Π½Π½Ρ‹Ρ…, сан. ΡƒΠ·Π»Π°Ρ…, ΠΊΠΎΡ€ΠΈΠ΄ΠΎΡ€Π°Ρ…, Ρ‚Π°ΠΌΠ±ΡƒΡ€Π°Ρ…, ΠΊΡƒΡ…Π½Π΅. По Ρ‚Π΅ΠΏΠ»ΠΎΠΌΡƒ ΠΏΠΎΠ»Ρƒ всСгда приятно ΠΈ ΠΊΠΎΠΌΡ„ΠΎΡ€Ρ‚Π½ΠΎ Ρ…ΠΎΠ΄ΠΈΡ‚ΡŒ, особСнно Π±Π΅Π· ΠΎΠ±ΡƒΠ²ΠΈ. Они ΡΠΎΠ·Π΄Π°ΡŽΡ‚ ΠΎΡΠΎΠ±ΡƒΡŽ Ρ‚Π΅ΠΏΠ»ΡƒΡŽ атмосфСру ΠΈ ΡƒΡŽΡ‚.

Π­Ρ‚Ρƒ ΠΏΡ€ΠΎΠ±Π»Π΅ΠΌΡƒ ΠΊΠ°ΠΊ Ρ€Π°Π· Π½Π΅ΠΎΠ±Ρ…ΠΎΠ΄ΠΈΠΌΠΎ Ρ€Π΅ΡˆΠ°Ρ‚ΡŒ ΠΈΠΌΠ΅Π½Π½ΠΎ ΠΏΡ€ΠΈ ΠΎΡ€Π³Π°Π½ΠΈΠ·Π°Ρ†ΠΈΠΈ ΠΈ установкС систСмы отоплСния.

Β 

Π§Ρ‚ΠΎ Ρ‚Π°ΠΊΠΎΠ΅ Ρ‚Π΅ΠΏΠ»Ρ‹Π΅ ΠΏΠΎΠ»Ρ‹?

Для ΠΎΡ€Π³Π°Π½ΠΈΠ·Π°Ρ†ΠΈΠΈ напольного отоплСния ΠΏΠΎΠ΄ ΠΏΠΎΠ»ΠΎΠΌ довольно ΠΏΠ»ΠΎΡ‚Π½ΠΎ укладываСтся Ρ‚Ρ€ΡƒΠ±Π°, ΠΏΠΎ ΠΊΠΎΡ‚ΠΎΡ€ΠΎΠΉ двиТСтся Ρ‚Π΅ΠΏΠ»ΠΎΠ½ΠΎΡΠΈΡ‚Π΅Π»ΡŒ, согрСвая ΠΏΠΎΠ» ΠΈ Π²ΠΎΠ·Π΄ΡƒΡ… помСщСния сразу Π½Π° большой ΠΏΠ»ΠΎΡ‰Π°Π΄ΠΈ. ΠŸΡ€ΠΈ устройствС напольного отоплСния Ρ‚Ρ€ΡƒΠ±Ρ‹ ΡƒΠΊΠ»Π°Π΄Ρ‹Π²Π°ΡŽΡ‚ΡΡ ΠΏΠΎ всСй повСрхности ΠΏΠΎΠ»Π° ΠΏΠ°Ρ€Π°Π»Π»Π΅Π»ΡŒΠ½ΠΎ (Π·ΠΈΠ³Π·Π°Π³ΠΎΠΌ) ΠΈΠ»ΠΈ ΡΠΏΠΈΡ€Π°Π»ΡŒΡŽ. Π˜ΡΠΏΠΎΠ»ΡŒΠ·ΡƒΡŽΡ‚ΡΡ Π³ΠΈΠ±ΠΊΠΈΠ΅ мСталлопластиковыС Ρ‚Ρ€ΡƒΠ±Ρ‹, ΠΊΠΎΡ‚ΠΎΡ€Ρ‹Π΅ ΡƒΠΊΠ»Π°Π΄Ρ‹Π²Π°ΡŽΡ‚ΡΡ ΠΈΠ· ΠΎΠ΄Π½ΠΎΠ³ΠΎ Ρ†Π΅Π»ΠΎΠ³ΠΎ куска Ρ‚Ρ€ΡƒΠ±Ρ‹, Π±Π΅Π· ΡΠΎΠ΅Π΄ΠΈΠ½ΠΈΡ‚Π΅Π»ΡŒΠ½Ρ‹Ρ… Π΄Π΅Ρ‚Π°Π»Π΅ΠΉ.

ΠžΠ±Ρ‹Ρ‡Π½ΠΎ стандартноС расстояниС ΠΌΠ΅ΠΆΠ΄Ρƒ Ρ‚Ρ€ΡƒΠ±Π°ΠΌΠΈ (шаг ΡƒΠΊΠ»Π°Π΄ΠΊΠΈ) – 20 см. Для Π±ΠΎΠ»Π΅Π΅ Ρ€Π°Π²Π½ΠΎΠΌΠ΅Ρ€Π½ΠΎΠ³ΠΎ Π½Π°Π³Ρ€Π΅Π²Π° ΠΏΠΎΠ»Π° шаг ΡƒΠΊΠ»Π°Π΄ΠΊΠΈ ΠΌΠΎΠΆΠ½ΠΎ ΡƒΠΌΠ΅Π½ΡŒΡˆΠΈΡ‚ΡŒ Ρƒ Π½Π°Ρ€ΡƒΠΆΠ½ΠΎΠΉ стСны ΠΈΠ»ΠΈ ΠΎΠΊΠΎΠ»ΠΎ ΠΎΠΊΠ½Π°. Π’Ρ€ΡƒΠ±Ρ‹ ΡƒΠΊΠ»Π°Π΄Ρ‹Π²Π°ΡŽΡ‚ΡΡ Π½Π° слой тСплоизолятора (Π½Π°ΠΏΡ€ΠΈΠΌΠ΅Ρ€, пСнопласта ΠΈΠ»ΠΈ Ρ„ΠΎΠ»ΡŒΠ³ΠΈΡ€ΠΎΠ²Π°Π½Π½ΠΎΠ³ΠΎ пСнофлСкса), ΠΏΠΎΠ΄ΡΠΎΠ΅Π΄ΠΈΠ½ΡΡŽΡ‚ΡΡ ΠΊ ΠΎΠ±Ρ‰Π΅ΠΉ систСмС отоплСния ΠΈ Π·Π°ΠΏΠΎΠ»Π½ΡΡŽΡ‚ΡΡ тСплоноситСлСм ΠΏΠΎΠ΄ Π΄Π°Π²Π»Π΅Π½ΠΈΠ΅ΠΌ. Π§Ρ‚ΠΎΠ±Ρ‹ «ΠΏΡ€ΠΎΠΊΠ°Ρ‡Π°Ρ‚ΡŒ» Ρ‚Π΅ΠΏΠ»ΠΎΠ½ΠΎΡΠΈΡ‚Π΅Π»ΡŒ ΠΏΠΎ этому довольно Π΄Π»ΠΈΠ½Π½ΠΎΠΌΡƒ участку Ρ‚ΠΎΠ½ΠΊΠΎΠΉ Ρ‚Ρ€ΡƒΠ±Ρ‹ ΠΈ Π½Π΅ Π΄ΠΎΠΏΡƒΡΡ‚ΠΈΡ‚ΡŒ ΠΏΠ΅Ρ€Π΅Π³Ρ€Π΅Π²Π° ΠΏΠΎΠ»ΠΎΠ² Π² Π΄ΠΎΠΌΠ΅, Π½ΡƒΠΆΠ½ΠΎ ΡΠΌΠΎΠ½Ρ‚ΠΈΡ€ΠΎΠ²Π°Ρ‚ΡŒ Π² ΠΊΠΎΡ‚Π΅Π»ΡŒΠ½ΠΎΠΉ Π΄ΠΎΠΏΠΎΠ»Π½ΠΈΡ‚Π΅Π»ΡŒΠ½ΡƒΡŽ Π½Π°ΡΠΎΡΠ½ΡƒΡŽ Π³Ρ€ΡƒΠΏΠΏΡƒ ΠΈ Π³Ρ€ΡƒΠΏΠΏΡƒ подмСшивания горячСго тСплоноситСля, Ρ‡Ρ‚ΠΎΠ±Ρ‹ Π±Ρ‹Π»Π° Π²ΠΎΠ·ΠΌΠΎΠΆΠ½ΠΎΡΡ‚ΡŒ рСгулирования Ρ‚Π΅ΠΌΠΏΠ΅Ρ€Π°Ρ‚ΡƒΡ€Ρ‹.

Волько послС этого ΠΏΠΎΠ²Π΅Ρ€Ρ…Π½ΠΎΡΡ‚ΡŒ ΠΏΠΎΠ»Π° с Π·Π°ΠΏΠΎΠ»Π½Π΅Π½Π½Ρ‹ΠΌΠΈ Ρ‚Ρ€ΡƒΠ±Π°ΠΌΠΈ армируСтся ΠΈ заливаСтся Π±Π΅Ρ‚ΠΎΠ½Π½Ρ‹ΠΌ ΠΈΠ»ΠΈ Ρ†Π΅ΠΌΠ΅Π½Ρ‚Π½Ρ‹ΠΌ ΠΏΠΎΠΊΡ€Ρ‹Ρ‚ΠΈΠ΅ΠΌ (стяТкой). Π—Π°Ρ‚Π΅ΠΌ ΡƒΠΊΠ»Π°Π΄Ρ‹Π²Π°ΡŽΡ‚ Π²Π΅Ρ€Ρ…Π½ΠΈΠΉ ΠΎΡ‚Π΄Π΅Π»ΠΎΡ‡Π½Ρ‹ΠΉ слой. Для покрытия Ρ‚Π΅ΠΏΠ»Ρ‹Ρ… ΠΏΠΎΠ»ΠΎΠ² Π»ΡƒΡ‡ΡˆΠ΅ ΠΈΡΠΏΠΎΠ»ΡŒΠ·ΠΎΠ²Π°Ρ‚ΡŒ ΠΊΠ΅Ρ€Π°ΠΌΠΈΡ‡Π΅ΡΠΊΡƒΡŽ ΠΏΠ»ΠΈΡ‚ΠΊΡƒ.

Π‘ΡƒΡ‰Π΅ΡΡ‚Π²ΡƒΡŽΡ‚ ограничСния для Ρ‚Π΅ΠΌΠΏΠ΅Ρ€Π°Ρ‚ΡƒΡ€Ρ‹ Ρ‚Π΅ΠΏΠ»ΠΎΠ³ΠΎ ΠΏΠΎΠ»Π°. Π₯ΠΎΠ΄ΠΈΡ‚ΡŒ ΠΏΠΎ ΠΏΠΎΠ»Ρƒ, Π½Π°Π³Ρ€Π΅Ρ‚ΠΎΠΌΡƒ Π²Ρ‹ΡˆΠ΅ +30 градусов, нСприятно ΠΈ Π½Π΅ΠΏΠΎΠ»Π΅Π·Π½ΠΎ. ΠŸΠΎΡΡ‚ΠΎΠΌΡƒ напольного отоплСния Π±ΡƒΠ΄Π΅Ρ‚ нСдостаточно для ΠΎΠ±ΠΎΠ³Ρ€Π΅Π²Π° помСщСния Π² самоС Ρ…ΠΎΠ»ΠΎΠ΄Π½ΠΎΠ΅ врСмя, ΠΈ ΠΎΠ½ΠΎ Π±ΡƒΠ΄Π΅Ρ‚ Ρ‚ΠΎΠ»ΡŒΠΊΠΎ Π΄ΠΎΠΏΠΎΠ»Π½ΠΈΡ‚Π΅Π»ΡŒΠ½Ρ‹ΠΌ, для ΠΏΠΎΠ²Ρ‹ΡˆΠ΅Π½ΠΈΡ комфортности. Π’ климатичСском поясС НиТСгородской области, ΠΊΠ°ΠΊ ΠΏΡ€Π°Π²ΠΈΠ»ΠΎ, Π²Β  ΠΆΠΈΠ»Ρ‹Ρ… помСщСниях вмСстС с Ρ‚Π΅ΠΏΠ»Ρ‹ΠΌΠΈ ΠΏΠΎΠ»Π°ΠΌΠΈ (ΠΊΡ€ΠΎΠΌΠ΅ Ρ‚ΡƒΠ°Π»Π΅Ρ‚ΠΎΠ², сан. ΡƒΠ·Π»ΠΎΠ², ΠΊΠ»Π°Π΄ΠΎΠ²ΠΎΠΊ) ΡƒΡΡ‚Π°Π½Π°Π²Π»ΠΈΠ²Π°ΡŽΡ‚ Π΅Ρ‰Π΅ ΠΈ Ρ€Π°Π΄ΠΈΠ°Ρ‚ΠΎΡ€Ρ‹.

Β 

Π¨Π°Π³ пятый: ΠŸΡ€ΠΈΠ΄Ρ‚ΠΈ ΠΈ ΠΊΡƒΠΏΠΈΡ‚ΡŒ!

Наша компания занимаСтся ΠΏΡ€ΠΎΠ΄Π°ΠΆΠ΅ΠΉ, ΠΏΡ€ΠΎΠ΅ΠΊΡ‚ΠΈΡ€ΠΎΠ²Π°Π½ΠΈΠ΅ΠΌ ΠΈΒ  ΠΌΠΎΠ½Ρ‚Π°ΠΆΠ΅ΠΌ ΠΎΡ‚ΠΎΠΏΠΈΡ‚Π΅Π»ΡŒΠ½ΠΎΠ³ΠΎ оборудования элитных СвропСйских Ρ‚ΠΎΡ€Π³ΠΎΠ²Ρ‹Ρ… ΠΌΠ°Ρ€ΠΎΠΊ: Π‘Π’Π‘ (ШвСция), Bentone (ШвСция), Buderus (ГСрмания), Viessmann (ГСрмания), являясь ΠΈΡ… ΠΎΡ„ΠΈΡ†ΠΈΠ°Π»ΡŒΠ½Ρ‹ΠΌ Π΄ΠΈΠ»Π΅Ρ€ΠΎΠΌ.

Π–Π΄Π΅ΠΌ Вас Ρƒ нас Π² салонС. О Ρ†Π΅Π½Π°Ρ… ΠΈ скидках договоримся!

Как ΠΏΠΎΠ²Ρ‹ΡΠΈΡ‚ΡŒ Ρ‚Π΅ΠΏΠ»ΠΎΠΎΡ‚Π΄Π°Ρ‡Ρƒ Ρ‡ΡƒΠ³ΡƒΠ½Π½Ρ‹Ρ… Ρ€Π°Π΄ΠΈΠ°Ρ‚ΠΎΡ€ΠΎΠ² отоплСния

Автор admin На Ρ‡Ρ‚Π΅Π½ΠΈΠ΅ 2 ΠΌΠΈΠ½. ΠŸΡ€ΠΎΡΠΌΠΎΡ‚Ρ€ΠΎΠ² 16 ОбновлСно

ΠŸΠΎΡ€ΠΎΠΉ Ρ‚Ρ€ΡƒΠ΄Π½ΠΎ ΠΏΠΎΠ΄ΠΎΠ±Ρ€Π°Ρ‚ΡŒ ΠΎΠΏΡ‚ΠΈΠΌΠ°Π»ΡŒΠ½ΡƒΡŽ модСль Ρ€Π°Π΄ΠΈΠ°Ρ‚ΠΎΡ€Π° отоплСния. Π’ Π±ΠΎΠ»ΡŒΡˆΠΈΠ½ΡΡ‚Π²Π΅ случаСв учитываСтся нСсколько Ρ„Π°ΠΊΡ‚ΠΎΡ€ΠΎΠ² – ΡΠ»ΠΎΠΆΠ½ΠΎΡΡ‚ΡŒ ΠΌΠΎΠ½Ρ‚Π°ΠΆΠ°, срок эксплуатации ΠΈ Ρ‚Π΅ΠΏΠ»ΠΎΠΎΡ‚Π΄Π°Ρ‡Π°. ПослСдний ΠΏΠΎΠΊΠ°Π·Π°Ρ‚Π΅Π»ΡŒ являСтся Π½Π°ΠΈΠ±ΠΎΠ»Π΅Π΅ Π²Π°ΠΆΠ½Ρ‹ΠΌ, Ρ‚Π°ΠΊ ΠΊΠ°ΠΊ ΠΈΠΌΠ΅Π½Π½ΠΎ ΠΎΡ‚ Π½Π΅Π³ΠΎ Π±ΡƒΠ΄Π΅Ρ‚ Π·Π°Π²ΠΈΡΠ΅Ρ‚ΡŒ ΡΡ„Ρ„Π΅ΠΊΡ‚ΠΈΠ²Π½ΠΎΡΡ‚ΡŒ Ρ€Π°Π±ΠΎΡ‚Ρ‹ ΠΏΡ€ΠΈΠ±ΠΎΡ€Π°.

Π‘ появлСниСм Π½ΠΎΠ²Ρ‹Ρ… ΠΌΠ°Ρ‚Π΅Ρ€ΠΈΠ°Π»ΠΎΠ² изготовлСния Ρ€Π°Π΄ΠΈΠ°Ρ‚ΠΎΡ€ΠΎΠ² (Π°Π»ΡŽΠΌΠΈΠ½ΠΈΠ΅Π²Ρ‹Π΅, бимСталличСскиС) Ρ‡ΡƒΠ³ΡƒΠ½Π½Ρ‹Π΅ ΠΎΡ‚ΠΎΡˆΠ»ΠΈ Π½Π° Β«Π²Ρ‚ΠΎΡ€ΠΎΠΉ ΠΏΠ»Π°Π½Β». Но ΠΈΡ… ΡƒΠ½ΠΈΠΊΠ°Π»ΡŒΠ½Ρ‹Π΅ эксплуатационныС характСристики вновь заставили ΠΏΠΎΠΊΡƒΠΏΠ°Ρ‚Π΅Π»Π΅ΠΉ ΠΎΠ±Ρ€Π°Ρ‚ΠΈΡ‚ΡŒ Π½Π° сСбя Π²Π½ΠΈΠΌΠ°Π½ΠΈΠ΅. ΠŸΡ€Π΅ΠΆΠ΄Π΅ всСго, это Ρ…ΠΎΡ€ΠΎΡˆΠΈΠ΅ эксплуатационныС свойства. Π’ ΠΎΡ‚Π»ΠΈΡ‡ΠΈΠ΅ ΠΎΡ‚ алюминия ΠΈ ΠΌΠ΅Ρ‚Π°Π»Π»Π° Ρ‡ΡƒΠ³ΡƒΠ½ ΠΌΠΎΠΆΠ΅Ρ‚ Π°ΠΊΠΊΡƒΠΌΡƒΠ»ΠΈΡ€ΠΎΠ²Π°Ρ‚ΡŒ Ρ‚Π΅ΠΏΠ»ΠΎ ΠΈ ΠΏΡ€ΠΈ ΠΏΠΎΠ½ΠΈΠΆΠ΅Π½ΠΈΠΈ Ρ‚Π΅ΠΌΠΏΠ΅Ρ€Π°Ρ‚ΡƒΡ€Ρ‹ Π²ΠΎΠ΄Ρ‹ Π΅Ρ‰Π΅ Π½Π΅ΠΊΠΎΡ‚ΠΎΡ€ΠΎΠ΅ врСмя Ρ€Π°Π΄ΠΈΠ°Ρ‚ΠΎΡ€Ρ‹ Π±ΡƒΠ΄ΡƒΡ‚ Ρ‚Π΅ΠΏΠ»Ρ‹ΠΌΠΈ.

Но вСрнСмся ΠΊ вопросу Ρ‚Π΅ΠΏΠ»ΠΎΠΎΡ‚Π΄Π°Ρ‡ΠΈ. Π‘ ΠΏΠΎΠ΄Ρ€ΠΎΠ±Π½ΠΎΠΉ ΠΌΠ΅Ρ‚ΠΎΠ΄ΠΈΠΊΠΎΠΉ расчСта ΠΌΠΎΠΆΠ½ΠΎ ΠΎΠ·Π½Π°ΠΊΠΎΠΌΠΈΡ‚ΡŒΡΡ Π² спСциализированной ΡΡ‚Π°Ρ‚ΡŒΠ΅ Π½Π° эту Ρ‚Π΅ΠΌΡƒ , Π² ΠΊΠΎΡ‚ΠΎΡ€ΠΎΠΉΒ ΠΏΠΎΠ΄Ρ€ΠΎΠ±Π½ΠΎ ΠΈΠ·Π»ΠΎΠΆΠ΅Π½Π° ΠΌΠ΅Ρ‚ΠΎΠ΄ΠΈΠΊΠ° расчСта ΠΈ ΡƒΠΊΠ°Π·Π°Π½Ρ‹ способы увСличСния этого показатСля.

[box type=”info” ]ΠŸΡ€Π°ΠΊΡ‚ΠΈΡ‡Π΅ΡΠΊΠΈ всС ΠΏΡ€ΠΎΠΈΠ·Π²ΠΎΠ΄ΠΈΡ‚Π΅Π»ΠΈ ΡƒΠΊΠ°Π·Ρ‹Π²Π°ΡŽΡ‚ Π½ΠΎΠΌΠΈΠ½Π°Π»ΡŒΠ½Ρ‹Π΅ Π·Π½Π°Ρ‡Π΅Π½ΠΈΠ΅ тСплопроводности ΠΏΡ€ΠΈ ΠΈΠ΄Π΅Π°Π»ΡŒΠ½Ρ‹Ρ… Ρ‚Π΅ΠΌΠΏΠ΅Ρ€Π°Ρ‚ΡƒΡ€Π½Ρ‹Ρ… Ρ€Π΅ΠΆΠΈΠΌΠ°Ρ… – 90Β°Π‘. Однако фактичСски Π΄ΠΎΠ±ΠΈΡ‚ΡŒΡΡ этого ΠΎΡ‚ поставщиков Ρ‚Π΅ΠΏΠ»Π° Π² ΠΌΠ½ΠΎΠ³ΠΎΠΊΠ²Π°Ρ€Ρ‚ΠΈΡ€Π½Ρ‹Ρ… Π΄ΠΎΠΌΠ°Ρ… ΠΏΡ€ΠΎΠ±Π»Π΅ΠΌΠ°Ρ‚ΠΈΡ‡Π½ΠΎ.[/box]

ВслСдствиС этого ΠΏΠΎΠΊΠ°Π·Π°Ρ‚Π΅Π»ΠΈ Π½Π°Π³Ρ€Π΅Π²Π° ΠΊΠΎΠΌΠ½Π°Ρ‚Ρ‹ сущСствСнно ΠΎΡ‚Π»ΠΈΡ‡Π°ΡŽΡ‚ΡΡ ΠΎΡ‚ расчСтных. Π’ этом случаС ΠΌΠΎΠΆΠ½ΠΎ Π²ΠΎΡΠΏΠΎΠ»ΡŒΠ·ΠΎΠ²Π°Ρ‚ΡŒΡΡ нСсколькими нСбольшими Β«ΡƒΠ»ΠΎΠ²ΠΊΠ°ΠΌΠΈΒ», ΠΊΠΎΡ‚ΠΎΡ€Ρ‹Π΅ ΠΌΠΎΠ³ΡƒΡ‚ ΠΏΠΎΠ²Ρ‹ΡΠΈΡ‚ΡŒ Ρ‚Π΅ΠΌΠΏΠ΅Ρ€Π°Ρ‚ΡƒΡ€Ρƒ Π² ΠΊΠΎΠΌΠ½Π°Ρ‚Π΅ ΠΏΡ€ΠΈ Ρ‚Π΅ΠΊΡƒΡ‰ΠΈΡ… показатСлях систСмы отоплСния.

Установка ΠΎΡ‚Ρ€Π°ΠΆΠ°ΡŽΡ‰Π΅Π³ΠΎ экрана

Для Ρ‚ΠΎΠ³ΠΎ Ρ‡Ρ‚ΠΎΠ±Ρ‹ тСпловая энСргия Π½Π΅ ΠΏΠΎΠ³Π»ΠΎΡ‰Π°Π»Π°ΡΡŒ стСной, Π½Π° Π½Π΅Π΅ ΠΌΠΎΠΆΠ½ΠΎ ΡƒΡΡ‚Π°Π½ΠΎΠ²ΠΈΡ‚ΡŒ ΠΎΡ‚Ρ€Π°ΠΆΠ°ΡŽΡ‰ΠΈΠΉ экран ΠΈΠ· Ρ„ΠΎΠ»ΡŒΠ³ΠΈ.

Π’ этом случаС повысится эффСктивная Ρ‚Π΅ΠΏΠ»ΠΎΠΎΡ‚Π΄Π°Ρ‡Π° Ρ€Π°Π΄ΠΈΠ°Ρ‚ΠΎΡ€Π° – Π½Π° 5-10%. Но ΠΏΡ€ΠΈ этом стоит ΠΏΠΎΠΌΠ½ΠΈΡ‚ΡŒ, Ρ‡Ρ‚ΠΎ Ссли стСна наруТная, Ρ‚ΠΎ Π±Π΅Π· Π΄ΠΎΠ»ΠΆΠ½ΠΎΠ³ΠΎ ΠΎΠ±ΠΎΠ³Ρ€Π΅Π²Π° ΠΎΠ½Π° ΠΌΠΎΠΆΠ΅Ρ‚ ΡΡ‚Π°Ρ‚ΡŒ ΠΏΡ€ΠΈΡ‡ΠΈΠ½ΠΎΠΉ Ρ‚Π΅ΠΏΠ»ΠΎΠ²Ρ‹Ρ… ΠΏΠΎΡ‚Π΅Ρ€ΡŒ Π² ΠΊΠΎΠΌΠ½Π°Ρ‚Π΅.

Установка вСнтилятора

ΠžΠ±ΠΎΠ³Ρ€Π΅Π² помСщСния ΠΎΡ‚ Ρ‡ΡƒΠ³ΡƒΠ½Π½Ρ‹Ρ… Ρ€Π°Π΄ΠΈΠ°Ρ‚ΠΎΡ€ΠΎΠ² происходит с ΠΏΠΎΠΌΠΎΡ‰ΡŒΡŽ СстСствСнной ΠΊΠΎΠ½Π²Π΅ΠΊΡ†ΠΈΠΈ. Для увСличСния прохоТдСния Π²ΠΎΠ·Π΄ΡƒΡˆΠ½Ρ‹Ρ… масс Ρ‡Π΅Ρ€Π΅Π· сСкции ΠΏΡ€ΠΈΠ±ΠΎΡ€Π° ΠΌΠΎΠΆΠ½ΠΎ ΡƒΡΡ‚Π°Π½ΠΎΠ²ΠΈΡ‚ΡŒ нСбольшой вСнтилятор Π½Π° стСну ΠΏΠΎΠ·Π°Π΄ΠΈ Ρ€Π°Π΄ΠΈΠ°Ρ‚ΠΎΡ€Π°. Π­Ρ‚ΠΎ нСсколько ΡƒΠ²Π΅Π»ΠΈΡ‡ΠΈΡ‚ Ρ‚Π΅ΠΌΠΏΠ΅Ρ€Π°Ρ‚ΡƒΡ€Ρƒ Π² ΠΊΠΎΠΌΠ½Π°Ρ‚Π΅, Π½ΠΎ ΠΈ ΠΎΠ΄Π½ΠΎΠ²Ρ€Π΅ΠΌΠ΅Π½Π½ΠΎ станСт ΠΏΡ€ΠΈΡ‡ΠΈΠ½ΠΎΠΉ остывания тСплоноситСля. ΠŸΠΎΠ΄ΠΎΠ±Π½Ρ‹ΠΉ ΠΌΠ΅Ρ‚ΠΎΠ΄ ΠΌΠΎΠΆΠ½ΠΎ ΠΏΡ€ΠΈΠΌΠ΅Π½ΡΡ‚ΡŒ для Ρ†Π΅Π½Ρ‚Ρ€Π°Π»ΡŒΠ½ΠΎΠΉ систСмы отоплСния.

Установка ΡΡ‚Π°Π»ΡŒΠ½Ρ‹Ρ… Π΄Π΅ΠΊΠΎΡ€Π°Ρ‚ΠΈΠ²Π½Ρ‹Ρ… ΠΊΠΎΠΆΡƒΡ…ΠΎΠ²

Они искусствСнно ΡƒΠ²Π΅Π»ΠΈΡ‡Π°Ρ‚ ΠΏΠ»ΠΎΡ‰Π°Π΄ΡŒ Ρ€Π°Π΄ΠΈΠ°Ρ‚ΠΎΡ€Π° ΠΈ Π±ΡƒΠ΄ΡƒΡ‚ ΡΠΏΠΎΡΠΎΠ±ΡΡ‚Π²ΠΎΠ²Π°Ρ‚ΡŒ Π»ΡƒΡ‡ΡˆΠ΅ΠΉ Ρ‚Π΅ΠΏΠ»ΠΎΠΎΡ‚Π΄Π°Ρ‡ΠΈ. ΠžΠ΄Π½ΠΎΠ²Ρ€Π΅ΠΌΠ΅Π½Π½ΠΎ с этим увСличится врСмя Π½Π°Π³Ρ€Π΅Π²Π°, Ρ‡Ρ‚ΠΎ скаТСтся Π½Π° инСрционности ΠΎΠ±ΠΎΠ³Ρ€Π΅Π²Π° ΠΊΠΎΠΌΠ½Π°Ρ‚Ρ‹ ΠΎΡ‚ Π°Π²Ρ‚ΠΎΠ½ΠΎΠΌΠ½ΠΎΠΉ систСмы отоплСния.

Π­Ρ‚ΠΎ лишь нСсколько ΠΌΠ΅Ρ‚ΠΎΠ΄ΠΎΠ² искусствСнного увСличСния Ρ‚Π΅ΠΏΠ»ΠΎΠΎΡ‚Π΄Π°Ρ‡ΠΈ Ρ‡ΡƒΠ³ΡƒΠ½Π½Ρ‹Ρ… Ρ€Π°Π΄ΠΈΠ°Ρ‚ΠΎΡ€ΠΎΠ². Но самым дСйствСнным Π±ΡƒΠ΄Π΅Ρ‚ соблюдСниС Ρ‚Π΅ΠΌΠΏΠ΅Ρ€Π°Ρ‚ΡƒΡ€Ρ‹ тСплоноситСля. Для этого Π½Π΅ΠΎΠ±Ρ…ΠΎΠ΄ΠΈΠΌΠΎ Π»ΠΈΠ±ΠΎ ΡƒΠ»ΡƒΡ‡ΡˆΠΈΡ‚ΡŒ качСство прСдоставляСмых услуг ΡƒΠΏΡ€Π°Π²Π»ΡΡŽΡ‰Π΅ΠΉ ΠΊΠΎΠΌΠΏΠ°Π½ΠΈΠΈ ΠΏΡ€ΠΈ Ρ†Π΅Π½Ρ‚Ρ€Π°Π»ΡŒΠ½ΠΎΠΉ систСмС отоплСния, Π»ΠΈΠ±ΠΎ Π΄Π΅Π»Π°Ρ‚ΡŒ Π°Π²Ρ‚ΠΎΠ½ΠΎΠΌΠ½ΠΎΠ΅.

ΠšΠΎΠ½Π²Π΅ΠΊΡ†ΠΈΡ | Π€ΠΈΠ·ΠΈΠΊΠ°

Π¦Π΅Π»ΠΈ обучСния

К ΠΊΠΎΠ½Ρ†Ρƒ этого Ρ€Π°Π·Π΄Π΅Π»Π° Π²Ρ‹ смоТСтС:

  • ΠžΠ±ΡΡƒΠ΄ΠΈΡ‚Π΅ ΠΌΠ΅Ρ‚ΠΎΠ΄ ΠΏΠ΅Ρ€Π΅Π΄Π°Ρ‡ΠΈ Ρ‚Π΅ΠΏΠ»Π° ΠΊΠΎΠ½Π²Π΅ΠΊΡ†ΠΈΠ΅ΠΉ.

ΠšΠΎΠ½Π²Π΅ΠΊΡ†ΠΈΡ вызываСтся ΠΊΡ€ΡƒΠΏΠ½ΠΎΠΌΠ°ΡΡˆΡ‚Π°Π±Π½Ρ‹ΠΌ ΠΏΠΎΡ‚ΠΎΠΊΠΎΠΌ вСщСства. Π’ случаС с Π—Π΅ΠΌΠ»Π΅ΠΉ атмосфСрная циркуляция вызываСтся ΠΏΠΎΡ‚ΠΎΠΊΠΎΠΌ горячСго Π²ΠΎΠ·Π΄ΡƒΡ…Π° ΠΎΡ‚ Ρ‚Ρ€ΠΎΠΏΠΈΠΊΠΎΠ² ΠΊ полюсам ΠΈ ΠΏΠΎΡ‚ΠΎΠΊΠΎΠΌ Ρ…ΠΎΠ»ΠΎΠ΄Π½ΠΎΠ³ΠΎ Π²ΠΎΠ·Π΄ΡƒΡ…Π° ΠΎΡ‚ полюсов ΠΊ Ρ‚Ρ€ΠΎΠΏΠΈΠΊΠ°ΠΌ. (ΠžΠ±Ρ€Π°Ρ‚ΠΈΡ‚Π΅ Π²Π½ΠΈΠΌΠ°Π½ΠΈΠ΅, Ρ‡Ρ‚ΠΎ Π²Ρ€Π°Ρ‰Π΅Π½ΠΈΠ΅ Π—Π΅ΠΌΠ»ΠΈ Π²Ρ‹Π·Ρ‹Π²Π°Π΅Ρ‚ Π½Π°Π±Π»ΡŽΠ΄Π°Π΅ΠΌΡ‹ΠΉ восточный ΠΏΠΎΡ‚ΠΎΠΊ Π²ΠΎΠ·Π΄ΡƒΡ…Π° Π² сСвСрном ΠΏΠΎΠ»ΡƒΡˆΠ°Ρ€ΠΈΠΈ).ΠΠ²Ρ‚ΠΎΠΌΠΎΠ±ΠΈΠ»ΡŒΠ½Ρ‹Π΅ Π΄Π²ΠΈΠ³Π°Ρ‚Π΅Π»ΠΈ ΠΎΡ…Π»Π°ΠΆΠ΄Π°ΡŽΡ‚ΡΡ ΠΏΠΎΡ‚ΠΎΠΊΠΎΠΌ Π²ΠΎΠ΄Ρ‹ Π² систСмС охлаТдСния, Π° водяной насос ΠΏΠΎΠ΄Π΄Π΅Ρ€ΠΆΠΈΠ²Π°Π΅Ρ‚ ΠΏΠΎΡ‚ΠΎΠΊ Ρ…ΠΎΠ»ΠΎΠ΄Π½ΠΎΠΉ Π²ΠΎΠ΄Ρ‹ ΠΊ ΠΏΠΎΡ€ΡˆΠ½ΡΠΌ. БистСма кровообращСния ΠΈΡΠΏΠΎΠ»ΡŒΠ·ΡƒΠ΅Ρ‚ΡΡ Ρ‚Π΅Π»ΠΎΠΌ: ΠΊΠΎΠ³Π΄Π° Ρ‚Π΅Π»ΠΎ пСрСгрСваСтся, кровСносныС сосуды Π² ΠΊΠΎΠΆΠ΅ Ρ€Π°ΡΡˆΠΈΡ€ΡΡŽΡ‚ΡΡ (Ρ€Π°ΡΡˆΠΈΡ€ΡΡŽΡ‚ΡΡ), Ρ‡Ρ‚ΠΎ ΡƒΠ²Π΅Π»ΠΈΡ‡ΠΈΠ²Π°Π΅Ρ‚ ΠΏΡ€ΠΈΡ‚ΠΎΠΊ ΠΊΡ€ΠΎΠ²ΠΈ ΠΊ ΠΊΠΎΠΆΠ΅, Π³Π΄Π΅ Π΅Π΅ ΠΌΠΎΠΆΠ½ΠΎ ΠΎΡ…Π»Π°Π΄ΠΈΡ‚ΡŒ Π·Π° счСт потоотдСлСния. Π­Ρ‚ΠΈ сосуды становятся мСньшС, ΠΊΠΎΠ³Π΄Π° Π½Π° ΡƒΠ»ΠΈΡ†Π΅ Ρ…ΠΎΠ»ΠΎΠ΄Π½ΠΎ, ΠΈ большС, ΠΊΠΎΠ³Π΄Π° ΠΆΠ°Ρ€ΠΊΠΎ (поэтому Ρ‚Π΅Ρ‡Π΅Ρ‚ большС Тидкости ΠΈ пСрСдаСтся большС энСргии).

Π’Π΅Π»ΠΎ Ρ‚Π°ΠΊΠΆΠ΅ тСряСт Π·Π½Π°Ρ‡ΠΈΡ‚Π΅Π»ΡŒΠ½ΡƒΡŽ Ρ‡Π°ΡΡ‚ΡŒ своСго Ρ‚Π΅ΠΏΠ»Π° Π² процСссС дыхания.

Π₯отя конвСкция ΠΎΠ±Ρ‹Ρ‡Π½ΠΎ слоТнСС, Ρ‡Π΅ΠΌ Ρ‚Π΅ΠΏΠ»ΠΎΠΏΡ€ΠΎΠ²ΠΎΠ΄Π½ΠΎΡΡ‚ΡŒ, ΠΌΡ‹ ΠΌΠΎΠΆΠ΅ΠΌ ΠΎΠΏΠΈΡΠ°Ρ‚ΡŒ ΠΊΠΎΠ½Π²Π΅ΠΊΡ†ΠΈΡŽ ΠΈ ΡΠ΄Π΅Π»Π°Ρ‚ΡŒ нСсколько простых, рСалистичных расчСтов Π΅Π΅ эффСктов. ЕстСствСнная конвСкция вызываСтся Π²Ρ‹Ρ‚Π°Π»ΠΊΠΈΠ²Π°ΡŽΡ‰ΠΈΠΌΠΈ силами: горячий Π²ΠΎΠ·Π΄ΡƒΡ… поднимаСтся Π²Π²Π΅Ρ€Ρ…, ΠΏΠΎΡ‚ΠΎΠΌΡƒ Ρ‡Ρ‚ΠΎ ΠΏΠ»ΠΎΡ‚Π½ΠΎΡΡ‚ΡŒ ΡƒΠΌΠ΅Π½ΡŒΡˆΠ°Π΅Ρ‚ΡΡ с ΡƒΠ²Π΅Π»ΠΈΡ‡Π΅Π½ΠΈΠ΅ΠΌ Ρ‚Π΅ΠΌΠΏΠ΅Ρ€Π°Ρ‚ΡƒΡ€Ρ‹. Π’Π°ΠΊΠΈΠΌ ΠΎΠ±Ρ€Π°Π·ΠΎΠΌ, Π΄ΠΎΠΌ Π½Π° РисункС 1 поддСрТиваСтся Π² Ρ‚Π΅ΠΏΠ»Π΅, ΠΊΠ°ΠΊ ΠΈ Π³ΠΎΡ€ΡˆΠΎΠΊ с Π²ΠΎΠ΄ΠΎΠΉ Π½Π° ΠΏΠ»ΠΈΡ‚Π΅ Π½Π° РисункС 2. ОкСанскиС тСчСния ΠΈ ΠΊΡ€ΡƒΠΏΠ½ΠΎΠΌΠ°ΡΡˆΡ‚Π°Π±Π½Π°Ρ атмосфСрная циркуляция ΠΏΠ΅Ρ€Π΅Π΄Π°ΡŽΡ‚ ΡΠ½Π΅Ρ€Π³ΠΈΡŽ ΠΈΠ· ΠΎΠ΄Π½ΠΎΠΉ части Π·Π΅ΠΌΠ½ΠΎΠ³ΠΎ ΡˆΠ°Ρ€Π° Π² Π΄Ρ€ΡƒΠ³ΡƒΡŽ. Оба ΡΠ²Π»ΡΡŽΡ‚ΡΡ ΠΏΡ€ΠΈΠΌΠ΅Ρ€Π°ΠΌΠΈ СстСствСнной ΠΊΠΎΠ½Π²Π΅ΠΊΡ†ΠΈΠΈ.

Рис. 1. Π’ΠΎΠ·Π΄ΡƒΡ…, Π½Π°Π³Ρ€Π΅Ρ‚Ρ‹ΠΉ Ρ‚Π°ΠΊ Π½Π°Π·Ρ‹Π²Π°Π΅ΠΌΠΎΠΉ Π³Ρ€Π°Π²ΠΈΡ‚Π°Ρ†ΠΈΠΎΠ½Π½ΠΎΠΉ ΠΏΠ΅Ρ‡ΡŒΡŽ, Ρ€Π°ΡΡˆΠΈΡ€ΡΠ΅Ρ‚ΡΡ ΠΈ поднимаСтся, образуя ΠΊΠΎΠ½Π²Π΅ΠΊΡ‚ΠΈΠ²Π½ΡƒΡŽ ΠΏΠ΅Ρ‚Π»ΡŽ, которая ΠΏΠ΅Ρ€Π΅Π΄Π°Π΅Ρ‚ ΡΠ½Π΅Ρ€Π³ΠΈΡŽ Π΄Ρ€ΡƒΠ³ΠΈΠΌ частям ΠΊΠΎΠΌΠ½Π°Ρ‚Ρ‹. По ΠΌΠ΅Ρ€Π΅ Ρ‚ΠΎΠ³ΠΎ, ΠΊΠ°ΠΊ Π²ΠΎΠ·Π΄ΡƒΡ… охлаТдаСтся Ρƒ ΠΏΠΎΡ‚ΠΎΠ»ΠΊΠ° ΠΈ Π²Π½Π΅ΡˆΠ½ΠΈΡ… стСн, ΠΎΠ½ сТимаСтся, Π² ΠΊΠΎΠ½Π΅Ρ‡Π½ΠΎΠΌ ΠΈΡ‚ΠΎΠ³Π΅ ΡΡ‚Π°Π½ΠΎΠ²ΡΡΡŒ Π±ΠΎΠ»Π΅Π΅ ΠΏΠ»ΠΎΡ‚Π½Ρ‹ΠΌ, Ρ‡Π΅ΠΌ Π²ΠΎΠ·Π΄ΡƒΡ… Π² ΠΏΠΎΠΌΠ΅Ρ‰Π΅Π½ΠΈΠΈ, ΠΈ опускаСтся Π½Π° ΠΏΠΎΠ». ΠŸΡ€Π°Π²ΠΈΠ»ΡŒΠ½ΠΎ спроСктированная систСма отоплСния с использованиСм СстСствСнной ΠΊΠΎΠ½Π²Π΅ΠΊΡ†ΠΈΠΈ, подобная этой, ΠΌΠΎΠΆΠ΅Ρ‚ Π±Ρ‹Ρ‚ΡŒ достаточно эффСктивной для Ρ€Π°Π²Π½ΠΎΠΌΠ΅Ρ€Π½ΠΎΠ³ΠΎ ΠΎΠ±ΠΎΠ³Ρ€Π΅Π²Π° Π΄ΠΎΠΌΠ°.

Рис. 2. ΠšΠΎΠ½Π²Π΅ΠΊΡ†ΠΈΡ ΠΈΠ³Ρ€Π°Π΅Ρ‚ Π²Π°ΠΆΠ½ΡƒΡŽ Ρ€ΠΎΠ»ΡŒ Π² Ρ‚Π΅ΠΏΠ»ΠΎΠΏΠ΅Ρ€Π΅Π΄Π°Ρ‡Π΅ Π²Π½ΡƒΡ‚Ρ€ΠΈ этого ΠΊΠΎΡ‚Π»Π° с Π²ΠΎΠ΄ΠΎΠΉ.Попадая Π²Π½ΡƒΡ‚Ρ€ΡŒ, ΠΏΠ΅Ρ€Π΅Π΄Π°Ρ‡Π° Ρ‚Π΅ΠΏΠ»Π° Π΄Ρ€ΡƒΠ³ΠΈΠΌ частям Π³ΠΎΡ€ΡˆΠΊΠ° происходит Π² основном Π·Π° счСт ΠΊΠΎΠ½Π²Π΅ΠΊΡ†ΠΈΠΈ. Π‘ΠΎΠ»Π΅Π΅ горячая Π²ΠΎΠ΄Π° Ρ€Π°ΡΡˆΠΈΡ€ΡΠ΅Ρ‚ΡΡ, ΡƒΠΌΠ΅Π½ΡŒΡˆΠ°Π΅Ρ‚ΡΡ ΠΏΠΎ плотности ΠΈ поднимаСтся, пСрСдавая Ρ‚Π΅ΠΏΠ»ΠΎ Π΄Ρ€ΡƒΠ³ΠΈΠΌ областям Π²ΠΎΠ΄Ρ‹, Π² Ρ‚ΠΎ врСмя ΠΊΠ°ΠΊ Π±ΠΎΠ»Π΅Π΅ холодная Π²ΠΎΠ΄Π° опускаСтся Π½Π° Π΄Π½ΠΎ. Π­Ρ‚ΠΎΡ‚ процСсс повторяСтся.

ЭкспСримСнт Π½Π° вынос: ΠΊΠΎΠ½Π²Π΅ΠΊΡ†ΠΈΠΎΠ½Π½Ρ‹Π΅ Ρ€ΠΎΠ»ΠΈΠΊΠΈ Π² ΠΏΠΎΠ΄ΠΎΠ³Ρ€Π΅Π²Π°Π΅ΠΌΠΎΠΉ сковородС

Π’ΠΎΠ·ΡŒΠΌΠΈΡ‚Π΅ Π΄Π²Π΅ малСнькиС Π³ΠΎΡ€ΡˆΠΊΠΈ с Π²ΠΎΠ΄ΠΎΠΉ ΠΈ с ΠΏΠΎΠΌΠΎΡ‰ΡŒΡŽ ΠΏΠΈΠΏΠ΅Ρ‚ΠΊΠΈ нанСситС каплю ΠΏΠΈΡ‰Π΅Π²ΠΎΠ³ΠΎ краситСля Π½Π° Π΄Π½ΠΎ ΠΊΠ°ΠΆΠ΄ΠΎΠΉ. ΠžΡΡ‚Π°Π²ΡŒΡ‚Π΅ ΠΎΠ΄ΠΈΠ½ Π½Π° ΡΡ‚ΠΎΠ»Π΅ΡˆΠ½ΠΈΡ†Π΅, Π° Π΄Ρ€ΡƒΠ³ΠΎΠΉ Π½Π°Π³Ρ€Π΅ΠΉΡ‚Π΅ Π½Π° ΠΏΠ»ΠΈΡ‚Π΅.ΠΠ°Π±Π»ΡŽΠ΄Π°ΠΉΡ‚Π΅, ΠΊΠ°ΠΊ Ρ†Π²Π΅Ρ‚ распространяСтся ΠΈ сколько Π²Ρ€Π΅ΠΌΠ΅Π½ΠΈ трСбуСтся, Ρ‡Ρ‚ΠΎΠ±Ρ‹ Ρ†Π²Π΅Ρ‚ достигал Π²Π΅Ρ€ΡˆΠΈΠ½Ρ‹. ΠΠ°Π±Π»ΡŽΠ΄Π°ΠΉΡ‚Π΅, ΠΊΠ°ΠΊ ΠΎΠ±Ρ€Π°Π·ΡƒΡŽΡ‚ΡΡ ΠΊΠΎΠ½Π²Π΅ΠΊΡ‚ΠΈΠ²Π½Ρ‹Π΅ ΠΏΠ΅Ρ‚Π»ΠΈ.

ΠŸΡ€ΠΈΠΌΠ΅Ρ€ 1. РасчСт Ρ‚Π΅ΠΏΠ»ΠΎΠΏΠ΅Ρ€Π΅Π΄Π°Ρ‡ΠΈ ΠΏΡƒΡ‚Π΅ΠΌ ΠΊΠΎΠ½Π²Π΅ΠΊΡ†ΠΈΠΈ: конвСкция Π²ΠΎΠ·Π΄ΡƒΡ…Π° Ρ‡Π΅Ρ€Π΅Π· стСны Π΄ΠΎΠΌΠ°

Π‘ΠΎΠ»ΡŒΡˆΠΈΠ½ΡΡ‚Π²ΠΎ Π΄ΠΎΠΌΠΎΠ² Π½Π΅ Π³Π΅Ρ€ΠΌΠ΅Ρ‚ΠΈΡ‡Π½Ρ‹: Π²ΠΎΠ·Π΄ΡƒΡ… Π²Ρ…ΠΎΠ΄ΠΈΡ‚ ΠΈ Π²Ρ‹Ρ…ΠΎΠ΄ΠΈΡ‚ Ρ‡Π΅Ρ€Π΅Π· Π΄Π²Π΅Ρ€ΠΈ ΠΈ ΠΎΠΊΠ½Π°, Ρ‡Π΅Ρ€Π΅Π· Ρ‚Ρ€Π΅Ρ‰ΠΈΠ½Ρ‹ ΠΈ Ρ‰Π΅Π»ΠΈ, ΠΏΠΎ ΠΏΡ€ΠΎΠ²ΠΎΠ΄ΠΊΠ΅ ΠΊ Π²Ρ‹ΠΊΠ»ΡŽΡ‡Π°Ρ‚Π΅Π»ΡΠΌ ΠΈ Ρ€ΠΎΠ·Π΅Ρ‚ΠΊΠ°ΠΌ ΠΈ Ρ‚Π°ΠΊ Π΄Π°Π»Π΅Π΅. Π’ΠΎΠ·Π΄ΡƒΡ… Π² Ρ‚ΠΈΠΏΠΈΡ‡Π½ΠΎΠΌ Π΄ΠΎΠΌΠ΅ ΠΏΠΎΠ»Π½ΠΎΡΡ‚ΡŒΡŽ замСняСтся ΠΌΠ΅Π½Π΅Π΅ Ρ‡Π΅ΠΌ Π·Π° час. ΠŸΡ€Π΅Π΄ΠΏΠΎΠ»ΠΎΠΆΠΈΠΌ, Ρ‡Ρ‚ΠΎ Π΄ΠΎΠΌ срСднСго Ρ€Π°Π·ΠΌΠ΅Ρ€Π° ΠΈΠΌΠ΅Π΅Ρ‚ Π²Π½ΡƒΡ‚Ρ€Π΅Π½Π½ΠΈΠ΅ Ρ€Π°Π·ΠΌΠ΅Ρ€Ρ‹ 12.0 ΠΌ Γ— 18,0 ΠΌ Γ— 3,00 ΠΌ Π² высоту, ΠΈ Ρ‡Ρ‚ΠΎ вСсь Π²ΠΎΠ·Π΄ΡƒΡ… замСняСтся Π·Π° 30,0 ΠΌΠΈΠ½. РассчитайтС Ρ‚Π΅ΠΏΠ»ΠΎΠΏΠ΅Ρ€Π΅Π΄Π°Ρ‡Ρƒ Π² Π²Π°Ρ‚Ρ‚Π°Ρ… Π·Π° Π΅Π΄ΠΈΠ½ΠΈΡ†Ρƒ Π²Ρ€Π΅ΠΌΠ΅Π½ΠΈ, Π½Π΅ΠΎΠ±Ρ…ΠΎΠ΄ΠΈΠΌΡƒΡŽ для Π½Π°Π³Ρ€Π΅Π²Π° входящСго Ρ…ΠΎΠ»ΠΎΠ΄Π½ΠΎΠ³ΠΎ Π²ΠΎΠ·Π΄ΡƒΡ…Π° Π½Π° 10,0 Β° C, замСняя Ρ‚Π΅ΠΌ самым Ρ‚Π΅ΠΏΠ»ΠΎ, ΠΏΠ΅Ρ€Π΅Π΄Π°Π²Π°Π΅ΠΌΠΎΠ΅ Ρ‚ΠΎΠ»ΡŒΠΊΠΎ ΠΊΠΎΠ½Π²Π΅ΠΊΡ†ΠΈΠ΅ΠΉ.

БтратСгия

Π’Π΅ΠΏΠ»ΠΎ ΠΈΡΠΏΠΎΠ»ΡŒΠ·ΡƒΠ΅Ρ‚ΡΡ для ΠΏΠΎΠ²Ρ‹ΡˆΠ΅Π½ΠΈΡ Ρ‚Π΅ΠΌΠΏΠ΅Ρ€Π°Ρ‚ΡƒΡ€Ρ‹ Π²ΠΎΠ·Π΄ΡƒΡ…Π° Ρ‚Π°ΠΊ, Ρ‡Ρ‚ΠΎΠ±Ρ‹ Q = mc Ξ” T . Π‘ΠΊΠΎΡ€ΠΎΡΡ‚ΡŒ Ρ‚Π΅ΠΏΠ»ΠΎΠΏΠ΅Ρ€Π΅Π΄Π°Ρ‡ΠΈ Ρ‚ΠΎΠ³Π΄Π° Ρ€Π°Π²Π½Π° [латСкс] \ frac {Q} {t} \\ [/ latex], Π³Π΄Π΅ t — врСмя ΠΎΠ±ΠΎΡ€ΠΎΡ‚Π° Π²ΠΎΠ·Π΄ΡƒΡ…Π°. Нам Π΄Π°Π½ΠΎ, Ρ‡Ρ‚ΠΎ Ξ” T Ρ€Π°Π²Π½ΠΎ 10.0ΒΊC, Π½ΠΎ ΠΌΡ‹ всС Ρ€Π°Π²Π½ΠΎ Π΄ΠΎΠ»ΠΆΠ½Ρ‹ Π½Π°ΠΉΡ‚ΠΈ значСния массы Π²ΠΎΠ·Π΄ΡƒΡ…Π° ΠΈ Π΅Π³ΠΎ ΡƒΠ΄Π΅Π»ΡŒΠ½ΠΎΠΉ тСплоСмкости, ΠΏΡ€Π΅ΠΆΠ΄Π΅ Ρ‡Π΅ΠΌ ΠΌΡ‹ смоТСм Π²Ρ‹Ρ‡ΠΈΡΠ»ΠΈΡ‚ΡŒ Q . УдСльная Ρ‚Π΅ΠΏΠ»ΠΎΠ΅ΠΌΠΊΠΎΡΡ‚ΡŒ Π²ΠΎΠ·Π΄ΡƒΡ…Π° прСдставляСт собой ΡΡ€Π΅Π΄Π½Π΅Π²Π·Π²Π΅ΡˆΠ΅Π½Π½ΠΎΠ΅ Π·Π½Π°Ρ‡Π΅Π½ΠΈΠ΅ ΡƒΠ΄Π΅Π»ΡŒΠ½ΠΎΠΉ Ρ‚Π΅ΠΏΠ»ΠΎΡ‚Ρ‹ Π°Π·ΠΎΡ‚Π° ΠΈ кислорода, Ρ‡Ρ‚ΠΎ Π΄Π°Π΅Ρ‚ c = c p 1000 Π”ΠΆ / ΠΊΠ³ Β· ΒΊC ΠΈΠ· Ρ‚Π°Π±Π»ΠΈΡ†Ρ‹ 1 (ΠΎΠ±Ρ€Π°Ρ‚ΠΈΡ‚Π΅ Π²Π½ΠΈΠΌΠ°Π½ΠΈΠ΅, Ρ‡Ρ‚ΠΎ ΡƒΠ΄Π΅Π»ΡŒΠ½Π°Ρ Ρ‚Π΅ΠΏΠ»ΠΎΠ΅ΠΌΠΊΠΎΡΡ‚ΡŒ ΠΏΡ€ΠΈ постоянном Π΄Π°Π²Π»Π΅Π½ΠΈΠΈ Π΄ΠΎΠ»ΠΆΠ½Π° ΠΈΡΠΏΠΎΠ»ΡŒΠ·ΠΎΠ²Π°Ρ‚ΡŒΡΡ для этого процСсса).

РСшСниС
  1. ΠžΠΏΡ€Π΅Π΄Π΅Π»ΠΈΡ‚Π΅ массу Π²ΠΎΠ·Π΄ΡƒΡ…Π° ΠΏΠΎ Π΅Π³ΠΎ плотности ΠΈ Π·Π°Π΄Π°Π½Π½ΠΎΠΌΡƒ ΠΎΠ±ΡŠΠ΅ΠΌΡƒ Π΄ΠΎΠΌΠ°.ΠŸΠ»ΠΎΡ‚Π½ΠΎΡΡ‚ΡŒ рассчитываСтся исходя ΠΈΠ· плотности ρ ΠΈ объСма ΠΌ = ρV = (1,29 ΠΊΠ³ / ΠΌ 3 ) (12,0 ΠΌ Γ— 18,0 ΠΌ Γ— 3,00 ΠΌ) = 836 ΠΊΠ³.
  2. РассчитайтС Ρ‚Π΅ΠΏΠ»ΠΎΠΏΠ΅Ρ€Π΅Π΄Π°Ρ‡Ρƒ ΠΏΡ€ΠΈ ΠΈΠ·ΠΌΠ΅Π½Π΅Π½ΠΈΠΈ Ρ‚Π΅ΠΌΠΏΠ΅Ρ€Π°Ρ‚ΡƒΡ€Ρ‹ Π²ΠΎΠ·Π΄ΡƒΡ…Π°: Q = ΠΌΠΊΠ» Ξ” T Ρ‚Π°ΠΊ, Ρ‡Ρ‚ΠΎΠ±Ρ‹ Q = (836 ΠΊΠ³) (1000 Π”ΠΆ / ΠΊΠ³ Β· ΒΊC) (10,0ΒΊC) = 8,36 Γ— 10 6 Π”ΠΆ.
  3. РассчитайтС Ρ‚Π΅ΠΏΠ»ΠΎΠΏΠ΅Ρ€Π΅Π΄Π°Ρ‡Ρƒ ΠΎΡ‚ Ρ‚Π΅ΠΏΠ»Π° Q ΠΈ врСмя ΠΎΠ±ΠΎΡ€ΠΎΡ‚Π° t .{6} \ text {J}} {1800 \ text {s}} = 4,64 \ text {ΠΊΠ’Ρ‚} \\ [/ latex].
ΠžΠ±ΡΡƒΠΆΠ΄Π΅Π½ΠΈΠ΅

Π­Ρ‚Π° ΡΠΊΠΎΡ€ΠΎΡΡ‚ΡŒ ΠΏΠ΅Ρ€Π΅Π΄Π°Ρ‡ΠΈ Ρ‚Π΅ΠΏΠ»Π° Ρ€Π°Π²Π½Π° мощности, потрСбляСмой ΠΏΡ€ΠΈΠΌΠ΅Ρ€Π½ΠΎ сорока ΡˆΠ΅ΡΡ‚ΡŒΡŽ Π»Π°ΠΌΠΏΠΎΡ‡ΠΊΠ°ΠΌΠΈ ΠΌΠΎΡ‰Π½ΠΎΡΡ‚ΡŒΡŽ 100 Π’Ρ‚. Π’Π½ΠΎΠ²ΡŒ построСнныС Π΄ΠΎΠΌΠ° рассчитаны Π½Π° врСмя ΠΎΠ±ΠΎΡ€ΠΎΡ‚Π° 2 часа ΠΈΠ»ΠΈ Π±ΠΎΠ»Π΅Π΅, Π° Π½Π΅ 30 ΠΌΠΈΠ½ΡƒΡ‚ для Π΄ΠΎΠΌΠ° Π² этом ΠΏΡ€ΠΈΠΌΠ΅Ρ€Π΅. ΠžΠ±Ρ‹Ρ‡Π½ΠΎ ΠΈΡΠΏΠΎΠ»ΡŒΠ·ΡƒΡŽΡ‚ΡΡ погодоустойчивыС уплотнСния, уплотнСния ΠΈ ΡƒΠ»ΡƒΡ‡ΡˆΠ΅Π½Π½Ρ‹Π΅ ΠΎΠΊΠΎΠ½Π½Ρ‹Π΅ уплотнСния. Π’ ΠΎΡ‡Π΅Π½ΡŒ Ρ…ΠΎΠ»ΠΎΠ΄Π½ΠΎΠΌ (ΠΈΠ»ΠΈ ΠΆΠ°Ρ€ΠΊΠΎΠΌ) ΠΊΠ»ΠΈΠΌΠ°Ρ‚Π΅ ΠΈΠ½ΠΎΠ³Π΄Π° ΠΏΡ€ΠΈΠ½ΠΈΠΌΠ°ΡŽΡ‚ΡΡ Π±ΠΎΠ»Π΅Π΅ ΠΊΡ€Π°ΠΉΠ½ΠΈΠ΅ ΠΌΠ΅Ρ€Ρ‹ для достиТСния ТСсткого стандарта Π±ΠΎΠ»Π΅Π΅ 6 часов Π½Π° ΠΎΠ΄ΠΈΠ½ ΠΎΠ±ΠΎΡ€ΠΎΡ‚ Π²ΠΎΠ·Π΄ΡƒΡ…Π°.Π•Ρ‰Π΅ Π±ΠΎΠ»Π΅Π΅ ΠΏΡ€ΠΎΠ΄ΠΎΠ»ΠΆΠΈΡ‚Π΅Π»ΡŒΠ½ΠΎΠ΅ врСмя ΠΎΠ±ΠΎΡ€ΠΎΡ‚Π° Π²Ρ€Π΅Π΄Π½ΠΎ для Π·Π΄ΠΎΡ€ΠΎΠ²ΡŒΡ, ΠΏΠΎΡ‚ΠΎΠΌΡƒ Ρ‡Ρ‚ΠΎ Π½Π΅ΠΎΠ±Ρ…ΠΎΠ΄ΠΈΠΌΠΎ минимальноС количСство свСТСго Π²ΠΎΠ·Π΄ΡƒΡ…Π° для ΠΏΠΎΠ΄Π°Ρ‡ΠΈ кислорода для дыхания ΠΈ разбавлСния Π±Ρ‹Ρ‚ΠΎΠ²Ρ‹Ρ… загрязнитСлСй. Π’Π΅Ρ€ΠΌΠΈΠ½, ΠΈΡΠΏΠΎΠ»ΡŒΠ·ΡƒΠ΅ΠΌΡ‹ΠΉ для процСсса проникновСния Π½Π°Ρ€ΡƒΠΆΠ½ΠΎΠ³ΠΎ Π²ΠΎΠ·Π΄ΡƒΡ…Π° Π² Π΄ΠΎΠΌ ΠΈΠ· Ρ‚Ρ€Π΅Ρ‰ΠΈΠ½ Π²ΠΎΠΊΡ€ΡƒΠ³ ΠΎΠΊΠΎΠ½, Π΄Π²Π΅Ρ€Π΅ΠΉ ΠΈ Ρ„ΡƒΠ½Π΄Π°ΠΌΠ΅Π½Ρ‚Π°, называСтся Β«ΠΏΡ€ΠΎΠ½ΠΈΠΊΠ½ΠΎΠ²Π΅Π½ΠΈΠ΅ Π²ΠΎΠ·Π΄ΡƒΡ…Π°Β».

Π₯ΠΎΠ»ΠΎΠ΄Π½Ρ‹ΠΉ Π²Π΅Ρ‚Π΅Ρ€ Π±ΠΎΠ»Π΅Π΅ Ρ…ΠΎΠ»ΠΎΠ΄Π½Ρ‹ΠΉ, Ρ‡Π΅ΠΌ Π½Π΅ΠΏΠΎΠ΄Π²ΠΈΠΆΠ½Ρ‹ΠΉ Ρ…ΠΎΠ»ΠΎΠ΄Π½Ρ‹ΠΉ Π²ΠΎΠ·Π΄ΡƒΡ…, ΠΏΠΎΡ‚ΠΎΠΌΡƒ Ρ‡Ρ‚ΠΎ конвСкция Π² сочСтании с ΠΏΡ€ΠΎΠ²ΠΎΠ΄ΠΈΠΌΠΎΡΡ‚ΡŒΡŽ Π² Ρ‚Π΅Π»Π΅ ΡƒΠ²Π΅Π»ΠΈΡ‡ΠΈΠ²Π°Π΅Ρ‚ ΡΠΊΠΎΡ€ΠΎΡΡ‚ΡŒ ΠΏΠ΅Ρ€Π΅Π΄Π°Ρ‡ΠΈ энСргии ΠΎΡ‚ Ρ‚Π΅Π»Π°.Π’ Ρ‚Π°Π±Π»ΠΈΡ†Π΅ Π½ΠΈΠΆΠ΅ ΠΏΡ€ΠΈΠ²Π΅Π΄Π΅Π½Ρ‹ ΠΏΡ€ΠΈΠ±Π»ΠΈΠ·ΠΈΡ‚Π΅Π»ΡŒΠ½Ρ‹Π΅ коэффициСнты охлаТдСния Π²Π΅Ρ‚Ρ€ΠΎΠΌ, ΠΊΠΎΡ‚ΠΎΡ€Ρ‹Π΅ ΠΏΡ€Π΅Π΄ΡΡ‚Π°Π²Π»ΡΡŽΡ‚ собой Ρ‚Π΅ΠΌΠΏΠ΅Ρ€Π°Ρ‚ΡƒΡ€Ρ‹ Π½Π΅ΠΏΠΎΠ΄Π²ΠΈΠΆΠ½ΠΎΠ³ΠΎ Π²ΠΎΠ·Π΄ΡƒΡ…Π°, ΠΎΠ±Π΅ΡΠΏΠ΅Ρ‡ΠΈΠ²Π°ΡŽΡ‰ΠΈΠ΅ Ρ‚Π°ΠΊΡƒΡŽ ​​ТС ΡΠΊΠΎΡ€ΠΎΡΡ‚ΡŒ охлаТдСния, ΠΊΠ°ΠΊ ΠΈ Π²ΠΎΠ·Π΄ΡƒΡ… с Π·Π°Π΄Π°Π½Π½ΠΎΠΉ Ρ‚Π΅ΠΌΠΏΠ΅Ρ€Π°Ρ‚ΡƒΡ€ΠΎΠΉ ΠΈ ΡΠΊΠΎΡ€ΠΎΡΡ‚ΡŒΡŽ. Π€Π°ΠΊΡ‚ΠΎΡ€Ρ‹ охлаТдСния Π²Π΅Ρ‚Ρ€ΠΎΠΌ ΡΠ²Π»ΡΡŽΡ‚ΡΡ ярким Π½Π°ΠΏΠΎΠΌΠΈΠ½Π°Π½ΠΈΠ΅ΠΌ ΠΎ способности ΠΊΠΎΠ½Π²Π΅ΠΊΡ†ΠΈΠΈ ΠΏΠ΅Ρ€Π΅Π΄Π°Π²Π°Ρ‚ΡŒ Ρ‚Π΅ΠΏΠ»ΠΎ быстрСС, Ρ‡Π΅ΠΌ Ρ‚Π΅ΠΏΠ»ΠΎΠΏΡ€ΠΎΠ²ΠΎΠ΄Π½ΠΎΡΡ‚ΡŒ. НапримСр, Π²Π΅Ρ‚Π΅Ρ€ со ΡΠΊΠΎΡ€ΠΎΡΡ‚ΡŒΡŽ 15,0 ΠΌ / с ΠΏΡ€ΠΈ 0ΒΊC ΠΈΠΌΠ΅Π΅Ρ‚ Ρ…ΠΎΠ»ΠΎΠ΄Π½Ρ‹ΠΉ эквивалСнт Π½Π΅ΠΏΠΎΠ΄Π²ΠΈΠΆΠ½ΠΎΠ³ΠΎ Π²ΠΎΠ·Π΄ΡƒΡ…Π° ΠΏΡ€ΠΈ Ρ‚Π΅ΠΌΠΏΠ΅Ρ€Π°Ρ‚ΡƒΡ€Π΅ ΠΎΠΊΠΎΠ»ΠΎ -18ΒΊC.

Π’Π°Π±Π»ΠΈΡ†Π° 1. Π€Π°ΠΊΡ‚ΠΎΡ€Ρ‹ охлаТдСния Π²Π΅Ρ‚Ρ€ΠΎΠΌ
Π’Π΅ΠΌΠΏΠ΅Ρ€Π°Ρ‚ΡƒΡ€Π° двиТущСгося Π²ΠΎΠ·Π΄ΡƒΡ…Π° Π‘ΠΊΠΎΡ€ΠΎΡΡ‚ΡŒ Π²Π΅Ρ‚Ρ€Π° (ΠΌ / с)
(ΒΊC) 2 5 10 15 0
5 3 -1 βˆ’8 βˆ’10 βˆ’12
2 0 βˆ’7 βˆ’12 βˆ’16 βˆ’18
0 βˆ’2 βˆ’9 βˆ’15 βˆ’18 βˆ’20
βˆ’5 βˆ’7 βˆ’15 βˆ’22 βˆ’26 βˆ’29
βˆ’10 βˆ’12 βˆ’21 βˆ’29 βˆ’34 βˆ’36
βˆ’20 βˆ’23 βˆ’34 βˆ’44 βˆ’50 βˆ’52
βˆ’10 βˆ’12 βˆ’21 βˆ’29 βˆ’34 βˆ’36
βˆ’20 βˆ’23 βˆ’34 βˆ’44 βˆ’50 βˆ’52
βˆ’40 βˆ’44 βˆ’59 βˆ’73 βˆ’82 βˆ’84

Π₯отя Π²ΠΎΠ·Π΄ΡƒΡ… ΠΌΠΎΠΆΠ΅Ρ‚ быстро ΠΏΠ΅Ρ€Π΅Π΄Π°Π²Π°Ρ‚ΡŒ Ρ‚Π΅ΠΏΠ»ΠΎ Π·Π° счСт ΠΊΠΎΠ½Π²Π΅ΠΊΡ†ΠΈΠΈ, ΠΎΠ½ являСтся ΠΏΠ»ΠΎΡ…ΠΈΠΌ ΠΏΡ€ΠΎΠ²ΠΎΠ΄Π½ΠΈΠΊΠΎΠΌ ΠΈ, ΡΠ»Π΅Π΄ΠΎΠ²Π°Ρ‚Π΅Π»ΡŒΠ½ΠΎ, Ρ…ΠΎΡ€ΠΎΡˆΠΈΠΌ изолятором.ΠšΠΎΠ»ΠΈΡ‡Π΅ΡΡ‚Π²ΠΎ доступного пространства для Π²ΠΎΠ·Π΄ΡƒΡˆΠ½ΠΎΠ³ΠΎ ΠΏΠΎΡ‚ΠΎΠΊΠ° опрСдСляСт, дСйствуСт Π»ΠΈ Π²ΠΎΠ·Π΄ΡƒΡ… ΠΊΠ°ΠΊ изолятор ΠΈΠ»ΠΈ ΠΏΡ€ΠΎΠ²ΠΎΠ΄Π½ΠΈΠΊ. НапримСр, расстояниС ΠΌΠ΅ΠΆΠ΄Ρƒ Π²Π½ΡƒΡ‚Ρ€Π΅Π½Π½Π΅ΠΉ ΠΈ внСшнСй стСнами Π΄ΠΎΠΌΠ° составляСт ΠΎΠΊΠΎΠ»ΠΎ 9 см (3,5 дюйма) — достаточно для эффСктивной Ρ€Π°Π±ΠΎΡ‚Ρ‹ ΠΊΠΎΠ½Π²Π΅ΠΊΡ†ΠΈΠΈ. Π”ΠΎΠ±Π°Π²Π»Π΅Π½ΠΈΠ΅ тСплоизоляции прСпятствуСт Π²ΠΎΠ·Π΄ΡƒΡˆΠ½ΠΎΠΌΡƒ ΠΏΠΎΡ‚ΠΎΠΊΡƒ, поэтому ΠΏΠΎΡ‚Π΅Ρ€ΠΈ (ΠΈΠ»ΠΈ ΠΏΡ€ΠΈΡ‚ΠΎΠΊ) Ρ‚Π΅ΠΏΠ»Π° ΡƒΠΌΠ΅Π½ΡŒΡˆΠ°ΡŽΡ‚ΡΡ. Π’ΠΎΡ‡Π½ΠΎ Ρ‚Π°ΠΊ ΠΆΠ΅ Π·Π°Π·ΠΎΡ€ ΠΌΠ΅ΠΆΠ΄Ρƒ двумя стСклами ΠΎΠΊΠ½Π° с Π΄Π²ΠΎΠΉΠ½Ρ‹ΠΌ остСклСниСм составляСт ΠΎΠΊΠΎΠ»ΠΎ 1 см, Ρ‡Ρ‚ΠΎ ΠΏΡ€Π΅Π΄ΠΎΡ‚Π²Ρ€Π°Ρ‰Π°Π΅Ρ‚ ΠΊΠΎΠ½Π²Π΅ΠΊΡ†ΠΈΡŽ ΠΈ ΠΈΡΠΏΠΎΠ»ΡŒΠ·ΡƒΠ΅Ρ‚ Π½ΠΈΠ·ΠΊΡƒΡŽ ΠΏΡ€ΠΎΠ²ΠΎΠ΄ΠΈΠΌΠΎΡΡ‚ΡŒ Π²ΠΎΠ·Π΄ΡƒΡ…Π° для прСдотвращСния Π±ΠΎΠ»ΡŒΡˆΠΈΡ… ΠΏΠΎΡ‚Π΅Ρ€ΡŒ.ΠœΠ΅Ρ…, Π²ΠΎΠ»ΠΎΠΊΠ½Π° ΠΈ стСкловолокно Ρ‚Π°ΠΊΠΆΠ΅ ΠΈΡΠΏΠΎΠ»ΡŒΠ·ΡƒΡŽΡ‚ прСимущСства Π½ΠΈΠ·ΠΊΠΎΠΉ проводимости Π²ΠΎΠ·Π΄ΡƒΡ…Π°, удСрТивая Π΅Π³ΠΎ Π² пространствах, слишком ΠΌΠ°Π»Π΅Π½ΡŒΠΊΠΈΡ… для поддСрТания ΠΊΠΎΠ½Π²Π΅ΠΊΡ†ΠΈΠΈ, ΠΊΠ°ΠΊ ΠΏΠΎΠΊΠ°Π·Π°Π½ΠΎ Π½Π° рисункС. ΠœΠ΅Ρ… ΠΈ ΠΏΠ΅Ρ€ΡŒΡ Π»Π΅Π³ΠΊΠΈΠ΅ ΠΈ поэтому идСально подходят для Π·Π°Ρ‰ΠΈΡ‚Ρ‹ ΠΆΠΈΠ²ΠΎΡ‚Π½Ρ‹Ρ….

Рисунок 3. ΠœΠ΅Ρ… Π½Π°ΠΏΠΎΠ»Π½Π΅Π½ Π²ΠΎΠ·Π΄ΡƒΡ…ΠΎΠΌ, ΠΊΠΎΡ‚ΠΎΡ€Ρ‹ΠΉ разбиваСтся Π½Π° мноТСство ΠΌΠ°Π»Π΅Π½ΡŒΠΊΠΈΡ… ΠΊΠ°Ρ€ΠΌΠ°Π½ΠΎΠ². ΠšΠΎΠ½Π²Π΅ΠΊΡ†ΠΈΡ здСсь ΠΎΡ‡Π΅Π½ΡŒ мСдлСнная, ΠΏΠΎΡ‚ΠΎΠΌΡƒ Ρ‡Ρ‚ΠΎ ΠΏΠ΅Ρ‚Π»ΠΈ Ρ‚Π°ΠΊΠΈΠ΅ малСнькиС. Низкая ΠΏΡ€ΠΎΠ²ΠΎΠ΄ΠΈΠΌΠΎΡΡ‚ΡŒ Π²ΠΎΠ·Π΄ΡƒΡ…Π° Π΄Π΅Π»Π°Π΅Ρ‚ ΠΌΠ΅Ρ… ΠΎΡ‡Π΅Π½ΡŒ Ρ…ΠΎΡ€ΠΎΡˆΠΈΠΌ Π»Π΅Π³ΠΊΠΈΠΌ изолятором.

НСкоторыС интСрСсныС явлСния происходят , ΠΊΠΎΠ³Π΄Π° конвСкция сопровоТдаСтся Ρ„Π°Π·ΠΎΠ²Ρ‹ΠΌ ΠΏΠ΅Ρ€Π΅Ρ…ΠΎΠ΄ΠΎΠΌ .Π­Ρ‚ΠΎ позволяСт Π½Π°ΠΌ ΠΎΡ…Π»Π°Π΄ΠΈΡ‚ΡŒΡΡ с ΠΏΠΎΠΌΠΎΡ‰ΡŒΡŽ потоотдСлСния, Π΄Π°ΠΆΠ΅ Ссли Ρ‚Π΅ΠΌΠΏΠ΅Ρ€Π°Ρ‚ΡƒΡ€Π° ΠΎΠΊΡ€ΡƒΠΆΠ°ΡŽΡ‰Π΅Π³ΠΎ Π²ΠΎΠ·Π΄ΡƒΡ…Π° ΠΏΡ€Π΅Π²Ρ‹ΡˆΠ°Π΅Ρ‚ Ρ‚Π΅ΠΌΠΏΠ΅Ρ€Π°Ρ‚ΡƒΡ€Ρƒ Ρ‚Π΅Π»Π°. Π’Π΅ΠΏΠ»ΠΎ ΠΎΡ‚ ΠΊΠΎΠΆΠΈ трСбуСтся для испарСния ΠΏΠΎΡ‚Π° с ΠΊΠΎΠΆΠΈ, Π½ΠΎ Π±Π΅Π· Π²ΠΎΠ·Π΄ΡƒΡˆΠ½ΠΎΠ³ΠΎ ΠΏΠΎΡ‚ΠΎΠΊΠ° Π²ΠΎΠ·Π΄ΡƒΡ… становится насыщСнным ΠΈ испарСниС прСкращаСтся. Π’ΠΎΠ·Π΄ΡƒΡˆΠ½Ρ‹ΠΉ ΠΏΠΎΡ‚ΠΎΠΊ, Π²Ρ‹Π·Π²Π°Π½Π½Ρ‹ΠΉ ΠΊΠΎΠ½Π²Π΅ΠΊΡ†ΠΈΠ΅ΠΉ, замСняСт насыщСнный Π²ΠΎΠ·Π΄ΡƒΡ… сухим, ΠΈ испарСниС продолТаСтся.

ΠŸΡ€ΠΈΠΌΠ΅Ρ€ 2. РасчСт ΠΏΠΎΡ‚ΠΎΠΊΠ° массы Π²ΠΎ врСмя ΠΊΠΎΠ½Π²Π΅ΠΊΡ†ΠΈΠΈ: Ρ‚Π΅ΠΏΠ»ΠΎΠΏΠ΅Ρ€Π΅Π΄Π°Ρ‡Π° ΠΏΠΎΡ‚Π° ΠΎΡ‚ Ρ‚Π΅Π»Π°

Π‘Ρ€Π΅Π΄Π½ΠΈΠΉ Ρ‡Π΅Π»ΠΎΠ²Π΅ΠΊ Π² состоянии покоя выдСляСт Ρ‚Π΅ΠΏΠ»ΠΎ ΠΌΠΎΡ‰Π½ΠΎΡΡ‚ΡŒΡŽ ΠΎΠΊΠΎΠ»ΠΎ 120 Π’Ρ‚.Π‘ ΠΊΠ°ΠΊΠΎΠΉ ΡΠΊΠΎΡ€ΠΎΡΡ‚ΡŒΡŽ Π΄ΠΎΠ»ΠΆΠ½Π° ΠΈΡΠΏΠ°Ρ€ΡΡ‚ΡŒΡΡ Π²ΠΎΠ΄Π° ΠΈΠ· Ρ‚Π΅Π»Π°, Ρ‡Ρ‚ΠΎΠ±Ρ‹ ΠΈΠ·Π±Π°Π²ΠΈΡ‚ΡŒΡΡ ΠΎΡ‚ всСй этой энСргии? (Π­Ρ‚ΠΎ испарСниС ΠΌΠΎΠΆΠ΅Ρ‚ ΠΏΡ€ΠΎΠΈΡΡ…ΠΎΠ΄ΠΈΡ‚ΡŒ, ΠΊΠΎΠ³Π΄Π° Ρ‡Π΅Π»ΠΎΠ²Π΅ΠΊ сидит Π² Ρ‚Π΅Π½ΠΈ ΠΈ Ρ‚Π΅ΠΌΠΏΠ΅Ρ€Π°Ρ‚ΡƒΡ€Π° ΠΎΠΊΡ€ΡƒΠΆΠ°ΡŽΡ‰Π΅ΠΉ срСды такая ΠΆΠ΅, ΠΊΠ°ΠΊ Ρ‚Π΅ΠΌΠΏΠ΅Ρ€Π°Ρ‚ΡƒΡ€Π° ΠΊΠΎΠΆΠΈ, Ρ‡Ρ‚ΠΎ ΠΈΡΠΊΠ»ΡŽΡ‡Π°Π΅Ρ‚ ΠΏΠ΅Ρ€Π΅Π΄Π°Ρ‡Ρƒ Ρ‚Π΅ΠΏΠ»Π° Π΄Ρ€ΡƒΠ³ΠΈΠΌΠΈ ΠΌΠ΅Ρ‚ΠΎΠ΄Π°ΠΌΠΈ.)

БтратСгия

ЭнСргия Π½Π΅ΠΎΠ±Ρ…ΠΎΠ΄ΠΈΠΌΠ° для Ρ„Π°Π·ΠΎΠ²ΠΎΠ³ΠΎ ΠΏΠ΅Ρ€Π΅Ρ…ΠΎΠ΄Π° ( Q = ΠΌΠ» v ). Π’Π°ΠΊΠΈΠΌ ΠΎΠ±Ρ€Π°Π·ΠΎΠΌ, ΠΏΠΎΡ‚Π΅Ρ€ΠΈ энСргии Π² Π΅Π΄ΠΈΠ½ΠΈΡ†Ρƒ Π²Ρ€Π΅ΠΌΠ΅Π½ΠΈ ΡΠΎΡΡ‚Π°Π²Π»ΡΡŽΡ‚

[латСкс] \ displaystyle \ frac {Q} {t} = \ frac {mL _ {\ text {v}}} {t} = 120 \ text {W} = 120 \ text {J / s} \\ [/ латСкс].

ΠœΡ‹ Ρ€Π°Π·Π΄Π΅Π»ΠΈΠΌ ΠΎΠ±Π΅ части уравнСния Π½Π° L v , Ρ‡Ρ‚ΠΎΠ±Ρ‹ Π½Π°ΠΉΡ‚ΠΈ, Ρ‡Ρ‚ΠΎ масса, ΠΈΡΠΏΠ°Ρ€ΠΈΠ²ΡˆΠ°ΡΡΡ Π·Π° Π΅Π΄ΠΈΠ½ΠΈΡ†Ρƒ Π²Ρ€Π΅ΠΌΠ΅Π½ΠΈ, Ρ€Π°Π²Π½Π° [латСксу] \ frac {m} {t} = \ frac {120 \ text {Π”ΠΆ / с}} { L _ {\ text {v}}} \\ [/ latex].

РСшСниС

Π’ΡΡ‚Π°Π²ΡŒΡ‚Π΅ Π·Π½Π°Ρ‡Π΅Π½ΠΈΠ΅ скрытой Ρ‚Π΅ΠΏΠ»ΠΎΡ‚Ρ‹ ΠΈΠ· Ρ‚Π°Π±Π»ΠΈΡ†Ρ‹ 1 Π² Ρ€Π°Π·Π΄Π΅Π» Β«Π€Π°Π·ΠΎΠ²Ρ‹ΠΉ ΠΏΠ΅Ρ€Π΅Ρ…ΠΎΠ΄ ΠΈ скрытая Ρ‚Π΅ΠΏΠ»ΠΎΡ‚Π°Β», L v = 2430 ΠΊΠ”ΠΆ / ΠΊΠ³ = 2430 Π”ΠΆ / Π³. Π­Ρ‚ΠΎ Π΄Π°Π΅Ρ‚

[латСкс] \ displaystyle \ frac {m} {t} = \ frac {120 \ text {J / s}} {2430 \ text {J / g}} = 0,0494 \ text {g / s} = 2,96 \ text {Π³ / ΠΌΠΈΠ½} \ [/ латСкс]

ΠžΠ±ΡΡƒΠΆΠ΄Π΅Π½ΠΈΠ΅

Π˜ΡΠΏΠ°Ρ€Π΅Π½ΠΈΠ΅ ΠΎΠΊΠΎΠ»ΠΎ 3 Π³ / ΠΌΠΈΠ½ каТСтся Ρ€Π°Π·ΡƒΠΌΠ½Ρ‹ΠΌ.Π­Ρ‚ΠΎ Π±ΡƒΠ΄Π΅Ρ‚ ΠΎΠΊΠΎΠ»ΠΎ 180 Π³ (ΠΎΠΊΠΎΠ»ΠΎ 7 ΡƒΠ½Ρ†ΠΈΠΉ) Π² час. Если Π²ΠΎΠ·Π΄ΡƒΡ… ΠΎΡ‡Π΅Π½ΡŒ сухой, ΠΏΠΎΡ‚ ΠΌΠΎΠΆΠ΅Ρ‚ ΠΈΡΠΏΠ°Ρ€ΠΈΡ‚ΡŒΡΡ, Π΄Π°ΠΆΠ΅ Ссли этого Π½Π΅ замСтят. Π—Π½Π°Ρ‡ΠΈΡ‚Π΅Π»ΡŒΠ½ΠΎΠ΅ количСство испарСний Ρ‚Π°ΠΊΠΆΠ΅ происходит Π² Π»Π΅Π³ΠΊΠΈΡ… ΠΈ Π΄Ρ‹Ρ…Π°Ρ‚Π΅Π»ΡŒΠ½Ρ‹Ρ… путях.

Рис. 4. ΠšΡƒΡ‡Π΅Π²Ρ‹Π΅ ΠΎΠ±Π»Π°ΠΊΠ° ΡΠΎΠ·Π΄Π°ΡŽΡ‚ΡΡ водяным ΠΏΠ°Ρ€ΠΎΠΌ, ΠΏΠΎΠ΄Π½ΠΈΠΌΠ°ΡŽΡ‰ΠΈΠΌΡΡ ΠΈΠ·-Π·Π° ΠΊΠΎΠ½Π²Π΅ΠΊΡ†ΠΈΠΈ. Π’ΠΎΠ·Π½ΠΈΠΊΠ½ΠΎΠ²Π΅Π½ΠΈΠ΅ ΠΎΠ±Π»Π°ΠΊΠΎΠ² происходит Π·Π° счСт ΠΌΠ΅Ρ…Π°Π½ΠΈΠ·ΠΌΠ° ΠΏΠΎΠ»ΠΎΠΆΠΈΡ‚Π΅Π»ΡŒΠ½ΠΎΠΉ ΠΎΠ±Ρ€Π°Ρ‚Π½ΠΎΠΉ связи. (ΠΊΡ€Π΅Π΄ΠΈΡ‚: Майк Π›Π°Π²)

Π”Ρ€ΡƒΠ³ΠΎΠΉ Π²Π°ΠΆΠ½Ρ‹ΠΉ ΠΏΡ€ΠΈΠΌΠ΅Ρ€ сочСтания Ρ„Π°Π·ΠΎΠ²ΠΎΠ³ΠΎ ΠΏΠ΅Ρ€Π΅Ρ…ΠΎΠ΄Π° ΠΈ ΠΊΠΎΠ½Π²Π΅ΠΊΡ†ΠΈΠΈ происходит ΠΏΡ€ΠΈ испарСнии Π²ΠΎΠ΄Ρ‹ ΠΈΠ· ΠΎΠΊΠ΅Π°Π½ΠΎΠ².Когда Π²ΠΎΠ΄Π° испаряСтся, Ρ‚Π΅ΠΏΠ»ΠΎ ΡƒΡ…ΠΎΠ΄ΠΈΡ‚ ΠΈΠ· ΠΎΠΊΠ΅Π°Π½Π°. Если водяной ΠΏΠ°Ρ€ кондСнсируСтся Π² ΠΆΠΈΠ΄ΠΊΠΈΠ΅ ΠΊΠ°ΠΏΠ»ΠΈ ΠΏΡ€ΠΈ ΠΎΠ±Ρ€Π°Π·ΠΎΠ²Π°Π½ΠΈΠΈ ΠΎΠ±Π»Π°ΠΊΠΎΠ², Π² атмосфСру выдСляСтся Ρ‚Π΅ΠΏΠ»ΠΎ. Π’Π°ΠΊΠΈΠΌ ΠΎΠ±Ρ€Π°Π·ΠΎΠΌ, происходит ΠΎΠ±Ρ‰ΠΈΠΉ пСрСнос Ρ‚Π΅ΠΏΠ»Π° ΠΎΡ‚ ΠΎΠΊΠ΅Π°Π½Π° Π² атмосфСру. Π­Ρ‚ΠΎΡ‚ процСсс являСтся Π΄Π²ΠΈΠΆΡƒΡ‰Π΅ΠΉ силой Π³Ρ€ΠΎΠ·ΠΎΠ²Ρ‹Ρ… ΠΎΠ±Π»Π°ΠΊΠΎΠ², Ρ‚Π΅Ρ… ΠΎΠ³Ρ€ΠΎΠΌΠ½Ρ‹Ρ… ΠΊΡƒΡ‡Π΅Π²Ρ‹Ρ… ΠΎΠ±Π»Π°ΠΊΠΎΠ², ΠΊΠΎΡ‚ΠΎΡ€Ρ‹Π΅ ΠΏΠΎΠ΄Π½ΠΈΠΌΠ°ΡŽΡ‚ΡΡ Π½Π° 20 ΠΊΠΌ Π² стратосфСру. Водяной ΠΏΠ°Ρ€, пСрСносимый ΠΊΠΎΠ½Π²Π΅ΠΊΡ†ΠΈΠ΅ΠΉ, кондСнсируСтся, высвобоТдая ΠΎΠ³Ρ€ΠΎΠΌΠ½ΠΎΠ΅ количСство энСргии. Π­Ρ‚Π° энСргия заставляСт Π²ΠΎΠ·Π΄ΡƒΡ… Ρ€Π°ΡΡˆΠΈΡ€ΡΡ‚ΡŒΡΡ ΠΈ ΠΏΠΎΠ΄Π½ΠΈΠΌΠ°Ρ‚ΡŒΡΡ Ρ‚Π°ΠΌ, Π³Π΄Π΅ ΠΎΠ½ Ρ…ΠΎΠ»ΠΎΠ΄Π½Π΅Π΅.Π’ этих Π±ΠΎΠ»Π΅Π΅ Ρ…ΠΎΠ»ΠΎΠ΄Π½Ρ‹Ρ… Ρ€Π΅Π³ΠΈΠΎΠ½Π°Ρ… происходит большС кондСнсации, Ρ‡Ρ‚ΠΎ, Π² свою ΠΎΡ‡Π΅Ρ€Π΅Π΄ΡŒ, ΠΏΠΎΠ΄Π½ΠΈΠΌΠ°Π΅Ρ‚ ΠΎΠ±Π»Π°ΠΊΠΎ Π΅Ρ‰Π΅ Π²Ρ‹ΡˆΠ΅. Π’Π°ΠΊΠΎΠΉ ΠΌΠ΅Ρ…Π°Π½ΠΈΠ·ΠΌ называСтся ΠΏΠΎΠ»ΠΎΠΆΠΈΡ‚Π΅Π»ΡŒΠ½ΠΎΠΉ ΠΎΠ±Ρ€Π°Ρ‚Π½ΠΎΠΉ связью, ΠΏΠΎΡΠΊΠΎΠ»ΡŒΠΊΡƒ процСсс усиливаСтся ΠΈ ускоряСтся.

Рис. 5. ΠšΠΎΠ½Π²Π΅ΠΊΡ†ΠΈΡ, ΡΠΎΠΏΡ€ΠΎΠ²ΠΎΠΆΠ΄Π°ΡŽΡ‰Π°ΡΡΡ Ρ„Π°Π·ΠΎΠ²Ρ‹ΠΌ ΠΏΠ΅Ρ€Π΅Ρ…ΠΎΠ΄ΠΎΠΌ, высвобоТдаСт ΡΠ½Π΅Ρ€Π³ΠΈΡŽ, Π½Π΅ΠΎΠ±Ρ…ΠΎΠ΄ΠΈΠΌΡƒΡŽ для Ρ‚ΠΎΠ³ΠΎ, Ρ‡Ρ‚ΠΎΠ±Ρ‹ Π·Π°Π³Π½Π°Ρ‚ΡŒ этот Π³Ρ€ΠΎΠ·ΠΎΠ²ΠΎΠΉ ΠΏΠΎΡ‚ΠΎΠΊ Π² стратосфСру. (ΠΊΡ€Π΅Π΄ΠΈΡ‚: Π₯Π΅Ρ€Π°Ρ€Π΄ΠΎ Гарсиа ΠœΠΎΡ€Π΅Ρ‚Ρ‚ΠΈ)

Π­Ρ‚ΠΈ систСмы ΠΈΠ½ΠΎΠ³Π΄Π° Π²Ρ‹Π·Ρ‹Π²Π°ΡŽΡ‚ ΡΠΈΠ»ΡŒΠ½Ρ‹Π΅ ΡˆΡ‚ΠΎΡ€ΠΌΡ‹ с молниями ΠΈ Π³Ρ€Π°Π΄ΠΎΠΌ ΠΈ ΠΏΡ€Π΅Π΄ΡΡ‚Π°Π²Π»ΡΡŽΡ‚ собой ΠΌΠ΅Ρ…Π°Π½ΠΈΠ·ΠΌ, Π²Ρ‹Π·Ρ‹Π²Π°ΡŽΡ‰ΠΈΠΉ ΡƒΡ€Π°Π³Π°Π½Ρ‹ (рис. 5).

Π”Π²ΠΈΠΆΠ΅Π½ΠΈΠ΅ айсбСргов (рис. 6) — Π΅Ρ‰Π΅ ΠΎΠ΄ΠΈΠ½ ΠΏΡ€ΠΈΠΌΠ΅Ρ€ ΠΊΠΎΠ½Π²Π΅ΠΊΡ†ΠΈΠΈ, ΡΠΎΠΏΡ€ΠΎΠ²ΠΎΠΆΠ΄Π°ΡŽΡ‰Π΅ΠΉΡΡ Ρ„Π°Π·ΠΎΠ²Ρ‹ΠΌ ΠΏΠ΅Ρ€Π΅Ρ…ΠΎΠ΄ΠΎΠΌ. ΠŸΡ€Π΅Π΄ΠΏΠΎΠ»ΠΎΠΆΠΈΠΌ, айсбСрг Π΄Ρ€Π΅ΠΉΡ„ΡƒΠ΅Ρ‚ ΠΈΠ· Π“Ρ€Π΅Π½Π»Π°Π½Π΄ΠΈΠΈ Π² Π±ΠΎΠ»Π΅Π΅ Ρ‚Π΅ΠΏΠ»Ρ‹Π΅ Π²ΠΎΠ΄Ρ‹ Атлантики. Π’Π΅ΠΏΠ»ΠΎ удаляСтся ΠΈΠ· Ρ‚Π΅ΠΏΠ»ΠΎΠΉ окСанской Π²ΠΎΠ΄Ρ‹, ΠΊΠΎΠ³Π΄Π° Π»Π΅Π΄ Ρ‚Π°Π΅Ρ‚, ΠΈ Ρ‚Π΅ΠΏΠ»ΠΎ выдСляСтся Π½Π° ΡΡƒΡˆΡƒ, ΠΊΠΎΠ³Π΄Π° айсбСрг формируСтся Π½Π° Π“Ρ€Π΅Π½Π»Π°Π½Π΄ΠΈΠΈ.

Рис. 6. Π€Π°Π·ΠΎΠ²ΠΎΠ΅ ΠΈΠ·ΠΌΠ΅Π½Π΅Π½ΠΈΠ΅, ΠΊΠΎΡ‚ΠΎΡ€ΠΎΠ΅ происходит ΠΏΡ€ΠΈ таянии этого айсбСрга, связано с ΠΎΠ³Ρ€ΠΎΠΌΠ½ΠΎΠΉ Ρ‚Π΅ΠΏΠ»ΠΎΠΏΠ΅Ρ€Π΅Π΄Π°Ρ‡Π΅ΠΉ. (ΠΊΡ€Π΅Π΄ΠΈΡ‚: Π”ΠΎΠΌΠΈΠ½ΠΈΠΊ АлвСс)

ΠŸΡ€ΠΎΠ²Π΅Ρ€ΡŒΡ‚Π΅ своС ΠΏΠΎΠ½ΠΈΠΌΠ°Π½ΠΈΠ΅

ΠžΠ±ΡŠΡΡΠ½ΠΈΡ‚Π΅, ΠΏΠΎΡ‡Π΅ΠΌΡƒ использованиС вСнтилятора Π»Π΅Ρ‚ΠΎΠΌ Π΄Π°Π΅Ρ‚ ΠΎΡ‰ΡƒΡ‰Π΅Π½ΠΈΠ΅ свСТСсти!

РСшСниС

ИспользованиС вСнтилятора ΡƒΠ²Π΅Π»ΠΈΡ‡ΠΈΠ²Π°Π΅Ρ‚ ΠΏΠΎΡ‚ΠΎΠΊ Π²ΠΎΠ·Π΄ΡƒΡ…Π°: Ρ‚Π΅ΠΏΠ»Ρ‹ΠΉ Π²ΠΎΠ·Π΄ΡƒΡ… рядом с вашим Ρ‚Π΅Π»ΠΎΠΌ замСняСтся Π±ΠΎΠ»Π΅Π΅ Ρ…ΠΎΠ»ΠΎΠ΄Π½Ρ‹ΠΌ Π²ΠΎΠ·Π΄ΡƒΡ…ΠΎΠΌ ΠΈΠ· Π΄Ρ€ΡƒΠ³ΠΎΠ³ΠΎ мСста.ΠšΠΎΠ½Π²Π΅ΠΊΡ†ΠΈΡ ΡƒΠ²Π΅Π»ΠΈΡ‡ΠΈΠ²Π°Π΅Ρ‚ ΡΠΊΠΎΡ€ΠΎΡΡ‚ΡŒ Ρ‚Π΅ΠΏΠ»ΠΎΠΏΠ΅Ρ€Π΅Π΄Π°Ρ‡ΠΈ, Ρ‚Π°ΠΊ Ρ‡Ρ‚ΠΎ двиТущийся Π²ΠΎΠ·Π΄ΡƒΡ… «каТСтся» Ρ…ΠΎΠ»ΠΎΠ΄Π½Π΅Π΅, Ρ‡Π΅ΠΌ Π½Π΅ΠΏΠΎΠ΄Π²ΠΈΠΆΠ½Ρ‹ΠΉ.

Π‘Π²ΠΎΠ΄ΠΊΠ° Ρ€Π°Π·Π΄Π΅Π»Π°

ΠšΠΎΠ½Π²Π΅ΠΊΡ†ΠΈΡ — это ΠΏΠ΅Ρ€Π΅Π΄Π°Ρ‡Π° Ρ‚Π΅ΠΏΠ»Π° Π·Π° счСт макроскопичСского двиТСния массы. ΠšΠΎΠ½Π²Π΅ΠΊΡ†ΠΈΡ ΠΌΠΎΠΆΠ΅Ρ‚ Π±Ρ‹Ρ‚ΡŒ СстСствСнной ΠΈΠ»ΠΈ ΠΏΡ€ΠΈΠ½ΡƒΠ΄ΠΈΡ‚Π΅Π»ΡŒΠ½ΠΎΠΉ ΠΈ ΠΎΠ±Ρ‹Ρ‡Π½ΠΎ ΠΏΠ΅Ρ€Π΅Π΄Π°Π΅Ρ‚ Ρ‚Π΅ΠΏΠ»ΠΎΠ²ΡƒΡŽ ΡΠ½Π΅Ρ€Π³ΠΈΡŽ быстрСС, Ρ‡Π΅ΠΌ Ρ‚Π΅ΠΏΠ»ΠΎΠΏΡ€ΠΎΠ²ΠΎΠ΄Π½ΠΎΡΡ‚ΡŒ. Π’ Ρ‚Π°Π±Π»ΠΈΡ†Π΅ 1 ΠΏΡ€ΠΈΠ²Π΅Π΄Π΅Π½Ρ‹ коэффициСнты охлаТдСния Π²Π΅Ρ‚Ρ€ΠΎΠΌ, ΡƒΠΊΠ°Π·Ρ‹Π²Π°ΡŽΡ‰ΠΈΠ΅ Π½Π° Ρ‚ΠΎ, Ρ‡Ρ‚ΠΎ двиТущийся Π²ΠΎΠ·Π΄ΡƒΡ… ΠΈΠΌΠ΅Π΅Ρ‚ Ρ‚Π°ΠΊΠΎΠΉ ΠΆΠ΅ ΠΎΡ…Π»Π°ΠΆΠ΄Π°ΡŽΡ‰ΠΈΠΉ эффСкт, ΠΊΠ°ΠΊ ΠΈ Π³ΠΎΡ€Π°Π·Π΄ΠΎ Π±ΠΎΠ»Π΅Π΅ Ρ…ΠΎΠ»ΠΎΠ΄Π½Ρ‹ΠΉ стационарный Π²ΠΎΠ·Π΄ΡƒΡ…. ΠšΠΎΠ½Π²Π΅ΠΊΡ†ΠΈΡ, Π²ΠΎΠ·Π½ΠΈΠΊΠ°ΡŽΡ‰Π°Ρ вмСстС с Ρ„Π°Π·ΠΎΠ²Ρ‹ΠΌ ΠΏΠ΅Ρ€Π΅Ρ…ΠΎΠ΄ΠΎΠΌ, ΠΌΠΎΠΆΠ΅Ρ‚ ΠΏΠ΅Ρ€Π΅Π΄Π°Π²Π°Ρ‚ΡŒ ΡΠ½Π΅Ρ€Π³ΠΈΡŽ ΠΈΠ· Ρ…ΠΎΠ»ΠΎΠ΄Π½Ρ‹Ρ… областСй Π² Ρ‚Π΅ΠΏΠ»Ρ‹Π΅.

ΠšΠΎΠ½Ρ†Π΅ΠΏΡ‚ΡƒΠ°Π»ΡŒΠ½Ρ‹Π΅ вопросы

  1. Один ΠΈΠ· способов ΡΠ΄Π΅Π»Π°Ρ‚ΡŒ ΠΊΠ°ΠΌΠΈΠ½ Π±ΠΎΠ»Π΅Π΅ энСргоэффСктивным — это ΠΈΡΠΏΠΎΠ»ΡŒΠ·ΠΎΠ²Π°Ρ‚ΡŒ внСшний Π²ΠΎΠ·Π΄ΡƒΡ… для сТигания Ρ‚ΠΎΠΏΠ»ΠΈΠ²Π°. Π”Ρ€ΡƒΠ³ΠΎΠΉ — ΠΎΠ±Π΅ΡΠΏΠ΅Ρ‡ΠΈΡ‚ΡŒ Ρ†ΠΈΡ€ΠΊΡƒΠ»ΡΡ†ΠΈΡŽ ΠΊΠΎΠΌΠ½Π°Ρ‚Π½ΠΎΠ³ΠΎ Π²ΠΎΠ·Π΄ΡƒΡ…Π° Π²ΠΎΠΊΡ€ΡƒΠ³ Ρ‚ΠΎΠΏΠΊΠΈ ΠΈ ΠΎΠ±Ρ€Π°Ρ‚Π½ΠΎ Π² ΠΊΠΎΠΌΠ½Π°Ρ‚Ρƒ. ΠŸΠΎΠ΄Ρ€ΠΎΠ±Π½ΠΎ ΠΎΠΏΠΈΡˆΠΈΡ‚Π΅ ΠΌΠ΅Ρ‚ΠΎΠ΄Ρ‹ Ρ‚Π΅ΠΏΠ»ΠΎΠΏΠ΅Ρ€Π΅Π΄Π°Ρ‡ΠΈ, задСйствованныС Π² ΠΊΠ°ΠΆΠ΄ΠΎΠΌ ΠΈΠ· Π½ΠΈΡ….
  2. Π₯ΠΎΠ»ΠΎΠ΄Π½Ρ‹ΠΌΠΈ ясными Π½ΠΎΡ‡Π°ΠΌΠΈ лошади Π±ΡƒΠ΄ΡƒΡ‚ ΡΠΏΠ°Ρ‚ΡŒ ΠΏΠΎΠ΄ ΠΏΠΎΠΊΡ€ΠΎΠ²ΠΎΠΌ Π±ΠΎΠ»ΡŒΡˆΠΈΡ… Π΄Π΅Ρ€Π΅Π²ΡŒΠ΅Π². Как это ΠΏΠΎΠΌΠΎΠ³Π°Π΅Ρ‚ ΠΈΠΌ ΡΠΎΠ³Ρ€Π΅Ρ‚ΡŒΡΡ?

Π—Π°Π΄Π°Ρ‡ΠΈ ΠΈ упраТнСния

  1. ΠŸΡ€ΠΈ ΠΊΠ°ΠΊΠΎΠΉ скорости Π²Π΅Ρ‚Ρ€Π° -10ΒΊC Π²ΠΎΠ·Π΄ΡƒΡ… Π²Ρ‹Π·Ρ‹Π²Π°Π΅Ρ‚ Ρ‚Π°ΠΊΠΎΠΉ ΠΆΠ΅ коэффициСнт охлаТдСния, ΠΊΠ°ΠΊ ΠΈ Π½Π΅ΠΏΠΎΠ΄Π²ΠΈΠΆΠ½Ρ‹ΠΉ Π²ΠΎΠ·Π΄ΡƒΡ… ΠΏΡ€ΠΈ -29ΒΊC?
  2. ΠŸΡ€ΠΈ ΠΊΠ°ΠΊΠΎΠΉ Ρ‚Π΅ΠΌΠΏΠ΅Ρ€Π°Ρ‚ΡƒΡ€Π΅ Π½Π΅ΠΏΠΎΠ΄Π²ΠΈΠΆΠ½Ρ‹ΠΉ Π²ΠΎΠ·Π΄ΡƒΡ… Π²Ρ‹Π·Ρ‹Π²Π°Π΅Ρ‚ Ρ‚Π°ΠΊΠΎΠΉ ΠΆΠ΅ коэффициСнт охлаТдСния, ΠΊΠ°ΠΊ βˆ’5ΒΊC, двиТущийся со ΡΠΊΠΎΡ€ΠΎΡΡ‚ΡŒΡŽ 15 ΠΌ / с?
  3. Β«ΠŸΠ°Ρ€Β» Π½Π°Π΄ Ρ‡Π°ΡˆΠΊΠΎΠΉ свСТСприготовлСнного растворимого ΠΊΠΎΡ„Π΅ — это Π½Π° самом Π΄Π΅Π»Π΅ ΠΊΠ°ΠΏΠ»ΠΈ водяного ΠΏΠ°Ρ€Π°, ΠΊΠΎΠ½Π΄Π΅Π½ΡΠΈΡ€ΡƒΡŽΡ‰ΠΈΠ΅ΡΡ послС испарСния горячСго ΠΊΠΎΡ„Π΅.Какова конСчная Ρ‚Π΅ΠΌΠΏΠ΅Ρ€Π°Ρ‚ΡƒΡ€Π° 250 Π³ горячСго ΠΊΠΎΡ„Π΅ ΠΏΡ€ΠΈ Π½Π°Ρ‡Π°Π»ΡŒΠ½ΠΎΠΉ Ρ‚Π΅ΠΌΠΏΠ΅Ρ€Π°Ρ‚ΡƒΡ€Π΅ 90,0 Β° C, Ссли ΠΈΠ· Π½Π΅Π³ΠΎ испаряСтся 2,00 Π³? ΠšΠΎΡ„Π΅ находится Π² Ρ‡Π°ΡˆΠΊΠ΅ ΠΈΠ· пСнополистирола, поэтому Π΄Ρ€ΡƒΠ³ΠΈΠΌΠΈ ΠΌΠ΅Ρ‚ΠΎΠ΄Π°ΠΌΠΈ ΠΏΠ΅Ρ€Π΅Π΄Π°Ρ‡ΠΈ Ρ‚Π΅ΠΏΠ»Π° ΠΌΠΎΠΆΠ½ΠΎ ΠΏΡ€Π΅Π½Π΅Π±Ρ€Π΅Ρ‡ΡŒ.
  4. (a) Бколько ΠΊΠΈΠ»ΠΎΠ³Ρ€Π°ΠΌΠΌΠΎΠ² Π²ΠΎΠ΄Ρ‹ Π΄ΠΎΠ»ΠΆΠ½ΠΎ ΠΈΡΠΏΠ°Ρ€ΠΈΡ‚ΡŒΡΡ ΠΆΠ΅Π½Ρ‰ΠΈΠ½ΠΎΠΉ с вСсом 60,0 ΠΊΠ³, Ρ‡Ρ‚ΠΎΠ±Ρ‹ Ρ‚Π΅ΠΌΠΏΠ΅Ρ€Π°Ρ‚ΡƒΡ€Π° Π΅Π΅ Ρ‚Π΅Π»Π° снизилась Π½Π° 0,750ΒΊC? (b) Достаточно Π»ΠΈ это количСства Π²ΠΎΠ΄Ρ‹ для испарСния Π² Π²ΠΈΠ΄Π΅ потоотдСлСния, Ссли ΠΎΡ‚Π½ΠΎΡΠΈΡ‚Π΅Π»ΡŒΠ½Π°Ρ Π²Π»Π°ΠΆΠ½ΠΎΡΡ‚ΡŒ ΠΎΠΊΡ€ΡƒΠΆΠ°ΡŽΡ‰Π΅Π³ΠΎ Π²ΠΎΠ·Π΄ΡƒΡ…Π° низкая?
  5. Π’ ΠΆΠ°Ρ€ΠΊΠΈΠΉ Π·Π°ΡΡƒΡˆΠ»ΠΈΠ²Ρ‹ΠΉ дСнь испарСниС ΠΈΠ· ΠΎΠ·Π΅Ρ€Π° ΠΈΠΌΠ΅Π΅Ρ‚ достаточно Ρ‚Π΅ΠΏΠ»ΠΎΠΏΠ΅Ρ€Π΅Π΄Π°Ρ‡ΠΈ, Ρ‡Ρ‚ΠΎΠ±Ρ‹ ΡƒΡ€Π°Π²Π½ΠΎΠ²Π΅ΡΠΈΡ‚ΡŒ 1.00 ΠΊΠ’Ρ‚ / ΠΌ 2 приходящСго Ρ‚Π΅ΠΏΠ»Π° ΠΎΡ‚ Π‘ΠΎΠ»Π½Ρ†Π°. Какая масса Π²ΠΎΠ΄Ρ‹ испаряСтся Π·Π° 1,00 Ρ‡ с ΠΊΠ°ΠΆΠ΄ΠΎΠ³ΠΎ ΠΊΠ²Π°Π΄Ρ€Π°Ρ‚Π½ΠΎΠ³ΠΎ ΠΌΠ΅Ρ‚Ρ€Π°?
  6. ΠžΠ΄Π½Π°ΠΆΠ΄Ρ‹ Π·ΠΈΠΌΠ½ΠΈΠΌ Π΄Π½Π΅ΠΌ ​​систСма ΠΊΠ»ΠΈΠΌΠ°Ρ‚-контроля Π² большом Π·Π΄Π°Π½ΠΈΠΈ унивСрситСтской Π°ΡƒΠ΄ΠΈΡ‚ΠΎΡ€ΠΈΠΈ Π²Ρ‹ΡˆΠ»Π° ΠΈΠ· строя. Π’ Ρ€Π΅Π·ΡƒΠ»ΡŒΡ‚Π°Ρ‚Π΅ ΠΊΠ°ΠΆΠ΄ΡƒΡŽ ΠΌΠΈΠ½ΡƒΡ‚Ρƒ вводится 500 ΠΌ 3 ΠΈΠ·Π±Ρ‹Ρ‚ΠΎΡ‡Π½ΠΎΠ³ΠΎ Ρ…ΠΎΠ»ΠΎΠ΄Π½ΠΎΠ³ΠΎ Π²ΠΎΠ·Π΄ΡƒΡ…Π°. Π‘ ΠΊΠ°ΠΊΠΎΠΉ ΡΠΊΠΎΡ€ΠΎΡΡ‚ΡŒΡŽ Π² ΠΊΠΈΠ»ΠΎΠ²Π°Ρ‚Ρ‚Π°Ρ… Π΄ΠΎΠ»ΠΆΠ½Π° ΠΏΡ€ΠΎΠΈΡΡ…ΠΎΠ΄ΠΈΡ‚ΡŒ Ρ‚Π΅ΠΏΠ»ΠΎΠΏΠ΅Ρ€Π΅Π΄Π°Ρ‡Π°, Ρ‡Ρ‚ΠΎΠ±Ρ‹ Π½Π°Π³Ρ€Π΅Ρ‚ΡŒ этот Π²ΠΎΠ·Π΄ΡƒΡ… Π½Π° 10,0ΒΊC (Ρ‚ΠΎ Π΅ΡΡ‚ΡŒ довСсти Π²ΠΎΠ·Π΄ΡƒΡ… Π΄ΠΎ ΠΊΠΎΠΌΠ½Π°Ρ‚Π½ΠΎΠΉ Ρ‚Π΅ΠΌΠΏΠ΅Ρ€Π°Ρ‚ΡƒΡ€Ρ‹)?
  7. Π’ΡƒΠ»ΠΊΠ°Π½ ΠšΠΈΠ»Π°ΡƒΡΠ° Π½Π° Гавайях — самый Π°ΠΊΡ‚ΠΈΠ²Π½Ρ‹ΠΉ Π² ΠΌΠΈΡ€Π΅, ΠΈΠ·Π²Π΅Ρ€Π³Π°ΡŽΡ‰ΠΈΠΉ ΠΎΠΊΠΎΠ»ΠΎ 5 Γ— 10 5 ΠΌ 3 Π»Π°Π²Ρ‹ 1200ΒΊC Π² дСнь.Какова ΡΠΊΠΎΡ€ΠΎΡΡ‚ΡŒ ΠΏΠ΅Ρ€Π΅Π΄Π°Ρ‡ΠΈ Ρ‚Π΅ΠΏΠ»Π° ΠΎΡ‚ Π—Π΅ΠΌΠ»ΠΈ Π·Π° счСт ΠΊΠΎΠ½Π²Π΅ΠΊΡ†ΠΈΠΈ, Ссли эта Π»Π°Π²Π° ΠΈΠΌΠ΅Π΅Ρ‚ ΠΏΠ»ΠΎΡ‚Π½ΠΎΡΡ‚ΡŒ 2700 ΠΊΠ³ / ΠΌ 3 ΠΈ Π² ΠΊΠΎΠ½Π΅Ρ‡Π½ΠΎΠΌ ΠΈΡ‚ΠΎΠ³Π΅ остываСт Π΄ΠΎ 30ΒΊC? ΠŸΡ€Π΅Π΄ΠΏΠΎΠ»ΠΎΠΆΠΈΠΌ, Ρ‡Ρ‚ΠΎ ΡƒΠ΄Π΅Π»ΡŒΠ½Π°Ρ Ρ‚Π΅ΠΏΠ»ΠΎΠ΅ΠΌΠΊΠΎΡΡ‚ΡŒ Π»Π°Π²Ρ‹ такая ΠΆΠ΅, ΠΊΠ°ΠΊ Ρƒ Π³Ρ€Π°Π½ΠΈΡ‚Π°.

    Рис. 7. Π›Π°Π²ΠΎΠ²Ρ‹ΠΉ ΠΏΠΎΡ‚ΠΎΠΊ Π½Π° Π²ΡƒΠ»ΠΊΠ°Π½Π΅ ΠšΠΈΠ»Π°ΡƒΡΠ° Π½Π° Гавайях. (Π˜ΡΡ‚ΠΎΡ‡Π½ΠΈΠΊ: Π”ΠΆ. П. Π˜Ρ‚ΠΎΠ½, ГСологичСская слуТба БША)

  8. Π’ΠΎ врСмя тяТСлых ΡƒΠΏΡ€Π°ΠΆΠ½Π΅Π½ΠΈΠΉ Ρ‚Π΅Π»ΠΎ ΠΏΠ΅Ρ€Π΅ΠΊΠ°Ρ‡ΠΈΠ²Π°Π΅Ρ‚ 2,00 Π» ΠΊΡ€ΠΎΠ²ΠΈ Π² ΠΌΠΈΠ½ΡƒΡ‚Ρƒ Π½Π° ΠΏΠΎΠ²Π΅Ρ€Ρ…Π½ΠΎΡΡ‚ΡŒ, Π³Π΄Π΅ ΠΎΠ½Π° охлаТдаСтся Π΄ΠΎ 2,00 Β° C. Какова ΡΠΊΠΎΡ€ΠΎΡΡ‚ΡŒ Ρ‚Π΅ΠΏΠ»ΠΎΠΏΠ΅Ρ€Π΅Π΄Π°Ρ‡ΠΈ Ρ‚ΠΎΠ»ΡŒΠΊΠΎ ΠΎΡ‚ этой ΠΏΡ€ΠΈΠ½ΡƒΠ΄ΠΈΡ‚Π΅Π»ΡŒΠ½ΠΎΠΉ ΠΊΠΎΠ½Π²Π΅ΠΊΡ†ΠΈΠΈ, Ссли ΠΏΡ€Π΅Π΄ΠΏΠΎΠ»ΠΎΠΆΠΈΡ‚ΡŒ, Ρ‡Ρ‚ΠΎ ΠΊΡ€ΠΎΠ²ΡŒ ΠΈΠΌΠ΅Π΅Ρ‚ Ρ‚Π°ΠΊΡƒΡŽ ​​ТС ΡƒΠ΄Π΅Π»ΡŒΠ½ΡƒΡŽ Ρ‚Π΅ΠΏΠ»ΠΎΠ΅ΠΌΠΊΠΎΡΡ‚ΡŒ, Ρ‡Ρ‚ΠΎ ΠΈ Π²ΠΎΠ΄Π°, ΠΈ Π΅Π΅ ΠΏΠ»ΠΎΡ‚Π½ΠΎΡΡ‚ΡŒ составляСт 1050 ΠΊΠ³ / ΠΌ 3 ?
  9. Π§Π΅Π»ΠΎΠ²Π΅ΠΊ Π²Π΄Ρ‹Ρ…Π°Π΅Ρ‚ ΠΈ Π²Ρ‹Π΄Ρ‹Ρ…Π°Π΅Ρ‚ 2.00 Π» Π²ΠΎΠ·Π΄ΡƒΡ…Π° 37,0ΒΊC, ΠΈΡΠΏΠ°Ρ€ΡΡŽΡ‰Π΅Π³ΠΎ 4,00 Γ— 10 βˆ’2 Π³ Π²ΠΎΠ΄Ρ‹ ΠΈΠ· Π»Π΅Π³ΠΊΠΈΡ… ΠΈ Π΄Ρ‹Ρ…Π°Ρ‚Π΅Π»ΡŒΠ½Ρ‹Ρ… ΠΏΡƒΡ‚Π΅ΠΉ ΠΏΡ€ΠΈ ΠΊΠ°ΠΆΠ΄ΠΎΠΌ Π²Π΄ΠΎΡ…Π΅. Π°) Бколько Ρ‚Π΅ΠΏΠ»Π° происходит Π·Π° счСт испарСния ΠΏΡ€ΠΈ ΠΊΠ°ΠΆΠ΄ΠΎΠΌ Π²Π΄ΠΎΡ…Π΅? Π±) Какова ΡΠΊΠΎΡ€ΠΎΡΡ‚ΡŒ Ρ‚Π΅ΠΏΠ»ΠΎΠΏΠ΅Ρ€Π΅Π΄Π°Ρ‡ΠΈ Π² Π²Π°Ρ‚Ρ‚Π°Ρ…, Ссли Ρ‡Π΅Π»ΠΎΠ²Π΅ΠΊ Π΄Ρ‹ΡˆΠΈΡ‚ со срСднСй ΡΠΊΠΎΡ€ΠΎΡΡ‚ΡŒΡŽ 18,0 Π²Π΄ΠΎΡ…ΠΎΠ² Π² ΠΌΠΈΠ½ΡƒΡ‚Ρƒ? (c) Если Π²Π΄Ρ‹Ρ…Π°Π΅ΠΌΡ‹ΠΉ Π²ΠΎΠ·Π΄ΡƒΡ… ΠΈΠΌΠ΅Π» Ρ‚Π΅ΠΌΠΏΠ΅Ρ€Π°Ρ‚ΡƒΡ€Ρƒ 20,0 Β° C, ΠΊΠ°ΠΊΠΎΠ²Π° ΡΠΊΠΎΡ€ΠΎΡΡ‚ΡŒ Ρ‚Π΅ΠΏΠ»ΠΎΠΏΠ΅Ρ€Π΅Π΄Π°Ρ‡ΠΈ для Π½Π°Π³Ρ€Π΅Π²Π° Π²ΠΎΠ·Π΄ΡƒΡ…Π°? (d) ΠžΠ±ΡΡƒΠ΄ΠΈΡ‚Π΅ ΠΎΠ±Ρ‰ΡƒΡŽ ΡΠΊΠΎΡ€ΠΎΡΡ‚ΡŒ Ρ‚Π΅ΠΏΠ»ΠΎΠΏΠ΅Ρ€Π΅Π΄Π°Ρ‡ΠΈ, ΠΏΠΎΡΠΊΠΎΠ»ΡŒΠΊΡƒ ΠΎΠ½Π° соотносится с Ρ‚ΠΈΠΏΠΈΡ‡Π½ΠΎΠΉ ΡΠΊΠΎΡ€ΠΎΡΡ‚ΡŒΡŽ ΠΌΠ΅Ρ‚Π°Π±ΠΎΠ»ΠΈΠ·ΠΌΠ°.Π‘ΡƒΠ΄Π΅Ρ‚ Π»ΠΈ это Π΄Ρ‹Ρ…Π°Π½ΠΈΠ΅ основной Ρ„ΠΎΡ€ΠΌΠΎΠΉ ΠΏΠ΅Ρ€Π΅Π΄Π°Ρ‡ΠΈ Ρ‚Π΅ΠΏΠ»Π° для этого Ρ‡Π΅Π»ΠΎΠ²Π΅ΠΊΠ°?
  10. БтСклянный ΠΊΠΎΡ„Π΅ΠΉΠ½ΠΈΠΊ ΠΈΠΌΠ΅Π΅Ρ‚ ΠΊΡ€ΡƒΠ³Π»ΠΎΠ΅ Π΄Π½ΠΎ Π΄ΠΈΠ°ΠΌΠ΅Ρ‚Ρ€ΠΎΠΌ 9,00 см, ΠΊΠΎΠ½Ρ‚Π°ΠΊΡ‚ΠΈΡ€ΡƒΡŽΡ‰Π΅Π΅ с Π½Π°Π³Ρ€Π΅Π²Π°Ρ‚Π΅Π»ΡŒΠ½Ρ‹ΠΌ элСмСнтом, ΠΊΠΎΡ‚ΠΎΡ€Ρ‹ΠΉ ΠΏΠΎΠ΄Π΄Π΅Ρ€ΠΆΠΈΠ²Π°Π΅Ρ‚ ΠΊΠΎΡ„Π΅ Π² Ρ‚Π΅ΠΏΠ»Π΅ с постоянной ΡΠΊΠΎΡ€ΠΎΡΡ‚ΡŒΡŽ Ρ‚Π΅ΠΏΠ»ΠΎΠΏΠ΅Ρ€Π΅Π΄Π°Ρ‡ΠΈ 50,0 Π’Ρ‚. (A) Какова Ρ‚Π΅ΠΌΠΏΠ΅Ρ€Π°Ρ‚ΡƒΡ€Π° Π΄Π½Π° ΠΊΠΎΡ„Π΅ΠΉΠ½ΠΈΠΊΠ°, Ссли ΠΎΠ½ ΠΈΠΌΠ΅Π΅Ρ‚ Ρ‚ΠΎΠ»Ρ‰ΠΈΠ½Ρƒ 3,00 ΠΌΠΌ ΠΈ внутрСнняя Ρ‚Π΅ΠΌΠΏΠ΅Ρ€Π°Ρ‚ΡƒΡ€Π° 60,0ΒΊC? (Π±) Если Ρ‚Π΅ΠΌΠΏΠ΅Ρ€Π°Ρ‚ΡƒΡ€Π° ΠΊΠΎΡ„Π΅ остаСтся постоянной ΠΈ вся Ρ‚Π΅ΠΏΠ»ΠΎΠΏΠ΅Ρ€Π΅Π΄Π°Ρ‡Π° устраняСтся испарСниСм, сколько Π³Ρ€Π°ΠΌΠΌΠΎΠ² Π² ΠΌΠΈΠ½ΡƒΡ‚Ρƒ испаряСтся? ΠŸΡ€ΠΈΠ½ΡΡ‚ΡŒ Ρ‚Π΅ΠΏΠ»ΠΎΡ‚Ρƒ испарСния 2340 ΠΊΠ”ΠΆ / ΠΊΠ³.

Π˜Π·Π±Ρ€Π°Π½Π½Ρ‹Π΅ Ρ€Π΅ΡˆΠ΅Π½ΠΈΡ ΠΏΡ€ΠΎΠ±Π»Π΅ΠΌ ΠΈ упраТнСния

1. 10 м / с

3. 85,7ΒΊC

5. 1,48 ΠΊΠ³

7. 2 Γ— 10 4 ΠœΠ’Ρ‚

9. (Π°) 97,2 Π”ΠΆ; (Π±) 29,2 Π’Ρ‚; (c) 9,49 Π’Ρ‚; (Π³) ΠžΠ±Ρ‰Π°Ρ ΡΠΊΠΎΡ€ΠΎΡΡ‚ΡŒ ΠΏΠΎΡ‚Π΅Ρ€ΠΈ Ρ‚Π΅ΠΏΠ»Π° составит 29,2 Π’Ρ‚ + 9,49 Π’Ρ‚ = 38,7 Π’Ρ‚. Π’ΠΎ врСмя сна нашС Ρ‚Π΅Π»ΠΎ потрСбляСт 83 Π’Ρ‚ энСргии, Π² Ρ‚ΠΎ врСмя ΠΊΠ°ΠΊ сидя ΠΎΠ½ΠΎ потрСбляСт ΠΎΡ‚ 120 Π΄ΠΎ 210 Π’Ρ‚. Π‘Π»Π΅Π΄ΠΎΠ²Π°Ρ‚Π΅Π»ΡŒΠ½ΠΎ, общая ΡΠΊΠΎΡ€ΠΎΡΡ‚ΡŒ ΠΏΠΎΡ‚Π΅Ρ€ΠΈ Ρ‚Π΅ΠΏΠ»Π° ΠΎΡ‚ дыхания Π½Π΅ Π±ΡƒΠ΄Π΅Ρ‚ ΡΠ΅Ρ€ΡŒΠ΅Π·Π½ΠΎΠΉ Ρ„ΠΎΡ€ΠΌΠΎΠΉ ΠΏΠΎΡ‚Π΅Ρ€ΠΈ Ρ‚Π΅ΠΏΠ»Π° для этого Ρ‡Π΅Π»ΠΎΠ²Π΅ΠΊΠ°.

4 Ρ‚ΠΈΠΏΠ° ΠΌΠ΅Ρ…Π°Π½ΠΈΠ·ΠΌΠΎΠ² Ρ‚Π΅ΠΏΠ»ΠΎΠΏΠ΅Ρ€Π΅Π΄Π°Ρ‡ΠΈ для охлаТдСния элСктричСских ΡˆΠΊΠ°Ρ„ΠΎΠ²

ΠžΡ…Π»Π°ΠΆΠ΄Π΅Π½ΠΈΠ΅ элСктричСского ΡˆΠΊΠ°Ρ„Π° Π²ΠΊΠ»ΡŽΡ‡Π°Π΅Ρ‚ Π² сСбя процСссы ΠΏΠ΅Ρ€Π΅Π΄Π°Ρ‡ΠΈ Ρ‚Π΅ΠΏΠ»Π° ΠΈΠ·Π½ΡƒΡ‚Ρ€ΠΈ ΡˆΠΊΠ°Ρ„Π° ΠΈ ΠΎΡ‚Π²ΠΎΠ΄Π° Π΅Π³ΠΎ Π² ΠΎΠΊΡ€ΡƒΠΆΠ°ΡŽΡ‰ΠΈΠΉ Π²ΠΎΠ·Π΄ΡƒΡ…. Π‘ΡƒΡ‰Π΅ΡΡ‚Π²ΡƒΡŽΡ‚ Ρ€Π°Π·Π»ΠΈΡ‡Π½Ρ‹Π΅ ΠΌΠ΅Ρ…Π°Π½ΠΈΠ·ΠΌΡ‹ Ρ‚Π΅ΠΏΠ»ΠΎΠΏΠ΅Ρ€Π΅Π΄Π°Ρ‡ΠΈ, Π²ΠΊΠ»ΡŽΡ‡Π°Ρ ΠΊΠΎΠ½Π²Π΅ΠΊΡ†ΠΈΡŽ, Ρ‚Π΅ΠΏΠ»ΠΎΠΏΡ€ΠΎΠ²ΠΎΠ΄Π½ΠΎΡΡ‚ΡŒ, Ρ‚Π΅ΠΏΠ»ΠΎΠ²ΠΎΠ΅ ΠΈΠ·Π»ΡƒΡ‡Π΅Π½ΠΈΠ΅ ΠΈ ΠΈΡΠΏΠ°Ρ€ΠΈΡ‚Π΅Π»ΡŒΠ½ΠΎΠ΅ ΠΎΡ…Π»Π°ΠΆΠ΄Π΅Π½ΠΈΠ΅.

ΠœΠ΅Ρ…Π°Π½ΠΈΠ·ΠΌΡ‹ Ρ‚Π΅ΠΏΠ»ΠΎΠΏΠ΅Ρ€Π΅Π΄Π°Ρ‡ΠΈ

ΠžΡ…Π»Π°ΠΆΠ΄Π΅Π½ΠΈΠ΅ корпуса Π²ΠΊΠ»ΡŽΡ‡Π°Π΅Ρ‚ ΠΊΠΎΠΌΠ±ΠΈΠ½Π°Ρ†ΠΈΡŽ ΠΌΠ΅Ρ…Π°Π½ΠΈΠ·ΠΌΠΎΠ² Ρ‚Π΅ΠΏΠ»ΠΎΠΏΠ΅Ρ€Π΅Π΄Π°Ρ‡ΠΈ. ΠžΡΠ½ΠΎΠ²Π½Ρ‹Π΅ ΠΌΠ΅Ρ…Π°Π½ΠΈΠ·ΠΌΡ‹, ΠΈΡΠΏΠΎΠ»ΡŒΠ·ΡƒΠ΅ΠΌΡ‹Π΅ для охлаТдСния элСктричСских ΡˆΠΊΠ°Ρ„ΠΎΠ², ΡΠ»Π΅Π΄ΡƒΡŽΡ‰ΠΈΠ΅:

  • ΠŸΡ€ΠΎΠ²ΠΎΠ΄ΠΈΠΌΠΎΡΡ‚ΡŒ: Π­Ρ‚ΠΎ ΠΏΠ΅Ρ€Π΅Π΄Π°Ρ‡Π° Ρ‚Π΅ΠΏΠ»Π° Ρ‡Π΅Ρ€Π΅Π· Ρ‚Π²Π΅Ρ€Π΄ΠΎΠ΅ Ρ‚Π΅Π»ΠΎ.НапримСр, Ρ‚Π΅ΠΏΠ»ΠΎ, Π³Π΅Π½Π΅Ρ€ΠΈΡ€ΡƒΠ΅ΠΌΠΎΠ΅ Π²Π½ΡƒΡ‚Ρ€ΠΈ корпуса, пСрСдаСтся Π½Π° внСшнюю ΠΏΠΎΠ²Π΅Ρ€Ρ…Π½ΠΎΡΡ‚ΡŒ посрСдством тСплопроводности.
  • ΠšΠΎΠ½Π²Π΅ΠΊΡ†ΠΈΡ: ΠšΠΎΠ½Π²Π΅ΠΊΡ†ΠΈΡ — это ΠΏΠ΅Ρ€Π΅Π΄Π°Ρ‡Π° Ρ‚Π΅ΠΏΠ»Π° ΠΎΡ‚ повСрхности с ΠΏΠΎΠΌΠΎΡ‰ΡŒΡŽ Тидкости, Ρ‚Π°ΠΊΠΎΠΉ ΠΊΠ°ΠΊ Π²ΠΎΠ·Π΄ΡƒΡ…. ЕстСствСнная конвСкция Π²ΠΎΠ·Π½ΠΈΠΊΠ°Π΅Ρ‚ ΠΏΡ€ΠΈ Π½Π°Π³Ρ€Π΅Π²Π°Π½ΠΈΠΈ Π²ΠΎΠ·Π΄ΡƒΡ…Π°: ΠΎΠ½ Ρ€Π°ΡΡˆΠΈΡ€ΡΠ΅Ρ‚ΡΡ, поднимаСтся Π²Π²Π΅Ρ€Ρ… ΠΈ замСняСтся Π±ΠΎΠ»Π΅Π΅ Ρ…ΠΎΠ»ΠΎΠ΄Π½Ρ‹ΠΌ Π²ΠΎΠ·Π΄ΡƒΡ…ΠΎΠΌ. Π‘Ρ‚Π΅ΠΏΠ΅Π½ΡŒ ΠΊΠΎΠ½Π²Π΅ΠΊΡ†ΠΈΠΈ ΠΌΠΎΠΆΠ½ΠΎ ΡƒΠ²Π΅Π»ΠΈΡ‡ΠΈΡ‚ΡŒ, ΠΈΡΠΏΠΎΠ»ΡŒΠ·ΡƒΡ вСнтилятор для увСличСния ΠΏΠΎΡ‚ΠΎΠΊΠ° Π²ΠΎΠ·Π΄ΡƒΡ…Π°.
  • Π˜Π·Π»ΡƒΡ‡Π΅Π½ΠΈΠ΅: Π­Ρ‚ΠΎ процСсс, ΠΏΡ€ΠΈ ΠΊΠΎΡ‚ΠΎΡ€ΠΎΠΌ энСргия излучаСтся Ρ‡Π΅Ρ€Π΅Π· Π²ΠΎΠ·Π΄ΡƒΡ… посрСдством элСктромагнитного излучСния.Π₯отя ΠΎΠ½ эффСктивСн для источников высокой Ρ‚Π΅ΠΌΠΏΠ΅Ρ€Π°Ρ‚ΡƒΡ€Ρ‹, Ρ‚Π°ΠΊΠΈΡ… ΠΊΠ°ΠΊ солнцС, ΠΎΠ½ ΠΌΠ΅Π½Π΅Π΅ эффСктивСн ΠΏΡ€ΠΈ Ρ‚Π΅ΠΌΠΏΠ΅Ρ€Π°Ρ‚ΡƒΡ€Π΅ ΠΎΠΊΡ€ΡƒΠΆΠ°ΡŽΡ‰Π΅ΠΉ срСды Π½Π° Π—Π΅ΠΌΠ»Π΅.
  • Π˜ΡΠΏΠ°Ρ€Π΅Π½ΠΈΠ΅: Бкрытая Ρ‚Π΅ΠΏΠ»ΠΎΡ‚Π° Тидкости ΠΌΠΎΠΆΠ΅Ρ‚ ΠΈΡΠΏΠΎΠ»ΡŒΠ·ΠΎΠ²Π°Ρ‚ΡŒΡΡ для ΠΏΠ΅Ρ€Π΅Π΄Π°Ρ‡ΠΈ Ρ‚Π΅ΠΏΠ»Π° ΠΏΡƒΡ‚Π΅ΠΌ поглощСния энСргии, Π½Π΅ΠΎΠ±Ρ…ΠΎΠ΄ΠΈΠΌΠΎΠΉ для испарСния этой Тидкости. ΠŸΠΎΠ³Π»ΠΎΡ‰Π΅Π½Π½ΠΎΠ΅ Ρ‚Π΅ΠΏΠ»ΠΎ высвобоТдаСтся, позволяя Тидкости ΠΊΠΎΠ½Π΄Π΅Π½ΡΠΈΡ€ΠΎΠ²Π°Ρ‚ΡŒΡΡ Π·Π° ΠΏΡ€Π΅Π΄Π΅Π»Π°ΠΌΠΈ корпуса.

Π­Ρ‚ΠΈ Ρ„ΠΎΡ€ΠΌΡ‹ Ρ‚Π΅ΠΏΠ»ΠΎΠΏΠ΅Ρ€Π΅Π΄Π°Ρ‡ΠΈ ΠΈΡΠΏΠΎΠ»ΡŒΠ·ΡƒΡŽΡ‚ΡΡ для охлаТдСния элСктричСских ΡˆΠΊΠ°Ρ„ΠΎΠ² нСсколькими способами.

ПассивноС ΠΎΡ…Π»Π°ΠΆΠ΄Π΅Π½ΠΈΠ΅

ПассивноС ΠΎΡ…Π»Π°ΠΆΠ΄Π΅Π½ΠΈΠ΅, основанноС Π½Π° СстСствСнной тСплопроводности, ΠΊΠΎΠ½Π²Π΅ΠΊΡ†ΠΈΠΈ ΠΈ ΠΈΠ·Π»ΡƒΡ‡Π΅Π½ΠΈΠΈ, ΠΏΠΎΠ΄Ρ…ΠΎΠ΄ΠΈΡ‚ для ΠΌΠ°Π»ΠΎΠ½Π°Π³Ρ€ΡƒΠΆΠ΅Π½Π½Ρ‹Ρ… ΡˆΠΊΠ°Ρ„ΠΎΠ² с ΠΎΡ‚Π½ΠΎΡΠΈΡ‚Π΅Π»ΡŒΠ½ΠΎ большой ΠΏΠ»ΠΎΡ‰Π°Π΄ΡŒΡŽ повСрхности ΠΈ Ρ…ΠΎΡ€ΠΎΡˆΠ΅ΠΉ вСнтиляциСй.Π’Π΅ΠΌΠΏΠ΅Ρ€Π°Ρ‚ΡƒΡ€Π° ΠΎΠΊΡ€ΡƒΠΆΠ°ΡŽΡ‰Π΅Π³ΠΎ Π²ΠΎΠ·Π΄ΡƒΡ…Π° Π΄ΠΎΠ»ΠΆΠ½Π° Π±Ρ‹Ρ‚ΡŒ Π½ΠΈΠΆΠ΅ Ρ‚Π΅ΠΌΠΏΠ΅Ρ€Π°Ρ‚ΡƒΡ€Ρ‹ корпуса. Π­Ρ‚ΠΎΡ‚ ΠΌΠ΅Ρ‚ΠΎΠ΄ Π½Π΅ ΠΏΠΎΠ΄Ρ…ΠΎΠ΄ΠΈΡ‚ для Ρ‚Π΅Ρ€ΠΌΠΎΡ‡ΡƒΠ²ΡΡ‚Π²ΠΈΡ‚Π΅Π»ΡŒΠ½Ρ‹Ρ… ΠΊΠΎΠΌΠΏΠΎΠ½Π΅Π½Ρ‚ΠΎΠ² ΠΏΡ€ΠΈ высоких Ρ‚Π΅ΠΌΠΏΠ΅Ρ€Π°Ρ‚ΡƒΡ€Π°Ρ… ΠΎΠΊΡ€ΡƒΠΆΠ°ΡŽΡ‰Π΅ΠΉ срСды.

ΠŸΡ€ΠΈΠ½ΡƒΠ΄ΠΈΡ‚Π΅Π»ΡŒΠ½Π°Ρ вСнтиляция

Π­Ρ„Ρ„Π΅ΠΊΡ‚ΠΈΠ²Π½ΠΎΡΡ‚ΡŒ ΠΊΠΎΠ½Π²Π΅ΠΊΡ†ΠΈΠΈ ΠΌΠΎΠΆΠ½ΠΎ ΠΏΠΎΠ²Ρ‹ΡΠΈΡ‚ΡŒ Π·Π° счСт использования вСнтиляторов, ΠΊΠΎΡ‚ΠΎΡ€Ρ‹Π΅ ΡƒΠ²Π΅Π»ΠΈΡ‡ΠΈΠ²Π°ΡŽΡ‚ ΠΏΠΎΡ‚ΠΎΠΊ Π²ΠΎΠ·Π΄ΡƒΡ…Π° Ρ‡Π΅Ρ€Π΅Π· ΡˆΠΊΠ°Ρ„. Π₯ΠΎΠ»ΠΎΠ΄Π½Ρ‹ΠΉ Π²ΠΎΠ·Π΄ΡƒΡ… втягиваСтся Π² ниТнюю Ρ‡Π°ΡΡ‚ΡŒ ΡˆΠΊΠ°Ρ„Π°, Π° горячий Π²ΠΎΠ·Π΄ΡƒΡ… Π²Ρ‹Ρ…ΠΎΠ΄ΠΈΡ‚ Π² Π²Π΅Ρ€Ρ…Π½ΡŽΡŽ Ρ‡Π°ΡΡ‚ΡŒ. ВСнтиляторы Π΄ΠΎΠ»ΠΆΠ½Ρ‹ Π±Ρ‹Ρ‚ΡŒ оснащСны Ρ„ΠΈΠ»ΡŒΡ‚Ρ€Π°ΠΌΠΈ, Ρ‡Ρ‚ΠΎΠ±Ρ‹ ΠΎΠ³Ρ€Π°Π½ΠΈΡ‡ΠΈΡ‚ΡŒ ΠΏΠΎΠΏΠ°Π΄Π°Π½ΠΈΠ΅ грязи, которая ΠΌΠΎΠΆΠ΅Ρ‚ ΠΏΠΎΠ²Ρ€Π΅Π΄ΠΈΡ‚ΡŒ ΠΊΠΎΠΌΠΏΠΎΠ½Π΅Π½Ρ‚Ρ‹.Π§Ρ‚ΠΎΠ±Ρ‹ элСктричСскиС ΠΊΠΎΠΌΠΏΠΎΠ½Π΅Π½Ρ‚Ρ‹ Π½Π΅ ΠΏΠ΅Ρ€Π΅Π³Ρ€Π΅Π²Π°Π»ΠΈΡΡŒ, Ρ‚Π΅ΠΌΠΏΠ΅Ρ€Π°Ρ‚ΡƒΡ€Π° ΠΎΠΊΡ€ΡƒΠΆΠ°ΡŽΡ‰Π΅ΠΉ срСды Π΄ΠΎΠ»ΠΆΠ½Π° Π±Ρ‹Ρ‚ΡŒ Π·Π½Π°Ρ‡ΠΈΡ‚Π΅Π»ΡŒΠ½ΠΎ Π½ΠΈΠΆΠ΅ максимальной ΠΆΠ΅Π»Π°Π΅ΠΌΠΎΠΉ Ρ‚Π΅ΠΌΠΏΠ΅Ρ€Π°Ρ‚ΡƒΡ€Ρ‹ корпуса.

ВСхнология Ρ‚Π΅ΠΏΠ»ΠΎΠ²Ρ‹Ρ… Ρ‚Ρ€ΡƒΠ±

Π’Π΅ΠΏΠ»ΠΎΠ²Ρ‹Π΅ Ρ‚Ρ€ΡƒΠ±ΠΊΠΈ, Π²ΠΏΠ΅Ρ€Π²Ρ‹Π΅ Ρ€Π°Π·Ρ€Π°Π±ΠΎΡ‚Π°Π½Π½Ρ‹Π΅ Π² 1960-Ρ… Π³ΠΎΠ΄Π°Ρ…, ΠΏΡ€Π΅Π΄ΡΡ‚Π°Π²Π»ΡΡŽΡ‚ собой практичСски бСзэнСргСтичСский ΠΌΠ΅Ρ‚ΠΎΠ΄ охлаТдСния корпуса. ВСпловая Ρ‚Ρ€ΡƒΠ±ΠΊΠ° состоит ΠΈΠ· Π²Π°ΠΊΡƒΡƒΠΌΠΈΡ€ΠΎΠ²Π°Π½Π½ΠΎΠΉ ΠΌΠ΅Π΄Π½ΠΎΠΉ Ρ‚Ρ€ΡƒΠ±ΠΊΠΈ, частично Π·Π°ΠΏΠΎΠ»Π½Π΅Π½Π½ΠΎΠΉ ΠΆΠΈΠ΄ΠΊΠΎΡΡ‚ΡŒΡŽ, Π½Π°ΠΏΡ€ΠΈΠΌΠ΅Ρ€ спиртом ΠΈΠ»ΠΈ Π²ΠΎΠ΄ΠΎΠΉ. Из-Π·Π° Π½ΠΈΠ·ΠΊΠΎΠ³ΠΎ давлСния ΠΆΠΈΠ΄ΠΊΠΎΡΡ‚ΡŒ Π½Π° Π΄Π½Π΅ Ρ‚Ρ€ΡƒΠ±Ρ‹ Π·Π°ΠΊΠΈΠΏΠ°Π΅Ρ‚, ΠΊΠΎΠ³Π΄Π° ΠΎΠ½Π° ΠΏΠΎΠ³Π»ΠΎΡ‰Π°Π΅Ρ‚ Ρ‚Π΅ΠΏΠ»ΠΎ ΠΈΠ· Π²ΠΎΠ·Π΄ΡƒΡ…Π° Π²Π½ΡƒΡ‚Ρ€ΠΈ помСщСния.ΠŸΠ°Ρ€ поднимаСтся ΠΊ Π²Π΅Ρ€Ρ…Ρƒ Ρ‚Ρ€ΡƒΠ±Ρ‹, Π³Π΄Π΅ ΠΎΠ½ охлаТдаСтся Π²ΠΎΠ·Π΄ΡƒΡ…ΠΎΠΌ Π·Π° ΠΏΡ€Π΅Π΄Π΅Π»Π°ΠΌΠΈ ΠΊΠ°ΠΌΠ΅Ρ€Ρ‹ ΠΈ кондСнсируСтся. Π—Π°Ρ‚Π΅ΠΌ кондСнсированная ΠΆΠΈΠ΄ΠΊΠΎΡΡ‚ΡŒ возвращаСтся Π½Π° Π΄Π½ΠΎ Ρ‚Ρ€ΡƒΠ±ΠΊΠΈ, ΠΈ Ρ†ΠΈΠΊΠ» повторяСтся.

Π’ Ρ‚Π΅ΠΏΠ»ΠΎΠΎΠ±ΠΌΠ΅Π½Π½ΠΈΠΊΠ°Ρ… Β«Π²ΠΎΠ·Π΄ΡƒΡ…-Π²ΠΎΠ·Π΄ΡƒΡ…Β» ΠΊΠΎΠΌΠΏΠ°Π½ΠΈΠΈ Thermal Edge ΠΈΡΠΏΠΎΠ»ΡŒΠ·ΡƒΠ΅Ρ‚ΡΡ эта новая тСхнология для охлаТдСния Π³Π΅Ρ€ΠΌΠ΅Ρ‚ΠΈΡ‡Π½Ρ‹Ρ… элСктричСских ΡˆΠΊΠ°Ρ„ΠΎΠ². ЕдинствСнная энСргия, нСобходимая малСньким вСнтиляторам, — это циркуляция Π²ΠΎΠ·Π΄ΡƒΡ…Π° Π²ΠΎΠΊΡ€ΡƒΠ³ горячих ΠΈ Ρ…ΠΎΠ»ΠΎΠ΄Π½Ρ‹Ρ… ΠΊΠΎΠ½Ρ†ΠΎΠ² Ρ‚Π΅ΠΏΠ»ΠΎΠ²ΠΎΠΉ Ρ‚Ρ€ΡƒΠ±ΠΊΠΈ.

ΠšΠΎΠ½Π΄ΠΈΡ†ΠΈΠΎΠ½Π΅Ρ€ Π² ΡˆΠΊΠ°Ρ„Ρƒ

Π’ ΠΊΠΎΠ½Π΄ΠΈΡ†ΠΈΠΎΠ½ΠΈΡ€ΠΎΠ²Π°Π½ΠΈΠΈ Π²ΠΎΠ·Π΄ΡƒΡ…Π° Ρ‚Π°ΠΊΠΆΠ΅ ΠΈΡΠΏΠΎΠ»ΡŒΠ·ΡƒΠ΅Ρ‚ΡΡ испарСниС, Π½ΠΎ Π½Π΅ΠΌΠ½ΠΎΠ³ΠΎ ΠΏΠΎ-Π΄Ρ€ΡƒΠ³ΠΎΠΌΡƒ.Π–ΠΈΠ΄ΠΊΠΈΠΉ Ρ…Π»Π°Π΄Π°Π³Π΅Π½Ρ‚ ΠΏΠΎΠ΄ Π΄Π°Π²Π»Π΅Π½ΠΈΠ΅ΠΌ пропускаСтся Ρ‡Π΅Ρ€Π΅Π· Ρ€Π°ΡΡˆΠΈΡ€ΠΈΡ‚Π΅Π»ΡŒΠ½ΠΎΠ΅ устройство. ПадСниС давлСния Π²Ρ‹Π·Ρ‹Π²Π°Π΅Ρ‚ испарСниС Тидкости Π² Π·ΠΌΠ΅Π΅Π²ΠΈΠΊΠ΅ испаритСля ΠΊΠΎΠ½Π΄ΠΈΡ†ΠΈΠΎΠ½Π΅Ρ€Π° ΠΈ ΠΏΠΎΠ³Π»ΠΎΡ‰Π΅Π½ΠΈΠ΅ Ρ‚Π΅ΠΏΠ»Π°, охлаТдая Π²ΠΎΠ·Π΄ΡƒΡ… Π²Π½ΡƒΡ‚Ρ€ΠΈ помСщСния. Π—Π°Ρ‚Π΅ΠΌ горячий Π³Π°Π· сТимаСтся ΠΈ ΠΏΡ€ΠΎΡ…ΠΎΠ΄ΠΈΡ‚ Ρ‡Π΅Ρ€Π΅Π· Π·ΠΌΠ΅Π΅Π²ΠΈΠΊ кондСнсатора, Π³Π΄Π΅ Π³Π°Π· сТиТаСтся, пСрСдавая своС Ρ‚Π΅ΠΏΠ»ΠΎ Π²ΠΎΠ·Π΄ΡƒΡ…Ρƒ Π·Π° ΠΏΡ€Π΅Π΄Π΅Π»Π°ΠΌΠΈ ΠΊΠ°ΠΌΠ΅Ρ€Ρ‹. ΠšΠΎΠΌΠ±ΠΈΠ½ΠΈΡ€ΠΎΠ²Π°Π½Π½Ρ‹ΠΉ ΠΊΠΎΠ½Π΄ΠΈΡ†ΠΈΠΎΠ½Π΅Ρ€ прСдставляСт собой Ρ‡Ρ€Π΅Π·Π²Ρ‹Ρ‡Π°ΠΉΠ½ΠΎ эффСктивный ΠΌΠ΅Ρ‚ΠΎΠ΄ охлаТдСния ΡˆΠΊΠ°Ρ„Π° ΠΈ Π±ΡƒΠ΄Π΅Ρ‚ Ρ€Π°Π±ΠΎΡ‚Π°Ρ‚ΡŒ эффСктивно, Π΄Π°ΠΆΠ΅ Ссли Ρ‚Π΅ΠΌΠΏΠ΅Ρ€Π°Ρ‚ΡƒΡ€Π° ΠΎΠΊΡ€ΡƒΠΆΠ°ΡŽΡ‰Π΅ΠΉ срСды Π½Π°ΠΌΠ½ΠΎΠ³ΠΎ Π²Ρ‹ΡˆΠ΅, Ρ‡Π΅ΠΌ Ρ‚Π΅ΠΌΠΏΠ΅Ρ€Π°Ρ‚ΡƒΡ€Π° Π²ΠΎΠ·Π΄ΡƒΡ…Π° Π² ΡˆΠΊΠ°Ρ„Ρƒ.

Π§Ρ‚ΠΎΠ±Ρ‹ ΡƒΠ·Π½Π°Ρ‚ΡŒ большС ΠΎ ΠΏΡ€ΠΎΠ΄ΡƒΠΊΡ‚Π°Ρ… Thermal Edge ΠΈ ΠΎ Ρ‚ΠΎΠΌ, ΠΊΠ°ΠΊ Π²Ρ‹Π±Ρ€Π°Ρ‚ΡŒ подходящий ΠΌΠ΅Ρ‚ΠΎΠ΄ охлаТдСния для вашСго элСктричСского ΡˆΠΊΠ°Ρ„Π°, ΡΠ²ΡΠΆΠΈΡ‚Π΅ΡΡŒ с нашим ΠΎΡ‚Π΄Π΅Π»ΠΎΠΌ ΠΏΡ€ΠΎΠ΄Π°ΠΆ сСгодня.

11.2 Π’Π΅ΠΏΠ»ΠΎ, ΡƒΠ΄Π΅Π»ΡŒΠ½Π°Ρ Ρ‚Π΅ΠΏΠ»ΠΎΠ΅ΠΌΠΊΠΎΡΡ‚ΡŒ ΠΈ Ρ‚Π΅ΠΏΠ»ΠΎΠΏΠ΅Ρ€Π΅Π΄Π°Ρ‡Π°

ΠŸΡ€ΠΎΠ²ΠΎΠ΄ΠΈΠΌΠΎΡΡ‚ΡŒ, конвСкция ΠΈ ΠΈΠ·Π»ΡƒΡ‡Π΅Π½ΠΈΠ΅

ΠŸΡ€ΠΈ Π²ΠΎΠ·Π½ΠΈΠΊΠ½ΠΎΠ²Π΅Π½ΠΈΠΈ Ρ€Π°Π·Π½ΠΈΡ†Ρ‹ Ρ‚Π΅ΠΌΠΏΠ΅Ρ€Π°Ρ‚ΡƒΡ€ происходит Ρ‚Π΅ΠΏΠ»ΠΎΠΏΠ΅Ρ€Π΅Π΄Π°Ρ‡Π°. ΠŸΠ΅Ρ€Π΅Π΄Π°Ρ‡Π° Ρ‚Π΅ΠΏΠ»Π° ΠΌΠΎΠΆΠ΅Ρ‚ ΠΏΡ€ΠΎΠΈΡΡ…ΠΎΠ΄ΠΈΡ‚ΡŒ быстро, Π½Π°ΠΏΡ€ΠΈΠΌΠ΅Ρ€, Ρ‡Π΅Ρ€Π΅Π· сковороду, ΠΈΠ»ΠΈ ΠΌΠ΅Π΄Π»Π΅Π½Π½ΠΎ, Π½Π°ΠΏΡ€ΠΈΠΌΠ΅Ρ€, Ρ‡Π΅Ρ€Π΅Π· стСнки ΠΈΠ·ΠΎΠ»ΠΈΡ€ΠΎΠ²Π°Π½Π½ΠΎΠ³ΠΎ Ρ…ΠΎΠ»ΠΎΠ΄ΠΈΠ»ΡŒΠ½ΠΈΠΊΠ°.

БущСствуСт Ρ‚Ρ€ΠΈ Ρ€Π°Π·Π»ΠΈΡ‡Π½Ρ‹Ρ… ΠΌΠ΅Ρ‚ΠΎΠ΄Π° Ρ‚Π΅ΠΏΠ»ΠΎΠΏΠ΅Ρ€Π΅Π΄Π°Ρ‡ΠΈ: Ρ‚Π΅ΠΏΠ»ΠΎΠΏΡ€ΠΎΠ²ΠΎΠ΄Π½ΠΎΡΡ‚ΡŒ, конвСкция ΠΈ ΠΈΠ·Π»ΡƒΡ‡Π΅Π½ΠΈΠ΅.Иногда всС Ρ‚Ρ€ΠΈ ΠΌΠΎΠ³ΡƒΡ‚ ΠΏΡ€ΠΎΠΈΡΡ…ΠΎΠ΄ΠΈΡ‚ΡŒ ΠΎΠ΄Π½ΠΎΠ²Ρ€Π΅ΠΌΠ΅Π½Π½ΠΎ. Π‘ΠΌ. Рисунок 11.3.

Рис. 11.3 Π’ ΠΊΠ°ΠΌΠΈΠ½Π΅ ΠΏΠ΅Ρ€Π΅Π΄Π°Ρ‡Π° Ρ‚Π΅ΠΏΠ»Π° происходит всСми трСмя способами: Ρ‚Π΅ΠΏΠ»ΠΎΠΏΡ€ΠΎΠ²ΠΎΠ΄Π½ΠΎΡΡ‚ΡŒΡŽ, ΠΊΠΎΠ½Π²Π΅ΠΊΡ†ΠΈΠ΅ΠΉ ΠΈ ΠΈΠ·Π»ΡƒΡ‡Π΅Π½ΠΈΠ΅ΠΌ. Π˜Π·Π»ΡƒΡ‡Π΅Π½ΠΈΠ΅ ΠΎΡ‚Π²Π΅Ρ‡Π°Π΅Ρ‚ Π·Π° Π±ΠΎΠ»ΡŒΡˆΡƒΡŽ Ρ‡Π°ΡΡ‚ΡŒ Ρ‚Π΅ΠΏΠ»Π°, ΠΏΠ΅Ρ€Π΅Π΄Π°Π²Π°Π΅ΠΌΠΎΠ³ΠΎ Π² ΠΊΠΎΠΌΠ½Π°Ρ‚Ρƒ. ΠŸΠ΅Ρ€Π΅Π΄Π°Ρ‡Π° Ρ‚Π΅ΠΏΠ»Π° Ρ‚Π°ΠΊΠΆΠ΅ происходит Ρ‡Π΅Ρ€Π΅Π· Ρ‚Π΅ΠΏΠ»ΠΎΠΏΡ€ΠΎΠ²ΠΎΠ΄Π½ΠΎΡΡ‚ΡŒ Π² ΠΊΠΎΠΌΠ½Π°Ρ‚Ρƒ, Π½ΠΎ Π³ΠΎΡ€Π°Π·Π΄ΠΎ ΠΌΠ΅Π΄Π»Π΅Π½Π½Π΅Π΅. Π’Π΅ΠΏΠ»ΠΎΠΎΠ±ΠΌΠ΅Π½ Π·Π° счСт ΠΊΠΎΠ½Π²Π΅ΠΊΡ†ΠΈΠΈ Ρ‚Π°ΠΊΠΆΠ΅ происходит Ρ‡Π΅Ρ€Π΅Π· Ρ…ΠΎΠ»ΠΎΠ΄Π½Ρ‹ΠΉ Π²ΠΎΠ·Π΄ΡƒΡ…, ΠΏΠΎΡΡ‚ΡƒΠΏΠ°ΡŽΡ‰ΠΈΠΉ Π² ΠΊΠΎΠΌΠ½Π°Ρ‚Ρƒ Π²ΠΎΠΊΡ€ΡƒΠ³ ΠΎΠΊΠΎΠ½, ΠΈ горячий Π²ΠΎΠ·Π΄ΡƒΡ…, ΠΏΠΎΠΊΠΈΠ΄Π°ΡŽΡ‰ΠΈΠΉ ΠΊΠΎΠΌΠ½Π°Ρ‚Ρƒ, поднимаясь Π²Π²Π΅Ρ€Ρ… ΠΏΠΎ Π΄Ρ‹ΠΌΠΎΡ…ΠΎΠ΄Ρƒ.

ΠŸΡ€ΠΎΠ²ΠΎΠ΄ΠΈΠΌΠΎΡΡ‚ΡŒ — это ΠΏΠ΅Ρ€Π΅Π΄Π°Ρ‡Π° Ρ‚Π΅ΠΏΠ»Π° ΠΏΡ€ΠΈ прямом физичСском ΠΊΠΎΠ½Ρ‚Π°ΠΊΡ‚Π΅.Π’Π΅ΠΏΠ»ΠΎ, ΠΏΠ΅Ρ€Π΅Π΄Π°Π²Π°Π΅ΠΌΠΎΠ΅ ΠΌΠ΅ΠΆΠ΄Ρƒ элСктричСской Π³ΠΎΡ€Π΅Π»ΠΊΠΎΠΉ ΠΏΠ»ΠΈΡ‚Ρ‹ ΠΈ Π΄Π½ΠΎΠΌ сковороды, пСрСдаСтся Π·Π° счСт тСплопроводности. Иногда ΠΌΡ‹ пытаСмся ΠΊΠΎΠ½Ρ‚Ρ€ΠΎΠ»ΠΈΡ€ΠΎΠ²Π°Ρ‚ΡŒ Ρ‚Π΅ΠΏΠ»ΠΎΠΏΡ€ΠΎΠ²ΠΎΠ΄Π½ΠΎΡΡ‚ΡŒ, Ρ‡Ρ‚ΠΎΠ±Ρ‹ Ρ‡ΡƒΠ²ΡΡ‚Π²ΠΎΠ²Π°Ρ‚ΡŒ сСбя Π±ΠΎΠ»Π΅Π΅ ΠΊΠΎΠΌΡ„ΠΎΡ€Ρ‚Π½ΠΎ. ΠŸΠΎΡΠΊΠΎΠ»ΡŒΠΊΡƒ ΡΠΊΠΎΡ€ΠΎΡΡ‚ΡŒ Ρ‚Π΅ΠΏΠ»ΠΎΠΏΠ΅Ρ€Π΅Π΄Π°Ρ‡ΠΈ Ρƒ Ρ€Π°Π·Π½Ρ‹Ρ… ΠΌΠ°Ρ‚Π΅Ρ€ΠΈΠ°Π»ΠΎΠ² разная, ΠΌΡ‹ Π²Ρ‹Π±ΠΈΡ€Π°Π΅ΠΌ Ρ‚Π°ΠΊΠΈΠ΅ Ρ‚ΠΊΠ°Π½ΠΈ, ΠΊΠ°ΠΊ толстый ΡˆΠ΅Ρ€ΡΡ‚ΡΠ½ΠΎΠΉ свитСр, ΠΊΠΎΡ‚ΠΎΡ€Ρ‹Π΅ Π·ΠΈΠΌΠΎΠΉ Π·Π°ΠΌΠ΅Π΄Π»ΡΡŽΡ‚ ΠΎΡ‚Π²ΠΎΠ΄ Ρ‚Π΅ΠΏΠ»Π° ΠΎΡ‚ нашСго Ρ‚Π΅Π»Π°.

Когда Π²Ρ‹ ΠΈΠ΄Π΅Ρ‚Π΅ босиком ΠΏΠΎ ΠΊΠΎΠ²Ρ€Ρƒ Π² гостиной, ваши Π½ΠΎΠ³ΠΈ Ρ‡ΡƒΠ²ΡΡ‚Π²ΡƒΡŽΡ‚ сСбя ΠΎΡ‚Π½ΠΎΡΠΈΡ‚Π΅Π»ΡŒΠ½ΠΎ комфортно… ΠΏΠΎΠΊΠ° Π²Ρ‹ Π½Π΅ ступитС Π½Π° ΠΊΠ°Ρ„Π΅Π»ΡŒΠ½Ρ‹ΠΉ ΠΏΠΎΠ» ΠΊΡƒΡ…Π½ΠΈ.ΠŸΠΎΡΠΊΠΎΠ»ΡŒΠΊΡƒ ΠΊΠΎΠ²Π΅Ρ€ ΠΈ ΠΊΠ°Ρ„Π΅Π»ΡŒΠ½Ρ‹ΠΉ ΠΏΠΎΠ» ΠΈΠΌΠ΅ΡŽΡ‚ ΠΎΠ΄ΠΈΠ½Π°ΠΊΠΎΠ²ΡƒΡŽ Ρ‚Π΅ΠΌΠΏΠ΅Ρ€Π°Ρ‚ΡƒΡ€Ρƒ, ΠΏΠΎΡ‡Π΅ΠΌΡƒ ΠΎΠ΄ΠΈΠ½ ΠΈΠ· Π½ΠΈΡ… Ρ…ΠΎΠ»ΠΎΠ΄Π½Π΅Π΅ Π΄Ρ€ΡƒΠ³ΠΎΠ³ΠΎ? Π­Ρ‚ΠΎ ΠΎΠ±ΡŠΡΡΠ½ΡΠ΅Ρ‚ΡΡ Ρ€Π°Π·Π½ΠΎΠΉ ΡΠΊΠΎΡ€ΠΎΡΡ‚ΡŒΡŽ Ρ‚Π΅ΠΏΠ»ΠΎΠΏΠ΅Ρ€Π΅Π΄Π°Ρ‡ΠΈ: ΠΌΠ°Ρ‚Π΅Ρ€ΠΈΠ°Π» ΠΏΠ»ΠΈΡ‚ΠΊΠΈ ΠΎΡ‚Π²ΠΎΠ΄ΠΈΡ‚ Ρ‚Π΅ΠΏΠ»ΠΎ ΠΎΡ‚ вашСй ΠΊΠΎΠΆΠΈ с большСй ΡΠΊΠΎΡ€ΠΎΡΡ‚ΡŒΡŽ, Ρ‡Π΅ΠΌ ΠΊΠΎΠ²Ρ€ΠΎΠ²ΠΎΠ΅ ΠΏΠΎΠΊΡ€Ρ‹Ρ‚ΠΈΠ΅, Ρ‡Ρ‚ΠΎ Π΄Π΅Π»Π°Π΅Ρ‚ Π΅Π³ΠΎ Π±ΠΎΠ»Π΅Π΅ Ρ…ΠΎΠ»ΠΎΠ΄Π½Ρ‹ΠΌ.

НСкоторыС ΠΌΠ°Ρ‚Π΅Ρ€ΠΈΠ°Π»Ρ‹ просто проводят Ρ‚Π΅ΠΏΠ»ΠΎΠ²ΡƒΡŽ ΡΠ½Π΅Ρ€Π³ΠΈΡŽ быстрСС, Ρ‡Π΅ΠΌ Π΄Ρ€ΡƒΠ³ΠΈΠ΅. Π’ ΠΎΠ±Ρ‰Π΅ΠΌ, ΠΌΠ΅Ρ‚Π°Π»Π»Ρ‹ (Π½Π°ΠΏΡ€ΠΈΠΌΠ΅Ρ€, мСдь, алюминий, Π·ΠΎΠ»ΠΎΡ‚ΠΎ ΠΈ сСрСбро) ΡΠ²Π»ΡΡŽΡ‚ΡΡ Ρ…ΠΎΡ€ΠΎΡˆΠΈΠΌΠΈ ΠΏΡ€ΠΎΠ²ΠΎΠ΄Π½ΠΈΠΊΠ°ΠΌΠΈ Ρ‚Π΅ΠΏΠ»Π°, Ρ‚ΠΎΠ³Π΄Π° ΠΊΠ°ΠΊ Ρ‚Π°ΠΊΠΈΠ΅ ΠΌΠ°Ρ‚Π΅Ρ€ΠΈΠ°Π»Ρ‹, ΠΊΠ°ΠΊ Π΄Π΅Ρ€Π΅Π²ΠΎ, пластик ΠΈ Ρ€Π΅Π·ΠΈΠ½Π°, ΠΏΠ»ΠΎΡ…ΠΎ проводят Ρ‚Π΅ΠΏΠ»ΠΎ.

На рисункС 11.4 ΠΏΠΎΠΊΠ°Π·Π°Π½Ρ‹ частицы (Π°Ρ‚ΠΎΠΌΡ‹ ΠΈΠ»ΠΈ ΠΌΠΎΠ»Π΅ΠΊΡƒΠ»Ρ‹) Π² Π΄Π²ΡƒΡ… Ρ‚Π΅Π»Π°Ρ… ΠΏΡ€ΠΈ Ρ€Π°Π·Π½Ρ‹Ρ… Ρ‚Π΅ΠΌΠΏΠ΅Ρ€Π°Ρ‚ΡƒΡ€Π°Ρ…. (БрСдняя) кинСтичСская энСргия частицы Π² горячСм Ρ‚Π΅Π»Π΅ Π²Ρ‹ΡˆΠ΅, Ρ‡Π΅ΠΌ Π² Π±ΠΎΠ»Π΅Π΅ Ρ…ΠΎΠ»ΠΎΠ΄Π½ΠΎΠΌ Ρ‚Π΅Π»Π΅. Если Π΄Π²Π΅ частицы ΡΡ‚Π°Π»ΠΊΠΈΠ²Π°ΡŽΡ‚ΡΡ, энСргия пСрСдаСтся ΠΎΡ‚ частицы с большСй кинСтичСской энСргиСй ΠΊ частицС с мСньшСй кинСтичСской энСргиСй. Когда Π΄Π²Π° Ρ‚Π΅Π»Π° находятся Π² ΠΊΠΎΠ½Ρ‚Π°ΠΊΡ‚Π΅, происходит ΠΌΠ½ΠΎΠ³ΠΎ столкновСний частиц, Ρ‡Ρ‚ΠΎ ΠΏΡ€ΠΈΠ²ΠΎΠ΄ΠΈΡ‚ ΠΊ чистому ΠΏΠΎΡ‚ΠΎΠΊΡƒ Ρ‚Π΅ΠΏΠ»Π° ΠΎΡ‚ Ρ‚Π΅Π»Π° с Π±ΠΎΠ»Π΅Π΅ высокой Ρ‚Π΅ΠΌΠΏΠ΅Ρ€Π°Ρ‚ΡƒΡ€ΠΎΠΉ ΠΊ Ρ‚Π΅Π»Ρƒ с Π±ΠΎΠ»Π΅Π΅ Π½ΠΈΠ·ΠΊΠΎΠΉ Ρ‚Π΅ΠΌΠΏΠ΅Ρ€Π°Ρ‚ΡƒΡ€ΠΎΠΉ. Π’Π΅ΠΏΠ»ΠΎΠ²ΠΎΠΉ ΠΏΠΎΡ‚ΠΎΠΊ зависит ΠΎΡ‚ разности Ρ‚Π΅ΠΌΠΏΠ΅Ρ€Π°Ρ‚ΡƒΡ€ Ξ”T = Thot-TcoldΞ”T = Thot-Tcold.Π’Π°ΠΊΠΈΠΌ ΠΎΠ±Ρ€Π°Π·ΠΎΠΌ, Π²Ρ‹ ΠΏΠΎΠ»ΡƒΡ‡ΠΈΡ‚Π΅ Π±ΠΎΠ»Π΅Π΅ ΡΠΈΠ»ΡŒΠ½Ρ‹ΠΉ ΠΎΠΆΠΎΠ³ ΠΎΡ‚ кипятка, Ρ‡Π΅ΠΌ ΠΎΡ‚ горячСй Π²ΠΎΠ΄Ρ‹ ΠΈΠ·-ΠΏΠΎΠ΄ ΠΊΡ€Π°Π½Π°.

Рис. 11.4 Частицы Π² Π΄Π²ΡƒΡ… Ρ‚Π΅Π»Π°Ρ… ΠΏΡ€ΠΈ Ρ€Π°Π·Π½Ρ‹Ρ… Ρ‚Π΅ΠΌΠΏΠ΅Ρ€Π°Ρ‚ΡƒΡ€Π°Ρ… ΠΈΠΌΠ΅ΡŽΡ‚ Ρ€Π°Π·Π½Ρ‹Π΅ срСдниС кинСтичСскиС энСргии. БтолкновСния, происходящиС Π½Π° ΠΊΠΎΠ½Ρ‚Π°ΠΊΡ‚Π½ΠΎΠΉ повСрхности, ΠΈΠΌΠ΅ΡŽΡ‚ Ρ‚Π΅Π½Π΄Π΅Π½Ρ†ΠΈΡŽ ΠΏΠ΅Ρ€Π΅Π΄Π°Π²Π°Ρ‚ΡŒ ΡΠ½Π΅Ρ€Π³ΠΈΡŽ ΠΈΠ· высокотСмпСратурных областСй Π² Π½ΠΈΠ·ΠΊΠΎΡ‚Π΅ΠΌΠΏΠ΅Ρ€Π°Ρ‚ΡƒΡ€Π½Ρ‹Π΅ области. На этой ΠΈΠ»Π»ΡŽΡΡ‚Ρ€Π°Ρ†ΠΈΠΈ частица Π² области Π±ΠΎΠ»Π΅Π΅ Π½ΠΈΠ·ΠΊΠΈΡ… Ρ‚Π΅ΠΌΠΏΠ΅Ρ€Π°Ρ‚ΡƒΡ€ (правая сторона) ΠΈΠΌΠ΅Π΅Ρ‚ Π½ΠΈΠ·ΠΊΡƒΡŽ ΠΊΠΈΠ½Π΅Ρ‚ΠΈΡ‡Π΅ΡΠΊΡƒΡŽ ΡΠ½Π΅Ρ€Π³ΠΈΡŽ ΠΏΠ΅Ρ€Π΅Π΄ столкновСниСм, Π½ΠΎ Π΅Π΅ кинСтичСская энСргия увСличиваСтся послС столкновСния с ΠΊΠΎΠ½Ρ‚Π°ΠΊΡ‚Π½ΠΎΠΉ ΠΏΠΎΠ²Π΅Ρ€Ρ…Π½ΠΎΡΡ‚ΡŒΡŽ.Напротив, частица Π² области Π±ΠΎΠ»Π΅Π΅ высоких Ρ‚Π΅ΠΌΠΏΠ΅Ρ€Π°Ρ‚ΡƒΡ€ (слСва) ΠΈΠΌΠ΅Π΅Ρ‚ Π±ΠΎΠ»ΡŒΡˆΡƒΡŽ ΠΊΠΈΠ½Π΅Ρ‚ΠΈΡ‡Π΅ΡΠΊΡƒΡŽ ΡΠ½Π΅Ρ€Π³ΠΈΡŽ Π΄ΠΎ столкновСния, Π½ΠΎ Π΅Π΅ энСргия ΡƒΠΌΠ΅Π½ΡŒΡˆΠ°Π΅Ρ‚ΡΡ послС столкновСния с ΠΊΠΎΠ½Ρ‚Π°ΠΊΡ‚Π½ΠΎΠΉ ΠΏΠΎΠ²Π΅Ρ€Ρ…Π½ΠΎΡΡ‚ΡŒΡŽ.

ΠšΠΎΠ½Π²Π΅ΠΊΡ†ΠΈΡ — это ΠΏΠ΅Ρ€Π΅Π΄Π°Ρ‡Π° Ρ‚Π΅ΠΏΠ»Π° Π΄Π²ΠΈΠΆΠ΅Π½ΠΈΠ΅ΠΌ Тидкости. Π­Ρ‚ΠΎΡ‚ Ρ‚ΠΈΠΏ Ρ‚Π΅ΠΏΠ»ΠΎΠΏΠ΅Ρ€Π΅Π΄Π°Ρ‡ΠΈ происходит, Π½Π°ΠΏΡ€ΠΈΠΌΠ΅Ρ€, Π² ΠΊΠΎΡ‚Π»Π΅, кипящСм Π½Π° ΠΏΠ»ΠΈΡ‚Π΅, ΠΈΠ»ΠΈ Π²ΠΎ врСмя Π³Ρ€ΠΎΠ·Ρ‹, ΠΊΠΎΠ³Π΄Π° горячий Π²ΠΎΠ·Π΄ΡƒΡ… поднимаСтся ΠΊ основанию ΠΎΠ±Π»Π°ΠΊΠΎΠ².

Π‘ΠΎΠ²Π΅Ρ‚Ρ‹ для успСха

Π’ ΠΎΠ±ΠΈΡ…ΠΎΠ΄Π΅ Ρ‚Π΅Ρ€ΠΌΠΈΠ½ ΠΆΠΈΠ΄ΠΊΠΎΡΡ‚ΡŒ ΠΎΠ±Ρ‹Ρ‡Π½ΠΎ ΠΎΠ·Π½Π°Ρ‡Π°Π΅Ρ‚ ΠΆΠΈΠ΄ΠΊΠΎΡΡ‚ΡŒ.НапримСр, ΠΊΠΎΠ³Π΄Π° Π²Ρ‹ Π·Π°Π±ΠΎΠ»Π΅Π»ΠΈ ΠΈ Π²Ρ€Π°Ρ‡ Π³ΠΎΠ²ΠΎΡ€ΠΈΡ‚ Π²Π°ΠΌ Β«Π²Ρ‹ΠΏΠΈΡ‚ΡŒ Тидкости», это ΠΎΠ·Π½Π°Ρ‡Π°Π΅Ρ‚ Ρ‚ΠΎΠ»ΡŒΠΊΠΎ ΠΏΠΈΡ‚ΡŒ большС Π½Π°ΠΏΠΈΡ‚ΠΊΠΎΠ², Π° Π½Π΅ Π²Π΄Ρ‹Ρ…Π°Ρ‚ΡŒ большС Π²ΠΎΠ·Π΄ΡƒΡ…Π°. Однако Π² Ρ„ΠΈΠ·ΠΈΠΊΠ΅ ΠΆΠΈΠ΄ΠΊΠΎΡΡ‚ΡŒ ΠΎΠ·Π½Π°Ρ‡Π°Π΅Ρ‚ ΠΆΠΈΠ΄ΠΊΠΎΡΡ‚ΡŒ ΠΈΠ»ΠΈ Π³Π°Π· . Жидкости двиТутся ΠΈΠ½Π°Ρ‡Π΅, Ρ‡Π΅ΠΌ Ρ‚Π²Π΅Ρ€Π΄Ρ‹Π΅ Ρ‚Π΅Π»Π°, ΠΈ Π΄Π°ΠΆΠ΅ ΠΈΠΌΠ΅ΡŽΡ‚ свой собствСнный Ρ€Π°Π·Π΄Π΅Π» Ρ„ΠΈΠ·ΠΈΠΊΠΈ, извСстный ΠΊΠ°ΠΊ Π³ΠΈΠ΄Ρ€ΠΎΠ΄ΠΈΠ½Π°ΠΌΠΈΠΊΠ° , ΠΊΠΎΡ‚ΠΎΡ€Ρ‹ΠΉ ΠΈΠ·ΡƒΡ‡Π°Π΅Ρ‚ ΠΈΡ… Π΄Π²ΠΈΠΆΠ΅Π½ΠΈΠ΅.

ΠŸΡ€ΠΈ ΠΏΠΎΠ²Ρ‹ΡˆΠ΅Π½ΠΈΠΈ Ρ‚Π΅ΠΌΠΏΠ΅Ρ€Π°Ρ‚ΡƒΡ€Ρ‹ Тидкости ΠΎΠ½ΠΈ Ρ€Π°ΡΡˆΠΈΡ€ΡΡŽΡ‚ΡΡ ΠΈ становятся ΠΌΠ΅Π½Π΅Π΅ ΠΏΠ»ΠΎΡ‚Π½Ρ‹ΠΌΠΈ. НапримСр, Π½Π° рис. 11.4 ΠΌΠΎΠΆΠ΅Ρ‚ Π±Ρ‹Ρ‚ΡŒ ΠΈΠ·ΠΎΠ±Ρ€Π°ΠΆΠ΅Π½Π° стСнка Π²ΠΎΠ·Π΄ΡƒΡˆΠ½ΠΎΠ³ΠΎ ΡˆΠ°Ρ€Π° с Π³Π°Π·Π°ΠΌΠΈ Π²Π½ΡƒΡ‚Ρ€ΠΈ Π²ΠΎΠ·Π΄ΡƒΡˆΠ½ΠΎΠ³ΠΎ ΡˆΠ°Ρ€Π° с Π΄Ρ€ΡƒΠ³ΠΎΠΉ Ρ‚Π΅ΠΌΠΏΠ΅Ρ€Π°Ρ‚ΡƒΡ€ΠΎΠΉ, Ρ‡Π΅ΠΌ снаруТи Π² ΠΎΠΊΡ€ΡƒΠΆΠ°ΡŽΡ‰Π΅ΠΉ срСдС.Π‘ΠΎΠ»Π΅Π΅ горячиС ΠΈ, ΡΠ»Π΅Π΄ΠΎΠ²Π°Ρ‚Π΅Π»ΡŒΠ½ΠΎ, быстро двиТущиСся частицы Π³Π°Π·Π° Π²Π½ΡƒΡ‚Ρ€ΠΈ Π²ΠΎΠ·Π΄ΡƒΡˆΠ½ΠΎΠ³ΠΎ ΡˆΠ°Ρ€Π° ΡƒΠ΄Π°Ρ€ΡΡŽΡ‚ΡΡ ΠΎ ΠΏΠΎΠ²Π΅Ρ€Ρ…Π½ΠΎΡΡ‚ΡŒ с большСй силой, Ρ‡Π΅ΠΌ Π±ΠΎΠ»Π΅Π΅ Ρ…ΠΎΠ»ΠΎΠ΄Π½Ρ‹ΠΉ Π²ΠΎΠ·Π΄ΡƒΡ… снаруТи, вызывая Ρ€Π°ΡΡˆΠΈΡ€Π΅Π½ΠΈΠ΅ Π²ΠΎΠ·Π΄ΡƒΡˆΠ½ΠΎΠ³ΠΎ ΡˆΠ°Ρ€Π°. Π­Ρ‚ΠΎ ΡƒΠΌΠ΅Π½ΡŒΡˆΠ΅Π½ΠΈΠ΅ плотности ΠΏΠΎ ΠΎΡ‚Π½ΠΎΡˆΠ΅Π½ΠΈΡŽ ΠΊ ΠΎΠΊΡ€ΡƒΠΆΠ°ΡŽΡ‰Π΅ΠΉ срСдС создаСт ΠΏΠ»Π°Π²ΡƒΡ‡Π΅ΡΡ‚ΡŒ (Ρ‚Π΅Π½Π΄Π΅Π½Ρ†ΠΈΡŽ ΠΊ ΠΏΠΎΠ²Ρ‹ΡˆΠ΅Π½ΠΈΡŽ). ΠšΠΎΠ½Π²Π΅ΠΊΡ†ΠΈΡ обусловлСна β€‹β€‹ΠΏΠ»Π°Π²ΡƒΡ‡Π΅ΡΡ‚ΡŒΡŽ — горячий Π²ΠΎΠ·Π΄ΡƒΡ… поднимаСтся Π²Π²Π΅Ρ€Ρ…, ΠΏΠΎΡ‚ΠΎΠΌΡƒ Ρ‡Ρ‚ΠΎ ΠΎΠ½ ΠΌΠ΅Π½Π΅Π΅ ΠΏΠ»ΠΎΡ‚Π΅Π½, Ρ‡Π΅ΠΌ ΠΎΠΊΡ€ΡƒΠΆΠ°ΡŽΡ‰ΠΈΠΉ Π²ΠΎΠ·Π΄ΡƒΡ….

Иногда ΠΌΡ‹ ΠΊΠΎΠ½Ρ‚Ρ€ΠΎΠ»ΠΈΡ€ΡƒΠ΅ΠΌ Ρ‚Π΅ΠΌΠΏΠ΅Ρ€Π°Ρ‚ΡƒΡ€Ρƒ своСго Π΄ΠΎΠΌΠ° ΠΈΠ»ΠΈ самих сСбя, контролируя Π΄Π²ΠΈΠΆΠ΅Π½ΠΈΠ΅ Π²ΠΎΠ·Π΄ΡƒΡ…Π°. ГСрмСтизация Π΄Π²Π΅Ρ€Π΅ΠΉ Π³Π΅Ρ€ΠΌΠ΅Ρ‚ΠΈΡ‡Π½ΠΎΠΉ изоляциСй Π·Π°Ρ‰ΠΈΡ‰Π°Π΅Ρ‚ ΠΎΡ‚ Ρ…ΠΎΠ»ΠΎΠ΄Π½ΠΎΠ³ΠΎ Π²Π΅Ρ‚Ρ€Π° Π·ΠΈΠΌΠΎΠΉ.Π”ΠΎΠΌ Π½Π° рис. 11.5 ΠΈ Π³ΠΎΡ€ΡˆΠΎΠΊ с Π²ΠΎΠ΄ΠΎΠΉ Π½Π° ΠΏΠ»ΠΈΡ‚Π΅ Π½Π° рис. 11.6 ΡΠ²Π»ΡΡŽΡ‚ΡΡ ΠΏΡ€ΠΈΠΌΠ΅Ρ€Π°ΠΌΠΈ ΠΊΠΎΠ½Π²Π΅ΠΊΡ†ΠΈΠΈ ΠΈ плавучСсти, созданными Ρ‡Π΅Π»ΠΎΠ²Π΅ΠΊΠΎΠΌ. ОкСанскиС тСчСния ΠΈ ΠΊΡ€ΡƒΠΏΠ½ΠΎΠΌΠ°ΡΡˆΡ‚Π°Π±Π½Π°Ρ атмосфСрная циркуляция пСрСносят ΡΠ½Π΅Ρ€Π³ΠΈΡŽ ΠΈΠ· ΠΎΠ΄Π½ΠΎΠΉ части Π·Π΅ΠΌΠ½ΠΎΠ³ΠΎ ΡˆΠ°Ρ€Π° Π² Π΄Ρ€ΡƒΠ³ΡƒΡŽ ΠΈ ΡΠ²Π»ΡΡŽΡ‚ΡΡ ΠΏΡ€ΠΈΠΌΠ΅Ρ€Π°ΠΌΠΈ СстСствСнной ΠΊΠΎΠ½Π²Π΅ΠΊΡ†ΠΈΠΈ.

Рис. 11.5 Π’ΠΎΠ·Π΄ΡƒΡ…, Π½Π°Π³Ρ€Π΅Ρ‚Ρ‹ΠΉ Ρ‚Π°ΠΊ Π½Π°Π·Ρ‹Π²Π°Π΅ΠΌΠΎΠΉ Π³Ρ€Π°Π²ΠΈΡ‚Π°Ρ†ΠΈΠΎΠ½Π½ΠΎΠΉ ΠΏΠ΅Ρ‡ΡŒΡŽ, Ρ€Π°ΡΡˆΠΈΡ€ΡΠ΅Ρ‚ΡΡ ΠΈ поднимаСтся, образуя ΠΊΠΎΠ½Π²Π΅ΠΊΡ‚ΠΈΠ²Π½ΡƒΡŽ ΠΏΠ΅Ρ‚Π»ΡŽ, которая ΠΏΠ΅Ρ€Π΅Π΄Π°Π΅Ρ‚ ΡΠ½Π΅Ρ€Π³ΠΈΡŽ Π΄Ρ€ΡƒΠ³ΠΈΠΌ частям ΠΊΠΎΠΌΠ½Π°Ρ‚Ρ‹. По ΠΌΠ΅Ρ€Π΅ Ρ‚ΠΎΠ³ΠΎ, ΠΊΠ°ΠΊ Π²ΠΎΠ·Π΄ΡƒΡ… охлаТдаСтся Ρƒ ΠΏΠΎΡ‚ΠΎΠ»ΠΊΠ° ΠΈ Π²Π½Π΅ΡˆΠ½ΠΈΡ… стСн, ΠΎΠ½ сТимаСтся, Π² ΠΊΠΎΠ½Π΅Ρ‡Π½ΠΎΠΌ ΠΈΡ‚ΠΎΠ³Π΅ ΡΡ‚Π°Π½ΠΎΠ²ΡΡΡŒ Π±ΠΎΠ»Π΅Π΅ ΠΏΠ»ΠΎΡ‚Π½Ρ‹ΠΌ, Ρ‡Π΅ΠΌ Π²ΠΎΠ·Π΄ΡƒΡ… Π² ΠΏΠΎΠΌΠ΅Ρ‰Π΅Π½ΠΈΠΈ, ΠΈ опускаСтся Π½Π° ΠΏΠΎΠ».ΠŸΡ€Π°Π²ΠΈΠ»ΡŒΠ½ΠΎ спроСктированная систСма отоплСния, подобная этой, Π² ΠΊΠΎΡ‚ΠΎΡ€ΠΎΠΉ ΠΈΡΠΏΠΎΠ»ΡŒΠ·ΡƒΠ΅Ρ‚ΡΡ СстСствСнная конвСкция, ΠΌΠΎΠΆΠ΅Ρ‚ Π±Ρ‹Ρ‚ΡŒ достаточно эффСктивной для Ρ€Π°Π²Π½ΠΎΠΌΠ΅Ρ€Π½ΠΎΠ³ΠΎ ΠΎΠ±ΠΎΠ³Ρ€Π΅Π²Π° Π΄ΠΎΠΌΠ°.

Рис. 11.6 ΠšΠΎΠ½Π²Π΅ΠΊΡ†ΠΈΡ ΠΈΠ³Ρ€Π°Π΅Ρ‚ Π²Π°ΠΆΠ½ΡƒΡŽ Ρ€ΠΎΠ»ΡŒ Π² Ρ‚Π΅ΠΏΠ»ΠΎΠΏΠ΅Ρ€Π΅Π΄Π°Ρ‡Π΅ Π²Π½ΡƒΡ‚Ρ€ΠΈ этого ΠΊΠΎΡ‚Π»Π° с Π²ΠΎΠ΄ΠΎΠΉ. Попав Π²Π½ΡƒΡ‚Ρ€ΡŒ Тидкости, Ρ‚Π΅ΠΏΠ»ΠΎΠΏΠ΅Ρ€Π΅Π΄Π°Ρ‡Π° ΠΊ Π΄Ρ€ΡƒΠ³ΠΈΠΌ частям ΠΊΠ°ΡΡ‚Ρ€ΡŽΠ»ΠΈ происходит Π² основном Π·Π° счСт ΠΊΠΎΠ½Π²Π΅ΠΊΡ†ΠΈΠΈ. Π‘ΠΎΠ»Π΅Π΅ горячая Π²ΠΎΠ΄Π° Ρ€Π°ΡΡˆΠΈΡ€ΡΠ΅Ρ‚ΡΡ, ΡƒΠΌΠ΅Π½ΡŒΡˆΠ°Π΅Ρ‚ΡΡ ΠΏΠΎ плотности ΠΈ поднимаСтся, пСрСдавая Ρ‚Π΅ΠΏΠ»ΠΎ Π΄Ρ€ΡƒΠ³ΠΈΠΌ областям Π²ΠΎΠ΄Ρ‹, Π² Ρ‚ΠΎ врСмя ΠΊΠ°ΠΊ Π±ΠΎΠ»Π΅Π΅ холодная Π²ΠΎΠ΄Π° опускаСтся Π½Π° Π΄Π½ΠΎ. Π­Ρ‚ΠΎΡ‚ процСсс повторяСтся, ΠΏΠΎΠΊΠ° Π² ΠΊΠ°ΡΡ‚Ρ€ΡŽΠ»Π΅ Π΅ΡΡ‚ΡŒ Π²ΠΎΠ΄Π°.

Π˜Π·Π»ΡƒΡ‡Π΅Π½ΠΈΠ΅ — это Ρ„ΠΎΡ€ΠΌΠ° ΠΏΠ΅Ρ€Π΅Π΄Π°Ρ‡ΠΈ Ρ‚Π΅ΠΏΠ»Π°, которая происходит ΠΏΡ€ΠΈ испускании ΠΈΠ»ΠΈ ΠΏΠΎΠ³Π»ΠΎΡ‰Π΅Π½ΠΈΠΈ элСктромагнитного излучСния. Π­Π»Π΅ΠΊΡ‚Ρ€ΠΎΠΌΠ°Π³Π½ΠΈΡ‚Π½ΠΎΠ΅ ΠΈΠ·Π»ΡƒΡ‡Π΅Π½ΠΈΠ΅ Π²ΠΊΠ»ΡŽΡ‡Π°Π΅Ρ‚ Ρ€Π°Π΄ΠΈΠΎΠ²ΠΎΠ»Π½Ρ‹, ΠΌΠΈΠΊΡ€ΠΎΠ²ΠΎΠ»Π½Ρ‹, инфракрасноС ΠΈΠ·Π»ΡƒΡ‡Π΅Π½ΠΈΠ΅, Π²ΠΈΠ΄ΠΈΠΌΡ‹ΠΉ свСт, ΡƒΠ»ΡŒΡ‚Ρ€Π°Ρ„ΠΈΠΎΠ»Π΅Ρ‚ΠΎΠ²ΠΎΠ΅ ΠΈΠ·Π»ΡƒΡ‡Π΅Π½ΠΈΠ΅, рСнтгСновскиС Π»ΡƒΡ‡ΠΈ ΠΈ Π³Π°ΠΌΠΌΠ°-Π»ΡƒΡ‡ΠΈ, всС ΠΈΠ· ΠΊΠΎΡ‚ΠΎΡ€Ρ‹Ρ… ΠΈΠΌΠ΅ΡŽΡ‚ Ρ€Π°Π·Π½Ρ‹Π΅ Π΄Π»ΠΈΠ½Ρ‹ Π²ΠΎΠ»Π½ ΠΈ количСство энСргии (Π±ΠΎΠ»Π΅Π΅ ΠΊΠΎΡ€ΠΎΡ‚ΠΊΠΈΠ΅ Π΄Π»ΠΈΠ½Ρ‹ Π²ΠΎΠ»Π½ ΠΈΠΌΠ΅ΡŽΡ‚ Π±ΠΎΠ»Π΅Π΅ Π²Ρ‹ΡΠΎΠΊΡƒΡŽ частоту ΠΈ Π±ΠΎΠ»ΡŒΡˆΡƒΡŽ ΡΠ½Π΅Ρ€Π³ΠΈΡŽ).

Π’Ρ‹ ΠΌΠΎΠΆΠ΅Ρ‚Π΅ ΠΏΠΎΡ‡ΡƒΠ²ΡΡ‚Π²ΠΎΠ²Π°Ρ‚ΡŒ Ρ‚Π΅ΠΏΠ»ΠΎΠΎΡ‚Π΄Π°Ρ‡Ρƒ ΠΎΡ‚ огня ΠΈ солнца. Π’ΠΎΡ‡Π½ΠΎ Ρ‚Π°ΠΊ ΠΆΠ΅ Π²Ρ‹ ΠΈΠ½ΠΎΠ³Π΄Π° ΠΌΠΎΠΆΠ΅Ρ‚Π΅ ΡΠΊΠ°Π·Π°Ρ‚ΡŒ, Ρ‡Ρ‚ΠΎ Π΄ΡƒΡ…ΠΎΠ²ΠΊΠ° горячая, Π½Π΅ касаясь Π΅Π΅ Π΄Π²Π΅Ρ€Ρ†Ρ‹ ΠΈ Π½Π΅ заглядывая Π²Π½ΡƒΡ‚Ρ€ΡŒ — ΠΎΠ½Π° ​​моТСт просто ΡΠΎΠ³Ρ€Π΅Ρ‚ΡŒ вас, ΠΊΠΎΠ³Π΄Π° Π²Ρ‹ ΠΏΡ€ΠΎΠΉΠ΄Π΅Ρ‚Π΅ ΠΌΠΈΠΌΠΎ.Π”Ρ€ΡƒΠ³ΠΎΠΉ ΠΏΡ€ΠΈΠΌΠ΅Ρ€ — Ρ‚Π΅ΠΏΠ»ΠΎΠ²ΠΎΠ΅ ΠΈΠ·Π»ΡƒΡ‡Π΅Π½ΠΈΠ΅ чСловСчСского Ρ‚Π΅Π»Π°; люди постоянно ΠΈΠ·Π»ΡƒΡ‡Π°ΡŽΡ‚ инфракрасноС ΠΈΠ·Π»ΡƒΡ‡Π΅Π½ΠΈΠ΅, ΠΊΠΎΡ‚ΠΎΡ€ΠΎΠ΅ Π½Π΅ Π²ΠΈΠ΄Π½ΠΎ чСловСчСскому Π³Π»Π°Π·Ρƒ, Π½ΠΎ ощущаСтся ΠΊΠ°ΠΊ Ρ‚Π΅ΠΏΠ»ΠΎ.

Π˜Π·Π»ΡƒΡ‡Π΅Π½ΠΈΠ΅ — СдинствСнный ΠΌΠ΅Ρ‚ΠΎΠ΄ ΠΏΠ΅Ρ€Π΅Π΄Π°Ρ‡ΠΈ Ρ‚Π΅ΠΏΠ»Π°, ΠΏΡ€ΠΈ ΠΊΠΎΡ‚ΠΎΡ€ΠΎΠΌ срСда Π½Π΅ трСбуСтся, Π° это ΠΎΠ·Π½Π°Ρ‡Π°Π΅Ρ‚, Ρ‡Ρ‚ΠΎ Ρ‚Π΅ΠΏΠ»ΠΎ Π½Π΅ Π΄ΠΎΠ»ΠΆΠ½ΠΎ Π²ΡΡ‚ΡƒΠΏΠ°Ρ‚ΡŒ Π² прямой ΠΊΠΎΠ½Ρ‚Π°ΠΊΡ‚ с ΠΊΠ°ΠΊΠΈΠΌΠΈ-Π»ΠΈΠ±ΠΎ ΠΏΡ€Π΅Π΄ΠΌΠ΅Ρ‚Π°ΠΌΠΈ ΠΈΠ»ΠΈ ΠΏΠ΅Ρ€Π΅Π½ΠΎΡΠΈΡ‚ΡŒΡΡ ΠΈΠΌΠΈ. ΠŸΡ€ΠΎΡΡ‚Ρ€Π°Π½ΡΡ‚Π²ΠΎ ΠΌΠ΅ΠΆΠ΄Ρƒ Π—Π΅ΠΌΠ»Π΅ΠΉ ΠΈ Π‘ΠΎΠ»Π½Ρ†Π΅ΠΌ Π² основном пусто, Π±Π΅Π· ΠΊΠ°ΠΊΠΎΠΉ-Π»ΠΈΠ±ΠΎ возмоТности Ρ‚Π΅ΠΏΠ»ΠΎΠΏΠ΅Ρ€Π΅Π΄Π°Ρ‡ΠΈ Π·Π° счСт ΠΊΠΎΠ½Π²Π΅ΠΊΡ†ΠΈΠΈ ΠΈΠ»ΠΈ тСплопроводности. ВмСсто этого Ρ‚Π΅ΠΏΠ»ΠΎ пСрСдаСтся Π·Π° счСт излучСния, ΠΈ ЗСмля нагрСваСтся, ΠΏΠΎΡΠΊΠΎΠ»ΡŒΠΊΡƒ ΠΎΠ½Π° ΠΏΠΎΠ³Π»ΠΎΡ‰Π°Π΅Ρ‚ элСктромагнитноС ΠΈΠ·Π»ΡƒΡ‡Π΅Π½ΠΈΠ΅, испускаСмоС Π‘ΠΎΠ»Π½Ρ†Π΅ΠΌ.

Рис. 11.7 Π‘ΠΎΠ»ΡŒΡˆΠ°Ρ Ρ‡Π°ΡΡ‚ΡŒ Ρ‚Π΅ΠΏΠ»Π° ΠΎΡ‚ этого ΠΏΠΎΠΆΠ°Ρ€Π° пСрСдаСтся Π½Π°Π±Π»ΡŽΠ΄Π°Ρ‚Π΅Π»ΡΠΌ Ρ‡Π΅Ρ€Π΅Π· инфракрасноС ΠΈΠ·Π»ΡƒΡ‡Π΅Π½ΠΈΠ΅. Π’ΠΈΠ΄ΠΈΠΌΡ‹ΠΉ свСт ΠΏΠ΅Ρ€Π΅Π΄Π°Π΅Ρ‚ ΠΎΡ‚Π½ΠΎΡΠΈΡ‚Π΅Π»ΡŒΠ½ΠΎ Π½Π΅Π±ΠΎΠ»ΡŒΡˆΡƒΡŽ Ρ‚Π΅ΠΏΠ»ΠΎΠ²ΡƒΡŽ ΡΠ½Π΅Ρ€Π³ΠΈΡŽ. ΠŸΠΎΡΠΊΠΎΠ»ΡŒΠΊΡƒ ΠΊΠΎΠΆΠ° ΠΎΡ‡Π΅Π½ΡŒ Ρ‡ΡƒΠ²ΡΡ‚Π²ΠΈΡ‚Π΅Π»ΡŒΠ½Π° ΠΊ инфракрасному ΠΈΠ·Π»ΡƒΡ‡Π΅Π½ΠΈΡŽ, Π²Ρ‹ ΠΌΠΎΠΆΠ΅Ρ‚Π΅ ΠΏΠΎΡ‡ΡƒΠ²ΡΡ‚Π²ΠΎΠ²Π°Ρ‚ΡŒ присутствиС огня, Π΄Π°ΠΆΠ΅ Π½Π΅ глядя Π½Π° Π½Π΅Π³ΠΎ. (ДэниСл X. Πžβ€™ΠΠΈΠ»)

ВсС ΠΎΠ±ΡŠΠ΅ΠΊΡ‚Ρ‹ ΠΏΠΎΠ³Π»ΠΎΡ‰Π°ΡŽΡ‚ ΠΈ ΠΈΠ·Π»ΡƒΡ‡Π°ΡŽΡ‚ элСктромагнитноС ΠΈΠ·Π»ΡƒΡ‡Π΅Π½ΠΈΠ΅ (см. Рисунок 11.7). Π‘ΠΊΠΎΡ€ΠΎΡΡ‚ΡŒ ΠΏΠ΅Ρ€Π΅Π΄Π°Ρ‡ΠΈ Ρ‚Π΅ΠΏΠ»Π° ΠΈΠ·Π»ΡƒΡ‡Π΅Π½ΠΈΠ΅ΠΌ Π² основном зависит ΠΎΡ‚ Ρ†Π²Π΅Ρ‚Π° ΠΎΠ±ΡŠΠ΅ΠΊΡ‚Π°. Π§Π΅Ρ€Π½Ρ‹ΠΉ — Π½Π°ΠΈΠ±ΠΎΠ»Π΅Π΅ эффСктивный ΠΏΠΎΠ³Π»ΠΎΡ‚ΠΈΡ‚Π΅Π»ΡŒ ΠΈ Ρ€Π°Π΄ΠΈΠ°Ρ‚ΠΎΡ€, Π° Π±Π΅Π»Ρ‹ΠΉ — Π½Π°ΠΈΠΌΠ΅Π½Π΅Π΅ эффСктивный.НапримСр, люди, ΠΆΠΈΠ²ΡƒΡ‰ΠΈΠ΅ Π² ΠΆΠ°Ρ€ΠΊΠΎΠΌ ΠΊΠ»ΠΈΠΌΠ°Ρ‚Π΅, ΠΎΠ±Ρ‹Ρ‡Π½ΠΎ ΠΈΠ·Π±Π΅Π³Π°ΡŽΡ‚ ношСния Ρ‡Π΅Ρ€Π½ΠΎΠΉ ΠΎΠ΄Π΅ΠΆΠ΄Ρ‹. Π’ΠΎΡ‡Π½ΠΎ Ρ‚Π°ΠΊ ΠΆΠ΅ Ρ‡Π΅Ρ€Π½Ρ‹ΠΉ Π°ΡΡ„Π°Π»ΡŒΡ‚ Π½Π° стоянкС Π±ΡƒΠ΄Π΅Ρ‚ горячСС, Ρ‡Π΅ΠΌ ΠΏΡ€ΠΈΠ»Π΅Π³Π°ΡŽΡ‰ΠΈΠ΅ участки Ρ‚Ρ€Π°Π²Ρ‹ Π² Π»Π΅Ρ‚Π½ΠΈΠΉ дСнь, ΠΏΠΎΡ‚ΠΎΠΌΡƒ Ρ‡Ρ‚ΠΎ Ρ‡Π΅Ρ€Π½Ρ‹ΠΉ ΠΏΠΎΠ³Π»ΠΎΡ‰Π°Π΅Ρ‚ Π»ΡƒΡ‡ΡˆΠ΅, Ρ‡Π΅ΠΌ Π·Π΅Π»Π΅Π½Ρ‹ΠΉ. Π’Π΅Ρ€Π½ΠΎ ΠΈ ΠΎΠ±Ρ€Π°Ρ‚Π½ΠΎΠ΅ — Ρ‡Π΅Ρ€Π½Ρ‹ΠΉ Ρ†Π²Π΅Ρ‚ ΠΈΠ·Π»ΡƒΡ‡Π°Π΅Ρ‚ Π»ΡƒΡ‡ΡˆΠ΅, Ρ‡Π΅ΠΌ Π·Π΅Π»Π΅Π½Ρ‹ΠΉ. Ясной Π»Π΅Ρ‚Π½Π΅ΠΉ Π½ΠΎΡ‡ΡŒΡŽ Ρ‡Π΅Ρ€Π½Ρ‹ΠΉ Π°ΡΡ„Π°Π»ΡŒΡ‚ Π±ΡƒΠ΄Π΅Ρ‚ Ρ…ΠΎΠ»ΠΎΠ΄Π½Π΅Π΅, Ρ‡Π΅ΠΌ Π·Π΅Π»Π΅Π½Ρ‹ΠΉ участок Ρ‚Ρ€Π°Π²Ρ‹, ΠΏΠΎΡ‚ΠΎΠΌΡƒ Ρ‡Ρ‚ΠΎ Ρ‡Π΅Ρ€Π½Ρ‹ΠΉ ΠΈΠ·Π»ΡƒΡ‡Π°Π΅Ρ‚ ΡΠ½Π΅Ρ€Π³ΠΈΡŽ быстрСС, Ρ‡Π΅ΠΌ Π·Π΅Π»Π΅Π½Ρ‹ΠΉ. Напротив, Π±Π΅Π»Ρ‹ΠΉ Ρ†Π²Π΅Ρ‚ — ΠΏΠ»ΠΎΡ…ΠΎΠΉ ΠΏΠΎΠ³Π»ΠΎΡ‚ΠΈΡ‚Π΅Π»ΡŒ ΠΈ ΠΏΠ»ΠΎΡ…ΠΎΠΉ Ρ€Π°Π΄ΠΈΠ°Ρ‚ΠΎΡ€. Π‘Π΅Π»Ρ‹ΠΉ ΠΎΠ±ΡŠΠ΅ΠΊΡ‚, ΠΊΠ°ΠΊ Π·Π΅Ρ€ΠΊΠ°Π»ΠΎ, ΠΎΡ‚Ρ€Π°ΠΆΠ°Π΅Ρ‚ ΠΏΠΎΡ‡Ρ‚ΠΈ всС ΠΈΠ·Π»ΡƒΡ‡Π΅Π½ΠΈΠ΅.

Π’ΠΈΡ€Ρ‚ΡƒΠ°Π»ΡŒΠ½Π°Ρ Ρ„ΠΈΠ·ΠΈΠΊΠ°

Π€ΠΎΡ€ΠΌΡ‹ ΠΈ измСнСния энСргии

Π’ этой Π°Π½ΠΈΠΌΠ°Ρ†ΠΈΠΈ Π²Ρ‹ исслСдуСтС Ρ‚Π΅ΠΏΠ»ΠΎΠΏΠ΅Ρ€Π΅Π΄Π°Ρ‡Ρƒ с Ρ€Π°Π·Π»ΠΈΡ‡Π½Ρ‹ΠΌΠΈ ΠΌΠ°Ρ‚Π΅Ρ€ΠΈΠ°Π»Π°ΠΌΠΈ. ΠŸΠΎΡΠΊΡΠΏΠ΅Ρ€ΠΈΠΌΠ΅Π½Ρ‚ΠΈΡ€ΡƒΠΉΡ‚Π΅ с Π½Π°Π³Ρ€Π΅Π²ΠΎΠΌ ΠΈ ΠΎΡ…Π»Π°ΠΆΠ΄Π΅Π½ΠΈΠ΅ΠΌ ΠΆΠ΅Π»Π΅Π·Π°, ΠΊΠΈΡ€ΠΏΠΈΡ‡Π° ΠΈ Π²ΠΎΠ΄Ρ‹. Для этого ΠΏΠ΅Ρ€Π΅Ρ‚Π°Ρ‰ΠΈΡ‚Π΅ ΠΎΠ±ΡŠΠ΅ΠΊΡ‚ Π½Π° ΠΏΡŒΠ΅Π΄Π΅ΡΡ‚Π°Π» ΠΈ Π·Π°Ρ‚Π΅ΠΌ ΡƒΠ΄Π΅Ρ€ΠΆΠΈΠ²Π°ΠΉΡ‚Π΅ Ρ€Ρ‹Ρ‡Π°Π³ Π² ΠΏΠΎΠ»ΠΎΠΆΠ΅Π½ΠΈΠΈ Β«ΠΠ°Π³Ρ€Π΅Ρ‚ΡŒΒ» ΠΈΠ»ΠΈ Β«ΠžΡ…Π»Π°ΠΆΠ΄Π°Ρ‚ΡŒΒ». ΠŸΠ΅Ρ€Π΅Ρ‚Π°Ρ‰ΠΈΡ‚Π΅ Ρ‚Π΅Ρ€ΠΌΠΎΠΌΠ΅Ρ‚Ρ€ рядом с ΠΊΠ°ΠΆΠ΄Ρ‹ΠΌ ΠΎΠ±ΡŠΠ΅ΠΊΡ‚ΠΎΠΌ, Ρ‡Ρ‚ΠΎΠ±Ρ‹ ΠΈΠ·ΠΌΠ΅Ρ€ΠΈΡ‚ΡŒ Π΅Π³ΠΎ Ρ‚Π΅ΠΌΠΏΠ΅Ρ€Π°Ρ‚ΡƒΡ€Ρƒ — Π²Ρ‹ ΠΌΠΎΠΆΠ΅Ρ‚Π΅ Π² Ρ€Π΅Π°Π»ΡŒΠ½ΠΎΠΌ Π²Ρ€Π΅ΠΌΠ΅Π½ΠΈ Π½Π°Π±Π»ΡŽΠ΄Π°Ρ‚ΡŒ Π·Π° Ρ‚Π΅ΠΌ, ΠΊΠ°ΠΊ быстро ΠΎΠ½ нагрСваСтся ΠΈΠ»ΠΈ охлаТдаСтся.

Π’Π΅ΠΏΠ΅Ρ€ΡŒ ΠΏΠΎΠΏΡ€ΠΎΠ±ΡƒΠ΅ΠΌ ΠΏΠ΅Ρ€Π΅Π΄Π°Ρ‚ΡŒ Ρ‚Π΅ΠΏΠ»ΠΎ ΠΌΠ΅ΠΆΠ΄Ρƒ ΠΎΠ±ΡŠΠ΅ΠΊΡ‚Π°ΠΌΠΈ.НагрСйтС ΠΊΠΈΡ€ΠΏΠΈΡ‡ ΠΈ помСститС Π΅Π³ΠΎ Π² ΠΏΡ€ΠΎΡ…Π»Π°Π΄Π½ΡƒΡŽ Π²ΠΎΠ΄Ρƒ. Π’Π΅ΠΏΠ΅Ρ€ΡŒ снова Π½Π°Π³Ρ€Π΅ΠΉΡ‚Π΅ ΠΊΠΈΡ€ΠΏΠΈΡ‡, Π½ΠΎ Π·Π°Ρ‚Π΅ΠΌ помСститС Π΅Π³ΠΎ ΠΏΠΎΠ²Π΅Ρ€Ρ… ΡƒΡ‚ΡŽΠ³Π°. Π§Ρ‚ΠΎ Ρ‚Ρ‹ Π·Π°ΠΌΠ΅Ρ‚ΠΈΠ»?

Π’Ρ‹Π±ΠΎΡ€ ΠΎΠΏΡ†ΠΈΠΈ быстрой ΠΏΠ΅Ρ€Π΅ΠΌΠΎΡ‚ΠΊΠΈ Π²ΠΏΠ΅Ρ€Π΅Π΄ позволяСт ΡƒΡΠΊΠΎΡ€ΠΈΡ‚ΡŒ ΠΏΠ΅Ρ€Π΅Π΄Π°Ρ‡Ρƒ Ρ‚Π΅ΠΏΠ»Π° ΠΈ ΡΡΠΊΠΎΠ½ΠΎΠΌΠΈΡ‚ΡŒ врСмя.

ΠŸΡ€ΠΎΠ²Π΅Ρ€ΠΊΠ° Π·Π°Ρ…Π²Π°Ρ‚Π°

Π‘Ρ€Π°Π²Π½ΠΈΡ‚Π΅, насколько быстро Ρ€Π°Π·Π»ΠΈΡ‡Π½Ρ‹Π΅ ΠΌΠ°Ρ‚Π΅Ρ€ΠΈΠ°Π»Ρ‹ Π½Π°Π³Ρ€Π΅Π²Π°ΡŽΡ‚ΡΡ ΠΈΠ»ΠΈ ΠΎΡ…Π»Π°ΠΆΠ΄Π°ΡŽΡ‚ΡΡ. ΠžΡΠ½ΠΎΠ²Ρ‹Π²Π°ΡΡΡŒ Π½Π° этих Ρ€Π΅Π·ΡƒΠ»ΡŒΡ‚Π°Ρ‚Π°Ρ…, ΠΊΠ°ΠΊΠΎΠΉ ΠΌΠ°Ρ‚Π΅Ρ€ΠΈΠ°Π», ΠΏΠΎ Π²Π°ΡˆΠ΅ΠΌΡƒ мнСнию, ΠΈΠΌΠ΅Π΅Ρ‚ Π½Π°ΠΈΠ±ΠΎΠ»ΡŒΡˆΡƒΡŽ ΡƒΠ΄Π΅Π»ΡŒΠ½ΡƒΡŽ Ρ‚Π΅ΠΏΠ»ΠΎΠ΅ΠΌΠΊΠΎΡΡ‚ΡŒ? ΠŸΠΎΡ‡Π΅ΠΌΡƒ? Какая ΠΈΠ· Π½ΠΈΡ… ΠΈΠΌΠ΅Π΅Ρ‚ Π½Π°ΠΈΠΌΠ΅Π½ΡŒΡˆΡƒΡŽ ΡƒΠ΄Π΅Π»ΡŒΠ½ΡƒΡŽ Ρ‚Π΅ΠΏΠ»ΠΎΠ΅ΠΌΠΊΠΎΡΡ‚ΡŒ? ΠœΠΎΠΆΠ΅Ρ‚Π΅ Π»ΠΈ Π²Ρ‹ ΠΏΡ€Π΅Π΄ΡΡ‚Π°Π²ΠΈΡ‚ΡŒ сСбС Ρ€Π΅Π°Π»ΡŒΠ½ΡƒΡŽ ΡΠΈΡ‚ΡƒΠ°Ρ†ΠΈΡŽ, Π² ΠΊΠΎΡ‚ΠΎΡ€ΠΎΠΉ Π²Ρ‹ Ρ…ΠΎΡ‚Π΅Π»ΠΈ Π±Ρ‹ ΠΈΡΠΏΠΎΠ»ΡŒΠ·ΠΎΠ²Π°Ρ‚ΡŒ ΠΎΠ±ΡŠΠ΅ΠΊΡ‚ с большой ΡƒΠ΄Π΅Π»ΡŒΠ½ΠΎΠΉ Ρ‚Π΅ΠΏΠ»ΠΎΠ΅ΠΌΠΊΠΎΡΡ‚ΡŒΡŽ?

  1. Π’ΠΎΠ΄Π° Π·Π°Π½ΠΈΠΌΠ°Π΅Ρ‚ большС всСго Π²Ρ€Π΅ΠΌΠ΅Π½ΠΈ, Π° ΠΆΠ΅Π»Π΅Π·Ρƒ Π½ΡƒΠΆΠ½ΠΎ мСньшС Π²Ρ€Π΅ΠΌΠ΅Π½ΠΈ, Ρ‡Ρ‚ΠΎΠ±Ρ‹ Π½Π°Π³Ρ€Π΅Ρ‚ΡŒΡΡ ΠΈ ΠΎΡΡ‚Ρ‹Ρ‚ΡŒ.Для тСплоизоляции ΠΆΠ΅Π»Π°Ρ‚Π΅Π»ΡŒΠ½Ρ‹ ΠΎΠ±ΡŠΠ΅ΠΊΡ‚Ρ‹ с большСй ΡƒΠ΄Π΅Π»ΡŒΠ½ΠΎΠΉ Ρ‚Π΅ΠΏΠ»ΠΎΠ΅ΠΌΠΊΠΎΡΡ‚ΡŒΡŽ. НапримСр, ΡˆΠ΅Ρ€ΡΡ‚ΡΠ½Π°Ρ ΠΎΠ΄Π΅ΠΆΠ΄Π° с большой ΡƒΠ΄Π΅Π»ΡŒΠ½ΠΎΠΉ Ρ‚Π΅ΠΏΠ»ΠΎΠ΅ΠΌΠΊΠΎΡΡ‚ΡŒΡŽ ΠΏΡ€Π΅Π΄ΠΎΡ‚Π²Ρ€Π°Ρ‚ΠΈΡ‚ ΠΏΠΎΡ‚Π΅Ρ€ΡŽ Ρ‚Π΅ΠΏΠ»Π° Ρ‚Π΅Π»ΠΎΠΌ.
  2. Π’ΠΎΠ΄Π° Π·Π°Π½ΠΈΠΌΠ°Π΅Ρ‚ мСньшС всСго Π²Ρ€Π΅ΠΌΠ΅Π½ΠΈ, Π° ΠΆΠ΅Π»Π΅Π·Ρƒ Π½ΡƒΠΆΠ½ΠΎ большС Π²Ρ€Π΅ΠΌΠ΅Π½ΠΈ, Ρ‡Ρ‚ΠΎΠ±Ρ‹ Π½Π°Π³Ρ€Π΅Ρ‚ΡŒΡΡ ΠΈ ΠΎΡΡ‚Ρ‹Ρ‚ΡŒ. Для тСплоизоляции ΠΆΠ΅Π»Π°Ρ‚Π΅Π»ΡŒΠ½Ρ‹ ΠΎΠ±ΡŠΠ΅ΠΊΡ‚Ρ‹ с большСй ΡƒΠ΄Π΅Π»ΡŒΠ½ΠΎΠΉ Ρ‚Π΅ΠΏΠ»ΠΎΠ΅ΠΌΠΊΠΎΡΡ‚ΡŒΡŽ. НапримСр, ΡˆΠ΅Ρ€ΡΡ‚ΡΠ½Π°Ρ ΠΎΠ΄Π΅ΠΆΠ΄Π° с большой ΡƒΠ΄Π΅Π»ΡŒΠ½ΠΎΠΉ Ρ‚Π΅ΠΏΠ»ΠΎΠ΅ΠΌΠΊΠΎΡΡ‚ΡŒΡŽ ΠΏΡ€Π΅Π΄ΠΎΡ‚Π²Ρ€Π°Ρ‚ΠΈΡ‚ ΠΏΠΎΡ‚Π΅Ρ€ΡŽ Ρ‚Π΅ΠΏΠ»Π° Ρ‚Π΅Π»ΠΎΠΌ.
  3. ΠšΠΈΡ€ΠΏΠΈΡ‡ Π·Π°ΠΉΠΌΠ΅Ρ‚ мСньшС всСго Π²Ρ€Π΅ΠΌΠ΅Π½ΠΈ, Π° ΠΆΠ΅Π»Π΅Π·Ρƒ Π½ΡƒΠΆΠ½ΠΎ большС Π²Ρ€Π΅ΠΌΠ΅Π½ΠΈ, Ρ‡Ρ‚ΠΎΠ±Ρ‹ Π½Π°Π³Ρ€Π΅Ρ‚ΡŒΡΡ ΠΈ ΠΎΡΡ‚Ρ‹Ρ‚ΡŒ.Для тСплоизоляции ΠΆΠ΅Π»Π°Ρ‚Π΅Π»ΡŒΠ½Ρ‹ ΠΎΠ±ΡŠΠ΅ΠΊΡ‚Ρ‹ с большСй ΡƒΠ΄Π΅Π»ΡŒΠ½ΠΎΠΉ Ρ‚Π΅ΠΏΠ»ΠΎΠ΅ΠΌΠΊΠΎΡΡ‚ΡŒΡŽ. НапримСр, ΡˆΠ΅Ρ€ΡΡ‚ΡΠ½Π°Ρ ΠΎΠ΄Π΅ΠΆΠ΄Π° с большой ΡƒΠ΄Π΅Π»ΡŒΠ½ΠΎΠΉ Ρ‚Π΅ΠΏΠ»ΠΎΠ΅ΠΌΠΊΠΎΡΡ‚ΡŒΡŽ ΠΏΡ€Π΅Π΄ΠΎΡ‚Π²Ρ€Π°Ρ‚ΠΈΡ‚ ΠΏΠΎΡ‚Π΅Ρ€ΡŽ Ρ‚Π΅ΠΏΠ»Π° Ρ‚Π΅Π»ΠΎΠΌ.
  4. Π’ΠΎΠ΄Π° Π·Π°Π½ΠΈΠΌΠ°Π΅Ρ‚ мСньшС всСго Π²Ρ€Π΅ΠΌΠ΅Π½ΠΈ, Π° ΠΊΠΈΡ€ΠΏΠΈΡ‡Ρƒ Π½ΡƒΠΆΠ½ΠΎ большС Π²Ρ€Π΅ΠΌΠ΅Π½ΠΈ, Ρ‡Ρ‚ΠΎΠ±Ρ‹ Π½Π°Π³Ρ€Π΅Ρ‚ΡŒΡΡ ΠΈ ΠΎΡΡ‚Ρ‹Ρ‚ΡŒ. Для тСплоизоляции ΠΆΠ΅Π»Π°Ρ‚Π΅Π»ΡŒΠ½Ρ‹ ΠΎΠ±ΡŠΠ΅ΠΊΡ‚Ρ‹ с большСй ΡƒΠ΄Π΅Π»ΡŒΠ½ΠΎΠΉ Ρ‚Π΅ΠΏΠ»ΠΎΠ΅ΠΌΠΊΠΎΡΡ‚ΡŒΡŽ. НапримСр, ΡˆΠ΅Ρ€ΡΡ‚ΡΠ½Π°Ρ ΠΎΠ΄Π΅ΠΆΠ΄Π° с большой ΡƒΠ΄Π΅Π»ΡŒΠ½ΠΎΠΉ Ρ‚Π΅ΠΏΠ»ΠΎΠ΅ΠΌΠΊΠΎΡΡ‚ΡŒΡŽ ΠΏΡ€Π΅Π΄ΠΎΡ‚Π²Ρ€Π°Ρ‚ΠΈΡ‚ ΠΏΠΎΡ‚Π΅Ρ€ΡŽ Ρ‚Π΅ΠΏΠ»Π° Ρ‚Π΅Π»ΠΎΠΌ.

Π’Π΅ΠΏΠ»ΠΎΠΎΠ±ΠΌΠ΅Π½

Π’Π΅ΠΏΠ»ΠΎ энСргия, Π° Ρ‚ΠΎΡ‡Π½Π΅Π΅ ΠΏΠ΅Ρ€Π΅Π΄Π°Ρ‡Π° Ρ‚Π΅ΠΏΠ»ΠΎΠ²ΠΎΠΉ энСргии.Как энСргия, Ρ‚Π΅ΠΏΠ»ΠΎ измСряСтся Π² Π²Π°Ρ‚Ρ‚Π°Ρ… (Π’Ρ‚), Π² Ρ‚ΠΎ врСмя ΠΊΠ°ΠΊ Ρ‚Π΅ΠΌΠΏΠ΅Ρ€Π°Ρ‚ΡƒΡ€Π° измСряСтся Π² градусах ЦСльсия (C) ΠΈΠ»ΠΈ КСльвин (К). Π’ слова горячиС ΠΈ Ρ…ΠΎΠ»ΠΎΠ΄Π½Ρ‹Π΅ ΠΈΠΌΠ΅ΡŽΡ‚ смысл Ρ‚ΠΎΠ»ΡŒΠΊΠΎ для родствСнников основаниС. ВСпловая энСргия пСрСдаСтся ΠΎΡ‚ горячСго ΠΌΠ°Ρ‚Π΅Ρ€ΠΈΠ°Π»Π° ΠΊ Ρ…ΠΎΠ»ΠΎΠ΄Π½ΠΎΠΌΡƒ ΠΌΠ°Ρ‚Π΅Ρ€ΠΈΠ°Π». Горячий ΠΌΠ°Ρ‚Π΅Ρ€ΠΈΠ°Π» Π½Π°Π³Ρ€Π΅Π²Π°Π΅Ρ‚ Ρ…ΠΎΠ»ΠΎΠ΄Π½Ρ‹ΠΉ ΠΌΠ°Ρ‚Π΅Ρ€ΠΈΠ°Π», Π° Ρ…ΠΎΠ»ΠΎΠ΄Π½Ρ‹ΠΉ ΠΌΠ°Ρ‚Π΅Ρ€ΠΈΠ°Π» ΠΎΡ…Π»Π°ΠΆΠ΄Π°Π΅Ρ‚ горячий ΠΌΠ°Ρ‚Π΅Ρ€ΠΈΠ°Π». Π­Ρ‚ΠΎ Π΄Π΅ΠΉΡΡ‚Π²ΠΈΡ‚Π΅Π»ΡŒΠ½ΠΎ Ρ‚ΠΎ просто. Когда Π²Ρ‹ чувствуСтС Ρ‚Π΅ΠΏΠ»ΠΎ , Ρ‡Ρ‚ΠΎ Π²Ρ‹ ΠΎΡ‰ΡƒΡ‰Π°Π΅Ρ‚Π΅ ΠΏΠ΅Ρ€Π΅Π΄Π°Ρ‡Ρƒ Ρ‚Π΅ΠΏΠ»ΠΎΠ²ΠΎΠΉ энСргии ΠΎΡ‚ ΠΎΡ‚ горячСго ΠΊ Ρ…ΠΎΠ»ΠΎΠ΄Π½ΠΎΠΌΡƒ.

Дисциплина Ρ‚Π΅ΠΏΠ»ΠΎΠΏΠ΅Ρ€Π΅Π΄Π°Ρ‡Π° являСтся касаСтся Ρ‚ΠΎΠ»ΡŒΠΊΠΎ Π΄Π²ΡƒΡ… Π²Π΅Ρ‰Π΅ΠΉ: Ρ‚Π΅ΠΌΠΏΠ΅Ρ€Π°Ρ‚ΡƒΡ€Ρ‹ ΠΈ ΠΏΠΎΡ‚ΠΎΠΊ Ρ‚Π΅ΠΏΠ»Π°. Π’Π΅ΠΌΠΏΠ΅Ρ€Π°Ρ‚ΡƒΡ€Π° прСдставляСт собой количСство доступная тСпловая энСргия, Ρ‚ΠΎΠ³Π΄Π° ΠΊΠ°ΠΊ Ρ‚Π΅ΠΏΠ»ΠΎΠ²ΠΎΠΉ ΠΏΠΎΡ‚ΠΎΠΊ прСдставляСт Π΄Π²ΠΈΠΆΠ΅Π½ΠΈΠ΅ Ρ‚Π΅ΠΏΠ»ΠΎΠ²ΠΎΠΉ энСргии с мСста Π½Π° мСсто. Π’ микроскопичСском ΠΌΠ°ΡΡˆΡ‚Π°Π±Π΅ тСпловая энСргия связана с кинСтичСская энСргия ΠΌΠΎΠ»Π΅ΠΊΡƒΠ».Π§Π΅ΠΌ большС ΠΌΠ°Ρ‚Π΅Ρ€ΠΈΠ°Π»ΠΎΠ² Ρ‚Π΅ΠΌΠΏΠ΅Ρ€Π°Ρ‚ΡƒΡ€Ρ‹, Ρ‚Π΅ΠΌ сильнСС тСрмичСскоС ΠΏΠ΅Ρ€Π΅ΠΌΠ΅ΡˆΠΈΠ²Π°Π½ΠΈΠ΅ Π΅Π΅ ΡΠΎΡΡ‚Π°Π²Π»ΡΡŽΡ‰ΠΈΠ΅ ΠΌΠΎΠ»Π΅ΠΊΡƒΠ»Ρ‹ (ΠΏΡ€ΠΎΡΠ²Π»ΡΡŽΡ‚ΡΡ ΠΊΠ°ΠΊ Π² Π»ΠΈΠ½Π΅ΠΉΠ½ΠΎΠΌ Π΄Π²ΠΈΠΆΠ΅Π½ΠΈΠΈ ΠΈ ΠΊΠΎΠ»Π΅Π±Π°Ρ‚Π΅Π»ΡŒΠ½Ρ‹Π΅ Ρ€Π΅ΠΆΠΈΠΌΡ‹).

ΠŸΡ€ΠΎΠ²ΠΎΠ΄ΠΈΠΌΠΎΡΡ‚ΡŒ

Π‘ΠΎΠ»ΡŒΡˆΠΈΠ½ΡΡ‚Π²ΠΎ Π­Ρ„Ρ„Π΅ΠΊΡ‚ΠΈΠ²Π½Ρ‹ΠΉ ΠΌΠ΅Ρ‚ΠΎΠ΄ ΠΏΠ΅Ρ€Π΅Π΄Π°Ρ‡ΠΈ Ρ‚Π΅ΠΏΠ»Π° — Ρ‚Π΅ΠΏΠ»ΠΎΠΏΡ€ΠΎΠ²ΠΎΠ΄Π½ΠΎΡΡ‚ΡŒ . Π­Ρ‚ΠΎΡ‚ Ρ€Π΅ΠΆΠΈΠΌ Ρ‚Π΅ΠΏΠ»ΠΎΠΏΠ΅Ρ€Π΅Π΄Π°Ρ‡ΠΈ Π²ΠΎΠ·Π½ΠΈΠΊΠ°Π΅Ρ‚, ΠΊΠΎΠ³Π΄Π° Π΅ΡΡ‚ΡŒ Π³Ρ€Π°Π΄ΠΈΠ΅Π½Ρ‚ Ρ‚Π΅ΠΌΠΏΠ΅Ρ€Π°Ρ‚ΡƒΡ€Ρ‹ ΠΏΠΎ Ρ‚Π΅Π»Ρƒ.Π’ Ρ‚Π°ΠΊΠΎΠΌ случаС, энСргия пСрСдаСтся ΠΈΠ· высокотСмпСратурной области Π² ΠΎΠ±Π»Π°ΡΡ‚ΡŒ Π½ΠΈΠ·ΠΊΠΈΡ… Ρ‚Π΅ΠΌΠΏΠ΅Ρ€Π°Ρ‚ΡƒΡ€ ΠΈΠ·-Π·Π° случайного двиТСния ΠΌΠΎΠ»Π΅ΠΊΡƒΠ» (диффузия). ΠŸΡ€ΠΎΠ²ΠΎΠ΄ΠΈΠΌΠΎΡΡ‚ΡŒ Π°Π½Π°Π»ΠΎΠ³ΠΈΡ‡Π½Ρ‹ΠΌ ΠΎΠ±Ρ€Π°Π·ΠΎΠΌ происходит Π² Тидкостях ΠΈ Π³Π°Π·Ρ‹. ΠžΠ±Π»Π°ΡΡ‚ΠΈ с большСй молСкулярной кинСтичСской энСргиСй пСрСдаст свою Ρ‚Π΅ΠΏΠ»ΠΎΠ²ΡƒΡŽ ΡΠ½Π΅Ρ€Π³ΠΈΡŽ Π² Ρ€Π΅Π³ΠΈΠΎΠ½Ρ‹ с мСньшими молСкулярная энСргия Π·Π° счСт прямых столкновСний ΠΌΠΎΠ»Π΅ΠΊΡƒΠ». Π’ ΠΌΠ΅Ρ‚Π°Π»Π»ΠΎΠ², Π·Π½Π°Ρ‡ΠΈΡ‚Π΅Π»ΡŒΠ½Π°Ρ Ρ‡Π°ΡΡ‚ΡŒ транспортируСмого Ρ‚Π΅ΠΏΠ»ΠΎΠ²ΠΎΠ³ΠΎ энСргия Ρ‚Π°ΠΊΠΆΠ΅ пСрСносится элСктронами Π·ΠΎΠ½Ρ‹ проводимости.Π Π°Π·Π»ΠΈΡ‡Π½Ρ‹Π΅ ΠΌΠ°Ρ‚Π΅Ρ€ΠΈΠ°Π»Ρ‹ ΠΎΠ±Π»Π°Π΄Π°ΡŽΡ‚ Ρ€Π°Π·Π½ΠΎΠΉ ΡΠΏΠΎΡΠΎΠ±Π½ΠΎΡΡ‚ΡŒΡŽ ΠΏΡ€ΠΎΠ²ΠΎΠ΄ΠΈΡ‚ΡŒ Π½Π°Π³Ρ€Π΅Π²Π°Ρ‚ΡŒ. ΠœΠ°Ρ‚Π΅Ρ€ΠΈΠ°Π»Ρ‹, ΠΏΠ»ΠΎΡ…ΠΎ проводящиС Ρ‚Π΅ΠΏΠ»ΠΎ (Π΄Π΅Ρ€Π΅Π²ΠΎ, пСнополистирол) часто Π½Π°Π·Ρ‹Π²Π°ΡŽΡ‚ изоляторами . Π’Π΅ΠΌ Π½Π΅ ΠΌΠ΅Π½ΠΈΠ΅, ΠΌΠ°Ρ‚Π΅Ρ€ΠΈΠ°Π»Ρ‹, ΠΊΠΎΡ‚ΠΎΡ€Ρ‹Π΅ Ρ…ΠΎΡ€ΠΎΡˆΠΎ проводят Ρ‚Π΅ΠΏΠ»ΠΎ (ΠΌΠ΅Ρ‚Π°Π»Π»Ρ‹, стСкло, Π½Π΅ΠΊΠΎΡ‚ΠΎΡ€Ρ‹Π΅ пластмассы) Π½Π΅ ΠΈΠΌΠ΅ΡŽΡ‚ ΡΠΏΠ΅Ρ†ΠΈΠ°Π»ΡŒΠ½ΠΎΠ³ΠΎ названия.

Π’ ΠΏΡ€ΠΎΡΡ‚Π΅ΠΉΡˆΠ°Ρ Ρ‚Π΅ΠΏΠ»ΠΎΠΏΡ€ΠΎΠ²ΠΎΠ΄Π½ΠΎΡΡ‚ΡŒ ΠΌΠΎΠΆΠ΅Ρ‚ Π±Ρ‹Ρ‚ΡŒ описана ΠΊΠ°ΠΊ ΠΎΠ΄Π½ΠΎΠΌΠ΅Ρ€Π½Ρ‹ΠΉ Ρ‚Π΅ΠΏΠ»ΠΎΠ²ΠΎΠΉ ΠΏΠΎΡ‚ΠΎΠΊ, ΠΊΠ°ΠΊ ΠΏΠΎΠΊΠ°Π·Π°Π½ΠΎ Π½ΠΈΠΆΠ΅ Ρ„ΠΈΠ³ΡƒΡ€Π°.Π‘ΠΊΠΎΡ€ΠΎΡΡ‚ΡŒ Ρ‚Π΅ΠΏΠ»ΠΎΠ²ΠΎΠ³ΠΎ ΠΏΠΎΡ‚ΠΎΠΊΠ° ΠΎΡ‚ ΠΎΠ΄Π½ΠΎΠΉ стороны ΠΎΠ±ΡŠΠ΅ΠΊΡ‚Π° ΠΊ Π΄Ρ€ΡƒΠ³ΠΎΠΉ ΠΈΠ»ΠΈ ΠΌΠ΅ΠΆΠ΄Ρƒ ΡΠΎΠΏΡ€ΠΈΠΊΠ°ΡΠ°ΡŽΡ‰ΠΈΠΌΠΈΡΡ ΠΎΠ±ΡŠΠ΅ΠΊΡ‚Π°ΠΌΠΈ, зависит ΠΎΡ‚ ΠΏΠΎΠΏΠ΅Ρ€Π΅Ρ‡Π½ΠΎΠ΅ сСчСниС ΠΏΠΎΡ‚ΠΎΠΊΠ°, ΠΏΡ€ΠΎΠ²ΠΎΠ΄ΠΈΠΌΠΎΡΡ‚ΡŒ ΠΌΠ°Ρ‚Π΅Ρ€ΠΈΠ°Π» ΠΈ Ρ€Π°Π·Π½ΠΈΡ†Π° Ρ‚Π΅ΠΌΠΏΠ΅Ρ€Π°Ρ‚ΡƒΡ€ ΠΌΠ΅ΠΆΠ΄Ρƒ двумя повСрхности ΠΈΠ»ΠΈ ΠΏΡ€Π΅Π΄ΠΌΠ΅Ρ‚Ρ‹.

ΠœΠ°Ρ‚Π΅ΠΌΠ°Ρ‚ΠΈΡ‡Π΅ΡΠΊΠΈ это ΠΌΠΎΠΆΠ½ΠΎ Π²Ρ‹Ρ€Π°Π·ΠΈΡ‚ΡŒ ΠΊΠ°ΠΊ

Π³Π΄Π΅ q — коэффициСнт Ρ‚Π΅ΠΏΠ»ΠΎΠΏΠ΅Ρ€Π΅Π΄Π°Ρ‡ΠΈ Π² Π²Π°Ρ‚Ρ‚Π°Ρ… (Π’Ρ‚), k — Ρ‚Π΅ΠΏΠ»ΠΎΠΏΡ€ΠΎΠ²ΠΎΠ΄Π½ΠΎΡΡ‚ΡŒ ΠΌΠ°Ρ‚Π΅Ρ€ΠΈΠ°Π»Π° (Π’Ρ‚ / ΠΌ.К), А — ΠΏΠ»ΠΎΡ‰Π°Π΄ΡŒ ΠΏΠΎΠΏΠ΅Ρ€Π΅Ρ‡Π½ΠΎΠ³ΠΎ сСчСния Ρ‚Π΅ΠΏΠ»ΠΎΠ²ΠΎΠ³ΠΎ Ρ‚Ρ€Π°ΠΊΡ‚Π°, Π° являСтся Π³Ρ€Π°Π΄ΠΈΠ΅Π½Ρ‚ Ρ‚Π΅ΠΌΠΏΠ΅Ρ€Π°Ρ‚ΡƒΡ€Ρ‹ Π² Π½Π°ΠΏΡ€Π°Π²Π»Π΅Π½ΠΈΠΈ ΠΏΠΎΡ‚ΠΎΠΊΠ° (К / ΠΌ).

ΠŸΡ€ΠΈΠ²Π΅Π΄Π΅Π½Π½ΠΎΠ΅ Π²Ρ‹ΡˆΠ΅ ΡƒΡ€Π°Π²Π½Π΅Π½ΠΈΠ΅ извСстСн ΠΊΠ°ΠΊ Π·Π°ΠΊΠΎΠ½ тСплопроводности Π€ΡƒΡ€ΡŒΠ΅. Π‘Π»Π΅Π΄ΠΎΠ²Π°Ρ‚Π΅Π»ΡŒΠ½ΠΎ, ΡΠΊΠΎΡ€ΠΎΡΡ‚ΡŒ Ρ‚Π΅ΠΏΠ»ΠΎΠΏΠ΅Ρ€Π΅Π΄Π°Ρ‡ΠΈ Π·Π° счСт тСплопроводности Ρ‡Π΅Ρ€Π΅Π· ΠΎΠ±ΡŠΠ΅ΠΊΡ‚ Π½Π° ΠΏΡ€ΠΈΠ²Π΅Π΄Π΅Π½Π½ΠΎΠΌ Π²Ρ‹ΡˆΠ΅ рисункС ΠΌΠΎΠΆΠ΅Ρ‚ Π±Ρ‹Ρ‚ΡŒ Π²Ρ‹Ρ€Π°ΠΆΠ΅Π½ΠΎ ΠΊΠ°ΠΊ

Π“Π΄Π΅ L Ρ‚ΠΎΠ»Ρ‰ΠΈΠ½Π° (ΠΈΠ»ΠΈ Π΄Π»ΠΈΠ½Π°) ΠΏΡ€ΠΎΠ²ΠΎΠ΄Π½ΠΈΠΊΠ°, Π” Π’ Ρ€Π°Π·Π½ΠΈΡ†Π° Ρ‚Π΅ΠΌΠΏΠ΅Ρ€Π°Ρ‚ΡƒΡ€ ΠΌΠ΅ΠΆΠ΄Ρƒ ΠΎΠ΄Π½ΠΎΠΉ стороной ΠΈ Π΄Ρ€ΡƒΠ³ΠΎΠ΅ (Π½Π°ΠΏΡ€ΠΈΠΌΠ΅Ρ€, Π” Π’ = T 1 T 2 — это Ρ€Π°Π·Π½ΠΈΡ†Π° Ρ‚Π΅ΠΌΠΏΠ΅Ρ€Π°Ρ‚ΡƒΡ€ ΠΌΠ΅ΠΆΠ΄Ρƒ стороной 1 ΠΈ стороной 2).

ΠšΠΎΠ»ΠΈΡ‡Π΅ΡΡ‚Π²ΠΎ (Π”Π’ / Π») Π² ΡƒΡ€Π°Π²Π½Π΅Π½ΠΈΠΈ (16.5) называСтся Π³Ρ€Π°Π΄ΠΈΠ΅Π½Ρ‚ΠΎΠΌ Ρ‚Π΅ΠΌΠΏΠ΅Ρ€Π°Ρ‚ΡƒΡ€Ρ‹ : ΠΎΠ½ сообщаСт, Π½Π° сколько 0C ΠΈΠ»ΠΈ K Ρ‚Π΅ΠΌΠΏΠ΅Ρ€Π°Ρ‚ΡƒΡ€Π° измСняСтся Π·Π° Π΅Π΄ΠΈΠ½ΠΈΡ†Π° расстояния, пСрСмСщСнная ΠΏΠΎ ΠΏΡƒΡ‚ΠΈ Ρ‚Π΅ΠΏΠ»ΠΎΠ²ΠΎΠ³ΠΎ ΠΏΠΎΡ‚ΠΎΠΊΠ°. Π’ Π²Π΅Π»ΠΈΡ‡ΠΈΠ½Π° L / kA называСтся тСрмичСским сопротивлСниСм

Π’Π΅ΠΏΠ»ΠΎΠ²ΠΎΠΉ сопротивлСниС измСряСтся Π² Π΅Π΄ΠΈΠ½ΠΈΡ†Π°Ρ… БИ — ΠΊΠ΅Π»ΡŒΠ²ΠΈΠ½Π°Ρ… Π½Π° Π²Π°Ρ‚Ρ‚ (К / Π’Ρ‚).ΠžΠ±Ρ€Π°Ρ‚ΠΈΡ‚Π΅ Π²Π½ΠΈΠΌΠ°Π½ΠΈΠ΅ Π½Π° ΡƒΡ€Π°Π²Π½Π΅Π½ΠΈΠ΅ (16.6), Ρ‡Ρ‚ΠΎ Ρ‚Π΅ΠΏΠ»ΠΎΠ²ΠΎΠ΅ сопротивлСниС зависит ΠΎΡ‚ Ρ…Π°Ρ€Π°ΠΊΡ‚Π΅Ρ€Π° ΠΌΠ°Ρ‚Π΅Ρ€ΠΈΠ°Π»Π° (тСрмичСский ΠΏΡ€ΠΎΠ²ΠΎΠ΄ΠΈΠΌΠΎΡΡ‚ΡŒ ΠΊ ΠΈ гСомСтрия корпуса Π΄ / А ). ΠœΡ‹ ΠΏΠΎΠ½ΠΈΠΌΠ°Π΅ΠΌ ΠΈΠ· ΠΏΡ€ΠΈΠ²Π΅Π΄Π΅Π½Π½Ρ‹Ρ… Π²Ρ‹ΡˆΠ΅ ΡƒΡ€Π°Π²Π½Π΅Π½ΠΈΠΉ, ΠΌΡ‹ ΠΏΠΎΠ½ΠΈΠΌΠ°Π΅ΠΌ Ρ‚Π΅ΠΏΠ»ΠΎ ΡΠΊΠΎΡ€ΠΎΡΡ‚ΡŒ ΠΏΠ΅Ρ€Π΅Π΄Π°Ρ‡ΠΈ ΠΊΠ°ΠΊ ΠΏΠΎΡ‚ΠΎΠΊΠ°, Ρ‚Π°ΠΊ ΠΈ сочСтаниС тСрмичСского ΠΏΡ€ΠΎΠ²ΠΎΠ΄ΠΈΠΌΠΎΡΡ‚ΡŒ, Ρ‚ΠΎΠ»Ρ‰ΠΈΠ½Π° ΠΌΠ°Ρ‚Π΅Ρ€ΠΈΠ°Π»Π° ΠΈ ΠΏΠ»ΠΎΡ‰Π°Π΄ΡŒ ΠΊΠ°ΠΊ сопротивлСниС этому ΠΏΠΎΡ‚ΠΎΠΊΡƒ.

Учитывая Ρ‚Π΅ΠΌΠΏΠ΅Ρ€Π°Ρ‚ΡƒΡ€Π° ΠΊΠ°ΠΊ ΠΏΠΎΡ‚Π΅Π½Ρ†ΠΈΠ°Π»ΡŒΠ½Π°Ρ функция Ρ‚Π΅ΠΏΠ»Π° Π·Π°ΠΊΠΎΠ½ Π€ΡƒΡ€ΡŒΠ΅ ΠΌΠΎΠΆΠ½ΠΎ Π·Π°ΠΏΠΈΡΠ°Ρ‚ΡŒ ΠΊΠ°ΠΊ

Если ΠΌΡ‹ ΠΎΠΏΡ€Π΅Π΄Π΅Π»ΠΈΠΌ сопротивлСниС ΠΊΠ°ΠΊ ΠΎΡ‚Π½ΠΎΡˆΠ΅Π½ΠΈΠ΅ ΠΏΠΎΡ‚Π΅Π½Ρ†ΠΈΠ°Π»Π° ΠΊ ΡΠΎΠΎΡ‚Π²Π΅Ρ‚ΡΡ‚Π²ΡƒΡŽΡ‰Π°Ρ ΡΠΊΠΎΡ€ΠΎΡΡ‚ΡŒ ΠΏΠ΅Ρ€Π΅Π΄Π°Ρ‡ΠΈ, Ρ‚Π΅ΠΏΠ»ΠΎΠ²ΠΎΠ΅ сопротивлСниС для ΠΏΡ€ΠΎΠ²ΠΎΠ΄ΠΈΠΌΠΎΡΡ‚ΡŒ ΠΌΠΎΠΆΠ½ΠΎ Π²Ρ‹Ρ€Π°Π·ΠΈΡ‚ΡŒ ΠΊΠ°ΠΊ

Ясно ΠΈΠ· ΠΏΡ€ΠΈΠ²Π΅Π΄Π΅Π½Π½ΠΎΠ³ΠΎ Π²Ρ‹ΡˆΠ΅ уравнСния, Ρ‡Ρ‚ΠΎ ΡƒΠΌΠ΅Π½ΡŒΡˆΠ΅Π½ΠΈΠ΅ Ρ‚ΠΎΠ»Ρ‰ΠΈΠ½Ρ‹ ΠΈΠ»ΠΈ ΡƒΠ²Π΅Π»ΠΈΡ‡Π΅Π½ΠΈΠ΅ ΠΏΠ»ΠΎΡ‰Π°Π΄ΠΈ ΠΏΠΎΠΏΠ΅Ρ€Π΅Ρ‡Π½ΠΎΠ³ΠΎ сСчСния ΠΈΠ»ΠΈ тСрмичСскоС ΠΏΡ€ΠΎΠ²ΠΎΠ΄ΠΈΠΌΠΎΡΡ‚ΡŒ ΠΎΠ±ΡŠΠ΅ΠΊΡ‚Π° снизит Π΅Π³ΠΎ Ρ‚Π΅ΠΏΠ»ΠΎΠ²ΡƒΡŽ сопротивлСниС ΠΈ ΡƒΠ²Π΅Π»ΠΈΡ‡ΠΈΡ‚ΡŒ ΡΠΊΠΎΡ€ΠΎΡΡ‚ΡŒ Ρ‚Π΅ΠΏΠ»ΠΎΠΏΠ΅Ρ€Π΅Π΄Π°Ρ‡ΠΈ.

ΠšΠΎΠ½Π²Π΅ΠΊΡ†ΠΈΡ

А Π±ΠΎΠ»Π΅Π΅ ΠΌΠ΅Π΄Π»Π΅Π½Π½Ρ‹ΠΌ ΠΌΠ΅Ρ‚ΠΎΠ΄ΠΎΠΌ Ρ‚Π΅ΠΏΠ»ΠΎΠΏΠ΅Ρ€Π΅Π΄Π°Ρ‡ΠΈ являСтся конвСкция, которая Π²ΠΊΠ»ΡŽΡ‡Π°Π΅Ρ‚ ΠΏΠΎΡ‚ΠΎΠΊΠΈ Тидкости, ΠΊΠΎΡ‚ΠΎΡ€Ρ‹Π΅ пСрСносят Ρ‚Π΅ΠΏΠ»ΠΎ ΠΈΠ· ΠΎΠ΄Π½ΠΎΠ³ΠΎ мСста ΠΊ Π΄Ρ€ΡƒΠ³ΠΎΠΌΡƒ. Π’ проводимости энСргия Ρ‚Π΅Ρ‡Π΅Ρ‚ Ρ‡Π΅Ρ€Π΅Π· ΠΌΠ°Ρ‚Π΅Ρ€ΠΈΠ°Π», Π½ΠΎ сам ΠΌΠ°Ρ‚Π΅Ρ€ΠΈΠ°Π» Π½Π΅ двиТСтся. Π’ конвСкция, сам ΠΌΠ°Ρ‚Π΅Ρ€ΠΈΠ°Π» пСрСмСщаСтся с ΠΎΠ΄Π½ΠΎΠ³ΠΎ мСста Π½Π° Π”Ρ€ΡƒΠ³ΠΎΠΉ. ΠšΠΎΠ½Π²Π΅ΠΊΡ†ΠΈΠΎΠ½Π½Π°Ρ Ρ‚Π΅ΠΏΠ»ΠΎΠΏΠ΅Ρ€Π΅Π΄Π°Ρ‡Π° состоит ΠΈΠ· Π΄Π²Π° ΠΌΠ΅Ρ…Π°Π½ΠΈΠ·ΠΌΠ°: случайноС Π΄Π²ΠΈΠΆΠ΅Π½ΠΈΠ΅ ΠΌΠΎΠ»Π΅ΠΊΡƒΠ» (диффузия) ΠΈ энСргия, пСрСдаваСмая ΠΎΠ±ΡŠΠ΅ΠΌΠ½Ρ‹ΠΌ ΠΈΠ»ΠΈ макроскопичСским Π΄Π²ΠΈΠΆΠ΅Π½ΠΈΠ΅ΠΌ ΠΆΠΈΠ΄ΠΊΠΎΡΡ‚ΡŒ.ΠŸΠ΅Ρ€Π΅Π΄Π°Ρ‡Π° Ρ‚Π΅ΠΏΠ»Π° ΠΎΡ‚ Ρ‚Π²Π΅Ρ€Π΄ΠΎΠ³ΠΎ Ρ‚Π΅Π»Π° ΠΊ Тидкости (Тидкости ΠΈΠ»ΠΈ Π³Π°Π·ΠΎΠΎΠ±Ρ€Π°Π·Π½Ρ‹ΠΉ) Π±ΠΎΠ»Π΅Π΅ слоТСн, Ρ‡Π΅ΠΌ пСрСнос Ρ‚Π²Π΅Ρ€Π΄ΠΎΠ΅ Ρ‚Π΅Π»ΠΎ-Ρ‚Π²Π΅Ρ€Π΄ΠΎΠ΅ Ρ‚Π΅Π»ΠΎ, ΠΏΠΎΡΠΊΠΎΠ»ΡŒΠΊΡƒ ΠΏΠ΅Ρ€Π΅ΠΏΠ°Π΄Ρ‹ Ρ‚Π΅ΠΏΠ»Π° Π² Тидкости ΠΎΠ±Ρ‹Ρ‡Π½ΠΎ Π²Ρ‹Π·Ρ‹Π²Π°ΡŽΡ‚ Π²Π½ΡƒΡ‚Ρ€Π΅Π½Π½Π΅Π΅ Π΄Π²ΠΈΠΆΠ΅Π½ΠΈΠ΅, извСстноС ΠΊΠ°ΠΊ ΠΊΠΎΠ½Π²Π΅ΠΊΡ†ΠΈΠΎΠ½Π½Ρ‹Π΅ Ρ‚ΠΎΠΊΠΈ. Π’ Π²ΠΈΠ΄Π΅ объСм увСличиваСтся с Ρ‚Π΅ΠΌΠΏΠ΅Ρ€Π°Ρ‚ΡƒΡ€ΠΎΠΉ, Π±ΠΎΠ»Π΅Π΅ Ρ‚Π΅ΠΏΠ»Ρ‹Π΅ области ΠΆΠΈΠ΄ΠΊΠΎΡΡ‚ΡŒ ΠΈΠΌΠ΅Π΅Ρ‚ ΠΌΠ΅Π½ΡŒΡˆΡƒΡŽ массу, Ρ‡Π΅ΠΌ Π±ΠΎΠ»Π΅Π΅ Ρ…ΠΎΠ»ΠΎΠ΄Π½Ρ‹Π΅ области. ΠŸΠ»ΠΎΡ…ΠΎΠΉ Π²ΠΎΠ·Π΄ΡƒΡ… ΠΏΡ€ΠΎΠ²ΠΎΠ΄Π½ΠΈΠΊ Ρ‚Π΅ΠΏΠ»Π°, Π½ΠΎ ΠΎΠ½ ΠΌΠΎΠΆΠ΅Ρ‚ Π»Π΅Π³ΠΊΠΎ Ρ‚Π΅Ρ‡ΡŒ ΠΈ ΠΏΠ΅Ρ€Π΅Π½ΠΎΡΠΈΡ‚ΡŒ Ρ‚Π΅ΠΏΠ»ΠΎ ΠΊΠΎΠ½Π²Π΅ΠΊΡ†ΠΈΠ΅ΠΉ. ИспользованиС Π³Π΅Ρ€ΠΌΠ΅Ρ‚ΠΈΡ‡Π½Ρ‹Ρ… стСклопакСтов замСняСт большой Π²ΠΎΠ·Π΄ΡƒΡˆΠ½Ρ‹ΠΉ Π·Π°Π·ΠΎΡ€ ΠΌΠ΅ΠΆΠ΄Ρƒ ΡˆΡ‚ΠΎΡ€ΠΌΠΎΠ²Ρ‹ΠΌ ΠΎΠΊΠ½ΠΎΠΌ ΠΈ ΠΎΠ±Ρ‹Ρ‡Π½ΠΎΠ΅ ΠΎΠΊΠ½ΠΎ с Π³ΠΎΡ€Π°Π·Π΄ΠΎ мСньшим Π·Π°Π·ΠΎΡ€ΠΎΠΌ.МСньший Π²ΠΎΠ·Π΄ΡƒΡ… Π·Π°Π·ΠΎΡ€ сводит ΠΊ ΠΌΠΈΠ½ΠΈΠΌΡƒΠΌΡƒ Ρ†ΠΈΡ€ΠΊΡƒΠ»ΠΈΡ€ΡƒΡŽΡ‰ΠΈΠ΅ ΠΊΠΎΠ½Π²Π΅ΠΊΡ†ΠΈΠΎΠ½Π½Ρ‹Π΅ Ρ‚ΠΎΠΊΠΈ ΠΌΠ΅ΠΆΠ΄Ρƒ Π΄Π²Π΅ ΠΏΠ°Π½Π΅Π»ΠΈ.

Π’Π΅Π»ΠΈΡ‡ΠΈΠ½Π° ΠΊΠΎΠ½Π²Π΅ΠΊΡ‚ΠΈΠ²Π½ΠΎΠ³ΠΎ Ρ‚Π΅ΠΏΠ»ΠΎΠ²ΠΎΠ³ΠΎ ΠΏΠΎΡ‚ΠΎΠΊΠ° Π² Тидкости зависит ΠΎΡ‚ ΠΏΠ»ΠΎΡ‰Π°Π΄ΠΈ ΠΊΠΎΠ½Ρ‚Π°ΠΊΡ‚Π° с Ρ‚Π²Π΅Ρ€Π΄ΠΎΠ΅ Ρ‚Π΅Π»ΠΎ, Π΅Π³ΠΎ Π²ΡΠ·ΠΊΠΎΡΡ‚ΡŒ, ΡΠΊΠΎΡ€ΠΎΡΡ‚ΡŒ двиТСния Ρ‚Π²Π΅Ρ€Π΄ΠΎΠ³ΠΎ Ρ‚Π΅Π»Π°, Ρ‚Π΅Ρ‡Π΅Π½ΠΈΠ΅ характСристики ΠΈ ΠΎΠ±Ρ‰ΠΈΠΉ ΠΏΠ΅Ρ€Π΅ΠΏΠ°Π΄ Ρ‚Π΅ΠΌΠΏΠ΅Ρ€Π°Ρ‚ΡƒΡ€ ΠΌΠ΅ΠΆΠ΄Ρƒ двумя. Π’Π΅Ρ€ΠΌΠΈΠ½ конвСкция Ρ‚Π°ΠΊΠΆΠ΅ использовался историчСски ΠΎΠΏΠΈΡΡ‹Π²Π°Ρ‚ΡŒ пСрСнос Ρ‚Π΅ΠΏΠ»Π° ΠΎΡ‚ ΠΎΠ΄Π½ΠΎΠ³ΠΎ Ρ‚Π²Π΅Ρ€Π΄ΠΎΠ΅ Ρ‚Π΅Π»ΠΎ ΠΊ Π΄Ρ€ΡƒΠ³ΠΎΠΌΡƒ Ρ€Π°Π·Π΄Π΅Π»Π΅Π½ΠΎ ΠΆΠΈΠ΄ΠΊΠΎΠΉ срСдой.

Π—Π°ΠΊΠΎΠ½ ΠΡŒΡŽΡ‚ΠΎΠ½Π° ΠΎΡ…Π»Π°ΠΆΠ΄Π΅Π½ΠΈΠ΅ Π²Ρ‹Ρ€Π°ΠΆΠ°Π΅Ρ‚ ΠΎΠ±Ρ‰ΠΈΠΉ эффСкт ΠΊΠΎΠ½Π²Π΅ΠΊΡ†ΠΈΠΈ:

Π“Π΄Π΅ Ρ… — коэффициСнт ΠΊΠΎΠ½Π²Π΅ΠΊΡ‚ΠΈΠ²Π½ΠΎΠΉ Ρ‚Π΅ΠΏΠ»ΠΎΠΎΡ‚Π΄Π°Ρ‡ΠΈ (Π’Ρ‚ / ΠΌ 2 K), A — ΠΏΠ»ΠΎΡ‰Π°Π΄ΡŒ повСрхности, Π” Π’ = T с T f — это Ρ€Π°Π·Π½ΠΈΡ†Π° Ρ‚Π΅ΠΌΠΏΠ΅Ρ€Π°Ρ‚ΡƒΡ€ ΠΌΠ΅ΠΆΠ΄Ρƒ Ρ‚Π΅ΠΌΠΏΠ΅Ρ€Π°Ρ‚ΡƒΡ€ΠΎΠΉ повСрхности T s , Π° Ρ‚Π΅ΠΌΠΏΠ΅Ρ€Π°Ρ‚ΡƒΡ€Π° Тидкости T f .Как ΠΈ Π² случаС проводимости, Ρ‚Π΅ΠΏΠ»ΠΎΠ²ΠΎΠ΅ сопротивлСниС Ρ€Π°Π²Π½ΠΎ Ρ‚Π°ΠΊΠΆΠ΅ связаны с ΠΊΠΎΠ½Π²Π΅ΠΊΡ†ΠΈΠΎΠ½Π½ΠΎΠΉ Ρ‚Π΅ΠΏΠ»ΠΎΠΏΠ΅Ρ€Π΅Π΄Π°Ρ‡Π΅ΠΉ ΠΈ ΠΌΠΎΠΆΠ½ΠΎ Π²Ρ‹Ρ€Π°Π·ΠΈΡ‚ΡŒ ΠΊΠ°ΠΊ

ΠšΠΎΠ½Π²Π΅ΠΊΡ†ΠΈΠΎΠ½Π½Π°Ρ Ρ‚Π΅ΠΏΠ»ΠΎΠΏΠ΅Ρ€Π΅Π΄Π°Ρ‡Π° ΠΌΠΎΠΆΠ΅Ρ‚ Π±Ρ‹Ρ‚ΡŒ ΠΊΠ»Π°ΡΡΠΈΡ„ΠΈΡ†ΠΈΡ€ΡƒΡŽΡ‚ΡΡ ΠΏΠΎ Ρ…Π°Ρ€Π°ΠΊΡ‚Π΅Ρ€Ρƒ ΠΏΠΎΡ‚ΠΎΠΊΠ° Тидкости.

ΠŸΡ€ΠΈΠ½ΡƒΠΆΠ΄Π΅Π½Π½Ρ‹ΠΉ конвСкция Π²ΠΎΠ·Π½ΠΈΠΊΠ°Π΅Ρ‚, ΠΊΠΎΠ³Π΄Π° ΠΏΠΎΡ‚ΠΎΠΊ Π²Ρ‹Π·Π²Π°Π½ внСшними срСдства, Ρ‚Π°ΠΊΠΈΠ΅ ΠΊΠ°ΠΊ вСнтилятор, насос ΠΈ Ρ‚. Π΄.

Радиация

Π’ НаимСнСС эффСктивным ΠΌΠ΅Ρ‚ΠΎΠ΄ΠΎΠΌ ΠΏΠ΅Ρ€Π΅Π΄Π°Ρ‡ΠΈ Ρ‚Π΅ΠΏΠ»Π° являСтся ΠΈΠ·Π»ΡƒΡ‡Π΅Π½ΠΈΠ΅. ЛучистоС Ρ‚Π΅ΠΏΠ»ΠΎ — это просто Ρ‚Π΅ΠΏΠ»ΠΎ энСргия Π² ΠΏΡƒΡ‚ΠΈ Π² Π²ΠΈΠ΄Π΅ элСктромагнитного излучСния. ВсС ΠΌΠ°Ρ‚Π΅Ρ€ΠΈΠ°Π»Ρ‹ ΠΈΠ·Π»ΡƒΡ‡Π°ΡŽΡ‚ Ρ‚Π΅ΠΏΠ»ΠΎΠ²ΡƒΡŽ ΡΠ½Π΅Ρ€Π³ΠΈΡŽ Π² ΠΎΠΏΡ€Π΅Π΄Π΅Π»Π΅Π½Π½Ρ‹Ρ… количСствах ΠΏΠΎ ΠΈΡ… Ρ‚Π΅ΠΌΠΏΠ΅Ρ€Π°Ρ‚ΡƒΡ€Π΅, Π³Π΄Π΅ энСргия пСрСносится Ρ„ΠΎΡ‚ΠΎΠ½Ρ‹ свСта Π² инфракрасной ΠΈ Π²ΠΈΠ΄ΠΈΠΌΠΎΠΉ частях свСта элСктромагнитный спСктр.Π’ этом случаС Ρ‚Π΅ΠΏΠ»ΠΎ двиТСтся Π² пространствС ΠΊΠ°ΠΊ элСктромагнитноС ΠΈΠ·Π»ΡƒΡ‡Π΅Π½ΠΈΠ΅ Π±Π΅Π· ΠΏΠΎΠΌΠΎΡ‰ΠΈ физичСская субстанция. ВсС ΠΎΠ±ΡŠΠ΅ΠΊΡ‚Ρ‹, содСрТащиС Ρ‚Π΅ΠΏΠ»ΠΎ ΠΈΡΠΏΡƒΡΠΊΠ°Ρ‚ΡŒ Π½Π΅ΠΊΠΎΡ‚ΠΎΡ€Ρ‹ΠΉ ΡƒΡ€ΠΎΠ²Π΅Π½ΡŒ лучистой энСргии. ΠšΠΎΠ»ΠΈΡ‡Π΅ΡΡ‚Π²ΠΎ ΠΈΠ·Π»ΡƒΡ‡Π΅Π½ΠΈΠ΅ ΠΎΠ±Ρ€Π°Ρ‚Π½ΠΎ ΠΏΡ€ΠΎΠΏΠΎΡ€Ρ†ΠΈΠΎΠ½Π°Π»ΡŒΠ½ΠΎ Π΅Π³ΠΎ Π΄Π»ΠΈΠ½Π΅ Π²ΠΎΠ»Π½Ρ‹ (Ρ‡Π΅ΠΌ ΠΊΠΎΡ€ΠΎΡ‡Π΅ Π΄Π»ΠΈΠ½Π° Π²ΠΎΠ»Π½Ρ‹, Ρ‚Π΅ΠΌ большС энСргия content), ΠΊΠΎΡ‚ΠΎΡ€Ρ‹ΠΉ, Π² свою ΠΎΡ‡Π΅Ρ€Π΅Π΄ΡŒ, ΠΎΠ±Ρ€Π°Ρ‚Π½ΠΎ ΠΏΡ€ΠΎΠΏΠΎΡ€Ρ†ΠΈΠΎΠ½Π°Π»Π΅Π½ Π΅Π³ΠΎ Ρ‚Π΅ΠΌΠΏΠ΅Ρ€Π°Ρ‚ΡƒΡ€Π° (Π² К).

Π‘ΠΎΠ»Π½Ρ†Π΅ Ρ‚Π΅ΠΏΠ»ΠΎ являСтся ΠΏΡ€ΠΈΠΌΠ΅Ρ€ΠΎΠΌ Ρ‚Π΅ΠΏΠ»ΠΎΠ²ΠΎΠ³ΠΎ излучСния, ΠΊΠΎΡ‚ΠΎΡ€ΠΎΠ΅ достигаСт ЗСмля.ЛучистоС Ρ‚Π΅ΠΏΠ»ΠΎ пСрСдаСтся нСпосрСдствСнно Π² ΠΏΠΎΠ²Π΅Ρ€Ρ…Π½ΠΎΡΡ‚ΡŒ любого Ρ‚Π²Π΅Ρ€Π΄ΠΎΠ³ΠΎ ΠΎΠ±ΡŠΠ΅ΠΊΡ‚Π°, с ΠΊΠΎΡ‚ΠΎΡ€Ρ‹ΠΌ ΠΎΠ½ сталкиваСтся (Ссли Ρ‚ΠΎΠ»ΡŒΠΊΠΎ ΠΎΠ½ Π½Π΅ сильно ΡΠ²Π΅Ρ‚ΠΎΠΎΡ‚Ρ€Π°ΠΆΠ°ΡŽΡ‰ΠΈΠΉ), Π½ΠΎ Π»Π΅Π³ΠΊΠΎ ΠΏΡ€ΠΎΡ…ΠΎΠ΄ΠΈΡ‚ сквозь ΠΏΡ€ΠΎΠ·Ρ€Π°Ρ‡Π½Ρ‹ΠΉ Ρ‚Π°ΠΊΠΈΠ΅ ΠΌΠ°Ρ‚Π΅Ρ€ΠΈΠ°Π»Ρ‹, ΠΊΠ°ΠΊ Π²ΠΎΠ·Π΄ΡƒΡ… ΠΈ стСкло. Π˜Π΄Π΅Π°Π»ΡŒΠ½Ρ‹ΠΉ Ρ‚Π΅ΠΏΠ»ΠΎΠ²ΠΎΠΉ Ρ€Π°Π΄ΠΈΠ°Ρ‚ΠΎΡ€ ΠΈΠ»ΠΈ Ρ‡Π΅Ρ€Π½ΠΎΠ΅ Ρ‚Π΅Π»ΠΎ Π±ΡƒΠ΄Π΅Ρ‚ ΠΈΠ·Π»ΡƒΡ‡Π°Ρ‚ΡŒ энСргии со ΡΠΊΠΎΡ€ΠΎΡΡ‚ΡŒΡŽ, ΠΏΡ€ΠΎΠΏΠΎΡ€Ρ†ΠΈΠΎΠ½Π°Π»ΡŒΠ½ΠΎΠΉ Ρ‡Π΅Ρ‚Π²Π΅Ρ€Ρ‚ΠΎΠΉ стСпСни Π΅Π΅ Π°Π±ΡΠΎΠ»ΡŽΡ‚Π½Π°Ρ Ρ‚Π΅ΠΌΠΏΠ΅Ρ€Π°Ρ‚ΡƒΡ€Π° ΠΈ ΠΏΠ»ΠΎΡ‰Π°Π΄ΡŒ Π΅Π΅ повСрхности. ΠœΠ°Ρ‚Π΅ΠΌΠ°Ρ‚ΠΈΡ‡Π΅ΡΠΊΠΈ это

Π³Π΄Π΅ s — константа ΠΏΡ€ΠΎΠΏΠΎΡ€Ρ†ΠΈΠΎΠ½Π°Π»ΡŒΠ½ΠΎΡΡ‚ΠΈ (постоянная Π‘Ρ‚Π΅Ρ„Π°Π½Π°-Π‘ΠΎΠ»ΡŒΡ†ΠΌΠ°Π½Π° = 5.669 10 -8 Π’Ρ‚ / ΠΌ 2 .K 4 ). Π’ ΠΏΡ€ΠΈΠ²Π΅Π΄Π΅Π½Π½ΠΎΠ΅ Π²Ρ‹ΡˆΠ΅ ΡƒΡ€Π°Π²Π½Π΅Π½ΠΈΠ΅ называСтся Π·Π°ΠΊΠΎΠ½ΠΎΠΌ Π‘Ρ‚Π΅Ρ„Π°Π½Π°-Π‘ΠΎΠ»ΡŒΡ†ΠΌΠ°Π½Π° Ρ‚Π΅ΠΏΠ»ΠΎΠ²ΠΎΠ³ΠΎ радиация, ΠΈ это относится Ρ‚ΠΎΠ»ΡŒΠΊΠΎ ΠΊ Ρ‡Π΅Ρ€Π½Ρ‹ΠΌ Ρ‚Π΅Π»Π°ΠΌ. Π’ ВСмпСратурная Π·Π°Π²ΠΈΡΠΈΠΌΠΎΡΡ‚ΡŒ Ρ‡Π΅Ρ‚Π²Π΅Ρ€Ρ‚ΠΎΠΉ стСпСни ΠΎΠ·Π½Π°Ρ‡Π°Π΅Ρ‚, Ρ‡Ρ‚ΠΎ Π˜Π·Π»ΡƒΡ‡Π°Π΅ΠΌΠ°Ρ ΠΌΠΎΡ‰Π½ΠΎΡΡ‚ΡŒ ΠΎΡ‡Π΅Π½ΡŒ Ρ‡ΡƒΠ²ΡΡ‚Π²ΠΈΡ‚Π΅Π»ΡŒΠ½Π° ΠΊ измСнСниям Ρ‚Π΅ΠΌΠΏΠ΅Ρ€Π°Ρ‚ΡƒΡ€Ρ‹. Если Π°Π±ΡΠΎΠ»ΡŽΡ‚Π½Π°Ρ Ρ‚Π΅ΠΌΠΏΠ΅Ρ€Π°Ρ‚ΡƒΡ€Π° Ρ‚Π΅Π»Π° увСличиваСтся Π²Π΄Π²ΠΎΠ΅, излучаСмая энСргия увСличиваСтся Π² 2 Ρ€Π°Π·Π° 4 = 16.

Для Ρ‚Π΅Π» Π½Π΅ ΠΏΠΎΠ²Π΅Π΄Π΅Π½ΠΈΠ΅ Ρ‡Π΅Ρ€Π½ΠΎΠ³ΠΎ Ρ‚Π΅Π»Π°, Ρ„Π°ΠΊΡ‚ΠΎΡ€, извСстный ΠΊΠ°ΠΊ коэффициСнт излучСния e , ΠΊΠΎΡ‚ΠΎΡ€Ρ‹ΠΉ связываСт ΠΈΠ·Π»ΡƒΡ‡Π΅Π½ΠΈΠ΅ повСрхности с этим идСальной Ρ‡Π΅Ρ€Π½ΠΎΠΉ повСрхности. Π£Ρ€Π°Π²Π½Π΅Π½ΠΈΠ΅ становится

ΠšΠΎΡΡ„Ρ„ΠΈΡ†ΠΈΠ΅Π½Ρ‚ излучСния колСблСтся ΠΎΡ‚ 0 Π΄ΠΎ 1; e = 1 для идСального Ρ€Π°Π΄ΠΈΠ°Ρ‚ΠΎΡ€Π° ΠΈ абсорбСра (Ρ‡Π΅Ρ€Π½ΠΎΠ΅ Ρ‚Π΅Π»ΠΎ) ΠΈ e = 0 для идСального Ρ€Π°Π΄ΠΈΠ°Ρ‚ΠΎΡ€Π°.КоТа Ρ‡Π΅Π»ΠΎΠ²Π΅ΠΊΠ°, для Π½Π°ΠΏΡ€ΠΈΠΌΠ΅Ρ€, нСзависимо ΠΎΡ‚ ΠΏΠΈΠ³ΠΌΠ΅Π½Ρ‚Π°Ρ†ΠΈΠΈ, ΠΈΠΌΠ΅Π΅Ρ‚ коэффициСнт излучСния ΠΎΠΊΠΎΠ»ΠΎ 0,97 Π² инфракрасной части спСктр. Π’ Ρ‚ΠΎ врСмя ΠΊΠ°ΠΊ ΠΏΠΎΠ»ΠΈΡ€ΠΎΠ²Π°Π½Π½Ρ‹ΠΉ алюминий ΠΈΠΌΠ΅Π΅Ρ‚ коэффициСнт излучСния ΠΎΠΊΠΎΠ»ΠΎ 0,05.

Π’Π΅ΠΏΠ»ΠΎΠ²ΠΎΠΉ ΠΈΠ·Π»ΡƒΡ‡Π΅Π½ΠΈΠ΅ ΠΎΡ‚ Ρ‚Π΅Π»Π° ΠΈΡΠΏΠΎΠ»ΡŒΠ·ΡƒΠ΅Ρ‚ΡΡ ΠΊΠ°ΠΊ диагностичСский инструмСнт Π² ΠΌΠ΅Π΄ΠΈΡ†ΠΈΠ½Π°. Π’Π΅Ρ€ΠΌΠΎΠ³Ρ€Π°ΠΌΠΌΠ° ΠΏΠΎΠΊΠ°Π·Ρ‹Π²Π°Π΅Ρ‚, являСтся Π»ΠΈ ΠΎΠ΄Π½Π° ΠΎΠ±Π»Π°ΡΡ‚ΡŒ ΠΈΠ·Π»ΡƒΡ‡Π°Π΅Ρ‚ большС Ρ‚Π΅ΠΏΠ»Π°, Ρ‡Π΅ΠΌ Π΄ΠΎΠ»ΠΆΠ½ΠΎ, Ρ‡Ρ‚ΠΎ ΡƒΠΊΠ°Π·Ρ‹Π²Π°Π΅Ρ‚ Π½Π° Π±ΠΎΠ»Π΅Π΅ Π²Ρ‹ΡΠΎΠΊΡƒΡŽ Ρ‚Π΅ΠΌΠΏΠ΅Ρ€Π°Ρ‚ΡƒΡ€Π° ΠΈΠ·-Π·Π° аномальной ΠΊΠ»Π΅Ρ‚ΠΎΡ‡Π½ΠΎΠΉ активности.ВСрмография ΠΈΠ»ΠΈ Ρ‚Π΅ΠΏΠ»ΠΎΠ²ΠΈΠ΄Π΅Π½ΠΈΠ΅ Π² ΠΌΠ΅Π΄ΠΈΡ†ΠΈΠ½Π΅ основано Π½Π° СстСствСнноС Ρ‚Π΅ΠΏΠ»ΠΎΠ²ΠΎΠ΅ ΠΈΠ·Π»ΡƒΡ‡Π΅Π½ΠΈΠ΅ ΠΊΠΎΠΆΠΈ. НаибольшСС прСимущСство сияниС, свободноС ΠΎΡ‚ ΠΏΡ€ΠΈΠ½Ρ†ΠΈΠΏΠ° измСрСния.

ΠžΠΏΡ€Π΅Π΄Π΅Π»Π΅Π½Π½Ρ‹ΠΉ области Ρ‚Π΅Π»Π° ΠΈΠΌΠ΅ΡŽΡ‚ Ρ€Π°Π·Π½Ρ‹ΠΉ Ρ‚Π΅ΠΌΠΏΠ΅Ρ€Π°Ρ‚ΡƒΡ€Π½Ρ‹ΠΉ Ρ€Π΅ΠΆΠΈΠΌ. Если ΠΎΠ΄ΠΈΠ½ ΠΎΠ±Π½Π°ΠΆΠ°Π΅Ρ‚ Ρ‚Π΅Π»ΠΎ, Π½Π°ΠΏΡ€ΠΈΠΌΠ΅Ρ€ ΠΊ ΠΎΡ…Π»Π°ΠΆΠ΄Π°ΡŽΡ‰Π΅ΠΌΡƒ Π°Ρ‚Ρ‚Ρ€Π°ΠΊΡ†ΠΈΠΎΠ½Ρƒ, Ρ‚ΠΎ Π·ΠΎΠ½Ρ‹ Ρ‚Π΅Π»Π° ΠΊΠΎΠΆΠΈ Ρ€Π΅Π°Π³ΠΈΡ€ΡƒΡŽΡ‚, Ρ‡Ρ‚ΠΎΠ±Ρ‹ Π²ΠΎΡΡΡ‚Π°Π½ΠΎΠ²ΠΈΡ‚ΡŒ Ρ‚Π΅ΠΏΠ»ΠΎΠ²ΠΎΠΉ баланс Ρ‚Π΅Π»Π°. Π’Π΅ΠΌ самым Ρ‚Π΅Ρ€ΠΌΠΎΡ€Π΅Π³ΡƒΠ»ΠΈΡ€ΠΎΠ²Π°Π½ΠΈΠ΅ ΠΏΠΎΡ€Π°ΠΆΠ΅Π½Π½Ρ‹Ρ… участков Ρ‚Π΅Π»Π° ΠΈ ΠΎΡ€Π³Π°Π½ΠΎΠ² отличаСтся ΠΎΡ‚ Π·Π΄ΠΎΡ€ΠΎΠ²Ρ‹ΠΉ.Π’Π°ΠΊ называСмая «Ρ€Π΅Π³ΡƒΠ»ΡΡ†ΠΈΠΎΠ½Π½Π°Ρ тСрмография» основанный Π½Π° этом ΠΏΡ€ΠΈΠ½Ρ†ΠΈΠΏΠ΅.

Π‘Π²ΠΎΠ΄ΠΊΠ°

Π€ΠΎΡ‚ΠΎ: УнивСрситСт Висконсина

Π’Π΅ΠΏΠ»ΠΎΠΎΠ±ΠΌΠ΅Π½

Бколько Ρ‚Π΅ΠΏΠ»Π° трСбуСтся для заТигания лСсного Ρ‚ΠΎΠΏΠ»ΠΈΠ²Π°? Π Π°ΡΡ‚ΠΈΡ‚Π΅Π»ΡŒΠ½Ρ‹ΠΉ ΠΌΠ°Ρ‚Π΅Ρ€ΠΈΠ°Π», Ρ‚Π°ΠΊΠΎΠΉ ΠΊΠ°ΠΊ лСсноС Ρ‚ΠΎΠΏΠ»ΠΈΠ²ΠΎ, воспламСняСтся ΠΏΡ€ΠΈ ΠΎΡ‚Π½ΠΎΡΠΈΡ‚Π΅Π»ΡŒΠ½ΠΎ Π½ΠΈΠ·ΠΊΠΈΡ… Ρ‚Π΅ΠΌΠΏΠ΅Ρ€Π°Ρ‚ΡƒΡ€Π°Ρ…. Ρ‚Π΅ΠΌΠΏΠ΅Ρ€Π°Ρ‚ΡƒΡ€Ρ‹ ΠΏΡ€ΠΈ условии Π½ΠΈΠ·ΠΊΠΎΠ³ΠΎ содСрТания Π²Π»Π°Π³ΠΈ Π² Ρ‚ΠΎΠΏΠ»ΠΈΠ²Π΅ ΠΈ подвСргаСтся Π²ΠΎΠ·Π΄Π΅ΠΉΡΡ‚Π²ΠΈΡŽ Π²ΠΎΠ·Π΄ΡƒΡ…Π°, Ρ‚Π°ΠΊ Ρ‡Ρ‚ΠΎ доступно достаточноС количСство кислорода.ЀактичСская ΠΏΠΎΡ‚Ρ€Π΅Π±Π½ΠΎΡΡ‚ΡŒ Π² Ρ‚Π΅ΠΏΠ»Π΅ для воспламСнСния Ρ‚ΠΎΠΏΠ»ΠΈΠ²Π° ΠΈΠ· ΠΌΠ΅Ρ€Ρ‚Π²ΠΎΠ³ΠΎ лСса Π²Π°Ρ€ΡŒΠΈΡ€ΡƒΠ΅Ρ‚ΡΡ ΠΎΡ‚ ΠΎΡ‚ 500 Π΄ΠΎ 750 F. МногиС ΠΎΠ±Ρ‹Ρ‡Π½Ρ‹Π΅ источники воспламСнСния обСспСчат достаточно Ρ‚Π΅ΠΏΠ»Π°, Π²ΠΊΠ»ΡŽΡ‡Π°Ρ Π³ΠΎΡ€ΡΡ‰ΡƒΡŽ спичку ΠΈ Π΄Π°ΠΆΠ΅ Ρ‚Π»Π΅ΡŽΡ‰Π°Ρ сигарСта ΠΏΡ€ΠΈ ΠΊΠΎΠ½Ρ‚Π°ΠΊΡ‚Π΅ с сухим мСлкодиспСрсным Ρ‚ΠΎΠΏΠ»ΠΈΠ²ΠΎΠΌ.

Π’Π•ΠœΠŸΠ•Π ΠΠ’Π£Π Π Π—ΠΠ–Π˜Π“ΠΠΠ˜Π― Π‘Π£Π₯ΠžΠ“Πž Π›Π•Π‘ΠΠžΠ“Πž Π’ΠžΠŸΠ›Π˜Π’Π = 500-750 F.

ΠœΡ‹ Π·Π½Π°Π΅ΠΌ ΠΎ ΠΌΠ½ΠΎΠ³ΠΈΡ… ΠΌΠ΅Ρ‚ΠΎΠ΄Π°Ρ…, с ΠΏΠΎΠΌΠΎΡ‰ΡŒΡŽ ΠΊΠΎΡ‚ΠΎΡ€Ρ‹Ρ… для Π½Π°Ρ‡Π°Π»Π° Ρ€Π°Π±ΠΎΡ‚Ρ‹ с лСсным Ρ‚ΠΎΠΏΠ»ΠΈΠ²ΠΎΠΌ ΠΌΠΎΠΆΠ½ΠΎ ΠΈΡΠΏΠΎΠ»ΡŒΠ·ΠΎΠ²Π°Ρ‚ΡŒ Ρ‚Π΅ΠΏΠ»ΠΎ. процСсс горСния; Π½ΠΎ ΠΊΠ°ΠΊ процСсс продолТаСтся? Огонь распространяСтся Π·Π° счСт ΠΏΠ΅Ρ€Π΅Π΄Π°Ρ‡ΠΈ Ρ‚Π΅ΠΏΠ»ΠΎΠ²ΠΎΠΉ энСргии трСмя способами: Радиация, конвСкция ΠΈ ΠŸΡ€ΠΎΠ²Π΅Π΄Π΅Π½ΠΈΠ΅.

Радиация

Радиация ΠΎΠ·Π½Π°Ρ‡Π°Π΅Ρ‚ ΠΈΠ·Π»ΡƒΡ‡Π΅Π½ΠΈΠ΅ энСргии Π² Π²ΠΈΠ΄Π΅ Π»ΡƒΡ‡Π΅ΠΉ ΠΈΠ»ΠΈ Π²ΠΎΠ»Π½. Π’Π΅ΠΏΠ»ΠΎ двиТСтся Π² пространствС Π² Π²ΠΈΠ΄Π΅ энСргСтичСских Π²ΠΎΠ»Π½. Π­Ρ‚ΠΎ Ρ‚ΠΎΡ‚ ΠΆΠ°Ρ€, ΠΊΠΎΡ‚ΠΎΡ€Ρ‹ΠΉ Ρ‡ΡƒΠ²ΡΡ‚Π²ΡƒΠ΅ΡˆΡŒ, сидя ΠΏΠ΅Ρ€Π΅Π΄ ΠΊΠ°ΠΌΠΈΠ½ΠΎΠΌ ΠΈΠ»ΠΈ Π²ΠΎΠΊΡ€ΡƒΠ³ костра. Он ΠΏΡƒΡ‚Π΅ΡˆΠ΅ΡΡ‚Π²ΡƒΠ΅Ρ‚ Π² прямыС со ΡΠΊΠΎΡ€ΠΎΡΡ‚ΡŒΡŽ свСта. Π­Ρ‚ΠΎΡ‚ это ΠΏΡ€ΠΈΡ‡ΠΈΠ½Π° Ρ‚ΠΎΠ³ΠΎ, Ρ‡Ρ‚ΠΎ ΠΏΡ€ΠΈ столкновСнии с ΠΎΠ³Π½Π΅ΠΌ согрСваСтся Ρ‚ΠΎΠ»ΡŒΠΊΠΎ фасад. Π—Π°Π΄ Π½Π΅ грССтся, ΠΏΠΎΠΊΠ° Ρ‡Π΅Π»ΠΎΠ²Π΅ΠΊ Π½Π΅ обСрнСтся. ЗСмля нагрСваСтся солнцСм Π·Π° счСт излучСния. Π‘ΠΎΠ»Π½Π΅Ρ‡Π½Ρ‹Π΅ ΠΎΠΆΠΎΠ³ΠΈ — это Ρ„Π°ΠΊΡ‚ ΠΆΠΈΠ·Π½ΠΈ, ΠΊΠΎΠ³Π΄Π° люди ΠΏΠΎΠ΄Π²Π΅Ρ€Π³Π°ΡŽΡ‚ΡΡ ΠΎΡ‡Π΅Π½ΡŒ ΡΠΈΠ»ΡŒΠ½ΠΎΠΌΡƒ Π²ΠΎΠ·Π΄Π΅ΠΉΡΡ‚Π²ΠΈΡŽ солнца. Π΄Π»ΠΈΠ½Π½Ρ‹ΠΉ.Π‘ΠΎΠ»ΡŒΡˆΠ°Ρ Ρ‡Π°ΡΡ‚ΡŒ ΠΏΠΎΠ΄ΠΎΠ³Ρ€Π΅Π²Π° Ρ‚ΠΎΠΏΠ»ΠΈΠ²Π° ΠΏΠ΅Ρ€Π΅Π΄ Π²ΠΎΠ·Π³ΠΎΡ€Π°Π½ΠΈΠ΅ΠΌ происходит Π·Π° счСт ΠΈΠ·Π»ΡƒΡ‡Π΅Π½ΠΈΠ΅ Ρ‚Π΅ΠΏΠ»Π° ΠΎΡ‚ огня. Π’ Π²ΠΈΠ΄Π΅ Ρ„Ρ€ΠΎΠ½Ρ‚ огня приблиТаСтся, количСство ΠΏΠΎΠ»ΡƒΡ‡Π°Π΅ΠΌΠΎΠ³ΠΎ лучистого Ρ‚Π΅ΠΏΠ»Π° увСличиваСтся.

ΠšΠΎΠ½Π²Π΅ΠΊΡ†ΠΈΡ

ΠšΠΎΠ½Π²Π΅ΠΊΡ†ΠΈΡ — это ΠΏΠ΅Ρ€Π΅Π΄Π°Ρ‡Π° Ρ‚Π΅ΠΏΠ»Π° ΠΏΡ€ΠΈ физичСском Π΄Π²ΠΈΠΆΠ΅Π½ΠΈΠΈ горячих масс. Π²ΠΎΠ·Π΄ΡƒΡ…Π°. Как Π²ΠΎΠ·Π΄ΡƒΡ… нагрСтая, ΠΎΠ½Π° Ρ€Π°ΡΡˆΠΈΡ€ΡΠ΅Ρ‚ΡΡ (ΠΊΠ°ΠΊ ΠΈ всС ΠΏΡ€Π΅Π΄ΠΌΠ΅Ρ‚Ρ‹). По ΠΌΠ΅Ρ€Π΅ Ρ€Π°ΡΡˆΠΈΡ€Π΅Π½ΠΈΡ ΠΎΠ½ становится свСтлСС ΠΎΠΊΡ€ΡƒΠΆΠ°ΡŽΡ‰Π΅Π³ΠΎ Π²ΠΎΠ·Π΄ΡƒΡ…Π° ΠΈ поднимаСтся Π²Π²Π΅Ρ€Ρ…. (Π’ΠΎΡ‚ ΠΏΠΎΡ‡Π΅ΠΌΡƒ Π²ΠΎΠ·Π΄ΡƒΡ… ΠΏΠΎΠ΄ ΠΏΠΎΡ‚ΠΎΠ»ΠΊΠΎΠΌ ΠΎΡ‚Π°ΠΏΠ»ΠΈΠ²Π°Π΅ΠΌΠΎΠ³ΠΎ помСщСния Ρ‚Π΅ΠΏΠ»Π΅Π΅, Ρ‡Π΅ΠΌ Ρ‡Ρ‚ΠΎ Π²ΠΎΠ·Π»Π΅ ΠΏΠΎΠ»Π°.) ΠšΡƒΠ»Π΅Ρ€ Π²ΠΎΠ·Π΄ΡƒΡ… врываСтся с Π±ΠΎΠΊΠΎΠ². это грССтся ΠΏΠΎ ΠΎΡ‡Π΅Ρ€Π΅Π΄ΠΈ ΠΈ Ρ‚ΠΎΠΆΠ΅ поднимаСтся. Π‘ΠΊΠΎΡ€ΠΎ Π½Π°Π΄ ΠΎΠ³Π½Π΅ΠΌ образуСтся конвСкционная ΠΊΠΎΠ»ΠΎΠ½Π½Π°, ΠΊΠΎΡ‚ΠΎΡ€ΡƒΡŽ Π²ΠΈΠ΄Π½ΠΎ ΠΏΠΎ Π΄Ρ‹ΠΌΡƒ, возносится Π² Π½Π΅ΠΌ. Π­Ρ‚ΠΎΡ‚ ΠΏΡ€ΠΈΡ‚ΠΎΠΊ Π±ΠΎΠ»Π΅Π΅ Ρ…ΠΎΠ»ΠΎΠ΄Π½ΠΎΠ³ΠΎ Π²ΠΎΠ·Π΄ΡƒΡ…Π° сбоку ΠΏΠΎΠΌΠΎΠ³Π°Π΅Ρ‚ ΠΏΠΎΠ΄Π°Π²Π°Ρ‚ΡŒ Π΄ΠΎΠΏΠΎΠ»Π½ΠΈΡ‚Π΅Π»ΡŒΠ½Ρ‹ΠΉ кислород для процСсс горСния для продолТСния.

ΠŸΡ€ΠΎΠ²ΠΎΠ΄ΠΈΠΌΠΎΡΡ‚ΡŒ

ΠŸΡ€ΠΎΠ²ΠΎΠ΄ΠΈΠΌΠΎΡΡ‚ΡŒ — это ΠΏΠ΅Ρ€Π΅Π΄Π°Ρ‡Π° Ρ‚Π΅ΠΏΠ»Π° Π²Π½ΡƒΡ‚Ρ€ΠΈ самого ΠΌΠ°Ρ‚Π΅Ρ€ΠΈΠ°Π»Π°. Π‘ΠΎΠ»ΡŒΡˆΠΈΠ½ΡΡ‚Π²ΠΎ ΠΌΠ΅Ρ‚Π°Π»Π»ΠΎΠ² ΡΠ²Π»ΡΡŽΡ‚ΡΡ Ρ…ΠΎΡ€ΠΎΡˆΠΈΠΌΠΈ ΠΏΡ€ΠΎΠ²ΠΎΠ΄Π½ΠΈΠΊΠ°ΠΌΠΈ Ρ‚Π΅ΠΏΠ»Π°. Π”Π΅Ρ€Π΅Π²ΠΎ — ΠΎΡ‡Π΅Π½ΡŒ ΠΏΠ»ΠΎΡ…ΠΎΠΉ ΠΏΡ€ΠΎΠ²ΠΎΠ΄Π½ΠΈΠΊ, поэтому ΠΎΡ‡Π΅Π½ΡŒ ΠΌΠ΅Π΄Π»Π΅Π½Π½ΠΎ ΠΏΠ΅Ρ€Π΅Π΄Π°Π΅Ρ‚ Ρ‚Π΅ΠΏΠ»ΠΎ. Π­Ρ‚ΠΎ ΠΌΠΎΠΆΠ½ΠΎ ΠΏΡ€ΠΎΠΈΠ»Π»ΡŽΡΡ‚Ρ€ΠΈΡ€ΠΎΠ²Π°Ρ‚ΡŒ Ρ‚Π΅ΠΌ, Ρ‡Ρ‚ΠΎ дСрСвянная Ρ€ΡƒΡ‡ΠΊΠ° ΠΏΡ€ΠΈ ΠΆΠ°Ρ€ΠΊΠ΅ сковорода остаСтся достаточно Ρ…ΠΎΠ»ΠΎΠ΄Π½ΠΎΠΉ, Ρ‡Ρ‚ΠΎΠ±Ρ‹ Π΅Π΅ ΠΌΠΎΠΆΠ½ΠΎ Π±Ρ‹Π»ΠΎ Π΄Π΅Ρ€ΠΆΠ°Ρ‚ΡŒ Π³ΠΎΠ»Ρ‹ΠΌΠΈ Ρ€ΡƒΠΊΠ°ΠΌΠΈ. ΠŸΡ€ΠΎΠ²Π΅Π΄Π΅Π½ΠΈΠ΅ Π½Π΅ являСтся Π²Π°ΠΆΠ½Ρ‹ΠΌ Ρ„Π°ΠΊΡ‚ΠΎΡ€ΠΎΠΌ распространСния лСсных ΠΏΠΎΠΆΠ°Ρ€ΠΎΠ².

ДСмонстрация

Π—Π°ΠΆΠ³ΠΈΡ‚Π΅ свСчу снова, ΠΊΠΎΡ‚ΠΎΡ€ΡƒΡŽ ΠΌΡ‹ использовали Π² ΠΏΡ€Π΅Π΄Ρ‹Π΄ΡƒΡ‰Π΅ΠΉ дСмонстрации. (ΠžΠ±Ρ€Π°Ρ‚ΠΈΡ‚Π΅ Π²Π½ΠΈΠΌΠ°Π½ΠΈΠ΅, Ρ‡Ρ‚ΠΎ Π²Ρ‹ ΠΌΠΎΠΆΠ΅Ρ‚Π΅ Π΄Π΅Ρ€ΠΆΠ°Ρ‚ΡŒ спичку, ΠΏΠΎΠΊΠ° Π΄Ρ€ΡƒΠ³ΠΎΠΉ ΠΊΠΎΠ½Π΅Ρ† Π³ΠΎΡ€ΠΈΡ‚, ΠΏΠΎΡ‚ΠΎΠΌΡƒ Ρ‡Ρ‚ΠΎ Π΄Π΅Ρ€Π΅Π²ΠΎ Π½Π΅ Ρ…ΠΎΡ€ΠΎΡˆΠΈΠΉ Π΄ΠΈΡ€ΠΈΠΆΠ΅Ρ€ Ρ‚Π΅ΠΏΠ»Π°.) Π’Π΅ΠΏΠ΅Ρ€ΡŒ протянитС Ρ€ΡƒΠΊΡƒ рядом со свСчой ΠΈ ΠΏΠΎΠ΄Π²ΠΈΠ½ΡŒΡ‚Π΅ Π΅Π΅ Π±Π»ΠΈΠΆΠ΅, ΠΏΠΎΠΊΠ° Ρ‚Π΅ΠΏΠ»ΠΎ ΠΌΠΎΠΆΠ½ΠΎ ΠΏΠΎΡ‡ΡƒΠ²ΡΡ‚Π²ΠΎΠ²Π°Ρ‚ΡŒ.Π’Π΅ΠΏΠ»ΠΎ ΠΎΡ‚ свСчи Π΄ΠΎΡ…ΠΎΠ΄ΠΈΡ‚ Π΄ΠΎ вашСй Ρ€ΡƒΠΊΠΈ. радиация. ΠŸΠΎΠ΄Π½Π΅ΡΠΈΡ‚Π΅ Ρ€ΡƒΠΊΡƒ Π±Π»ΠΈΠΆΠ΅ ΠΊ свСчС. Π§Ρ‚ΠΎ происходит с Ρ€ΡƒΠΊΠΎΠΉ? Π­Ρ‚ΠΎ становится Ρ‚Π΅ΠΏΠ»Π΅Π΅, ΠΏΠΎΡ‚ΠΎΠΌΡƒ Ρ‡Ρ‚ΠΎ лучистому Ρ‚Π΅ΠΏΠ»Ρƒ Π½Π΅ Π½ΡƒΠΆΠ½ΠΎ Π±Ρ‹Π»ΠΎ Ρ€Π°ΡΠΏΡ€ΠΎΡΡ‚Ρ€Π°Π½ΡΡ‚ΡŒΡΡ Ρ‚Π°ΠΊ Π΄Π°Π»Π΅ΠΊΠΎ. Π’Π΅ΠΏΠ΅Ρ€ΡŒ Π΄Π΅Ρ€ΠΆΠΈ ΠΏΠΎΠ»ΠΎΠΆΠΈΡ‚Π΅ Ρ€ΡƒΠΊΡƒ Π½Π° свСчу ΠΈ пСрСмСститС Π΅Π΅ ΠΊΠ°ΠΊ ΠΌΠΎΠΆΠ½ΠΎ Π±Π»ΠΈΠΆΠ΅. ΠœΠΎΠΆΠ΅Ρ‚Π΅ Π»ΠΈ Π²Ρ‹ Π΄Π΅Ρ€ΠΆΠ°Ρ‚ΡŒ это ΠΊΠ°ΠΊ Π·Π°ΠΊΡ€Ρ‹Ρ‚ΡŒ ΠΊΠ°ΠΊ ΠΌΠΎΠΆΠ½ΠΎ сбоку? Π’Ρ‹ Π½Π΅ ΠΌΠΎΠΆΠ΅Ρ‚Π΅ ΠΈΠ·-Π·Π° ΠΊΠΎΠ½Π²Π΅ΠΊΡ†ΠΈΠΎΠ½Π½ΠΎΠ³ΠΎ Π½Π°Π³Ρ€Π΅Π²Π° ΠΎΡ‚ свСчи Π² Π΄ΠΎΠΏΠΎΠ»Π½Π΅Π½ΠΈΠ΅ ΠΊ лучистому Ρ‚Π΅ΠΏΠ»Ρƒ.

1. Π’Ρ€ΠΈ способа ΠΏΠ΅Ρ€Π΅Π΄Π°Ρ‡ΠΈ Ρ‚Π΅ΠΏΠ»Π°: — Π²Ρ‹Π±Π΅Ρ€ΠΈΡ‚Π΅ ΠΎΡ‚Π²Π΅Ρ‚ -a. ΠΏΡ€ΠΎΠ²ΠΎΠ΄ΠΈΠΌΠΎΡΡ‚ΡŒ, ΠΈΠ·Π»ΡƒΡ‡Π΅Π½ΠΈΠ΅, конвСкция b.ΠΏΡ€ΠΎΠ²ΠΎΠ΄ΠΈΠΌΠΎΡΡ‚ΡŒ, конвСкция, свСрткаc. ΠΏΡ€ΠΎΠ²ΠΎΠ΄ΠΈΠΌΠΎΡΡ‚ΡŒ, ΠΎΡ‰ΡƒΠΏΡ‹Π²Π°Π½ΠΈΠ΅, ΠΈΠ·Π»ΡƒΡ‡Π΅Π½ΠΈΠ΅. кондукция, конвСкция, остаточная 7. Π’Π΅ΠΌΠΏΠ΅Ρ€Π°Ρ‚ΡƒΡ€Π° воспламСнСния сухого лСсного Ρ‚ΠΎΠΏΠ»ΠΈΠ²Π° находится Π² ΠΏΡ€Π΅Π΄Π΅Π»Π°Ρ… — Π²Ρ‹Π±Π΅Ρ€ΠΈΡ‚Π΅ ΠΎΡ‚Π²Π΅Ρ‚ -a. 400 ΠΈ 1000b. 500 ΠΈ 750 F500 ΠΈ 750 C1000 ΠΈ 1750

ΠšΠΎΠ½Π²Π΅ΠΊΡ‚ΠΈΠ²Π½Π°Ρ Ρ‚Π΅ΠΏΠ»ΠΎΠΏΠ΅Ρ€Π΅Π΄Π°Ρ‡Π°

ВСпловая энСргия, пСрСдаваСмая ΠΌΠ΅ΠΆΠ΄Ρƒ ΠΏΠΎΠ²Π΅Ρ€Ρ…Π½ΠΎΡΡ‚ΡŒΡŽ ΠΈ двиТущСйся ΠΆΠΈΠ΄ΠΊΠΎΡΡ‚ΡŒΡŽ с Ρ€Π°Π·Π½Ρ‹ΠΌΠΈ Ρ‚Π΅ΠΌΠΏΠ΅Ρ€Π°Ρ‚ΡƒΡ€Π°ΠΌΠΈ, извСстна ΠΊΠ°ΠΊ конвСкция .

На самом Π΄Π΅Π»Π΅ это комбинация Π΄ΠΈΡ„Ρ„ΡƒΠ·ΠΈΠΈ ΠΈ объСмного двиТСния ΠΌΠΎΠ»Π΅ΠΊΡƒΠ».Π’Π±Π»ΠΈΠ·ΠΈ повСрхности ΡΠΊΠΎΡ€ΠΎΡΡ‚ΡŒ Тидкости ΠΌΠ°Π»Π°, ΠΈ ΠΏΡ€Π΅ΠΎΠ±Π»Π°Π΄Π°Π΅Ρ‚ диффузия. На расстоянии ΠΎΡ‚ повСрхности объСмноС Π΄Π²ΠΈΠΆΠ΅Π½ΠΈΠ΅ усиливаСт влияниС ΠΈ ΠΏΡ€Π΅ΠΎΠ±Π»Π°Π΄Π°Π΅Ρ‚.

ΠšΠΎΠ½Π²Π΅ΠΊΡ‚ΠΈΠ²Π½Π°Ρ Ρ‚Π΅ΠΏΠ»ΠΎΠΏΠ΅Ρ€Π΅Π΄Π°Ρ‡Π° ΠΌΠΎΠΆΠ΅Ρ‚ Π±Ρ‹Ρ‚ΡŒ

  • ΠΏΡ€ΠΈΠ½ΡƒΠ΄ΠΈΡ‚Π΅Π»ΡŒΠ½Π°Ρ ΠΈΠ»ΠΈ Π²ΡΠΏΠΎΠΌΠΎΠ³Π°Ρ‚Π΅Π»ΡŒΠ½Π°Ρ конвСкция
  • СстСствСнная ΠΈΠ»ΠΈ свободная конвСкция

ΠŸΡ€ΠΈΠ½ΡƒΠ΄ΠΈΡ‚Π΅Π»ΡŒΠ½Π°Ρ ΠΈΠ»ΠΈ Π²ΡΠΏΠΎΠΌΠΎΠ³Π°Ρ‚Π΅Π»ΡŒΠ½Π°Ρ конвСкция

ΠŸΡ€ΠΈΠ½ΡƒΠ΄ΠΈΡ‚Π΅Π»ΡŒΠ½Π°Ρ конвСкция

ΠŸΡ€ΠΈΠ½ΡƒΠ΄ΠΈΡ‚Π΅Π»ΡŒΠ½Π°Ρ конвСкция Тидкости внСшняя сила, такая ΠΊΠ°ΠΊ насос, вСнтилятор ΠΈΠ»ΠΈ ΡΠΌΠ΅ΡΠΈΡ‚Π΅Π»ΡŒ.

ЕстСствСнная ΠΈΠ»ΠΈ свободная конвСкция

ЕстСствСнная конвСкция вызываСтся силами плавучСсти ΠΈΠ·-Π·Π° Ρ€Π°Π·Π½ΠΈΡ†Ρ‹ плотности, Π²Ρ‹Π·Π²Π°Π½Π½ΠΎΠΉ колСбаниями Ρ‚Π΅ΠΌΠΏΠ΅Ρ€Π°Ρ‚ΡƒΡ€Ρ‹ Π² Тидкости. ΠŸΡ€ΠΈ Π½Π°Π³Ρ€Π΅Π²Π°Π½ΠΈΠΈ ΠΈΠ·ΠΌΠ΅Π½Π΅Π½ΠΈΠ΅ плотности Π² ΠΏΠΎΠ³Ρ€Π°Π½ΠΈΡ‡Π½ΠΎΠΌ слоС ΠΏΡ€ΠΈΠ²Π΅Π΄Π΅Ρ‚ ΠΊ Ρ‚ΠΎΠΌΡƒ, Ρ‡Ρ‚ΠΎ ΠΆΠΈΠ΄ΠΊΠΎΡΡ‚ΡŒ поднимСтся ΠΈ Π±ΡƒΠ΄Π΅Ρ‚ Π·Π°ΠΌΠ΅Π½Π΅Π½Π° Π±ΠΎΠ»Π΅Π΅ Ρ…ΠΎΠ»ΠΎΠ΄Π½ΠΎΠΉ ΠΆΠΈΠ΄ΠΊΠΎΡΡ‚ΡŒΡŽ, которая Ρ‚Π°ΠΊΠΆΠ΅ Π±ΡƒΠ΄Π΅Ρ‚ Π½Π°Π³Ρ€Π΅Π²Π°Ρ‚ΡŒΡΡ ΠΈ ΠΏΠΎΠ΄Π½ΠΈΠΌΠ°Ρ‚ΡŒΡΡ. Π­Ρ‚ΠΎ ΠΏΡ€ΠΎΠ΄ΠΎΠ»ΠΆΠ°ΡŽΡ‰Π΅Π΅ΡΡ явлСниС называСтся свободной ΠΈΠ»ΠΈ СстСствСнной ΠΊΠΎΠ½Π²Π΅ΠΊΡ†ΠΈΠ΅ΠΉ.

ΠŸΡ€ΠΎΡ†Π΅ΡΡΡ‹ кипСния ΠΈΠ»ΠΈ кондСнсации Ρ‚Π°ΠΊΠΆΠ΅ Π½Π°Π·Ρ‹Π²Π°ΡŽΡ‚ ΠΊΠΎΠ½Π²Π΅ΠΊΡ‚ΠΈΠ²Π½Ρ‹ΠΌΠΈ процСссами Ρ‚Π΅ΠΏΠ»ΠΎΠΏΠ΅Ρ€Π΅Π΄Π°Ρ‡ΠΈ.

  • Π’Π΅ΠΏΠ»ΠΎΠΏΠ΅Ρ€Π΅Π΄Π°Ρ‡Π° Π½Π° Π΅Π΄ΠΈΠ½ΠΈΡ†Ρƒ повСрхности Π·Π° счСт ΠΊΠΎΠ½Π²Π΅ΠΊΡ†ΠΈΠΈ Π±Ρ‹Π»Π° Π²ΠΏΠ΅Ρ€Π²Ρ‹Π΅ описана ΠΡŒΡŽΡ‚ΠΎΠ½ΠΎΠΌ, ΠΈ это ΡΠΎΠΎΡ‚Π½ΠΎΡˆΠ΅Π½ΠΈΠ΅ извСстно ΠΊΠ°ΠΊ Π—Π°ΠΊΠΎΠ½ охлаТдСния ΠΡŒΡŽΡ‚ΠΎΠ½Π° .

Π£Ρ€Π°Π²Π½Π΅Π½ΠΈΠ΅ ΠΊΠΎΠ½Π²Π΅ΠΊΡ†ΠΈΠΈ ΠΌΠΎΠΆΠ΅Ρ‚ Π±Ρ‹Ρ‚ΡŒ Π²Ρ‹Ρ€Π°ΠΆΠ΅Π½ΠΎ ΠΊΠ°ΠΊ:

q = h c A dT (1)

Π³Π΄Π΅

q = Ρ‚Π΅ΠΏΠ»ΠΎΠΏΠ΅Ρ€Π΅Π΄Π°Ρ‡Π° Π·Π° Π΅Π΄ΠΈΠ½ΠΈΡ†Ρƒ Π²Ρ€Π΅ΠΌΠ΅Π½ΠΈ (Π’Ρ‚, Π‘Π’Π• / Ρ‡)

A = ΠΏΠ»ΠΎΡ‰Π°Π΄ΡŒ Ρ‚Π΅ΠΏΠ»ΠΎΠΎΠ±ΠΌΠ΅Π½Π° повСрхности (ΠΌ 2 , Ρ„ΡƒΡ‚ 2 )

Ρ‡ c = коэффициСнт ΠΊΠΎΠ½Π²Π΅ΠΊΡ‚ΠΈΠ²Π½ΠΎΠΉ Ρ‚Π΅ΠΏΠ»ΠΎΠΏΠ΅Ρ€Π΅Π΄Π°Ρ‡ΠΈ процСсса ( Π’Ρ‚ / (ΠΌ 2o C, Btu / (Ρ„ΡƒΡ‚ 2 h o F) )

dT = Ρ€Π°Π·Π½ΠΈΡ†Π° Ρ‚Π΅ΠΌΠΏΠ΅Ρ€Π°Ρ‚ΡƒΡ€ ΠΌΠ΅ΠΆΠ΄Ρƒ ΠΏΠΎΠ²Π΅Ρ€Ρ…Π½ΠΎΡΡ‚ΡŒΡŽ ΠΈ основной ΠΆΠΈΠ΄ΠΊΠΎΡΡ‚ΡŒΡŽ ( o C, F)

ΠšΠΎΡΡ„Ρ„ΠΈΡ†ΠΈΠ΅Π½Ρ‚Ρ‹ Ρ‚Π΅ΠΏΠ»ΠΎΠΏΠ΅Ρ€Π΅Π΄Π°Ρ‡ΠΈ — Π΅Π΄ΠΈΠ½ΠΈΡ†Ρ‹

ΠšΠΎΡΡ„Ρ„ΠΈΡ†ΠΈΠ΅Π½Ρ‚Ρ‹ ΠΊΠΎΠ½Π²Π΅ΠΊΡ‚ΠΈΠ²Π½ΠΎΠΉ Ρ‚Π΅ΠΏΠ»ΠΎΠΏΠ΅Ρ€Π΅Π΄Π°Ρ‡ΠΈ

ΠšΠΎΡΡ„Ρ„ΠΈΡ†ΠΈΠ΅Π½Ρ‚Ρ‹ ΠΊΠΎΠ½Π²Π΅ΠΊΡ‚ΠΈΠ²Π½ΠΎΠΉ Ρ‚Π΅ΠΏΠ»ΠΎΠΏΠ΅Ρ€Π΅Π΄Π°Ρ‡ΠΈ — Ρ‡ c Π² зависимости ΠΎΡ‚ t Ρ‚ΠΈΠΏ срСды, Π±ΡƒΠ΄ΡŒ Ρ‚ΠΎ Π³Π°Π· ΠΈΠ»ΠΈ ΠΆΠΈΠ΄ΠΊΠΎΡΡ‚ΡŒ, ΠΈ свойства ΠΏΠΎΡ‚ΠΎΠΊΠ°, Ρ‚Π°ΠΊΠΈΠ΅ ΠΊΠ°ΠΊ ΡΠΊΠΎΡ€ΠΎΡΡ‚ΡŒ, Π²ΡΠ·ΠΊΠΎΡΡ‚ΡŒ ΠΈ Π΄Ρ€ΡƒΠ³ΠΈΠ΅ свойства, зависящиС ΠΎΡ‚ ΠΏΠΎΡ‚ΠΎΠΊΠ° ΠΈ Ρ‚Π΅ΠΌΠΏΠ΅Ρ€Π°Ρ‚ΡƒΡ€Ρ‹.

Π’ΠΈΠΏΠΈΡ‡Π½Ρ‹Π΅ коэффициСнты ΠΊΠΎΠ½Π²Π΅ΠΊΡ‚ΠΈΠ²Π½ΠΎΠΉ Ρ‚Π΅ΠΏΠ»ΠΎΠΏΠ΅Ρ€Π΅Π΄Π°Ρ‡ΠΈ для Π½Π΅ΠΊΠΎΡ‚ΠΎΡ€Ρ‹Ρ… распространСнных ΠΏΡ€ΠΈΠΌΠ΅Π½Π΅Π½ΠΈΠΉ ΠΏΠΎΡ‚ΠΎΠΊΠ° Тидкости:

  • Бвободная конвСкция — Π²ΠΎΠ·Π΄ΡƒΡ…, Π³Π°Π·Ρ‹ ΠΈ сухиС ΠΏΠ°Ρ€Ρ‹: 0,5 — 1000 (Π’Ρ‚ / (ΠΌ 2 K))
  • Бвободная конвСкция — Π²ΠΎΠ΄Π° ΠΈ Тидкости: 50 — 3000 (Π’Ρ‚ / (ΠΌ 2 K))
  • ΠŸΡ€ΠΈΠ½ΡƒΠ΄ΠΈΡ‚Π΅Π»ΡŒΠ½Π°Ρ конвСкция — Π²ΠΎΠ·Π΄ΡƒΡ…, Π³Π°Π·Ρ‹ ΠΈ сухиС ΠΏΠ°Ρ€Ρ‹: 10 — 1000 (Π’Ρ‚ / (ΠΌ 2 K))
  • ΠŸΡ€ΠΈΠ½ΡƒΠ΄ΠΈΡ‚Π΅Π»ΡŒΠ½Π°Ρ конвСкция — Π²ΠΎΠ΄Π° ΠΈ Тидкости: 50 — 10000 (Π’Ρ‚ / (ΠΌ 2 K))
  • ΠŸΡ€ΠΈΠ½ΡƒΠ΄ΠΈΡ‚Π΅Π»ΡŒΠ½Π°Ρ конвСкция — ΠΆΠΈΠ΄ΠΊΠΈΠ΅ ΠΌΠ΅Ρ‚Π°Π»Π»Ρ‹: 5000 — 40000 (Π’Ρ‚ / (ΠΌ 2 K))
  • ΠšΠΈΠΏΡΡ‰Π°Ρ Π²ΠΎΠ΄Π°: 3.000 — 100,000 (Π’Ρ‚ / (ΠΌ 2 K))
  • Водяной кондСнсат: 5.000 — 100,000 (Π’Ρ‚ / (ΠΌ 2 K))
ΠšΠΎΡΡ„Ρ„ΠΈΡ†ΠΈΠ΅Π½Ρ‚ ΠΊΠΎΠ½Π²Π΅ΠΊΡ‚ΠΈΠ²Π½ΠΎΠΉ Ρ‚Π΅ΠΏΠ»ΠΎΠΏΠ΅Ρ€Π΅Π΄Π°Ρ‡ΠΈ для Π²ΠΎΠ·Π΄ΡƒΡ…Π°

ΠšΠΎΡΡ„Ρ„ΠΈΡ†ΠΈΠ΅Π½Ρ‚ ΠΊΠΎΠ½Π²Π΅ΠΊΡ‚ΠΈΠ²Π½ΠΎΠΉ Ρ‚Π΅ΠΏΠ»ΠΎΠΏΠ΅Ρ€Π΅Π΄Π°Ρ‡ΠΈ для ΠΏΠΎΡ‚ΠΎΠΊΠ° Π²ΠΎΠ·Π΄ΡƒΡ…Π° ΠΌΠΎΠΆΠ΅Ρ‚ Π±Ρ‹Ρ‚ΡŒ ΠΏΡ€ΠΈΠ±Π»ΠΈΠ·ΠΈΡ‚Π΅Π»ΡŒΠ½ΠΎ Ρ€Π°Π²Π΅Π½

Ρ‡ c = 10,45 — v + 10 v 1/2 (2)

Π³Π΄Π΅

h c = коэффициСнт Ρ‚Π΅ΠΏΠ»ΠΎΠΏΠ΅Ρ€Π΅Π΄Π°Ρ‡ΠΈ (кКал / ΠΌ 2 Ρ‡ Β° C)

v = ΠΎΡ‚Π½ΠΎΡΠΈΡ‚Π΅Π»ΡŒΠ½Π°Ρ ΡΠΊΠΎΡ€ΠΎΡΡ‚ΡŒ ΠΌΠ΅ΠΆΠ΄Ρƒ ΠΏΠΎΠ²Π΅Ρ€Ρ…Π½ΠΎΡΡ‚ΡŒΡŽ ΠΎΠ±ΡŠΠ΅ΠΊΡ‚Π° ΠΈ Π²ΠΎΠ·Π΄ΡƒΡ…ΠΎΠΌ (ΠΌ / с)

Начиная с

1 ΠΊΠΊΠ°Π» / ΠΌ 2 час Β° Π‘ = 1.16 Π’Ρ‚ / ΠΌ 2 Β° C

— (2) ΠΌΠΎΠΆΠ½ΠΎ ΠΈΠ·ΠΌΠ΅Π½ΠΈΡ‚ΡŒ Π½Π°

Ρ‡ cW = 12,12 — 1,16 Π² + 11,6 Π² 1/2 (2b)

Π³Π΄Π΅

h cW = коэффициСнт Ρ‚Π΅ΠΏΠ»ΠΎΠΏΠ΅Ρ€Π΅Π΄Π°Ρ‡ΠΈ (Π’Ρ‚ / ΠΌ 2 Β° C )

ΠŸΡ€ΠΈΠΌΠ΅Ρ‡Π°Π½ΠΈΠ΅! — это эмпиричСскоС ΡƒΡ€Π°Π²Π½Π΅Π½ΠΈΠ΅, ΠΊΠΎΡ‚ΠΎΡ€ΠΎΠ΅ ΠΌΠΎΠΆΠ΅Ρ‚ ΠΈΡΠΏΠΎΠ»ΡŒΠ·ΠΎΠ²Π°Ρ‚ΡŒΡΡ для скоростСй ΠΎΡ‚ 2 Π΄ΠΎ 20 ΠΌ / с .

ΠŸΡ€ΠΈΠΌΠ΅Ρ€ — конвСктивная Ρ‚Π΅ΠΏΠ»ΠΎΠΏΠ΅Ρ€Π΅Π΄Π°Ρ‡Π°

Π–ΠΈΠ΄ΠΊΠΎΡΡ‚ΡŒ Ρ‚Π΅Ρ‡Π΅Ρ‚ ΠΏΠΎ плоской повСрхности 1 ΠΌ Π½Π° 1 ΠΌ. Π’Π΅ΠΌΠΏΠ΅Ρ€Π°Ρ‚ΡƒΡ€Π° повСрхности 50 o C , Ρ‚Π΅ΠΌΠΏΠ΅Ρ€Π°Ρ‚ΡƒΡ€Π° Тидкости 20 o C ΠΈ коэффициСнт ΠΊΠΎΠ½Π²Π΅ΠΊΡ‚ΠΈΠ²Π½ΠΎΠΉ Ρ‚Π΅ΠΏΠ»ΠΎΠΏΠ΅Ρ€Π΅Π΄Π°Ρ‡ΠΈ 2000 Π’Ρ‚ / ΠΌ 2o Π‘ . ΠšΠΎΠ½Π²Π΅ΠΊΡ‚ΠΈΠ²Π½Ρ‹ΠΉ Ρ‚Π΅ΠΏΠ»ΠΎΠΎΠ±ΠΌΠ΅Π½ ΠΌΠ΅ΠΆΠ΄Ρƒ Π±ΠΎΠ»Π΅Π΅ горячСй ΠΏΠΎΠ²Π΅Ρ€Ρ…Π½ΠΎΡΡ‚ΡŒΡŽ ΠΈ Π±ΠΎΠ»Π΅Π΅ Ρ…ΠΎΠ»ΠΎΠ΄Π½Ρ‹ΠΌ Π²ΠΎΠ·Π΄ΡƒΡ…ΠΎΠΌ ΠΌΠΎΠΆΠ½ΠΎ Ρ€Π°ΡΡΡ‡ΠΈΡ‚Π°Ρ‚ΡŒ ΠΊΠ°ΠΊ

q = (2000 Π’Ρ‚ / (ΠΌ 2o C)) ((1 ΠΌ) (1 ΠΌ)) ((50 o C) — (20 o C))

= 60000 (Π’Ρ‚)

= 60 (ΠΊΠ’Ρ‚)

ΠšΠ°Π»ΡŒΠΊΡƒΠ»ΡΡ‚ΠΎΡ€ ΠΊΠΎΠ½Π²Π΅ΠΊΡ‚ΠΈΠ²Π½ΠΎΠΉ Ρ‚Π΅ΠΏΠ»ΠΎΠΏΠ΅Ρ€Π΅Π΄Π°Ρ‡ΠΈ

Π’Π°Π±Π»ΠΈΡ†Π° ΠΊΠΎΠ½Π²Π΅ΠΊΡ‚ΠΈΠ²Π½ΠΎΠΉ Ρ‚Π΅ΠΏΠ»ΠΎΠΏΠ΅Ρ€Π΅Π΄Π°Ρ‡ΠΈ

Π’Π΅ΠΏΠ»ΠΎ ВрансфСр Π ΠΎΠ½Π° ΠšΡƒΡ€Ρ‚ΡƒΡΠ°

SfC Home> Π€ΠΈΠ·ΠΈΠΊΠ°> ВСпловая энСргия>

, Π°Π²Ρ‚ΠΎΡ€: Π ΠΎΠ½ ΠšΡƒΡ€Ρ‚ΡƒΡ (ΠΎΡ‚ 16 января 2019 Π³.)

Π’Π΅ΠΏΠ»ΠΎΠΏΠ΅Ρ€Π΅Π΄Π°Ρ‡Π° — это ΠΏΠ΅Ρ€Π΅Π΄Π°Ρ‡Π° Ρ‚Π΅ΠΏΠ»ΠΎΠ²ΠΎΠΉ энСргии ΠΎΡ‚ ΠΎΠ΄Π½ΠΎΠ³ΠΎ ΠΎΠ±ΡŠΠ΅ΠΊΡ‚Π° ΠΈΠ»ΠΈ систСмы ΠΊ Π΄Ρ€ΡƒΠ³ΠΎΠΌΡƒ с ΠΈΠ·ΠΌΠ΅Π½Π΅Π½ΠΈΠ΅ΠΌ Ρ‚Π΅ΠΌΠΏΠ΅Ρ€Π°Ρ‚ΡƒΡ€Ρ‹ ΠΊΠ°ΠΆΠ΄ΠΎΠ³ΠΎ ΠΎΠ±ΡŠΠ΅ΠΊΡ‚Π°.Π­Ρ‚ΠΎΡ‚ процСсс измСняСт Ρ‚Π΅ΠΏΠ»ΠΎΠ²ΡƒΡŽ ΡΠ½Π΅Ρ€Π³ΠΈΡŽ ΠΎΠ±Π΅ΠΈΡ… задСйствованных систСм Π΄ΠΎ Ρ‚Π΅Ρ… ΠΏΠΎΡ€, ΠΏΠΎΠΊΠ° Π½Π΅ Π±ΡƒΠ΄Π΅Ρ‚ достигнуто Ρ‚Π΅ΠΏΠ»ΠΎΠ²ΠΎΠ΅ равновСсиС .

ВСпловая энСргия ΠΌΠΎΠΆΠ΅Ρ‚ ΠΏΠ΅Ρ€Π΅Π΄Π°Π²Π°Ρ‚ΡŒΡΡ Π²Π½ΡƒΡ‚Ρ€ΠΈ Π΄Π°Π½Π½ΠΎΠ³ΠΎ ΠΌΠ°Ρ‚Π΅Ρ€ΠΈΠ°Π»Π° ΠΈΠ»ΠΈ ΠΎΡ‚ ΠΎΠ΄Π½ΠΎΠ³ΠΎ ΠΌΠ°Ρ‚Π΅Ρ€ΠΈΠ°Π»Π° ΠΊ Π΄Ρ€ΡƒΠ³ΠΎΠΌΡƒ посрСдством тСплопроводности. Для Π³Π°Π·ΠΎΠ² ΠΈΠ»ΠΈ ТидкостСй Ρ‚Π΅ΠΏΠ»ΠΎ ΠΌΠΎΠΆΠ΅Ρ‚ ΠΏΠ΅Ρ€Π΅Π΄Π°Π²Π°Ρ‚ΡŒΡΡ Π²Π½ΡƒΡ‚Ρ€ΠΈ ΠΌΠ°Ρ‚Π΅Ρ€ΠΈΠ°Π»Π° ΠΏΡƒΡ‚Π΅ΠΌ ΠΊΠΎΠ½Π²Π΅ΠΊΡ†ΠΈΠΈ. Он Ρ‚Π°ΠΊΠΆΠ΅ ΠΌΠΎΠΆΠ΅Ρ‚ ΠΏΠ΅Ρ€Π΅Π΄Π°Π²Π°Ρ‚ΡŒΡΡ косвСнно посрСдством излучСния.

Вопросы, ΠΊΠΎΡ‚ΠΎΡ€Ρ‹Π΅ ΠΌΠΎΠ³ΡƒΡ‚ Ρƒ вас Π²ΠΎΠ·Π½ΠΈΠΊΠ½ΡƒΡ‚ΡŒ:

  • Как пСрСдаСтся энСргия Π·Π° счСт проводимости?
  • Π§Ρ‚ΠΎ Ρ‚Π°ΠΊΠΎΠ΅ конвСкция?
  • Как Ρ‚Π΅ΠΏΠ»ΠΎ пСрСдаСтся ΠΈΠ·Π»ΡƒΡ‡Π΅Π½ΠΈΠ΅ΠΌ?

Π­Ρ‚ΠΎΡ‚ ΡƒΡ€ΠΎΠΊ ΠΎΡ‚Π²Π΅Ρ‚ΠΈΡ‚ Π½Π° эти вопросы.ΠŸΠΎΠ»Π΅Π·Π½Ρ‹ΠΉ инструмСнт: ΠšΠΎΠ½Π²Π΅Ρ€Ρ‚Π°Ρ†ΠΈΡ Π΅Π΄ΠΈΠ½ΠΈΡ†



ΠŸΡ€ΠΎΠ²ΠΎΠ΄ΠΈΠΌΠΎΡΡ‚ΡŒ

ВСпловая энСргия ΠΌΠΎΠΆΠ΅Ρ‚ ΠΏΠ΅Ρ€Π΅Π΄Π°Π²Π°Ρ‚ΡŒΡΡ ΠΎΡ‚ ΠΎΠ΄Π½ΠΎΠ³ΠΎ вСщСства ΠΊ Π΄Ρ€ΡƒΠ³ΠΎΠΌΡƒ, ΠΊΠΎΠ³Π΄Π° ΠΎΠ½ΠΈ находятся Π² прямом ΠΊΠΎΠ½Ρ‚Π°ΠΊΡ‚Π΅. ДвиТущиСся ΠΌΠΎΠ»Π΅ΠΊΡƒΠ»Ρ‹ ΠΎΠ΄Π½ΠΎΠ³ΠΎ ΠΌΠ°Ρ‚Π΅Ρ€ΠΈΠ°Π»Π° ΠΌΠΎΠ³ΡƒΡ‚ ΡƒΠ²Π΅Π»ΠΈΡ‡ΠΈΠ²Π°Ρ‚ΡŒ ΡΠ½Π΅Ρ€Π³ΠΈΡŽ ΠΌΠΎΠ»Π΅ΠΊΡƒΠ» Π΄Ρ€ΡƒΠ³ΠΎΠ³ΠΎ. Π’Π΅ΠΏΠ»ΠΎ Ρ‚Π°ΠΊΠΆΠ΅ ΠΌΠΎΠΆΠ΅Ρ‚ ΠΏΠ΅Ρ€Π΅ΠΌΠ΅Ρ‰Π°Ρ‚ΡŒΡΡ ΠΏΠΎ ΠΌΠ°Ρ‚Π΅Ρ€ΠΈΠ°Π»Ρƒ, ΠΏΠΎΡΠΊΠΎΠ»ΡŒΠΊΡƒ ΠΎΠ΄Π½Π° ΠΌΠΎΠ»Π΅ΠΊΡƒΠ»Π° ΠΏΠ΅Ρ€Π΅Π΄Π°Π΅Ρ‚ ΡΠ½Π΅Ρ€Π³ΠΈΡŽ сосСднСй. Π’Π°ΠΊΠΎΠΉ Ρ‚ΠΈΠΏ Ρ‚Π΅ΠΏΠ»ΠΎΠΏΠ΅Ρ€Π΅Π΄Π°Ρ‡ΠΈ называСтся Ρ‚Π΅ΠΏΠ»ΠΎΠΏΡ€ΠΎΠ²ΠΎΠ΄Π½ΠΎΡΡ‚ΡŒΡŽ , .

( Π‘ΠΌ. ΠšΠΈΠ½Π΅Ρ‚ΠΈΡ‡Π΅ΡΠΊΠ°Ρ ВСория ΠœΠ°Ρ‚Π΅Ρ€ΠΈΠΈ для Π΄Ρ€ΡƒΠ³ΠΎΠ³ΠΎ объяснСния этого.)

ΠŸΡ€ΠΎΠ²ΠΎΠ΄ΠΈΠΌΠΎΡΡ‚ΡŒ Π² основном Π½Π°Π±Π»ΡŽΠ΄Π°Π΅Ρ‚ΡΡ с Ρ‚Π²Π΅Ρ€Π΄Ρ‹ΠΌΠΈ ΠΏΡ€Π΅Π΄ΠΌΠ΅Ρ‚Π°ΠΌΠΈ, Π½ΠΎ ΠΌΠΎΠΆΠ΅Ρ‚ Π²ΠΎΠ·Π½ΠΈΠΊΠ°Ρ‚ΡŒ ΠΏΡ€ΠΈ ΠΊΠΎΠ½Ρ‚Π°ΠΊΡ‚Π΅ с Π»ΡŽΠ±Ρ‹ΠΌΠΈ ΠΌΠ°Ρ‚Π΅Ρ€ΠΈΠ°Π»Π°ΠΌΠΈ. Когда Π²Ρ‹ опускаСтС Ρ€ΡƒΠΊΡƒ Π² Π΅ΠΌΠΊΠΎΡΡ‚ΡŒ с Ρ‚Π΅ΠΏΠ»ΠΎΠΉ Π²ΠΎΠ΄ΠΎΠΉ, ΠΎΠ½Π° нагрСваСтся Π·Π° счСт тСплопроводности Π²ΠΎΠ΄Ρ‹.

НСкоторыС ΠΌΠ°Ρ‚Π΅Ρ€ΠΈΠ°Π»Ρ‹ Π»ΡƒΡ‡ΡˆΠ΅ проводят Ρ‚Π΅ΠΏΠ»ΠΎ, Ρ‡Π΅ΠΌ Π΄Ρ€ΡƒΠ³ΠΈΠ΅. НапримСр, ΠΌΠ΅Ρ‚Π°Π»Π»Ρ‹ ΡΠ²Π»ΡΡŽΡ‚ΡΡ Ρ…ΠΎΡ€ΠΎΡˆΠΈΠΌΠΈ ΠΏΡ€ΠΎΠ²ΠΎΠ΄Π½ΠΈΠΊΠ°ΠΌΠΈ Ρ‚Π΅ΠΏΠ»Π°, Π° Π΄Π΅Ρ€Π΅Π²ΠΎ — Π½Π΅Ρ‚. ΠœΠ΅Ρ‚Π°Π»Π», Π½Π°Π³Ρ€Π΅Ρ‚Ρ‹ΠΉ Π½Π° ΠΎΠ΄Π½ΠΎΠΌ ΠΊΠΎΠ½Ρ†Π΅, скоро станСт горячим ΠΈ Π½Π° Π΄Ρ€ΡƒΠ³ΠΎΠΌ ΠΊΠΎΠ½Ρ†Π΅, Ρ‡Π΅Π³ΠΎ нСльзя ΡΠΊΠ°Π·Π°Ρ‚ΡŒ ΠΎ Π΄Π΅Ρ€Π΅Π²Π΅.Π₯ΠΎΡ€ΠΎΡˆΠΈΠ΅ ΠΏΡ€ΠΎΠ²ΠΎΠ΄Π½ΠΈΠΊΠΈ элСктричСства часто ΡΠ²Π»ΡΡŽΡ‚ΡΡ Ρ…ΠΎΡ€ΠΎΡˆΠΈΠΌΠΈ ΠΏΡ€ΠΎΠ²ΠΎΠ΄Π½ΠΈΠΊΠ°ΠΌΠΈ Ρ‚Π΅ΠΏΠ»Π°.

ΠŸΠΎΡΠΊΠΎΠ»ΡŒΠΊΡƒ Π°Ρ‚ΠΎΠΌΡ‹ располоТСны Π±Π»ΠΈΠΆΠ΅ Π΄Ρ€ΡƒΠ³ ΠΊ Π΄Ρ€ΡƒΠ³Ρƒ, Ρ‚Π²Π΅Ρ€Π΄Ρ‹Π΅ Ρ‚Π΅Π»Π° проводят Ρ‚Π΅ΠΏΠ»ΠΎ Π»ΡƒΡ‡ΡˆΠ΅, Ρ‡Π΅ΠΌ Тидкости ΠΈΠ»ΠΈ Π³Π°Π·Ρ‹. Π­Ρ‚ΠΎ ΠΎΠ·Π½Π°Ρ‡Π°Π΅Ρ‚, Ρ‡Ρ‚ΠΎ Π΄Π²Π° ΡΠΎΠΏΡ€ΠΈΠΊΠ°ΡΠ°ΡŽΡ‰ΠΈΡ…ΡΡ Ρ‚Π²Π΅Ρ€Π΄Ρ‹Ρ… ΠΌΠ°Ρ‚Π΅Ρ€ΠΈΠ°Π»Π° Π±ΡƒΠ΄ΡƒΡ‚ ΠΏΠ΅Ρ€Π΅Π΄Π°Π²Π°Ρ‚ΡŒ Ρ‚Π΅ΠΏΠ»ΠΎ ΠΎΡ‚ ΠΎΠ΄Π½ΠΎΠ³ΠΎ ΠΊ Π΄Ρ€ΡƒΠ³ΠΎΠΌΡƒ Π»ΡƒΡ‡ΡˆΠ΅, Ρ‡Π΅ΠΌ Ρ‚Π²Π΅Ρ€Π΄ΠΎΠ΅ Ρ‚Π΅Π»ΠΎ ΠΏΡ€ΠΈ ΠΊΠΎΠ½Ρ‚Π°ΠΊΡ‚Π΅ с Π³Π°Π·ΠΎΠΌ ΠΈΠ»ΠΈ Π³Π°Π· с ΠΆΠΈΠ΄ΠΊΠΎΡΡ‚ΡŒΡŽ.

ΠšΠΎΠ½Π²Π΅ΠΊΡ†ΠΈΡ

Когда Π³Π°Π· ΠΈΠ»ΠΈ ΠΆΠΈΠ΄ΠΊΠΎΡΡ‚ΡŒ Π½Π°Π³Ρ€Π΅Π²Π°ΡŽΡ‚ΡΡ, горячиС участки ΠΌΠ°Ρ‚Π΅Ρ€ΠΈΠ°Π»Π° Ρ‚Π΅ΠΊΡƒΡ‚ ΠΈ ΡΠΌΠ΅ΡˆΠΈΠ²Π°ΡŽΡ‚ΡΡ с Ρ…ΠΎΠ»ΠΎΠ΄Π½Ρ‹ΠΌΠΈ участками. Π’Π°ΠΊΠΎΠΉ Π²ΠΈΠ΄ отоплСния называСтся ΠΊΠΎΠ½Π²Π΅ΠΊΡ†ΠΈΠΎΠ½Π½Ρ‹ΠΌ .ΠŸΡ€ΠΈΠ½ΡƒΠ΄ΠΈΡ‚Π΅Π»ΡŒΠ½ΠΎΠ΅ Π²ΠΎΠ·Π΄ΡƒΡˆΠ½ΠΎΠ΅ ΠΎΡ‚ΠΎΠΏΠ»Π΅Π½ΠΈΠ΅ ΠΈ ΠΊΠΎΠ½Π΄ΠΈΡ†ΠΈΠΎΠ½ΠΈΡ€ΠΎΠ²Π°Π½ΠΈΠ΅ Π²ΠΎΠ·Π΄ΡƒΡ…Π° ΡΠ²Π»ΡΡŽΡ‚ΡΡ ΠΏΡ€ΠΈΠΌΠ΅Ρ€Π°ΠΌΠΈ Π½Π°Π³Ρ€Π΅Π²Π° (ΠΈΠ»ΠΈ охлаТдСния) Π·Π° счСт ΠΊΠΎΠ½Π²Π΅ΠΊΡ†ΠΈΠΈ.

Π­Ρ‚ΠΎ эффСктивный способ пСрСноса горячСй (ΠΈΠ»ΠΈ Ρ…ΠΎΠ»ΠΎΠ΄Π½ΠΎΠΉ) Тидкости Π² Π΄Ρ€ΡƒΠ³ΠΎΠ΅ мСсто. ΠšΠΎΠ½Π²Π΅ΠΊΡ†ΠΈΡ ΠΏΠ΅Ρ€Π΅Π΄Π°Π΅Ρ‚ Ρ‚Π΅ΠΏΠ»ΠΎ Π½Π° расстояниС быстрСС, Ρ‡Π΅ΠΌ Ρ‚Π΅ΠΏΠ»ΠΎΠΏΡ€ΠΎΠ²ΠΎΠ΄Π½ΠΎΡΡ‚ΡŒ. Но Π² ΠΊΠΎΠ½Π΅Ρ‡Π½ΠΎΠΌ ΠΈΡ‚ΠΎΠ³Π΅ Ρ‚Π΅ΠΏΠ»ΠΎΠΏΡ€ΠΎΠ²ΠΎΠ΄Π½ΠΎΡΡ‚ΡŒ Π΄ΠΎΠ»ΠΆΠ½Π° ΠΏΠ΅Ρ€Π΅Π΄Π°Π²Π°Ρ‚ΡŒ Ρ‚Π΅ΠΏΠ»ΠΎ ΠΎΡ‚ Π³Π°Π·Π° Π΄Ρ€ΡƒΠ³ΠΎΠΌΡƒ ΠΎΠ±ΡŠΠ΅ΠΊΡ‚Ρƒ Ρ‡Π΅Ρ€Π΅Π· молСкулярный ΠΊΠΎΠ½Ρ‚Π°ΠΊΡ‚.

Радиация

Π’Ρ€Π΅Ρ‚ΠΈΠΉ ΠΌΠ΅Ρ‚ΠΎΠ΄ ΠΏΠ΅Ρ€Π΅Π΄Π°Ρ‡ΠΈ Ρ‚Π΅ΠΏΠ»Π° — ΠΈΠ·Π»ΡƒΡ‡Π΅Π½ΠΈΠ΅ . Π’Π΅ΠΏΠ»Ρ‹ΠΉ ΠΈΠ»ΠΈ горячий ΠΎΠ±ΡŠΠ΅ΠΊΡ‚ испускаСт инфракрасноС элСктромагнитноС ΠΈΠ·Π»ΡƒΡ‡Π΅Π½ΠΈΠ΅, ΠΊΠΎΡ‚ΠΎΡ€ΠΎΠ΅ ΠΌΠΎΠΆΠ΅Ρ‚ ΠΏΠΎΠ³Π»ΠΎΡ‰Π°Ρ‚ΡŒΡΡ Π΄Ρ€ΡƒΠ³ΠΈΠΌ ΠΎΠ±ΡŠΠ΅ΠΊΡ‚ΠΎΠΌ Π½Π° Π½Π΅ΠΊΠΎΡ‚ΠΎΡ€ΠΎΠΌ расстоянии, нагрСвая Π΅Π³ΠΎ.Однако ΠΏΠ΅Ρ€Π΅Π΄Π°Ρ‡Π° Ρ‚Π΅ΠΏΠ»Π° Ρ€Π°Π±ΠΎΡ‚Π°Π΅Ρ‚ Ρ‚ΠΎΠ»ΡŒΠΊΠΎ Π² ΠΎΠ΄Π½ΠΎΠΌ Π½Π°ΠΏΡ€Π°Π²Π»Π΅Π½ΠΈΠΈ. Π’Ρ‹ Π½Π΅ ΠΌΠΎΠΆΠ΅Ρ‚Π΅ ΠΎΡ…Π»Π°Π΄ΠΈΡ‚ΡŒ ΠΎΠ±ΡŠΠ΅ΠΊΡ‚ ΠΈΠ·Π»ΡƒΡ‡Π΅Π½ΠΈΠ΅ΠΌ, Π² ΠΎΡ‚Π»ΠΈΡ‡ΠΈΠ΅ ΠΎΡ‚ тСплопроводности ΠΈΠ»ΠΈ ΠΊΠΎΠ½Π²Π΅ΠΊΡ†ΠΈΠΈ.

ЭлСктричСскиС ΠΎΠ±ΠΎΠ³Ρ€Π΅Π²Π°Ρ‚Π΅Π»ΠΈ ΠΈΡΠΏΠΎΠ»ΡŒΠ·ΡƒΡŽΡ‚ ΠΈΠ·Π»ΡƒΡ‡Π΅Π½ΠΈΠ΅ для ΠΎΠ±ΠΎΠ³Ρ€Π΅Π²Π° помСщСния. Если Π΄ΠΎΠ±Π°Π²Π»Π΅Π½ вСнтилятор, устройство ΠΈΡΠΏΠΎΠ»ΡŒΠ·ΡƒΠ΅Ρ‚ ΠΊΠ°ΠΊ ΠΈΠ·Π»ΡƒΡ‡Π΅Π½ΠΈΠ΅, Ρ‚Π°ΠΊ ΠΈ ΠΊΠΎΠ½Π²Π΅ΠΊΡ†ΠΈΡŽ для ΠΎΠ±ΠΎΠ³Ρ€Π΅Π²Π° помСщСния.

ΠŸΠ΅Ρ€Π΅Π΄Π°Ρ‡Π° Ρ‚Π΅ΠΏΠ»Π° ΠΈΠ·Π»ΡƒΡ‡Π΅Π½ΠΈΠ΅ΠΌ распространяСтся со ΡΠΊΠΎΡ€ΠΎΡΡ‚ΡŒΡŽ свСта ΠΈ распространяСтся Π½Π° большиС расстояния, Π΄Π°ΠΆΠ΅ Π² Π²Π°ΠΊΡƒΡƒΠΌΠ΅. ΠœΡ‹ Π½Π°Π³Ρ€Π΅Π²Π°ΡŽΡ‚ΡΡ ΠΎΡ‚ Π‘ΠΎΠ»Π½Ρ†Π° Π·Π° счСт Ρ€Π°Π΄ΠΈΠ°Ρ†ΠΈΠΎΠ½Π½ΠΎΠΉ ΠΏΠ΅Ρ€Π΅Π΄Π°Ρ‡ΠΈ Ρ‚Π΅ΠΏΠ»Π°.

( Π‘ΠΌ. Π˜Π½Ρ„Ρ€Π°ΠΊΡ€Π°ΡΠ½ΠΎΠ΅ ΠΈΠ·Π»ΡƒΡ‡Π΅Π½ΠΈΠ΅ для получСния Π΄ΠΎΠΏΠΎΠ»Π½ΠΈΡ‚Π΅Π»ΡŒΠ½ΠΎΠΉ ΠΈΠ½Ρ„ΠΎΡ€ΠΌΠ°Ρ†ΠΈΠΈ ΠΏΠΎ этому вопросу.)

РСзюмС

ВСпловая энСргия ΠΌΠΎΠΆΠ΅Ρ‚ ΠΏΠ΅Ρ€Π΅Π΄Π°Π²Π°Ρ‚ΡŒΡΡ ΠΎΡ‚ ΠΎΠ΄Π½ΠΎΠ³ΠΎ ΠΊΠΎΠ½Ρ†Π° ΠΌΠ°Ρ‚Π΅Ρ€ΠΈΠ°Π»Π° ΠΊ Π΄Ρ€ΡƒΠ³ΠΎΠΌΡƒ ΠΈΠ»ΠΈ ΠΎΡ‚ ΠΎΠ΄Π½ΠΎΠ³ΠΎ ΠΌΠ°Ρ‚Π΅Ρ€ΠΈΠ°Π»Π° ΠΊ Π΄Ρ€ΡƒΠ³ΠΎΠΌΡƒ посрСдством тСплопроводности. Он Ρ‚Π°ΠΊΠΆΠ΅ косвСнно пСрСносится ΠΊΠΎΠ½Π²Π΅ΠΊΡ†ΠΈΠ΅ΠΉ ΠΈ ΠΈΠ·Π»ΡƒΡ‡Π΅Π½ΠΈΠ΅ΠΌ.

Атомы ΠΈΠ»ΠΈ ΠΌΠΎΠ»Π΅ΠΊΡƒΠ»Ρ‹ ΠΏΠ΅Ρ€Π΅Π΄Π°ΡŽΡ‚ ΠΊΠΈΠ½Π΅Ρ‚ΠΈΡ‡Π΅ΡΠΊΡƒΡŽ ΡΠ½Π΅Ρ€Π³ΠΈΡŽ своим сосСдям посрСдством столкновСний. Когда ΠΎΠ±ΡŠΠ΅ΠΊΡ‚Ρ‹ ΡΠΎΠΏΡ€ΠΈΠΊΠ°ΡΠ°ΡŽΡ‚ΡΡ, Ρ‚Π΅ΠΏΠ»ΠΎ пСрСдаСтся Π·Π° счСт тСплопроводности. Жидкости ΠΈ Π³Π°Π·Ρ‹ ΠΌΠΎΠ³ΡƒΡ‚ ΠΏΠ΅Ρ€Π΅ΠΌΠ΅Ρ‰Π°Ρ‚ΡŒ ΠΌΠΎΠ»Π΅ΠΊΡƒΠ»Ρ‹ с высокой ΠΈΠ»ΠΈ Π½ΠΈΠ·ΠΊΠΎΠΉ энСргиСй Π² Π΄Ρ€ΡƒΠ³ΡƒΡŽ ΠΎΠ±Π»Π°ΡΡ‚ΡŒ посрСдством ΠΊΠΎΠ½Π²Π΅ΠΊΡ†ΠΈΠΈ.Атомы ΠΌΠΎΠ³ΡƒΡ‚ ΠΈΠ·Π»ΡƒΡ‡Π°Ρ‚ΡŒ ΡΠ½Π΅Ρ€Π³ΠΈΡŽ, которая ΠΌΠΎΠΆΠ΅Ρ‚ Π²ΠΎΠ·Π±ΡƒΠΆΠ΄Π°Ρ‚ΡŒ ΡΠ½Π΅Ρ€Π³ΠΈΡŽ ΡƒΠ΄Π°Π»Π΅Π½Π½ΠΎΠ³ΠΎ Π°Ρ‚ΠΎΠΌΠ°, Ρ‡Ρ‚ΠΎ ΠΏΡ€ΠΈΠ²ΠΎΠ΄ΠΈΡ‚ ΠΊ ΠΏΠ΅Ρ€Π΅Π΄Π°Ρ‡Π΅ Ρ‚Π΅ΠΏΠ»Π° Π·Π° счСт излучСния.


ΠŸΠ΅Ρ€Π΅Π΄Π°ΠΉΡ‚Π΅ свои знания Π΄Ρ€ΡƒΠ³ΠΈΠΌ


РСсурсы ΠΈ ссылки

ΠŸΠΎΠ»Π½ΠΎΠΌΠΎΡ‡ΠΈΡ Π ΠΎΠ½Π° ΠšΡƒΡ€Ρ‚ΡƒΡΠ°

Π‘Π°ΠΉΡ‚ΠΎΠ²

Π’Π΅ΠΏΠ»ΠΎΠΏΠ΅Ρ€Π΅Π΄Π°Ρ‡Π° — ВикипСдия

ЀизичСскиС рСсурсы

Книги

Книги ΠΏΠΎ Ρ‚Π΅ΠΏΠ»ΠΎΠ²ΠΎΠΉ энСргии с самым высоким Ρ€Π΅ΠΉΡ‚ΠΈΠ½Π³ΠΎΠΌ

Π›ΡƒΡ‡ΡˆΠΈΠ΅ ΠΊΠ½ΠΈΠ³ΠΈ ΠΏΠΎ Ρ„ΠΈΠ·ΠΈΠΊΠ΅ Ρ‚Π΅ΠΏΠ»ΠΎΠΏΠ΅Ρ€Π΅Π΄Π°Ρ‡ΠΈ


Вопросы ΠΈ ΠΊΠΎΠΌΠΌΠ΅Π½Ρ‚Π°Ρ€ΠΈΠΈ

Π•ΡΡ‚ΡŒ Π»ΠΈ Ρƒ вас ΠΊΠ°ΠΊΠΈΠ΅-Π»ΠΈΠ±ΠΎ вопросы, ΠΊΠΎΠΌΠΌΠ΅Π½Ρ‚Π°Ρ€ΠΈΠΈ ΠΈΠ»ΠΈ мнСния ΠΏΠΎ этой Ρ‚Π΅ΠΌΠ΅? Если Π΄Π°, ΠΎΡ‚ΠΏΡ€Π°Π²ΡŒΡ‚Π΅ свой ΠΎΡ‚Π·Ρ‹Π² ΠΏΠΎ элСктронной ΠΏΠΎΡ‡Ρ‚Π΅.Π― ΠΏΠΎΡΡ‚Π°Ρ€Π°ΡŽΡΡŒ Π²Π΅Ρ€Π½ΡƒΡ‚ΡŒΡΡ ΠΊ Π²Π°ΠΌ ΠΊΠ°ΠΊ ΠΌΠΎΠΆΠ½ΠΎ скорСС.


ΠŸΠΎΠ΄Π΅Π»ΠΈΡ‚ΡŒΡΡ страницСй

НаТмитС ΠΊΠ½ΠΎΠΏΠΊΡƒ, Ρ‡Ρ‚ΠΎΠ±Ρ‹ Π΄ΠΎΠ±Π°Π²ΠΈΡ‚ΡŒ эту страницу Π² Π·Π°ΠΊΠ»Π°Π΄ΠΊΠΈ ΠΈΠ»ΠΈ ΠΏΠΎΠ΄Π΅Π»ΠΈΡ‚ΡŒΡΡ Сю Ρ‡Π΅Ρ€Π΅Π· Twitter, Facebook, ΡΠ»Π΅ΠΊΡ‚Ρ€ΠΎΠ½Π½ΡƒΡŽ ΠΏΠΎΡ‡Ρ‚Ρƒ ΠΈΠ»ΠΈ Π΄Ρ€ΡƒΠ³ΠΈΠ΅ слуТбы:


Π‘Ρ‚ΡƒΠ΄Π΅Π½Ρ‚Ρ‹ ΠΈ исслСдоватСли

Π’Π΅Π±-адрСс этой страницы:
www.

alexxlab

Π”ΠΎΠ±Π°Π²ΠΈΡ‚ΡŒ ΠΊΠΎΠΌΠΌΠ΅Π½Ρ‚Π°Ρ€ΠΈΠΉ

Π’Π°Ρˆ адрСс email Π½Π΅ Π±ΡƒΠ΄Π΅Ρ‚ ΠΎΠΏΡƒΠ±Π»ΠΈΠΊΠΎΠ²Π°Π½. ΠžΠ±ΡΠ·Π°Ρ‚Π΅Π»ΡŒΠ½Ρ‹Π΅ поля ΠΏΠΎΠΌΠ΅Ρ‡Π΅Π½Ρ‹ *